Sunteți pe pagina 1din 287

[G.R. No. 81561. January 18, 1991.

PEOPLE OF THE PHILIPPINES, plaintiff-appellee, vs. ANDRE


MARTI, accused-appellant.

The Solicitor General for plaintiff-appellee.


Reynaldo B . Tatoy and Abelardo E . Rogacion for accused-appellant.

SYLLABUS

1. CONSTITUTIONAL LAW; BILL OF RIGHTS; GUARANTEE AGAINST


UNREASONABLE SEARCHES AND SEIZURES; PRONOUNCEMENT OF
UNITED STATES FEDERAL SUPREME COURT AND STATE APPELLATE
COURTS, DOCTRINAL IN THIS JURISDICTION. — Our present
constitutional provision on the guarantee against unreasonable search
and seizure had its origin in the 1935 Charter which was in turn derived
almost verbatim from the Fourth Amendment to the United
States Constitution. As such, the Court may turn to the pronouncements
of the United States Federal Supreme Court and State Appellate Courts
which are considered doctrinal in this jurisdiction.
2. REMEDIAL LAW; EVIDENCE; ADMISSIBILITY; EXCLUSIONARY RULE ON
EVIDENCE OBTAINED IN VIOLATION OF THE GUARANTEE AGAINST
UNREASONABLE SEARCHES AND SEIZURES. — In a number of cases, the
Court strictly adhered to the exclusionary rule and has struck down the
admissibility of evidence obtained in violation of the constitutional
safeguard against unreasonable searches and seizures. (Bache & Co.,
(Phil.), Inc., v. Ruiz, 37 SCRA 823 [1971]; Lim v. Ponce de Leon, 66 SCRA
299 [1975]; People v. Burgos, 144 SCRA 1 [1986]; Roan v. Gonzales, 145
SCRA 687 [1987]; See also Salazar v. Hon. Achacoso, et al., GR No. 81510,
March 14, 1990).
3. CONSTITUTIONAL LAW; BILL OF RIGHTS; LIBERTIES MAY BE INVOKED
ONLY AGAINST THE STATE, NOT UPON PRIVATE INDIVIDUALS. — In the
absence of governmental interference, the liberties guaranteed by the
Constitution cannot be invoked against the State. As this Court held in
Villanueva v. Querubin (48 SCRA 345 [1972]: 1. This constitutional
right (against unreasonable search and seizure) refers to the immunity of
one's person, whether citizen or alien, from interference by
government, included in which is his residence, his papers, and other
possessions . . . That the Bill of Rights embodied in the Constitution is
not meant to be invoked against acts of private individuals finds support
in the deliberations of the Constitutional Commission. The constitutional
proscription against unlawful searches and seizures therefore applies as
a restraint directed only against the government and its agencies tasked
with the enforcement of the law. Thus, it could only be invoked against
the State to whom the restraint against arbitrary and unreasonable
exercise of power is imposed.
4. ID.; ID.; ID.; GUARANTEE AGAINST UNREASONABLE SEARCH AND
SEIZURE; CASE AT BAR. — The contraband in the case at bar having
come into possession of the Government without the latter transgressing
appellant's rights against unreasonable search and seizure, the Court
sees no cogent reason why the same should not be admitted against him
in the prosecution of the offense charged. If the search is made upon the
request of law enforcers, a warrant must generally be first secured if it is
to pass the test of constitutionality. However, if the search is made at the
behest or initiative of the proprietor of a private establishment for its
own and private purposes, as in the case at bar, and without the
intervention of police authorities, the right against unreasonable search
and seizure cannot be invoked for only the act of private individual, not
the law enforcers, is involved. In sum, the protection against
unreasonable searches and seizures cannot be extended to acts
committed by private individuals so as to bring it within the ambit of
alleged unlawful intrusion by the government.
5. ID.; ID.; ID.; ID.; MERE PRESENCE OF NBI AGENTS TO OBSERVE AND
LOOK AT WHICH IS IN PLAIN SIGHT IS NOT A SEARCH. — The mere
presence of the NBI agents did not convert the reasonable search
effected by Reyes into a warrantless search and seizure proscribed
by the Constitution. Merely to observe and look at that which is in plain
sight is not a search. Having observed that which is open, where no
trespass has been committed in aid thereof, is not search (Chadwick v.
State, 429 SW2d 135). Where the contraband articles are identified
without a trespass on the part of the arresting officer, there is not the
search that is prohibited by the constitution (USv. Lee 274 US 559, 71
L.Ed. 1202 [1927]; Ker v. State of California 374 US 23, 10 L. Ed. 2d. 726
[1963]; Moore v. State, 429 SW2d 122 [1968]).
6. ID.; CONSTITUTION; DOES NOT GOVERN RELATIONSHIP BETWEEN
INDIVIDUALS. — The constitution, in laying down the principles of the
government and fundamental liberties of the people, does not govern
relationships between individuals.
7. REMEDIAL LAW; EVIDENCE; ADMISSIBILITY; EVIDENCE PROCURED BY
INDIVIDUALS EFFECTED THROUGH PRIVATE SEIZURE, ADMISSIBLE. —
Similarly, the admissibility of the evidence procured by an individual
effected through private seizure equally applies, in pari passu, to the
alleged violation, non-governmental as it is, of appellant's constitutional
rights to privacy and communication.
8. ID.; ID.; CREDIBILITY; SELF-SERVING DENIALS, DESERVE NO WEIGHT
IN LAW. — Denials, if unsubstantiated by clear and convincing evidence,
are negative self-serving evidence which deserve no weight in law and
cannot be given greater evidentiary weight than the testimony of
credible witnesses who testify on affirmative matters (People v. Esquillo,
171 SCRA 571 [1989]; People vs. Sariol, 174 SCRA 237 [1989]).
9. ID.; ID.; ID.; REQUISITE FOR EVIDENCE TO BE BELIEVED. — Evidence,
to be believed, must not only proceed from the mouth of a credible
witness, but it must be credible in itself such as the common experience
and observation of mankind can approve as probable under the
circumstances.
10. ID.; ID.; BURDEN OF PROOF AND PRESUMPTIONS; THINGS WHICH A
PERSON POSSESSES ARE PRESUMED OWNED BY HIM; CASE AT BAR. —
As records further show, appellant did not even bother to ask Michael's
full name, his complete address or passport number. Furthermore, if
indeed, the German national was the owner of the merchandise,
appellant should have so indicated in the contract of shipment. On the
contrary, appellant signed the contract as the owner and shipper thereof
giving more weight to the presumption that things which a person
possesses, or exercises acts of ownership over, are owned by him (Sec. 5
[j], Rule 131). At this point, appellant is therefore estopped to claim
otherwise.

DECISION

BIDIN, J :
p

This is an appeal from a decision * rendered by the Special Criminal


Court of Manila (Regional Trial Court, Branch XLIX) convicting accused-
appellant of violation ofSection 21 (b), Article IV in relation to Section 4,
Article II and Section 2 (e)(i), Article I of Republic Act 6425, as amended,
otherwise known as the Dangerous Drugs Act.
The facts as summarized in the brief of the prosecution are as follows:
"On August 14, 1987, between 10:00 and 11:00 a.m., the appellant
and his common-law wife, Shirley Reyes, went to the booth of the
"Manila Packing and Export Forwarders" in the Pistang Pilipino
Complex, Ermita, Manila, carrying with them four (4) gift-wrapped
packages. Anita Reyes (the proprietress and no relation to Shirley
Reyes) attended to them. The appellant informed Anita Reyes that
he was sending the packages to a friend in Zurich, Switzerland.
Appellant filled up the contract necessary for the transaction,
writing therein his name, passport number, the date of shipment
and the name and address of the consignee, namely, "WALTER
FIERZ, Mattacketr II, 8052 Zurich, Switzerland" (Decision, p. 6)
"Anita Reyes then asked the appellant if she could examine and
inspect the packages. Appellant, however, refused, assuring her
that the packages simply contained books, cigars, and gloves and
were gifts to his friend in Zurich. In view of appellant's
representation, Anita Reyes no longer insisted on inspecting the
packages. The four (4) packages were then placed inside a brown
corrugated box one by two feet in size (1' x 2'). Styro-foam was
placed at the bottom and on top of the packages before the box
was sealed with masking tape, thus making the box ready for
shipment (Decision, p. 8).LLpr

"Before delivery of appellant's box to the Bureau of Customs and/or


Bureau of Posts, Mr. Job Reyes (proprietor) and husband of Anita
(Reyes), following standard operating procedure, opened the boxes
for final inspection. When he opened appellant's box, a peculiar
odor emitted therefrom. His curiosity aroused, he squeezed one of
the bundles allegedly containing gloves and felt dried leaves
inside. Opening one of the bundles, he pulled out a cellophane
wrapper protruding from the opening of one of the gloves. He made
an opening on one of the cellophane wrappers and took several
grams of the contents thereof (tsn, pp. 29-30, October 6, 1987;
Emphasis supplied).
"Job Reyes forthwith prepared a letter reporting the shipment to
the NBI and requesting a laboratory examination of the samples he
extracted from the cellophane wrapper (tsn, pp. 5-6, October 6,
1987).
"He brought the letter and a sample of appellant's shipment to the
Narcotics Section of the National Bureau of Investigation (NBI), at
about 1:30 o'clock in the afternoon of that date, i.e., August 14,
1987. He was interviewed by the Chief of Narcotics Section. Job
Reyes informed the NBI that the rest of the shipment was still in
his office. Therefore, Job Reyes and three (3) NBI agents, and a
photographer, went to the Reyes' office at Ermita, Manila (tsn, p.
30, October 6, 1987).
"Job Reyes brought out the box in which appellant's packages
were placed and, in the presence of the NBI agents, opened the
top flaps, removed the styro-foam and took out the cellophane
wrappers from inside the gloves. Dried marijuana leaves were
found to have been contained inside the cellophane wrappers (tsn,
p. 38, October 6, 1987; Emphasis supplied).
"The package which allegedly contained books was likewise
opened by Job Reyes. He discovered that the package contained
bricks or cake-like dried marijuana leaves. The package which
allegedly contained tabacalera cigars was also opened. It turned
out that dried marijuana leaves were neatly stocked underneath
the cigars (tsn, p. 39, October 6, 1987).

"The NBI agents made an inventory and took charge of the box and
of the contents thereof, after signing a "Receipt"
acknowledging custody of the said effects (tsn, pp. 2-3, October 7,
1987).
Thereupon, the NBI agents tried to locate appellant but to no avail.
Appellant's stated address in his passport being the Manila Central Post
Office, the agents requested assistance from the latter's Chief Security.
On August 27, 1987, appellant, while claiming his mail at the Central Post
Office, was invited by the NBI to shed light on the attempted shipment of
the seized dried leaves. On the same day the Narcotics Section of the
NBI submitted the dried leaves to the Forensic Chemistry Section for
laboratory examination. It turned out that the dried leaves were
marijuana flowering tops as certified by the forensic chemist. (Appellee's
Brief, pp. 9-11, Rollo, pp. 132-134).
Thereafter, an Information was filed against appellant for violation of RA
6425, otherwise known as the Dangerous Drugs Act.
After trial, the court a quo rendered the assailed decision.
In this appeal, accused/appellant assigns the following errors, to wit:
"THE LOWER COURT ERRED IN ADMITTING IN EVIDENCE THE
ILLEGALLY SEARCHED AND SEIZED OBJECTS CONTAINED IN THE
FOUR PARCELS.
"THE LOWER COURT ERRED IN CONVICTING APPELLANT DESPITE
THE UNDISPUTED FACT THAT HIS RIGHTS UNDER THE
CONSTITUTION WHILE UNDER CUSTODIAL PROCEEDINGS WERE
NOT OBSERVED. cdphil

"THE LOWER COURT ERRED IN NOT GIVING CREDENCE TO THE


EXPLANATION OF THE APPELLANT ON HOW THE FOUR PARCELS
CAME INTO HIS POSSESSION." (Appellant's Brief, p. 1; Rollo, p. 55)
1. Appellant contends that the evidence subject of the imputed offense
had been obtained in violation of his constitutional rights against
unreasonable search and seizure and privacy of communication (Secs. 2
and 3, Art. III, Constitution) and therefore argues that the same should be
held inadmissible in evidence (Sec. 3 (2), Art. III).
Sections 2 and 3, Article III of the Constitution provide:
"Section 2. The right of the people to be secure in their persons,
houses, papers and effects against unreasonable searches and
seizures of whatever nature and for any purpose shall be
inviolable, and no search warrant or warrant of arrest shall issue
except upon probable cause to be determined personally by the
judge after examination under oath or affirmation of the
complainant and the witnesses he may produce, and particularly
describing the place to be searched and the persons or things to
be seized.
"Section 3. (1) The privacy of communication and correspondence
shall be inviolable except upon lawful order of the court, or when
public safety or order requires otherwise as prescribed by law.
"(2) Any evidence obtained in violation of this or the preceding
section shall be inadmissible for any purpose in any proceeding."
Our present constitutional provision on the guarantee against
unreasonable search and seizure had its origin in the 1935 Charter
which, worded as follows:
"The right of the people to be secure in their persons, houses,
papers and effects against unreasonable searches and seizures
shall not be violated, and no warrants shall issue but
upon probable cause, to be determined by the judge after
examination under oath or affirmation of the complainant and the
witnesses he may produce, and particularly describing the place to
be searched, and the persons or things to be seized." (Sec. 1 [3],
Article III).
was in turn derived almost verbatim from the Fourth Amendment ** to
the United States Constitution. As such, the Court may turn to the
pronouncements of the United States Federal Supreme Court and
State Appellate Courts which are considered doctrinal in this
jurisdiction.
Thus, following the exclusionary rule laid down in Mapp v. Ohio by the US
Federal Supreme Court (367 US 643, 81 S.Ct. 1684, 6 L.Ed. 1081 2d
[1961]), this Court, in Stonehill v. Diokno (20 SCRA 383 [1967]), declared
as inadmissible any evidence obtained by virtue of a defective search
and seizure warrant, abandoning in the process the ruling earlier adopted
in Moncado v. People's Court (80 Phil. 1 [1948]) wherein the admissibility
of evidence was not affected by the illegality of its seizure. The 1973
Charter (Sec. 4 [2], Art. IV) constitutionalized the Stonehill ruling and is
carried over up to the present with the advent of the 1987 Constitution.
In a number of cases, the Court strictly adhered to the exclusionary rule
and has struck down the admissibility of evidence obtained in violation
of the constitutional safeguard against unreasonable searches and
seizures. (Bache & Co., (Phil.), Inc., v. Ruiz, 37 SCRA 823 [1971]; Lim v.
Ponce de Leon, 66 SCRA 299 [1975]; People v. Burgos, 144 SCRA 1 [1986];
Roan v. Gonzales, 145 SCRA 687 [1987]; See also Salazar v. Hon.
Achacoso, et al., GR No. 81510, March 14, 1990).
It must be noted, however, that in all those cases adverted to, the
evidence so obtained were invariably procured by the State acting
through the medium of its law enforcers or other authorized government
agencies.LLpr

On the other hand, the case at bar assumes a peculiar character since
the evidence sought to be excluded was primarily discovered and
obtained by a private person, acting in a private capacity and without the
intervention and participation of State authorities. Under the
circumstances, can accused/appellant validly claim that his
constitutional right against unreasonable searches and seizure has been
violated? Stated otherwise, may an act of a private individual, allegedly
in violation of appellant's constitutional rights, be invoked against the
State?
We hold in the negative. In the absence of governmental interference, the
liberties guaranteed by the Constitution cannot be invoked against the
State.
As this Court held in Villanueva v. Querubin (48 SCRA 345 [1972]:
"1. This constitutional right (against unreasonable search and
seizure) refers to the immunity of one's person, whether citizen or
alien, from interference by government, included in which is his
residence, his papers, and other possessions . . .
". . . There the state, however powerful, does not as such have the
access except under the circumstances above noted, for in the
traditional formulation, his house, however humble, is his
castle. Thus is outlawed any unwarranted intrusion by government,
which is called upon to refrain from any invasion of his dwelling
and to respect the privacies of his life . . ." (Cf. Schermerber v.
California, 384 US 757 [1966] and Boyd v. United States, 116 US
616 [1886]; Emphasis supplied).
In Burdeau v. McDowell (256 US 465 (1921), 41 S Ct. 547; 65 L.Ed. 1048),
the Court there in construing the right against unreasonable searches
and seizures declared that:
"(t)he Fourth Amendment gives protection against unlawful
searches and seizures, and as shown in previous cases, its
protection applies to governmental action. Its origin and history
clearly show that it was intended as a restraint upon the activities
of sovereign authority, and was not intended to be a limitation
upon other than governmental agencies; as against such authority
it was the purpose of the Fourth Amendment to secure the citizen
in the right of unmolested occupation of his dwelling and the
possession of his property, subject to the right of seizure by
process duly served."
The above ruling was reiterated in State v. Bryan (457 P.2d 661 [1968])
where a parking attendant who searched the automobile to ascertain the
owner thereof found marijuana instead, without the knowledge and
participation of police authorities, was declared admissible in
prosecution for illegal possession of narcotics.
And again in the 1969 case of Walker v. State (429 S.W.2d 121), it was
held that the search and seizure clauses are restraints upon the
government and its agents, not upon private individuals, (citing People v.
Potter, 240 Cal. App.2d 621, 49 Cap. Rptr, 892 (1966); State v. Brown, Mo.,
391 S.W.2d 903 (1965); State v. Olsen, Or., 317 P.2d 938 (1957).
Likewise appropos is the case of Bernas v. US (373 F.2d 517 (1967). The
Court there said:
"The search of which appellant complains, however, was made by a
private citizen — the owner of a motel in which appellant stayed
overnight and in which he left behind a travel case containing the
evidence *** complained of. The search was made on the motel
owner's own initiative. Because of it, he became suspicious, called
the local police, informed them of the bag's contents, and made it
available to the authorities.
"The fourth amendment and the case law applying it do not require
exclusion of evidence obtained through a search by a private
citizen. Rather, the amendment only proscribes governmental
action."
The contraband in the case at bar having come into possession of the
Government without the latter transgressing appellant's rights against
unreasonable search and seizure, the Court sees no cogent reason why
the same should not be admitted against him in the prosecution of the
offense charged. LLphil

Appellant, however, would like this court to believe that NBI agents made
an illegal search and seizure of the evidence later on used in prosecuting
the case which resulted in his conviction.
The postulate advanced by accused/appellant needs to be clarified in
two days. In both instances, the argument stands to fall on its own
weight, or the lack of it.
First, the factual considerations of the case at bar readily foreclose the
proposition that NBI agents conducted an illegal search and seizure of
the prohibited merchandise. Records of the case clearly indicate that it
was Mr. Job Reyes, the proprietor of the forwarding agency, who made
search/inspection of the packages. Said inspection was reasonable and a
standard operating procedure on the part of Mr. Reyes as a precautionary
measure before delivery of packages to the Bureau of Customs or the
Bureau of Posts (TSN, October 6 & 7, 1987, pp. 15-18; pp. 7-8; Original
Records, pp. 119-122; 167-168).
It will be recalled that after Reyes opened the box containing the illicit
cargo, he took samples of the same to the NBI and later summoned the
agents to his place of business. Thereafter, he opened the parcels
containing the rest of the shipment and entrusted the care and custody
thereof to the NBI agents. Clearly, the NBI agents made no search and
seizure, much less an illegal one, contrary to the postulate of
accused/appellant.

Second, the mere presence of the NBI agents did not convert the
reasonable search effected by Reyes into a warrantless search and
seizure proscribed by the Constitution. Merely to observe and look at
that which is in plain sight is not a search. Having observed that which is
open, where no trespass has been committed in aid thereof, is not search
(Chadwick v. State, 429 SW2d 135). Where the contraband articles are
identified without a trespass on the part of the arresting officer, there is
not the search that is prohibited by the constitution (US v. Lee 274 US
559, 71 L.Ed. 1202 [1927]; Ker v. State of California 374 US 23, 10 L.Ed.2d.
726 [1963]; Moore v. State, 429 SW2d 122 [1968]).
In Gandy v. Watkins (237 F. Supp. 266 [1964]), it was likewise held that
where the property was taken into custody of the police at the specific
request of the manager and where the search was initially made by the
owner there is no unreasonable search and seizure within the
constitutional meaning of the term.
That the Bill of Rights embodied in the Constitution is not meant to be
invoked against acts of private individuals finds support in the
deliberations of the Constitutional Commission. True, the liberties
guaranteed by the fundamental law of the land must always be subject to
protection. But protection against whom? Commissioner Bernas in his
sponsorship speech in the Bill of Rights answers the query which he
himself posed, as follows:
"First, the general reflections. The protection of fundamental
liberties in the essence of constitutional democracy. Protection
against whom? Protection against the state. The Bill of Rights
governs the relationship between the individual and the state . Its
concern is not the relation between individuals, between a private
individual and other individuals. What the Bill of Rights does is to
declare some forbidden zones in the private sphere inaccessible to
any power holder." (Sponsorship Speech of Commissioner Bernas;
Record of the Constitutional Commission, Vol. 1, p. 674; July 17,
1986; Emphasis supplied)
The constitutional proscription against unlawful searches and seizures
therefore applies as a restraint directed only against the government and
its agencies tasked with the enforcement of the law. Thus, it could only
be invoked against the State to whom the restraint against arbitrary and
unreasonable exercise of power is imposed. cdphil

If the search is made upon the request of law enforcers, a warrant must
generally be first secured if it is to pass the test of constitutionality.
However, if the search is made at the behest or initiative of the proprietor
of a private establishment for its own and private purposes, as in the
case at bar, and without the intervention of police authorities, the right
against unreasonable search and seizure cannot be invoked for only the
act of private individual, not the law enforcers, is involved. In sum, the
protection against unreasonable searches and seizures cannot be
extended to acts committed by private individuals so as to bring it within
the ambit of alleged unlawful intrusion by the government.
Appellant argues, however, that since the provisions of the 1935
Constitution has been modified by the present phraseology found in the
1987 Charter, expressly declaring as inadmissible any evidence obtained
in violation of the constitutional prohibition against illegal search and
seizure, it matters not whether the evidence was procured by police
authorities or private individuals (Appellant's Brief, p. 8, Rollo, p. 62).
The argument is untenable. For one thing, the constitution, in laying
down the principles of the government and fundamental liberties of the
people, does not govern relationships between individuals. Moreover, it
must be emphasized that the modifications introduced in the 1987
Constitution (re: Sec. 2, Art. III) relate to the issuance of either a search
warrant or warrant of arrest vis-a-vis the responsibility of the judge in the
issuance thereof (See Soliven v. Makasiar, 167 SCRA 393 [1988]; Circular
No. 13 [October 1, 1985] and Circular No. 12 [June 30, 1987]. The
modifications introduced deviate in no manner as to whom the
restriction or inhibition against unreasonable search and seizure is
directed against. The restraint stayed with the State and did not shift to
anyone else.
Corollarily, alleged violations against unreasonable search and seizure
may only be invoked against the State by an individual unjustly traduced
by the exercise of sovereign authority. To agree with appellant that an
act of a private individual in violation of the Bill of Rights should also be
construed as an act of the State would result in serious legal
complications and an absurd interpretation of the constitution.
Similarly, the admissibility of the evidence procured by an individual
effected through private seizure equally applies, in pari passu, to the
alleged violation, non-governmental as it is, of appellant's constitutional
rights to privacy and communication.
2. In his second assignment of error, appellant contends that the lower
court erred in convicting him despite the undisputed fact that his rights
under the constitution while under custodial investigation were not
observed.
Again, the contention is without merit, We have carefully examined the
records of the case and found nothing to indicate, as an "undisputed
fact", that appellant was not informed of his constitutional rights or that
he gave statements without the assistance of counsel. The law enforcers
testified that accused/appellant was informed of his constitutional
rights. It is presumed that they have regularly performed their duties
(Sec. 5(m), Rule 131) and their testimonies should be given full faith and
credence, there being no evidence to the contrary. What is clear from the
records, on the other hand, is that appellant refused to give any written
statement while under investigation as testified by Atty. Lastimoso of the
NBI, Thus:
"Fiscal Formoso:
"You said that you investigated Mr. and Mrs. Job Reyes.
What about the accused here, did you investigate the
accused together with the girl?
"WITNESS:
"Yes, we have interviewed the accused together with the
girl but the accused availed of his constitutional right not
to give any written statement, sir." (TSN, October 8, 1987,
p. 62; Original Records, p. 240)
The above testimony of the witness for the prosecution was not
contradicted by the defense on cross-examination. As borne out by the
records, neither was there any proof by the defense that appellant gave
uncounselled confession while being investigated. What is more, we have
examined the assailed judgment of the trial court and nowhere is there
any reference made to the testimony of appellant while under custodial
investigation which was utilized in the finding of conviction. Appellant's
second assignment of error is therefore misplaced. cdphil

3. Coming now to appellant's third assignment of error, appellant would


like us to believe that he was not the owner of the packages which
contained prohibited drugs but rather a certain Michael, a German
national, whom appellant met in a pub along Ermita, Manila; that in the
course of their 30-minute conversation, Michael requested him to ship
the packages and gave him P2,000.00 for the cost of the shipment since
the German national was about to leave the country the next day
(October 15, 1987, TSN, pp. 2-10).
Rather than give the appearance of veracity, we find appellant's
disclaimer as incredulous, self-serving and contrary to human
experience. It can easily be fabricated. An acquaintance with a complete
stranger struck in half an hour could not have pushed a man to entrust
the shipment of four (4) parcels and shell out P2,000.00 for the purpose
and for appellant to readily accede to comply with the undertaking
without first ascertaining its contents. As stated by the trial court, "(a)
person would not simply entrust contraband and of considerable value at
that as the marijuana flowering tops, and the cash amount of P2,000.00
to a complete stranger like the Accused. The Accused, on the other hand,
would not simply accept such undertaking to take custody of the
packages and ship the same from a complete stranger on his mere say-
so" (Decision, p. 19, Rollo, p. 91). As to why he readily agreed to do the
errand, appellant failed to explain. Denials, if unsubstantiated by clear
and convincing evidence, are negative self-serving evidence which
deserve no weight in law and cannot be given greater evidentiary weight
than the testimony of credible witnesses who testify on affirmative
matters (People v. Esquillo, 171 SCRA 571 [1989]; People vs. Sariol, 174
SCRA 237 [1989]).
Appellant's bare denial is even made more suspect considering that, as
per records of the Interpol, he was previously convicted of possession of
hashish by the Kleve Court in the Federal Republic of Germany on
January 1, 1982 and that the consignee of the frustrated shipment,
Walter Fierz, also a Swiss national, was likewise convicted for drug
abuse and is just about an hour's drive from appellant's residence in
Zurich, Switzerland (TSN, October 8, 1987, p. 66; Original Records, p. 244;
Decision, p. 21; Rollo, p. 93).
Evidence to be believed, must not only proceed from the mouth of a
credible witness, but it must be credible in itself such as the common
experience and observation of mankind can approve as probable under
the circumstances (People v. Alto, 26 SCRA 342 [1968], citing Daggers v.
Van Dyke, 37 N.J. Eg. 130; see also People v. Sarda, 172 SCRA 651 [1989];
People v. Sunga, 123 SCRA 327 [1983]); Castañares v. CA, 92 SCRA 567
[1979]). As records further show, appellant did not even bother to ask
Michael's full name, his complete address or passport number.
Furthermore, if indeed, the German national was the owner of the
merchandise, appellant should have so indicated in the contract of
shipment (Exh. "B", Original Records, p. 40). On the contrary, appellant
signed the contract as the owner and shipper thereof giving more weight
to the presumption that things which a person possesses, or exercises
acts of ownership over, are owned by him (Sec. 5 [j], Rule 131). At this
point, appellant is therefore estopped to claim otherwise. LexLib
Premises considered, we see no error committed by the trial court in
rendering the assailed judgment.
WHEREFORE, the judgment of conviction finding appellant guilty beyond
reasonable doubt of the crime charged is hereby AFFIRMED. No costs.
SO ORDERED.
||| (People v. Marti, G.R. No. 81561, [January 18, 1991], 271 PHIL 51-65)

[G.R. No. L-19550. June 19, 1967.]

HARRY S. STONEHILL, ROBERT P. BROOKS, JOHN J. BROOKS


and KARL BECK, petitioners, vs. HON. JOSE W. DIOKNO, in his
capacity as SECRETARY OF JUSTICE, JOSE LUKBAN, in his
capacity as Acting Director of the National Bureau of
Investigation; SPECIAL PROSECUTORS PEDRO D. CENZON,
EFREN I. PLANA and MANUEL VILLAREAL, JR. and ASST.
FISCAL MANASES G. REYES, JUDGE AMADO ROAN, Municipal
Court of Manila, JUDGE ROMAN CANSINO, Municipal Court of
Manila, JUDGE HERMOGENES CALUAG, Court of First
Instance of Rizal-Quezon City Branch, and JUDGE DAMIAN
JIMENEZ, Municipal Court of Quezon City, respondents.

Paredes, Poblador, Cruz & Nazareno and Meer, Meer &


Meer and Juan T . David for petitioners.
Solicitor General Arturo A. Alafriz, Assistant Solicitor General
Pacifico P. de Castro, Assistant Solicitor General Frine C . Zaballero,
Solicitor Camilo D. Quiasonand Solicitor C . Padua for respondents.

SYLLABUS

1. CONSTITUTIONAL LAW; SEARCH AND SEIZURE; WHO MAY


CONTEST LEGALITY THEREOF CASE AT BAR. — It is well settled that
the legality of a seizure can be contested only by the party whose
rights have been impaired thereby (Lewis vs. U.S., 6 F. 2d. 22) and that
the objection to an unlawful search and seizure is purely personal and
cannot be availed of by third parties (In. re Dooley, 48 F. 2d. 121:
Rouda vs. U.S., 10 F. 2d. 916; Lusco vs. U.S., 287 F. 69; Ganci vs. U.S.,
287 F, 60; Morizvs. U.S., 26 F. 2d. 444). Consequently, petitioner in the
case at bar may not validly object to the use in evidence against them
of the document, papers, and things seized from the offices and
premises of the corporation adverted to, since the right to object to
the admission of said papers in evidence belongs exclusively to the
corporations, to whom the seized effects belong, and may not be
invoked by the corporate officers in proceedings against them in their
individual capacity U.S., vs.Gaas, 17 F. 2d. 997; People vs. Rubio, 57
Phil., 384).
2. ID.; ID.; REQUISITES FOR ISSUANCE OF SEARCH WARRANT. —
Two points must be stressed in connection with this constitutional
mandate, namely: (1) that no warrant issue but upon probable cause,
to be determined by the judge in the manner set forth in said provision;
and (2) that the warrant shall particularly describe the things to be
seized. None of these requirements has been complied with in the
contested warrants. Indeed, the same were issued upon applications
stating that the natural and juridical persons therein named had
committed a "violation of Central Bank Laws, Tariff and Customs Laws,
Internal Revenue (Code) and Revised Penal Code." In other words, no
specific offense had been alleged in said applications. The averments
thereof with respect to the offense committed were abstract. As a
consequence, it was impossible for the judges who issued the
warrants to have found the existence of probable cause, for the same
presupposes the introduction of competent proof that the party
against whom it is sought has performed particular acts, or committed
specific omissions, violating a given provision of our criminal laws. As
a matter of fact, the applications involved in the case at bar do not
allege any specific acts performed by herein petitioners. It would be a
legal heresy, of the highest order, to convict anybody of a "violation of
Central Bank Laws, Tariff and Customs Laws, Internal Revenue (Code)
and Revised Penal Code", — as alleged in aforementioned applications
— without reference to any determine provision of said laws or coders.
3. ID.; ID.; ID.; GENERAL WARRANTS ARE OUTLAWED BY THE
CONSTITUTION. — To uphold the validity of the warrants in question,
would be to wipe out completely one of the most fundamental rights
guaranteed in our Constitution, for it would place the sanctity of the
domicile and the privacy of communication and correspondence at the
mercy of the victims, caprice or passion of peace officers. This is
precisely the evil sought to be remedied by the constitutional
provision Sec. 1, par. 3 Art. III, Const.) — to outlaw the so-called
general warrants. It is not difficult to imagine what would happen, in
times of keen political strife, when the party in power feels that the
minority is likely to wrest it, even though by legal means. Such is the
seriousness of the irregularities committed in connection with the
disputed search warrants, that this Court deemed it fit to amend
Section 3 of Rule 122 of the former Rules of Court, by providing in its
counterpart, under the Revised Rules of Court (Sec. 3, Rule 126) that
"a search warrant shall not issue but upon probable cause in
connection with one specific offense." Not satisfied with this
qualification, the Court added thereto paragraph, directing that "no
search warrant shall issue for more than one specific offense."
4. ID.; ID.; ID.; ID.; CASE AT BAR. — The grave violation of the
Constitution made in the application for the contested search
warrants was compounded by the description therein made of the
effects to be searched for and seized, to wit: "Books of accounts,
Financial records, vouchers, journals, correspondence, receipts,
ledgers, portfolios, credit journals, typewriters, and other documents
and/or papers, showing all business transactions including
disbursement receipts, balance sheets and related profit and loss
statements." Thus, the warrants authorized the search for and seizure
of records pertaining to all business transactions petitioners herein,
regardless of whether the transaction were legal or illegal. The
warrants sanctioned the seizure of all records of the petitioners and
the aforementioned corporations, whatever their nature, thus openly
contravening the explicit command of our Bill of Rights — that the
things to be seized be particularly described — as well as tending to
defeat its major objective: the elimination of general warrants.
5. ID.; ID.; ID.; NON-EXCLUSIONARY RULE CONTRAVENES THE
CONSTITUTIONAL PROHIBITIONS AGAINST UNREASONABLE SEARCH
AND SEIZURES. — Indeed, the non-exclusionary rule is contrary, not
only to the letter, but also to the spirit of the constitutional injunction
against unreasonable searches and seizures. To be sure, if the
applicant for a search warrant has competent evidence to establish
probable cause of the commission of a given crime by the party
against whom the warrant is intended, then there is no reason why the
applicant should not comply with the requirements of the fundamental
law. Upon the other hand, if he has no such competent evidence, then
it is not possible for the Judge to find that there is probable cause and
only possible for the Judge to find that there is probable cause and
hence, no justification for the issuance of the warrant. The only
possible explanation (not justification) for its issuance is the necessity
of fishing evidence of the commission of crime. But when this fishing
expedition is indicative of the absence of evidence to establish a
probable cause.
6. ID.; ID.; ID.; ID.; PROSECUTION OF THOSE WHO SECURE
ILLEGAL SEARCH WARRANT OR MAKE UNREASONABLE SEARCH OR
SEIZURE IS NO EXCUSE. — The theory that the criminal prosecution of
those who secure an illegal search warrant and/or make unreasonable
searches or seizures would suffice to protect the constitutional
guarantee under consideration, overlooks the fact that violations
thereof are, in general, committed by agents of the party in power, for
certainly, those belonging to the minority could not possibly abuse a
power they do not have. Regardless of the handicap under which the
minority usually but understandably finds itself in prosecuting agents
of the majority, one must not lose sight of the fact that the
psychological and moral effect of the possibility of securing their
conviction, is watered down by the pardoning power of the party for
whose benefit the illegality had been committed.
7. ID.; ID.; ID.; MONCADO DOCTRINE ABANDONED. — The doctrine
adopted in the Moncado case must be, as it is hereby, abandoned; the
warrants for the search of 3 residences of petitioners, as specified in
the Resolution of June 29, 1962, are null and void; the searches and
seizures therein made are illegal.

DECISION

CONCEPCION, C .J : p

Upon application of the officers of the government named on the


margin 1 — hereinafter referred to as Respondent-Prosecutors —
several judges 2 — hereinafter referred to as Respondent-Judges —
issued, on different dates, 3 a total of 42 search warrants against
petitioners herein 4 and/or the corporations of which they were
officers, 5 directed to any peace officer, to search the persons above-
named and/or the premises of their offices, warehouses and/or
residences, and to seize and take possession of the following personal
property to wit:
"Books of accounts, financial records, vouchers,
correspondence, receipts, ledgers, journals, portfolios, credit
journals, typewriters, and other documents and/or papers showing
all business transactions including disbursements receipts,
balance sheets and profit and loss statements and Bobbins
(cigarette wrappers)."
as "the subject of the offense; stolen or embezzled and proceeds or
fruits of the offense," or "used or intended to be used as the means of
committing the offense," which is described in the applications
adverted to above as "violation of Central Bank Laws, Tariff and
Customs Laws, Internal Revenue (Code) and the Revised Penal Code."
Alleging that the aforementioned search warrants are null and
void, as contravening the Constitution and the Rules of Court —
because, inter alia: (1) they do not describe with particularity the
documents, books and things to be seized; (2) cash money, not
mentioned in the warrants, were actually seized; (3) the warrants were
issued to fish evidence against the aforementioned petitioners in
deportation cases filed against them; (4) the searches and seizures
were made in an illegal manner; and (5) the documents, papers and
cash money seized were not delivered to the courts that issued the
warrants, to be disposed of in accordance with law — on March 20,
1962, said petitioners filed with the Supreme Court this original action
for certiorari, prohibition, mandamus and injunction, and prayed that,
pending final disposition of the present case, a writ of preliminary
injunction be issued restraining Respondent-Prosecutors, their agents
and or representatives from using the effects seized as
aforementioned, or any copies thereof, in the deportation cases
already adverted to, and that, in due course, thereafter, decision be
rendered quashing the contested search warrants and declaring the
same null and void, and commanding the respondents, their agents or
representatives to return to petitioners herein, in accordance with
Section 3, Rule 67, of the Rules of Court, the documents, papers,
things and cash moneys seized or confiscated under the search
warrants in question.
In their answer, respondents-prosecutors alleged 6 (1) that the
contested search warrants are valid and have been issued in
accordance with law; (2) that the defects of said warrants, if any, were
cured by petitioners' consent; and (3) that, in any event, the effects
seized are admissible in evidence against herein petitioners,
regardless of the alleged illegality of the aforementioned searches and
seizures.
On March 22, 1962, this Court issued the writ of preliminary
injunction prayed for in the petition. However, by resolution dated June
29, 1962, the writ was partially lifted or dissolved, insofar as the
papers, documents and things seized from the offices of the
corporations above mentioned are concerned; but, the injunction was
maintained as regards the papers, documents and things found and
seized in the residences of petitioners herein. 7
Thus, the documents, papers, and things seized under the alleged
authority of the warrants in question may be split into (2) major
groups, namely: (a) those found and seized in the offices of the
aforementioned corporations and (b) those found seized in the
residences of petitioners herein.
As regards the first group, we hold that petitioners herein
have no cause of action to assail the legality of the contested
warrants and of the seizures made in pursuance thereof, for the simple
reason that said corporations have their respective personalities,
separate and distinct from the personality of herein petitioners,
regardless of the amount of shares of stock or of the interest of each
of them in said corporations, and whatever the offices they hold
therein may be. 8 Indeed, it is well settled that the legality of a seizure
can be contested only by the party whose rights have been impaired
thereby, 9 and that the objection to an unlawful search and seizure is
purely personal and cannot be availed of by third
parties. 10 Consequently, petitioners herein may not validly object to
the use in evidence against them of the documents, papers and things
seized from the offices and premises of the corporations adverted to
above, since the right to object to the admission of said papers in
evidence belongs exclusively to the corporations, to whom the seized
effects belong, and may not be invoked by the corporate officers in
proceedings against them in their individual capacity. 11 Indeed, it has
been held:
". . . that the Government's action in gaining possession of
papers belonging to the corporation did not relate to nor did it
affect the personal defendants. If these papers were unlawfully
seized and thereby the constitutional rights of or any one were
invaded, they were the rights of the corporation and not the rights
of the other defendants. Next, it is clear that a question of the
lawfulness of a seizure can be raised only by one whose rights
have been invaded. Certainly, such a seizure, if unlawful, could not
affect the constitutional rights of defendants whose property had
not been seized or the privacy of whose homes had not been
disturbed; nor could they claim for themselves the benefits of the
Fourth Amendment, when its violation, if any, was with reference
to the rights of another. Remus vs. United States (C.C.A.) 291 F.
501, 511. It follows, therefore, that the question of the admissibility
of the evidence based on an alleged unlawful search and seizure
does not extend to the personal defendants but embraces only the
corporation whose property was taken . . ." (A. Guckenheimer &
Bros. Co. vs. United States, [1925] 3 F. 2d, 786, 789, Emphasis
supplied.)
With respect to the documents, papers and things seized in the
residences of petitioners herein, the aforementioned resolution of
June 29, 1962, denied the lifting of the writ of preliminary injunction
previously issued by this Court, 12 thereby, in effect, restraining herein
Respondent-Prosecutors from using them in evidence against
petitioners herein.
In connection with said documents, papers and things, two (2)
important questions need be settled, namely: (1) whether the search
warrants in question, and the searches and seizures made under the
authority thereof, are valid or not; and (2) if the answer to the
preceding question is in the negative, whether said documents, papers
and things may be used in evidence against petitioners herein.
Petitioners maintain that the aforementioned search warrants are
in the nature of general warrants and that, accordingly, the seizures
effected upon the authority thereof are null and void. In this
connection, the Constitution 13 provides:
"The right of the people to be secure in their persons, houses,
papers, and effects against unreasonable searches and seizures
shall not be violated, and no warrants shall issue but upon
probable cause, to be determined by the judge after examination
under oath or affirmation of the complainant and the witnesses he
may produce, and particularly describing the place to be searched,
and the persons or things to be seized."
Two points must be stressed in connection with this
constitutional mandate, namely: (1) that no warrant shall issue
but upon probable cause, to be determined by the judge in the manner
set forth in said provision; and (2) that the warrant
shall particularly describe the things to be seized.
None of these requirements has been complied with in the
contested warrants. Indeed, the same were issued upon applications
stating that the natural and juridical persons therein named had
committed a "violation of Central Bank Laws, Tariff and Customs Laws,
Internal Revenue (Code) and Revised Penal Code." In other words,
no specific offense had been alleged in said applications. The
averments thereof with respect to the offense committed
were abstract. As a consequence, it was impossible for the judges
who issued the warrants to have found the existence of probable
cause, for the same presupposes the introduction of competent proof
that the party against whom it is sought has
performed particular acts, or committed specific omissions, violating
a given provision of our criminal laws. As a matter of fact, the
applications involved in this case do not allege any specific acts
performed by herein petitioners. It would be a legal heresy, of the
highest order, to convict anybody of a "violation of Central Bank Laws,
Tariff and Customs Laws, Internal Revenue (Code) and Revised Penal
Code," — as alleged in the aforementioned applications — without
reference to any determinate provision of said laws or codes.
To uphold the validity of the warrants in question would be to
wipe out completely one of the most fundamental rights guaranteed in
our Constitution, for it would place the sanctity of the domicile and the
privacy of communication and correspondence at the mercy of the
whims, caprice or passion of peace officers. This is precisely the evil
sought to be remedied by the constitutional provision above quoted —
to outlaw the so-called general warrants. It is not difficult to imagine
what would happen, in times of keen political strife, when the party in
power feels that the minority is likely to wrest it, even though by legal
means.
Such is the seriousness of the irregularities committed in
connection with the disputed search warrants, that this Court deemed
it fit to amend Section 3 of Rule 122 of the former Rules of Court 14 by
providing in its counterpart, under the Revised Rules of Court 15 that "a
search warrant shall not issue upon probable cause in connection
with one specific offense." Not satisfied with this qualification, the
Court added thereto a paragraph, directing that "no search warrant
shall issue for more than one specific offense."
The grave violation of the Constitution made in the application for
the contested search warrants was compounded by the description
therein made of the effects to be searched for and seized, to wit:
"Books of accounts, financial records, vouchers, journals,
correspondence, receipts, ledgers, portfolios, credit journals,
typewriters, and other documents and/or papers showing all
business transactions including disbursement receipts, balance
sheets and related profit and loss statements."
Thus, the warrants authorized the search for and seizure of
records pertaining to all business transactions of petitioners herein,
regardless of whether the transactions were legal or illegal. The
warrants sanctioned the seizure of all records of the petitioners and
the aforementioned corporations, whatever their nature, thus openly
contravening the explicit command of our Bill of Rights — that the
things to be seized be particularly described — as well as tending to
defeat its major objective: the elimination of general warrants.
Relying upon Moncado vs. People's Court (80 Phil. 1), Respondent-
Prosecutors maintain that, even if the searches and seizures under
consideration were unconstitutional, the documents, papers and
things thus seized are admissible in evidence against petitioners
herein. Upon mature deliberation, however, we are unanimously of the
opinion that the position taken in the Moncado case must be
abandoned. Said position was in line with the American common law
rule, that the criminal should not be allowed to go free merely
"because the constable has blundered," 16 upon the theory that the
constitutional prohibition against unreasonable searches and seizures
is protected by means other than the exclusion of evidence unlawfully
obtained, 17 such as the common-law action for damages against the
searching officer, against the party who procured the issuance of the
search warrant and against those assisting in the execution of an
illegal search, their criminal punishment, resistance, without liability
to an unlawful seizure, and such other legal remedies as may be
provided by other laws.
However, most common law jurisdictions have already given up
this approach and eventually adopted the exclusionary rule, realizing
that this is the only practical means of enforcing the constitutional
injunction against unreasonable searches and seizures. In the
language of Judge Learned Hand:
"As we understand it, the reason for the exclusion of
evidence competent as such, which has been unlawfully acquired,
is that exclusion is the only practical way of enforcing the
constitutional privilege. In earlier times the action of trespass
against the offending official may have been protection enough;
but that is true no longer. Only in case the prosecution which itself
controls the seizing officials, knows that it cannot profit by their
wrong, will that wrong be repressed". 18
In fact, over thirty (30) years before, the Federal Supreme Court
had already declared:
"If letters and private documents can thus be seized and held
and used in evidence against a citizen accused of an offense, the
protection of the 4th Amendment, declaring his rights to be secure
against such searches and seizures, is of no value, and, so far as
those thus placed are concerned, might as well be stricken from
the Constitution. The efforts of the courts and their officials to
bring the guilty to punishment, praiseworthy as they are, are not to
be aided by the sacrifice of those great principles established by
years of endeavor and suffering which have resulted in their
embodiment in the fundamental law of the land." 19
This view was, not only reiterated, but, also, broadened in
subsequent decisions of the same Federal Court. 20 After reviewing
previous decisions thereon, said Court held, in Mapp vs. Ohio (supra.):
". . . Today we once again examine the Wolf's constitutional
documentation of the right of privacy free from unreasonable state
intrusion, and, after its dozen years on our books, are led by it to
close the only courtroom door remaining open to evidence secured
by official lawlessness in flagrant abuse of that basic right,
reserved to all persons as a specific guarantee against that very
same unlawful conduct. We held that all evidence obtained by
searches and seizures in violation of the Constitution is, by that
same authority, inadmissible in a State court.
"Since the Fourth Amendment's right of privacy has been
declared enforceable against the States through the Due Process
Clause of the Fourteenth, it is enforceable against them by the
same sanction of exclusion as it used against the Federal
Government. Were it otherwise, then just as without the Weeks
rule the assurance against unreasonable federal searches and
seizures would be 'a form of words', valueless and undeserving of
mention in a perpetual charter of inestimable human liberties, so
too, 'without that rule the freedom from state invasions of privacy
would be so ephemeral and so neatly severed from its conceptual
nexus with the freedom from all brutish means of coercing
evidence as not to permit this Court's high regard as a freedom
implicit in the concept of ordered liberty.' At the time that the
Court held in Wolf that the Amendment was applicable to the
States through the Due Process Clause, the cases of this Court as
we have seen, had steadfastly held that as to federal officers the
Fourth Amendment included the exclusion of the evidence seized
in violation of its provisions. Even Wolf 'stoutly adhered' to that
proposition. The right to privacy, when conceded operatively
enforceable against the States, was not susceptible of destruction
by avulsion of the sanction upon which its protection and
enjoyment had always been deemed dependent under the Boyd,
Weeks and Silverthorne Cases. Therefore, in extending the
substantive protections of due process to all constitutionally
unreasonable searches — state or federal — it was logically and
constitutionally necessary that the exclusion doctrine — an
essential part of the right to privacy — be also insisted upon as an
essential ingredient of the right newly recognized by the Wolf Case.
In short, the admission of the new constitutional right by Wolf
could not consistently tolerate denial of its most important
constitutional privilege, namely, the exclusion of the evidence
which an accused had been forced to give by reason of the
unlawful seizure. To hold otherwise is to grant the right but in
reality to withhold its privilege and enjoinment. Only last year the
Court itself recognized that the purpose of the exclusionary rule 'is
to deter — to compel respect for the constitutional guaranty in the
only effectively available way — by removing the incentive to
disregard it.' . . .
"The ignoble shortcut to conviction left open to the State
tends to destroy the entire system of constitutional restraints on
which the liberties of the people rest. Having once recognized that
the right to privacy embodied in the Fourth Amendment is
enforceable against the States, and that the right to be secure
against rude invasions of privacy by state officers is, therefore
constitutional in origin, we can no longer permit that right to
remain an empty promise. Because it is enforceable in the same
manner and to like effect as other basic rights secured by the Due
Process Clause, we can no longer permit it to be revocable at the
whim of any police officer who, in the name of law enforceable
itself, chooses to suspend its enjoinment. Our decision, founded on
reason and truth, gives to the individual no more than that which
the Constitution guarantees him, to the police officer no less than
that to which honest law enforcement is entitled, and, to the
courts, that judicial integrity so necessary in the true
administration of justice." (Emphasis ours.)
Indeed, the non-exclusionary rule is contrary, not only to the
letter, but, also, to spirit of the constitutional injunction against
unreasonable searches and seizures. To be sure, if the applicant for a
search warrant has competent evidence to establish probable cause
of the commission of a given crime by the party against whom the
warrant is intended, then there is no reason why the applicant should
not comply with the requirements of the fundamental law. Upon the
other hand, if he has no such competent evidence, then it is not
possible for the judge to find that there is probable cause, and, hence,
no justification for the issuance of the warrant. The only possible
explanation (not justification) for its issuance is the necessity
of fishing evidence of the commission of a crime. But, then, this
fishing expedition is indicative of the absence of evidence to establish
a probable cause.
Moreover, the theory that the criminal prosecution of those who
secure an illegal search warrant and/or make unreasonable searches
or seizures would suffice to protect the constitutional guarantee under
consideration, overlooks the fact that violations thereof are, in
general, committed by agents of the party in power, for, certainly,
those belonging to the minority could not possibly abuse a power they
do not have. Regardless of the handicap under which the minority
usually — but, understandably — finds itself in prosecuting agents of
the majority, one must not lose sight of the fact that the psychological
and moral effect of the possibility 21 of securing their conviction, is
watered down by the pardoning, power of the party for whose benefit
the illegality had been committed.
In their Motion for Reconsideration and Amendment of the
Resolution of this Court dated June 29, 1962, petitioners allege that
Room Nos. 81 and 91 of Carmen Apartments, House No. 2008, Dewey
Boulevard, House No. 1436, Colorado Street, and Room No. 304 of the
Army-Navy Club, should be included among the premises considered in
said Resolution as residences of herein petitioners, Harry S. Stonehill,
Robert P. Brook, John J. Brooks and Karl Beck, respectively, and that,
furthermore, the records, papers and other effects seized in the
offices of the corporations above referred to include personal
belongings of said petitioners and other effects under their exclusive
possession and control, for the exclusion of which they have a
standing under the latest rulings of the federal courts of the United
States. 22
We note, however, that petitioners' theory, regarding their alleged
possession of and control over the aforementioned records, papers
and effects, and the alleged "personal" nature thereof, has been
advanced, not in their petition or amended petition herein, but in the
Motion for Reconsideration and Amendment of the Resolution of June
29, 1962. In other words, said theory would appear to be a
readjustment of that followed in said petitions, to suit the approach
intimated in the Resolution sought to be reconsidered and amended.
Then, too, some of the affidavits or copies of alleged affidavits
attached to said motion for reconsideration, or submitted in support
thereof, contain either inconsistent allegations, or allegations
inconsistent with the theory now advanced by petitioners herein.
Upon the other hand, we are not satisfied that the allegations of
said petitions and motion for reconsideration, and the contents of the
aforementioned affidavits and other papers submitted in support of
said motion, have sufficiently established the facts or conditions
contemplated in the cases relied upon by the petitioners, to warrant
application of the views therein expressed, should we agree thereto.
At any rate, we do not deem it necessary to express our opinion
thereon, it being best to leave the matter open for determination in
appropriate cases in the future.
We hold, therefore, that the doctrine adopted in the Moncado
case must be, as it is hereby, abandoned; that the warrants for the
search of three (3) residences of herein petitioners, as specified in the
Resolution of June 29, 1962 are null and void; that the searches and
seizures therein made are illegal; that the writ of preliminary
injunction heretofore issued, in connection with the documents,
papers and other effects thus seized in said residences of herein
petitioners is hereby made permanent, that the writs prayed for are
granted, insofar as the documents, papers and other effects so seized
in the aforementioned residences are concerned; that the
aforementioned motion for Reconsideration and Amendment should
be, as it is hereby, denied; and that the petition herein is dismissed
and the writs prayed for denied, as regards the documents, papers and
other effects seized in the twenty-nine (29) places, offices and other
premises enumerated in the same Resolution, without special
pronouncement as to costs.
It is so ordered.
(Stonehill v. Diokno, G.R. No. L-19550, [June 19, 1967], 126 PHIL 738-
|||

766)

[A.M. No. RTJ-93-964. February 28, 1996.]

LEOVIGILDO U. MANTARING, complainant, vs. JUDGE


MANUEL A. ROMAN, JR., RTC, Branch 42, Pinamalayan,
Oriental Mindoro; and JUDGE IRENEO B. MOLATO, MTC,
Bongabon, Oriental Mindoro, respondents.

SYLLABUS

1. REMEDIAL LAW; NEW TRIAL; REMEDY AVAILABLE TO PARTIES


DENIED OF A FAIR AND IMPARTIAL TRIAL. — There is a remedy available
to the party seeking the disqualification of the judge. If he is denied a fair
and impartial trial, caused by the judge's bias or prejudice, he can ask for
a new trial in the interest of justice which will be granted if that is really
the case.
2. ID.; CRIMINAL PROCEDURE; SEARCH WARRANT; WARRANT OF
ARREST; BASIS FOR ISSUANCE. — The issuance of a search warrant and
of a warrant of arrest requires the showing of probabilities as to different
facts. In the case of search warrants, the determination is based on the
finding that (1) the articles to be seized are connected to a criminal
activity and (2) they are found in the place to be searched. It is not
necessary that a particular person be implicated. On the other hand, in
arrest cases, the determination of probable cause is based on a finding
that a crime has been committed and that the person to be arrested has
committed it.
3. ID.; ID.; WARRANT OF ARREST; REQUIREMENTS FOR ISSUANCE.
— It is now settled that in issuing warrants of arrest in preliminary
investigations, the investigating judge must: (a) have examined in writing
and under oath the complainant and his witnesses by searching
questions and answers; (b) be satisfied that probable cause exists; and
(c) that there is a need to place the respondent under immediate custody
in order not to frustrate the ends of justice.

DECISION

MENDOZA, J : p

Respondent Judge Ireneo B. Molato is the presiding judge of the


Municipal Trial Court of Bongabon, Oriental Mindoro. On January 7, 1993,
an administrative complaint was filed against him and Judge Manuel A.
Roman, Jr., presiding judge of the Regional Trial Court of Pinamalayan,
Oriental Mindoro, Branch 42, by Leovigildo U. Mantaring, Sr., who charged
them with conduct unbecoming of members of the judiciary. On February
21, 1994, after the parties had filed their respective pleadings and
supporting documents, this Court dismissed the complaint against the
two for lack of merit. The motion for reconsideration filed by complainant
was subsequently denied.
What is before us now is the Supplemental Complaint filed by
Leovigildo U. Mantaring, Sr. against Judge Ireneo B. Molato, which
charges him with harassment. It is alleged that because of the filing of
the first complaint against him, respondent Judge Ireneo B. Molato
should have inhibited himself from conducting the preliminary
investigation of a criminal case considering that the respondents in that
case were complainant and his son. Instead, it is alleged, he took
cognizance of the case and ordered the arrest of complainant and his
son, Leovigildo Mantaring, Jr., out of hatred and revenge for them
because of the filing of the first case by the complainant.
The Supplemental Complaint was referred to the Office of the Court
Administrator which, in a Memorandum dated 25 November 1994,
recommended the dismissal of the case for lack of merit. Nonetheless,
the Court required the respondent Judge Ireneo B. Molato to comment.
In his Comment dated July 6, 1995, respondent judge denies the
allegations against him. He avers that on the application by SPO4
Pacifico L. Fradejas, he issued a search warrant which resulted in the
seizure from a certain Joel Gamo of a home-made gun, a hand grenade,
five live ammunitions for Cal. 38 and three live ammunitions for 12 gauge
shotgun; that on August 25, 1993, a complaint for Illegal Possession of
Firearms and Ammunition was filed against Joel Gamo in which the
herein complainant Leovigildo, Sr. and his son, Leovigildo, Jr., were
included; that finding that the house in which the firearms and
ammunition had been found was owned by complainant and his son, he
concluded that there was probable cause to believe that complainant
and his son were guilty of illegal possession of firearms and ammunition
and accordingly ordered their arrest. Respondent judge claims that he
inhibited himself from the case after he was ordered by the Executive
Judge, RTC, Branch 41, Pinamalayan, Oriental Mindoro.
In his Reply complainant contends that as the search warrant was
issued only against Joel Gamo and Mantaring, Jr. it was wrong for
respondent judge to find probable cause against him on the theory that,
as owners of the house in which the firearms and ammunition were
found, they had constructive possession of the same. He likewise
contends that respondent judge did not inhibit himself until after the
preliminary examination was terminated and the warrant of arrest
issued, and only after complainant had filed a petition for inhibition
which the Executive Judge found to be well taken.
On October 16, 1995, this case was referred to the OCA for
reevaluation, report and recommendation. On January 12, 1996, the OCA
submitted a Memorandum, recommending dismissal of the supplemental
complaint for lack of merit, for the following reasons:
(1) It is erroneous for herein complainant to equate the
application for the issuance of search warrant with the institution
and prosecution of criminal action in a trial court. ( Malaloan vs.
Court of Appeals, 232 SCRA 249). Complainant cannot insist that
since his name was not included in the search warrant, the house
designated to be searched did not belong to him, and that he was
not present at the preliminary investigation of witnesses
preparatory to the issuance of the questioned warrant of arrest,
there was no basis for respondent judge to order his arrest.
(2) No taint of irregularity attended the issuance by
respondent judge of the warrant of arrest against complainant and
his son. Neither was the charge that the warrant of arrest was
issued by respondent judge in the spirit of anger, hatred or
harassment purposes substantiated.
To begin with, it cannot be contended that complainant Leovigildo
Mantaring, Sr. could not be proceeded against simply because he was
not included in the search warrant issued against Gamo and Leovigildo
Mantaring, Jr., who is apparently his son. The determination of probable
cause in preliminary investigations is based solely on the evidence
presented by the complainant, regardless of whether or not the
respondent in that case is named in the proceedings for a search
warrant. As correctly pointed out by, the OCA, 1 the issuance of a search
warrant and of a warrant of arrest requires the showing of probabilities
as to different facts. In the case of search warrants, the determination is
based on the finding that (1) the articles to be seized are connected to a
criminal activity and (2) they are found in the place to be searched. It is
not necessary that a particular person be implicated. On the other hand,
in arrest cases, the determination of probable cause is based on a
finding that a crime has been committed and that the person to be
arrested has committed it.
In this case, the arrest of herein complainant and his son, together
with Joel Gamo, was ordered on the basis of respondent's finding that
the place from where the guns and ammunition were seized belonged to
complainant Leovigildo Mantaring, Sr. and the testimonies of witnesses
presented by SPO4 Fradejas. Of course complainant denies that the
house in which the firearms and ammunition were found belonged to him
and claims that at the time of the search he was in Manila. The provincial
prosecutor subsequently dismissed the case against complainant on
precisely these grounds, i.e., that the house did not belong to
complainant and he was in Manila at the time the search and seizure
were conducted. But to say this is not to say that respondent acted
arbitrarily or that he abused his powers so as to give ground for
administrative disciplinary action against him. It is only to say that he
committed an error of judgment for which complainant's remedy is
judicial.
What we think requires serious consideration is the contention by
the complainant that respondent judge should have inhibited himself
from conducting the preliminary investigation of the criminal case,
considering that the respondent was the present complainant, who had
earlier filed an administrative case against the judge and another one.
We are not unmindful of the cases in which it was stated that the
mere filing of an administrative case against a judge by one of the
parties before him is not a ground for disqualifying him from hearing a
case. 2 An examination of these cases reveals, however, that the
administrative cases were filed during the pendency of the cases, and it
is evident that the administrative cases were filed only to force the judge
to inhibit himself from the consideration of the case before him. As this
Court held, if on every occasion the party apparently aggrieved were
allowed to stop the proceedings in order to await the final decision on
the desired disqualification, or demand the immediate inhibition of the
judge on the basis alone of his being so charged, many cases would have
to be kept pending or perhaps there would not be enough judges left to
handle all the cases pending in all the courts. 3 On the other hand, there
is a remedy available to the party seeking the disqualification of the
judge. If he is denied a fair and impartial trial, caused by the judge's bias
or prejudice, he can ask for a new trial in the interest of justice which
will be granted if that is really the case. 4
But, in the case at bar, an administrative complaint against
respondent and Judge Manuel A. Roman, Jr. had previously been filed and
it was paramount that respondent was free from any appearance of bias
against, or hostility toward, the complainant. The impression could not
be helped that his action in that case was dictated by a spirit of revenge
against complainant for the latter's having filed an administrative
disciplinary action against the judge. The situation called for sedulous
regard on his part for the principle that a party is entitled to nothing less
than the cold neutrality of an impartial judge.
This circumstance should have underscored for respondent the
need of steering clear of the case because he might be perceived, rightly
or wrongly, to be susceptible to bias and partiality. For his judgment must
not be tainted by even the slightest suspicion of improbity or
preconceived interest in order to preserve at all times the faith and
confidence in courts of justice by any party to the litigation. 5
Indeed prudence should have made respondent judge heed the
admonition that "a spotless dispensation of justice requires not only that
the decision rendered be intrinsically fair but that the judge rendering it
must at all times maintain the appearance of fairness and impartiality." 6
Moreover, we think it was improper for respondent judge to have
issued the warrants of arrest against complainant and his son without
any finding that it was necessary to place them in immediate custody in
order to prevent a frustration of justice. It is now settled 7 that in issuing
warrants of arrest in preliminary investigations, the investigating judge
must:
(a) have examined in writing and under oath the complainant
and his witnesses by searching questions and answers;
(b) be satisfied that probable cause exists; and
(c) that there is a need to place the respondent under
immediate custody in order not to frustrate the ends of justice.
In this case, respondent judge justified the issuance of the warrant
of arrest on the following ground:
In view of the above considerations [referring to the
antecedent facts], it is the honest belief and finding of the Court
that there is sufficient probable cause that the crime of Illegal
Possession of Firearms and Ammunition was committed and that
the named three (3) accused Joel Gamo, Leovigildo Mantaring, Sr.
and Leovigildo Mantaring Jr. are the ones probably guilty thereof
for which reason Warrant of Arrest was issued by undersigned
against them.
He thus ordered the issuance of warrant of arrest solely on his finding
of probable cause, totally omitting to consider the third requirement
that there must be a need to place the respondent under immediate
custody "in order not to frustrate the ends of justice."
The framers of the Constitution confined the determination of
probable cause as basis for the issuance of warrants of arrest and
search warrants to judges the better to secure the people against
unreasonable searches and seizures. Respondent judge failed to live up
to this expectation by refusing to inhibit himself even when his very
impartiality was in question and worse by issuing a warrant of arrest
without determining whether or not it was justified by the need to
prevent a frustration of the ends of justice. Parenthetically, the records
show that the criminal complaints against herein complainant and his
son were eventually dismissed by the Provincial Prosecutor, but not
without the following parting words:
It cannot be gainsaid that respondents Mantarings were
greatly prejudiced and suffered damages as a consequence of
their inclusion in the criminal complaint. The unfortunate incident
could have been avoided had the Honorable Municipal Trial Judge
exercised the necessary prudence and judicial perpecuity [sic]
expected of an impartial Judge in the conduct of preliminary
investigation before issuance of warrant of arrest.
WHEREFORE, respondent judge Ireneo B. Molato is REPRIMANDED
and WARNED that commission of similar acts in the future will be dealt
with more severely. All other charges are dismissed for lack of merit.
SO ORDERED.
(Mantaring v. Roman, Jr., A.M. No. RTJ-93-964, [February 28, 1996], 324
|||

PHIL 387-395)

[G.R. No. 82585. November 14, 1988.]

MAXIMO V. SOLIVEN, ANTONIO V. ROCES, FREDERICK K.


AGCAOILI, and GODOFREDO L.
MANZANAS, petitioners, vs. THE HON. RAMON P. MAKASIAR,
Presiding Judge of the Regional Trial Court of Manila, Branch
35, UNDERSECRETARY SILVESTRE BELLO III, of the
Department of Justice, LUIS C. VICTOR, THE CITY FISCAL OF
MANILA AND PRESIDENT CORAZON C. AQUINO, respondents.

[G.R. No. 82827. November 14, 1988.]

LUIS D. BELTRAN, petitioner, vs. THE HON. RAMON P.


MAKASIAR, Presiding Judge of Branch 35 of the Regional Trial
Court, at Manila, THE HON. LUIS VICTOR CITY FISCAL OF
MANILA, PEOPLE OF THE PHILIPPINES, SUPERINTENDENT OF
THE WESTERN POLICE DISTRICT, AND THE MEMBERS OF THE
PROCESS SERVING UNIT AT THE REGIONAL TRIAL COURT OF
MANILA, respondents.

[G.R. No. 83979. November 14, 1988.]

LUIS D. BELTRAN, petitioner, vs. EXECUTIVE SECRETARY


CATALINO MACARAIG, SECRETARY OF JUSTICE SEDFREY
ORDOÑEZ, UNDERSECRETARY OF JUSTICE SILVESTRE
BELLO III, THE FISCAL OF MANILA JESUS F. GUERRERO, AND
JUDGE RAMON P. MAKASIAR, Presiding Judge of Branch 35 of
the Regional Trial Court, at Manila, respondents.

Angara, Abello, Concepcion, Regala and Cruz for petitioners in G.R.


No. 82585.
Perfecto V . Fernandez, Jose P. Fernandez and Cristobal P. Fernandez for
petitioner in G.R. No. 82827 and 83979.

SYLLABUS

1. CONSTITUTIONAL LAW; BILL OF RIGHTS; DUE PROCESS OF LAW;


RESPONDENT IN A CRIMINAL CASE NEED NOT FILE HIS COUNTER-
AFFIDAVITS BEFORE PRELIMINARY INVESTIGATION IS DEEMED
COMPLETED. — Due process of law does not require that the respondent
in a criminal case actually file his counter-affidavits before the
preliminary investigation is deemed completed. All that is required is that
the respondent be given the opportunity to submit counter-affidavits if he
is so minded.
2. ID.; ID.; RIGHT OF THE PEOPLE TO BE SECURE IN THEIR PERSONS,
HOUSES, PAPERS AND EFFECTS; ISSUANCE OF WARRANT OF ARREST;
PROBABLE CAUSE; THE JUDGE HAS EXCLUSIVE AND PERSONAL
RESPONSIBILITY TO DETERMINE EXISTENCE OF; THE PRESIDENT. —
This case is not a simple prosecution for libel. We have as complainant a
powerful and popular President who heads the investigation and
prosecution service and appoints members of appellate courts but who
feels so terribly maligned that she has taken the unorthodox step of
going to court inspite of the invocations of freedom of the press which
would inevitably follow.
3. ID.; ID.; ID.; ID.; HARASSMENT INHERENT IN ANY CRIMINAL
PROSECUTION; SUPREME COURT SHOULD DRAW THE DEMARCATION
LINE WHERE HARASSMENT GOES BEYOND USUAL DIFFICULTIES
ENCOUNTERED BY ANY ACCUSED. — There is always bound to be
harassment inherent in any criminal prosecution. Where the harassment
goes beyond the usual difficulties encountered by any accused and
results in an unwillingness of media to freely criticize government or to
question government handling of sensitive issues and public affairs, this
Court and not a lower tribunal should draw the demarcation line.
4. CONSTITUTIONAL LAW; BILL OF RIGHTS; FREEDOM OF SPEECH;
WHILE DEFAMATION IS NOT AUTHORIZED, CRITICISM IS TO BE
EXPECTED AND SHOULD BE BORNE FOR THE COMMON GOOD. — As early
as March 8, 1918, the decision in United States v. Bustos (37 Phil. 731)
stated that "(c)omplete liberty to comment on the conduct of public men
is a scalpel in the case of free speech. The sharp incision of its probe
relieves the abscesses of officialdom. Men in public life may suffer under
a hostile and unjust accusation; the wound can be assuaged with the
balm of a clear conscience." The Court pointed out that while defamation
is not authorized, criticism is to be expected and should be borne for the
common good.
5. REMEDIAL LAW; CRIMINAL PROCEDURE; PROSECUTION OF
OFFENSES; LIBEL; RULES THEREON SHOULD BE EXAMINED FROM
VARIOUS PERSPECTIVES IF DIRECTED AT A HIGH GOVERNMENT
OFFICIAL; THE SUPREME COURT SHOULD DRAW A FINE LINE INSTEAD
OF LEAVING IT TO A LOWER TRIBUNAL. — In fact, the Court observed
that high official position, instead of affording immunity from slanderous
and libelous charges would actually invite attacks by those who desire
to create sensation. It would seem that what would ordinarily be slander
if directed at the typical person should be examined from various
perspectives if directed at a high government official. Again, the Supreme
Court should draw this fine line instead of leaving it to lower tribunals.
6. ID.; ID.; FREEDOM OF EXPRESSION; SAFEGUARDS IN THE NAME
THEREOF SHOULD BE FAITHFULLY APPLIED IN TRIAL OF LIBEL CASE. —
In the trial of the libel case against the petitioners, the safeguards in the
name of freedom of expression should be faithfully applied.
GUTIERREZ, JR., J., concurring:
1. REMEDIAL LAW; CRIMINAL PROCEDURE; MOTION TO QUASH; COURT
SHOULD NOT HESITATE TO QUASH A CRIMINAL PROSECUTION IN
INTEREST OF MORE ENLIGHTENED AND SUBSTANTIAL JUSTICE. —
Consistent with our decision in Salonga v. Cruz Paño (134 SCRA 438
[1985]), the Court should not hesitate to quash a criminal prosecution in
the interest of more enlightened and substantial justice where it is not
alone the criminal liability of an accused in a seemingly minor libel case
which is involved but broader considerations of governmental power
versus a preferred freedom.
2. ID.; ID.; PROSECUTION OF OFFENSES; LIBEL; CASE NOT A SIMPLE
PROSECUTION THEREFOR WHERE COMPLAINANT IS THE PRESIDENT;
JUDGE NOT REQUIRED TO PERSONALLY EXAMINE COMPLAINANT AND
HIS WITNESSES. — What the Constitution underscores is the exclusive
and personal responsibility of the issuing judge to satisfy himself the
existence of probable cause. In satisfying himself of the existence of
probable cause for the issuance of a warrant of arrest, the judge is not
required to personally examine the complainant and his witnesses.
Following established doctrine and procedure, he shall: (1) personally
evaluate the report and the supporting documents submitted by the
fiscal regarding the existence of probable cause and, on the basis
thereof, issue a warrant of arrest; or (2) if on the basis thereof he finds no
probable cause, he may disregard the fiscal's report and require the
submission of supporting affidavits of witnesses to aid him in arriving at
a conclusion as to the existence of probable cause.
3. ID.; EXECUTIVE DEPARTMENT; PRESIDENT; IMMUNITY FROM SUIT;
RATIONALE. — The rationale for the grant to the President of the
privilege of immunity from suit is to assure the exercise of Presidential
duties and functions free from any hindrance or distraction, considering
that being the Chief Executive of the Government is a job that, aside
from requiring all of the office-holder's time, also demands undivided
attention.
4. ID.; ID.; ID.; ID.; PRIVILEGE PERTAINS TO PRESIDENT BY VIRTUE OF
THE OFFICE AND MAY BE INVOKED ONLY BY HOLDER OF OFFICE. — But
this privilege of immunity from suit, pertains to the President by virtue of
the office and may be invoked only by the holder of the office; not by any
other person in the President's behalf. Thus, an accused in a criminal
case in which the President is complainant cannot raise the presidential
privilege as a defense to prevent the case from proceeding against such
accused.
5. ID.; ID.; ID.; ID.; EXERCISE OF PRIVILEGE IS SOLELY THE PRESIDENT'S
PREROGATIVE. — Moreover, there is nothing in our laws that would
prevent the President from waiving the privilege. Thus, if so minded the
President may shed the protection afforded by the privilege and submit
to the court's jurisdiction. The choice of whether to exercise the privilege
or to waive it is solely the President's prerogative. It is a decision that
cannot be assumed and imposed by any other person.

RESOLUTION

PER CURIAM : p

In these consolidated cases, three principal issues were raised: (1)


whether or not petitioners were denied due process when informations
for libel were filed against them although the finding of the existence of
a prima facie case was still under review by the Secretary of Justice and,
subsequently, by the President; (2) whether or not the constitutional
rights of Beltran were violated when respondent RTC judge issued a
warrant for his arrest without personally examining the complainant and
the witnesses, if any, to determine probable cause; and (3) whether or
not the President of the Philippines, under the Constitution, may initiate
criminal proceedings against the petitioners through the filing of a
complaint-affidavit.
Subsequent events have rendered the first issue moot and academic. On
March 30, 1988, the Secretary of Justice denied petitioners' motion for
reconsideration and upheld the resolution of the Undersecretary of
Justice sustaining the City Fiscal's finding of a prima facie case against
petitioners. A second motion for reconsideration filed by petitioner
Beltran was denied by the Secretary of Justice on April 7, 1988. On
appeal, the President, through the Executive Secretary, affirmed the
resolution of the Secretary of Justice on May 2, 1988. The motion for
reconsideration was denied by the Executive Secretary on May 16, 1988.
With these developments, petitioner's contention that they have been
denied the administrative remedies available under the law has lost
factual support.
It may also be added that with respect to petitioner Beltran, the
allegation of denial of due process of law in the preliminary investigation
is negated by the fact that instead of submitting his counter-affidavits,
he filed a "Motion to Declare Proceeding Closed", in effect waiving his
right to refute the complaint by filing counter-affidavits. Due process of
law does not require that the respondent in a criminal case actually file
his counter-affidavits before the preliminary investigation completed. All
that is required is that the respondent be given the opportunity to submit
counter-affidavits if he is so minded.
The second issue, raised by petitioner Beltran, calls for an interpretation
of the constitutional provision on the issuance of warrants of arrest. The
pertinent provision reads:
Art. III, Sec. 2. The right of the people to be secure in their
persons, houses, papers and effects against unreasonable
searches and seizures of whatever nature and for any purpose
shall be inviolable, and no search warrant or warrant of arrest shall
issue except upon probable cause to be determined personally by
the judge after examination under oath or affirmation of the
complainant and the witnesses he may produce, and particularly
describing the place to be searched and the persons or things to
be seized.
The addition of the word "personally" after the word "determined" and
the deletion of the grant of authority by the 1973 Constitution to issue
warrants to "other responsible officers as may be authorized by law", has
apparently convinced petitioner Beltran that the Constitution now
requires the judge to personally examine the complainant and his
witnesses determination of probable cause for the issuance of warrants
of arrest. This is not an accurate interpretation.
What the Constitution underscores is the exclusive and personal
responsibility of the issuing judge to satisfy himself the existence of
probable cause. In satisfying himself of the existence of probable cause
for the issuance of a warrant of arrest, the judge is not required to
personally examine the complainant and his witnesses. Following
established doctrine and procedure, he shall: (1) personally evaluate the
report and the supporting documents submitted by the fiscal regarding
the existence of probable cause and, on the basis thereof, issue a
warrant of arrest; or (2) if on the basis thereof he finds no probable
cause, he may disregard the fiscal's report and require the submission of
supporting affidavits of witnesses to aid him in arriving at a conclusion
as to the existence of probable cause.
Sound policy dictates this procedure, otherwise judges would be unduly
laden with the preliminary examination and investigation of criminal
complaints instead of concentrating on hearing and deciding cases filed
before their courts.
On June 30, 1987, the Supreme Court unanimously adopted Circular No.
12, setting down guidelines for the issuance of warrants of arrest. The
procedure therein provided is reiterated and clarified in this resolution.
It has not been shown that respondent judge has deviated from the
prescribed procedure. Thus, with regard to the issuance of the warrants
of arrest, a finding of grave abuse of discretion amounting to lack or
excess of jurisdiction cannot be sustained.
Anent the third issue, petitioner Beltran argues that "the reasons which
necessitate presidential immunity from suit impose a correlative
disability to file suit". He contends that if criminal proceedings ensue by
virtue of the President's filing of her complaint-affidavit, she may
subsequently have to be a witness for the prosecution, bringing her
under the trial court's jurisdiction. This, continues Beltran, would in an
indirect way defeat her privilege of immunity from suit, as by testifying
on the witness stand, she would be exposing herself to possible
contempt of court or perjury.
The rationale for the grant to the President of the privilege of immunity
from suit is to assure the exercise of Presidential duties and functions
free from any hindrance or distraction, considering that being the Chief
Executive of the Government is a job that, aside from requiring all of the
office-holder's time, also demands undivided attention.
But this privilege of immunity from suit, pertains to the President by
virtue of the office and may be invoked only by the holder of the office;
not by any other person in the President's behalf Thus, an accused in a
criminal case in which the President is complainant cannot raise the
presidential privilege as a defense to prevent the case from proceeding
against such accused.
Moreover, there is nothing in our laws that would prevent the President
from waiving the privilege. Thus, if so minded the President may shed the
protection afforded by the privilege and submit to the court's jurisdiction.
The choice of whether to exercise the privilege or to waive it is solely the
President's prerogative. It is a decision that cannot be assumed and
imposed by any other person.
As regards the contention of petitioner Beltran that he could not be held
liable for libel because of the privileged character or the publication, the
Court reiterates that it is not a trier of facts and that such a defense is
best left to the trial court to appreciate after receiving the evidence of
the parties.
As to petitioner Beltran's claim that to allow the libel case to proceed
would produce a "chilling effect" on press freedom, the Court finds no
basis at this stage to rule on the point.
The petitions fail to establish that public respondents, through their
separate acts, gravely abused their discretion as to amount to lack of
jurisdiction. Hence, the writs of certiorari and prohibition prayed for
cannot issue.
WHEREFORE, finding no grave abuse of discretion amounting to excess
or lack of jurisdiction on the part of the public respondents, the Court
Resolved to DISMISS the petitions in G. R. Nos. 82585, 82827 and 83979.
The Order to maintain status quo contained in the Resolution of the
Court en banc dated April 7, 1988 and reiterated in the Resolution dated
April 26, 1988 is LIFTED.
Fernan, C.J., Narvasa, Melencio-Herrera, Cruz, Paras, Feliciano,
Gancayco, Padilla, Bidin, Sarmiento, Cortes, Griño-Aquino,
Medialdea and Regalado, JJ., concur.
(Soliven v. Makasiar, G.R. No. 82585, 82827, 83979 (Resolution),
|||

[November 14, 1988], 249 PHIL 394-406)

[G.R. No. 81756. October 21, 1991.]

NICOMEDES SILVA @ "Comedes", MARLON SILVA @ "Tama"


and ANTONIETA SILVA, petitioners, vs. THE HONORABLE
PRESIDING JUDGE, REGIONAL TRIAL COURT OF NEGROS
ORIENTAL, BRANCH XXXIII, DUMAGUETE CITY, respondent.

Marcelo G. Flores for petitioners.


SYLLABUS

1. CONSTITUTIONAL LAW; BILL OF RIGHTS; RIGHT AGAINST UNLAWFUL


SEARCH AND SEIZURE; PURPOSE. — The purpose of the constitutional
provision against unlawful searches and seizures is to prevent violations
of private security in person and property, and unlawful invasion of the
sanctity of the home, by officers of the law acting under legislative or
judicial sanction, and to give remedy against such usurpations when
attempted. (Alvero vs. Dizon, 76 Phil. 637 [1946]).
2. ID.; ID.; ID.; SEARCH WARRANT; REQUISITES FOR ISSUANCE THEREOF.
— Based on Section 2, Article III of the 1987 Constitution and Sections 3
and 4, Rule 126 of the Rules of Court, the judge must, before issuing a
search warrant, determine whether there is probable cause by examining
the complainant and witnesses through searching questions and
answers.
3. ID.; ID.; ID.; ID.; ID.; PROBABLE CAUSE; DEFINED. — In the case
of Prudente vs. Dayrit, G.R. No. 82870, December 14, 1989, 180 SCRA 69,
767 this Court defined "probable cause" as follows: "The 'probable cause'
for a valid search warrant, has been defined 'as such facts and
circumstances which would lead a reasonably discreet and prudent man
to believe that an offense has been committed, and that objects sought
in connection with the offense are in the place sought to be searched'.
This probable cause must be shown to be within the personal knowledge
of the complainant or the witnesses he may produce and not based on
mere hearsay."
4. ID.; ID.; ID.; ID.; ID.; DUTY OF JUDGE TO PERSONALLY EXAMINE THE
APPLICANT AND THE WITNESSES; EFFECT OF FAILURE TO COMPLY. — In
issuing a search warrant, the judge must strictly comply with the
constitutional and statutory requirement that he must determine the
existence of probable cause by personally examining the applicant and
his witnesses in the form of searching questions and answers. His failure
to comply with this requirement constitutes grave abuse of discretion. As
declared in Marcelo vs. De Guzman, G.R. No. L-29077, June 29, 1982, 114
SCRA 667, "the capricious disregard by the judge in not complying with
the requirements before issuance of search warrants constitutes abuse
of discretion".
5. ID.; ID.; ID.; ID.; SEIZURE OF PROPERTY NOT SPECIFIED IN THE
WARRANT; RULE. — The officers implementing the search warrant
clearly abused their authority when they seized the money of Antonieta
Silva. This is highly irregular considering that Antonieta Silva was not
even named as one of the respondents, that the warrant did not indicate
the seizure of money but only of marijuana leaves, cigarettes and joints,
and that the search warrant was issued for the seizure of personal
property (a) subject of the offense and (b) used or intended to be used as
means of committing an offense and NOT for personal property stolen or
embezzled or other proceeds of fruits of the offense.

DECISION

FERNAN, C.J : p

In this special civil action for certiorari, petitioners seek the nullification
of Search Warrant No. 1 issued by respondent Judge as well as the return
of the money in the amount of P1,231.00 seized from petitioner Antonieta
Silva.
The antecedent facts are as follows:
On June 13, 1986, M/Sgt. Ranulfo Villamor, Jr., as chief of the PC Narcom
Detachment in Dumaguete City, Negros Oriental, filed an "Application for
Search Warrant" with the Regional Trial Court, Branch XXXIII, Dumaguete
City against petitioners Nicomedes Silva and Marlon Silva. 1 This
application was accompanied by a "Deposition of Witness" executed by
Pfc. Arthur M. Alcoran and Pat. Leon T. Quindo, also dated June 13,
1986. 2
On the same day, Judge Nickarter A. Ontal, then Presiding Judge of the
Regional Trial Court, Branch XXXIII, Dumaguete City, pursuant to the said
"Application for Search Warrant" and "Deposition of Witness", issued
Search Warrant No. 1, directing the aforesaid police officers to search
the room of Marlon Silva in the residence of Nicomedes Silva for violation
of Republic Act No. 6425, otherwise known as the Dangerous Drugs Act
of 1972, as amended. Pertinent portions of Search Warrant No. 1 read as
follows:prLL

"It appearing to the satisfaction of the undersigned after


examining oath (sic) MSGT. Ranulfo T. Villamor, Jr. and his
witnesses (sic) Pfc. Arthur M. Alcoran and Pat. Leon T. Quindo that
there is probable cause to believe that possession and control of
Marijuana dried leaves, cigarettes, joint has been committed or is
about to be committed and that there are good and sufficient
reasons to believe that marijuana dried leaves, cigarettes,
joint has in possession and/or control at Tama's Room (Rgt. side
1st Floor) located at Nono-Limbaga Drive, Tanjay, Neg. Or. which
is/are:
"X (Subject of the offense stated above
(Stolen or embezzled or other proceeds of fruits of the
offense;
"X (Used or intended to be used as means of committing an
offense.
"You are hereby commanded to make an immediate search at any
time of the day (night) of the room of Tama Silva residence of his
father Comedes Silva to open (sic) aparadors, lockers, cabinets,
cartoons, containers, forthwith seize and take possession of the
following property Marijuana dried leaves, cigarettes, joint and
bring the said property to the undersigned to be dealt with as the
law directs." 3
In the course of the search, the serving officers also seized money
belonging to Antonieta Silva in the amount of P1,231.40.
On June 16, 1986, Antonieta Silva filed a motion for the return of the said
amount on the grounds that the search warrant only authorized the
serving officers to seize marijuana dried leaves, cigarettes and joint, and
that said officers failed or refused to make a return of the said search
warrant in gross violation of Section 11, Rule 126 of the Rules of Court. 4
Acting on said motion, Judge Ontal issued an Order dated July 1, 1986,
stating that the court "holds in abeyance the disposition of the said
amount of P1,231.40 pending the filing of appropriate charges in
connection with the search warrant." 5
On July 28, 1987, petitioners filed a motion to quash Search Warrant No. 1
on the grounds that (1) it was issued on the sole basis of a
mimeographed "Application for Search Warrant" and "Deposition of
Witness", which were accomplished by merely filling in the blanks and
(2) the judge failed to personally examine the complainant and witnesses
by searching questions and answers in violation of Section 3, Rule 126 of
the Rules of Court. 6
On August 11, 1987, respondent trial court, through Judge Eugenio M.
Cruz, who, by then, had replaced retired Judge Ontal, issued an Order
denying the motion for lack of merit, finding the requisites necessary for
the issuance of a valid search warrant duly complied with. 7
A motion for reconsideration dated September 1, 1987 filed by petitioners
was likewise denied by Judge Cruz in an order dated October 19, 1987.
Hence, this special civil action for certiorari.
Petitioners allege that the issuance of Search Warrant No. 1 was tainted
with illegality and that respondent Judge should be viewed to have acted
without or in excess of jurisdiction, or committed grave abuse of
discretion amounting to lack of jurisdiction when he issued the Order
dated August 11, 1987, denying their motion to quash Search Warrant No.
1.
We rule for petitioners.
Section 2, Article III (Bill of Rights) of the 1987 Constitution guarantees
the right to personal liberty and security of homes against unreasonable
searches and seizures. This section provides: LLpr

"SECTION 2. The right of the people to be secure in their persons,


houses, papers, and effects against unreasonable searches and
seizures of whatever nature and for any purpose shall be
inviolable, and no search warrant or warrant of arrest shall issue
except upon probable cause to be determined personally by the
judge after examination under oath or affirmation of the
complainant and the witnesses he may produce, and particularly
describing the place to be searched and the persons or things to
be seized."
The purpose of the constitutional provision against unlawful searches
and seizures is to prevent violations of private security in person and
property, and unlawful invasion of the sanctity of the home, by officers of
the law acting under legislative or judicial sanction, and to give remedy
against such usurpations when attempted. 8
Thus, Sections 3 and 4, Rule 126 of the Rules of Court provide for the
requisites for the issuance of a search warrant, to wit:
"SECTION 3. Requisite for issuing search warrant. — A search
warrant shall not issue but upon probable cause in connection with
one specific offense to be determined personally by the judge after
examination under oath or affirmation of the complainant and the
witnesses he may produce, and particularly describing the place to
be searched and the things to be seized.
"SECTION 4. Examination of complainant; record. — The judge
must, before issuing the warrant, personally examine in the form of
searching questions and answers, in writing and under oath the
complainant and any witnesses he may produce on facts
personally known to them and attach to the record their sworn
statements together with any affidavits submitted."
Based on the aforecited constitutional and statutory provisions, the
judge must, before issuing a search warrant, determine whether there is
probable cause by examining the complainant and witnesses through
searching questions and answers.
In the case of Prudente vs. Dayrit, G.R. No. 82870, December 14, 1989,
180 SCRA 69, 767 this Court defined "probable cause" as follows:
"The 'probable cause' for a valid search warrant, has been defined
'as such facts and circumstances which would lead a reasonably
discreet and prudent man to believe that an offense has been
committed, and that objects sought in connection with the offense
are in the place sought to be searched'. This probable cause must
be shown to be within the personal knowledge of the complainant
or the witnesses he may produce and not based on mere hearsay."

In the case at bar, we have carefully examined the questioned search


warrant as well as the "Application for Search Warrant" and "Deposition
of Witness", and found that Judge Ontal failed to comply with the legal
requirement that he must examine the applicant and his witnesses in the
form of searching questions and answers in order to determine the
existence of probable cause. The joint "Deposition of Witness" executed
by Pfc. Alcoran and Pat. Quindo, which was submitted together with the
"Application for Search Warrant" contained, for the most part, suggestive
questions answerable by merely placing "yes" or "no" in the blanks
provided thereon. In fact there were only four (4) questions asked, to wit:
"Q Do you personally know M/Sgt. Ranulfo Villamor, Jr. the
applicant for a search warrant?"
A Yes, sir.
"Q Do you have personal knowledge that the said premises subject
of the offense stated above, and other proceeds of fruit of the
offense, used or obtain (sic) or intended to be used as means
of committing an offense?"
A Yes, sir. LexLib

"Q Do you know personally who is/are the person who has have the
property in his/their possession and control?"
A Yes, sir.
"Q How did you know all this (sic) things?"
A Through discreet surveillance ." 9

The above deposition did not only contain leading questions but it was
also very broad. The questions propounded to the witnesses were in fact,
not probing but were merely routinary. The deposition was already
mimeographed and all that the witnesses had to do was fill in their
answers on the blanks provided.
In the case of Nolasco vs. Paño, G.R. No. 69803, October 8, 1986, 139
SCRA 152, 163, this Court held:
"The 'probable cause' required to justify the issuance of a search
warrant comprehends such facts and circumstances as will induce
a cautious man to rely upon them and act in pursuant thereof Of
the 8 questions asked, the 1st, 2nd and 4th pertain to identity. The
3rd and 5th are leading not searching questions. The 6th, 7th and
8th refer to the description of the personalities to be seized, which
is identical to that in the Search Warrant and suffers from the
same lack of particularity. The examination conducted was general
in nature and merely repetitious of the deposition of said witness.
Mere generalization will not suffice and does not satisfy the
requirements or probable cause upon which a warrant may issue."
Likewise, in the Prudente case cited earlier, this Court declared the
search warrant issued as invalid due to the failure of the judge to
examine the witness in the form of searching questions and answers.
Pertinent portion of the decision reads:
"Moreover, a perusal of the deposition of P/Lt. Florencio Angeles
shows that it was too brief and short. Respondent Judge did not
examine him 'in the form of searching questions and answers'. On
the contrary, the questions asked were leading as they called for a
simple 'yes' or 'no' answer. As held in Quintero vs. NBI, 'the
questions propounded by respondent Executive Judge to the
applicant's witness are not sufficiently searching to establish
probable cause. Asking of leading questions to the deponent in an
application for search warrant, and conducting of examination in a
general manner, would not satisfy the requirements for issuance of
a valid search warrant." 10
Thus, in issuing a search warrant, the judge must strictly comply with
the constitutional and statutory requirement that he must determine the
existence of probable cause by personally examining the applicant and
his witnesses in the form of searching questions and answers. His failure
to comply with this requirement constitutes grave abuse of discretion. As
declared in Marcelo vs. De Guzman, G.R. No. L-29077, June 29, 1982, 114
SCRA 667, "the capricious disregard by the judge in not complying with
the requirements before issuance of search warrants constitutes abuse
of discretion".
The officers implementing the search warrant clearly abused their
authority when they seized the money of Antonieta Silva. This is highly
irregular considering that Antonieta Silva was not even named as one of
the respondents, that the warrant did not indicate the seizure of money
but only of marijuana leaves, cigarettes and joints, and that the search
warrant was issued for the seizure of personal property (a) subject of the
offense and (b) used or intended to be used as means of committing an
offense and NOT for personal property stolen or embezzled or other
proceeds of fruits of the offense. Thus, the then presiding Judge Ontal
likewise abused his discretion when he rejected the motion of petitioner
Antonieta Silva seeking the return of her seized money.
WHEREFORE, the petition is granted. Search Warrant No. 1 is hereby
declared null and void. Respondent Judge of the Regional Trial Court of
Negros Oriental, Branch XXXIII is directed to order the return to
petitioner Antonieta Silva of the amount of P1,231.40 which had earlier
been seized from her by virtue of the illegal search warrant. This
decision is immediately executory. No costs. LexLib

SO ORDERED.
(Silva v. Presiding Judge, RTC of Negros Oriental, Br. XXXIII, Dumaguete
|||

City, G.R. No. 81756, [October 21, 1991], 280 PHIL 151-159)

[G.R. No. L-22196. June 30, 1967.]

ESTEBAN MORANO, CHAN SAU WAH and FU YAN


FUN, petitioners-appellants, vs. HON. MARTINIANO VIVO, in
his capacity as Acting Commissioner of
Immigration, respondent-appellant.

Engracio Fabre Law Office for petitioners-appellants.


Solicitor General Arturo A. Alafriz, and Solicitor A. M. Amores for
respondent-appellant.

SYLLABUS

1. CITIZENSHIP; MARRIAGE OF AN ALIEN WOMAN TO A FILIPINO


CITIZEN; EFFECT THEREOF; CASE AT BAR. — In a series of cases, this
Court has declared that the marriage of an alien woman to a Filipino
citizen does not ipso facto make her a Filipino citizen. She must
satisfactorily show that she has all the qualifications and none of the
disqualifications required by the Naturalization Law. Considering that
in the additional stipulation of facts of July 3, 1963, petitioners admit
that Chan-Sau Wah is not possessed of all the qualifications required
by the Naturalization Law there can be no doubt that petitioner Chan
Sau Wah did not become a Filipino citizen.
2. DEPORTATION PROCEEDINGS; SECTION I(3) ARTICLE III OF
THE CONSTITUTION NOT APPLICABLE. — The constitutional guarantee
set forth in Section 1(3), Article III of the Constitution, requiring that
the issue of probable cause be determined by a judge, does not extend
to deportation proceedings (Tu Chuan Hai vs.Commissioner of
Immigration, 55 Off. Gaz., No. 28, pp. 681-683).
3. ID.; RULE AS TO TEMPORARY VISITORS UPON EXPIRATION OF
PERIOD OF STAY. — The law is to the effect that temporary visitors
who do not depart upon the expiration of the period of stay granted'
them are subject to deportation by the Commissioner of Immigration,
for having violated the limitation or condition under which they were
admitted as non-immigrants (Immigration Law, Sec. 37 (a), subsection
(7); C.A. 613, as amended.
4. ID.; ID.; CHANGE OF STATUS; REQUISITES THEREFOR. — This
Court in a number of cases has ruled, and consistently too, that an
alien admitted as a temporary visitor cannot change his or her status
without first departing from the country and complying with the
requirements of Section 9 of the Immigration Act. Thus, the marriage
of a temporary alien visitor to a Filipino does not ipso facto make her a
permanent resident in this country.
5. ID.; CITIZENSHIP; MEANING OF "CHILD" IN SECTION 15
PARAGRAPH 3, COMMONWEALTH ACT 473. — The word child we are
certain means legitimate child, not a step-child. Thus, when the
Constitution provides that "those whose fathers are citizens of the
Philippines are citizens thereof" the fundamental charter intends
"those" to apply to legitimate children. In another case, the term
"minor children" or "minor child" in Section 15 of the Revised
Naturalization Law was interpreted to refer only to legitimate children
of Filipino citizens.
6. ID.; BONDS; RULE REQUIRING OFFICIAL APPROVAL THEREOF
MERELY DIRECTORY. — The provision requiring official approval of a
bond is merely directory. "Irregularity or entire failure in this respect
does not affect the validity of the bond." (9 C.J., p. 25).
7. ID.; ID.; ID.; ESTOPPEL ON EQUITABLE CONSIDERATIONS. —
Equitable consideration will stop petitioners from pleading invalidity of
the bond. They offered that bond to enable them to enter and stay in
this country. They enjoyed benefits therefrom. They cannot "in law and
good conscience be allowed to reap the fruits" of that bond, and then
jettison the same. They are "precluded from attacking the validity" of
such bond.

DECISION
SANCHEZ, J : p

Chan Sau Wah, a Chinese citizen born in Fukien, China on January


6, 1932, arrived in the Philippines on November 23, 1961 to visit her
cousin, Samuel Lee Malaps. She left in mainland China two of her
children by a first marriage: Fu Tse Haw and Fu Yan Kai. With her was
Fu Yan Fun, her minor son also by the first marriage, born in Hongkong
on September 11, 1957.
Chan Sau Wah and her minor son Fu Yan Fun were permitted entry
into the Philippines under a temporary visitor's visa for two (2) months
and after they posted a cash bond of P4,000.00.
On January 24, 1962, Chan Sau Wah married Esteban Morano, a
native-born Filipino citizen. Born to this union on September 16, 1962
was Esteban Morano, Jr.
To prolong their stay in the Philippines, Chan Sau Wah and Fu Yan
Fun obtained several extensions. The last extension expired on
September 10, 1962.
In a letter dated August 31, 1962, the Commissioner of
Immigration ordered Chan Sau Wah and her son, Fu Yan Fun, to leave
the country on or before September 10, 1962 with a warning that upon
failure to do so, he will issue a warrant for their arrest and will cause
the confiscation of their bond.
Instead of leaving the country, on September 10, 1962, Chan Sau
Wah (with her husband Esteban Morano) and Fu Yan Fun petitioned the
Court of First Instance of Manila for mandamus to compel the
Commissioner of Immigration to cancel petitioner's Alien Certificates
of Registration; prohibition to stop the Commissioner from issuing
warrants of arrest pending resolution of this case. 1 The trial court, on
November 3, 1962, issued the writ of preliminary injunction prayed for,
upon a P2,000-bond. After trial and the stipulations of facts filed by the
parties, the Court of First Instance rendered judgment, viz:
"IN VIEW OF ALL THE FOREGOING, judgment is hereby
rendered as follows:
(a) Granting this petition for Mandamus and Prohibition with
respect to petitioner CHAN SAU WAH, who is hereby declared a
citizen of the Philippines; ordering the respondent to cancel her
Alien Certificate of Registration and other immigration papers
upon the payment of proper dues; and declaring the preliminary
injunction with respect to her permanent admission, prohibiting
the respondent, his representatives or subordinates from arresting
and/or deporting said petitioner;
(b) Dismissing this petition with respect to petitioner FU YAN
FUN, and dissolving the writ of preliminary injunction issued
herein, restraining the respondent, his representatives or
subordinates from arresting and/or deporting said petitioner;
(c) Authorizing respondent Commissioner to forfeit the bond
filed by herein petitioners CHAN SAU WAH and FU YAN FUN in the
amount of P4,000.00; and
(d) Denying, for lack of merit, the prayer to declare Sec. 37 (a)
of the Philippine Immigration Act of 1940 unconstitutional
Without pronouncement as to costs."
Petitioners and respondent Commissioner both appealed.
We will deal with the claims of both appellants in their proper
sequence.
1. The Solicitor General's brief assails the trial court's
declaration that Chan Sau Wah is a citizen of the Philippines. The
court a quo took the position that "Chan Sau Wah became, by virtue of,
and upon, her marriage to Esteban Morano, a natural-born Filipino, a
Filipino citizen." 2
Placed to the fore is paragraph 1, Section 15 of Commonwealth
Act 473 [Revised Naturalization Act], which reads:
'Sec. 15. Effect of the naturalization on wife and children . —
Any woman who is now or may hereafter be married to a citizen of
the Philippines, and who might herself be lawfully naturalized shall
be deemed a citizen of the Philippines."
To apply this provision, two requisites must concur: (a) a valid
marriage of an alien woman to a citizen of the Philippines; and (b) the
alien woman herself might be lawfully naturalized.
We may concede that the first requisite has been properly met.
The validity of the marriage is presumed.
But can the same be said of the second requisite? This question
by all means is not new. In a series of cases, this court has declared
that the marriage of an alien woman to a Filipino citizen does not ipso
facto make her a Filipino citizen. She must satisfactorily show that she
has all the qualifications and none of the disqualifications required by
the Naturalization Law. 3 Ly Giok Ha alias Wy Giok Ha, et al. vs. Emilio
Galang, L-21332, March 18, 1966, clearly writes down the philosophy
behind the rule in the following expressive language, viz:
"Reflection will reveal why this must be so. The qualifications
prescribed under section 2 of the Naturalization Act, and the
disqualifications enumerated in its section 4, are not mutually
exclusive; and if all that were to be required is that the wife of a
Filipino be not disqualified under section 4, the result might well
he that citizenship would be conferred upon persons in violation of
the policy of the statute. For example, section 4 disqualified only —
(c) Polygamists or believers in the practice of polygamy;
and
(d) Persons convicted of crimes involving moral
turpitude',
so that a blackmailer, or a maintainer of gambling or bawdy
houses, not previously convicted by a competent court, would not
be thereby disqualified; still, it is certain that the law did not
intend such a person to be admitted as a citizen in view of the
requirement of section 2 that an applicant for citizenship 'must be
of good moral character'.
Similarly, the citizen's wife might be a convinced believer in
racial supremacy, in government by certain selected classes, in
the right to vote exclusively by certain "herrenvolk', and thus
disbelieve in the principles underlying the Philippine Constitution;
yet she would not be disqualified under section 4, as long as she is
not 'opposed to organized government', nor affiliated to groups
'upholding or teaching doctrines opposing all organized
governments', nor 'defending or teaching the necessity or propriety
of violence, personal assault or assassination for the success or
predominance of their ideas'. Et sic de caeteris."
Upon the principle of selective citizenship, we cannot afford to
depart from the wise precept affirmed and reaffirmed in the cases
heretofore noted.
In the additional stipulation of facts of July 3, 1963, petitioners
admit that Chan Sau Wah is not possessed of all the qualifications
required by the Naturalization Law.
Because of all these, we are left under no doubt that petitioner
Chan Sau Wah did not become a Filipino citizen.

2. Squarely put in issue by petitioners is the constitutionality of


Section 37(a) of the Immigration Act of 1940, which reads:
"Sec. 37. (a) The following aliens shall be arrested upon the
warrant of the Commissioner of Immigration or of any other officer
designated by him for the purpose and deported upon the warrant
of the Commission of Immigration after a determination by the
Board of Commissioners of the existence of the ground for
deportation as charged against the alien:
xxx xxx xxx
(7) Any alien who remains in the Philippines in violation of
any limitation or condition under which he was admitted as a
nonimmigrant."
Petitioners argue that the legal precept just quoted trenches
upon the constitutional mandate in Section 1 (3), Article III [Bill of
Right] of the Constitution, to wit:
"(3) The right of the people to be secure in their persons,
houses, papers, and effects against unreasonable searches and
seizures shall not be violated, and no warrants shall issue but upon
probable cause, to be determined by the judge after examination
under oath or affirmation of the complainant and the witnesses he
may produce, and particularly describing the place to be searched,
and the persons or things to be seized."
They say that the Constitution limits to judges the authority to issue
warrants of arrest and that the legislative delegation of such power to
the Commissioner of Immigration is thus violative of the Bill of Rights.
Section 1 (3), Article III of the Constitution, we perceive, does not
require judicial intervention in the execution of a final order of
deportation issued in accordance with law. The constitutional
limitation contemplates an order of arrest in the exercise of judicial
power 4 as a step preliminary or incidental to prosecution or
proceedings for a given offense or administrative action, not as a
measure indispensable to carry out a valid decision by a competent
official, such as a legal order of deportation, issued by the
Commissioner of Immigration, in pursuance of a valid legislation.
The following from American Jurisprudence, 5 is illuminating:
"It is thoroughly established that Congress has power to
order the deportation of aliens whose presence in the country it
deems hurtful. Owing to the nature of the proceeding, the
deportation of an alien who is found in this country in violation of
law is not a deprivation of liberty without due process of law. This
is so, although the inquiry devolves upon executive officers, and
their findings of fact, after A fair though summary hearing, are
made conclusive."
xxx xxx xxx
"The determination of the propriety of deportation is not a
prosecution for, or a conviction of, crime; nor is the deportation a
punishment, even though the facts underlying the decision may
constitute a crime under local law. The proceeding is in effect
simply a refusal by the government to harbor persons whom it does
not want. The coincidence of local penal law with the policy of
congress is purely accidental, and, though supported by the same
facts, a criminal prosecution and a proceeding for deportation are
separate and independent."
In consequence, the constitutional guarantee set forth in Section
1(3), Article III of the Constitution aforesaid requiring that the issue of
probable cause be determined by a judge, does not extend to
deportation proceedings. 6
The view, we, here express funds support in the discussions
during the constitutional convention. The convention recognized, as
sanctioned by due process, possibilities and cases of deprivation of
liberty, other than by order of a competent court. 7
Indeed, the power to deport or expel aliens is an attribute of
sovereignty. Such power is planted on the "accepted maxim of
international law, that every sovereign nation has the power, as
inherent in sovereignty, and essential to self-preservation, to forbid the
entrance of foreigners within its dominions." 8 So it is, that this Court
once aptly remarked that there can be no controversy on the fact that
where aliens are admitted as temporary visitors, "the law is to the
effect that temporary visitors who do not depart upon the expiration of
the period of stay granted them are subject to deportation by the
Commissioner of Immigration, for having violated the limitation or
condition under which they were admitted as non-immigrants
(Immigration Law, Sec. 37(a), subsection (7) C.A. 613, as amended)" 9
And, in a case directly in point, where the power of the
Commissioner to issue warrants of arrest was challenged as
unconstitutional because "such power is only vested in a judge by
Section 1, paragraph 3, Article III of our Constitution", this Court
declared —
"This argument overlooks the fact that the stay of appellant
Ng Hua To as temporary visitor is subject to certain contractual
stipulations as contained in the cash bond put up by him, among
them, that in case of breach the Commissioner may require the
recommitment of the person in whose favor the bond has been
filed. The Commissioner did nothing but to enforce such condition.
Such a step is necessary to enable the Commissioner to prepare
the ground for his deportation under section 37 (a)
of Commonwealth Act 613. A contrary interpretation would render
such power nugatory to the detriment of the State." 10
It is in this context that we rule that Section 37 (a) of
the Immigration Act of 1940 is not constitutionally proscribed.
3. A sequel to the question just discussed is the second error set
forth in the government's brief. The Solicitor General balks at the
lower court's ruling that petitioner Chan Sau Wah is entitled to
permanent residence in the Philippines without first complying with
the requirements of Sections 9 and 13 of the Immigration Act of 1940,
as amended by Republic Act 503.
We first go to the law, viz:
"SEC. 9 [last paragraph]
An alien who is admitted as a nonimmigrant cannot remain in
the Philippines permanently. To obtain permanent admission, a
nonimmigrant alien must depart voluntarily to some foreign
country and procure from the appropriate Philippine consul the
proper visa and thereafter undergo examination by the officers of
the Bureau of Immigration at a Philippine port of entry for
determination of his admissibility in accordance with the
requirements of this Act."
xxx xxx xxx
"Sec. 13. Under the conditions set forth in this Act, there may
be admitted into the Philippines immigrants, termed 'quota
immigrants' not in excess of fifty (50) of any one nationality or
without nationality for any one calendar year, except that the
following immigrants, termed 'nonquota immigrants, may be
admitted without regard to such numerical limitations.
The corresponding Philippine Consular representative abroad
shall investigate and certify the eligibility of a quota immigrant
previous to his admission into the Philippines. Qualified and
desirable aliens who are in the Philippines under temporary stay
may be admitted within the quota, subject to the provision of the
last paragraph of section 9 of this Act.
(a) The wife or the husband or the unmarried child under
twenty- one years of age of a Philippine citizen, if accompanying or
following to join such citizen:
(b) A child of alien parents born during the temporary visit
abroad of the mother, the mother having been previously lawfully
admitted into the Philippines for permanent residence, if the child
is accompanying or coming to join a parent and applies for
admission within five years from the date of its birth;"
Concededly, Chan Sau Wah entered the Philippines on a tourist
temporary visitor's visa. She is a non-immigrant. Under Section 15 just
quoted, she may therefore be admitted if she were a qualified and
desirable alien and subject to the provisions of the last paragraph of
Section 9. Therefore, first, she must depart voluntarily to some foreign
country; second, she must procure from the appropriate consul the
proper visa; and third, she must thereafter undergo examination by the
officials of the Bureau of Immigration at the port of entry for
determination of her admissibility in accordance with the
requirements of the Immigration Act.
This Court in a number of cases has ruled, and consistently too,
that an alien admitted as a temporary visitor cannot change his or her
status without first departing from the country and complying with the
requirements of Section 9 of the Immigration Act. 11
The gravamen of petitioner's argument is that Chan Sau Wah has,
since her entry, married in Manila a native-born Filipino, Esteban
Morano. It will not particularly help analysis for petitioners to appeal
to family solidarity in an effort to thwart her deportation. Chan Sau
Wah, seemingly is not one who has a high regard for such solidarity.
Proof: She left two of her children by the first marriage, both minors, in
the care of neighbors in Fukien, China.
Then, the wording of the statute heretofore adverted to is a
forbidding obstacle which will prevent this Court from writing into the
law an additional provision that marriage of a temporary alien visitor
to a Filipino would ipso facto make her a permanent resident in this
country. This is a field closed to judicial action. No breadth of
discretion is allowed us. We cannot insulate her from the State's power
of deportation.
Really, it would be an easy matter for an alien woman to enter the
Philippines as a temporary visitor, go through a mock marriage, but
actually live with another man as husband and wife, and thereby skirt
the provisions of our immigration law. Also, a woman of undesirable
character may enter this country, ply a pernicious trade, marry a
Filipino, and again throw overboard Sections 9 and 13 of the Act. Such
a flanking movement, we are confident, is impermissible.
Recently we confirmed the rule that an alien wife of a Filipino
may not stay permanently without first departing from the Philippines.
Reason: Discourage entry under false pretenses. 12
The ruling of the trial court on this score should be reversed.
4. It is petitioner's turn to point as error the dismissal of the
petition for mandamus and prohibition with respect to petitioner Fu
Yan Fun. Petitioner's line of thought is this: Fu Yan Fun follows the
citizenship of his mother. They cite Section 15, paragraph
3, Commonwealth Act 473, which says that:
"A foreign-born minor child, if dwelling in the Philippines at
the time of the naturalization of the parent, shall automatically
become a Philippine citizen. . . ."
Petitioner's position is based on the assumption that Chan Sau
Wah, the mother, is a Filipino citizen. We have held that she is not. At
best, Fu Yan Fun is a step-son of Esteban Morano, husband of Chan Sau
Wah. A step-son is not a foreign-born child of the step-father. The
word child, we are certain, means legitimate child, not a step- child.
We are not wanting in precedents. Thus, when the Constitution
provides that "[t]hose whose fathers are citizens of the Philippines"
are citizens thereof, 13 the fundamental charter intends "those" to
apply to legitimate children. 14 In another case, the term "minor
children" or "minor child" in Section 15 of theRevised Naturalization
Law refers only to legitimate children of Filipino citizens. This Court,
thru Mr. Chief Justice Roberto Concepcion, there said: 15
"It is claimed that the phrases 'minor children' and 'minor
child', used in these provisions, include adopted children. The
argument is predicated upon the theory that an adopted child is,
for all intents and purposes, a legitimate child. Whenever, the word
'children' or 'child' is used in statutes, it is generally understood,
however, to refer to legitimate children, unless the context of the
law and its spirit indicate clearly the contrary. Thus, for instance,
when the Constitution provides that 'those whose fathers are
citizens of the Philippines', and 'those whose mothers are citizens
of the Philippines' who shall elect Philippine citizenship upon
reaching the age of majority are citizens of the Philippines Article
IV, Section 1, subdivisions [3] and (4]), our fundamental law clearly
refers to legitimate children (Chiongbian vs. De Leon, 46 Off. Gaz.,
3652-3654; Serra vs. Republic, L-4223, May 12, 1952)."
At any rate, Fu Yan Fun entered the Philippines as a temporary
visitor. The status of a temporary visitor cannot be converted into that
of a permanent resident, as we have heretofore held, without first
complying with Section 9 of the Immigration Law.
5. Petitioners finally aver that the lower court erred in authorizing
respondent Commissioner to forfeit the bond filed by petitioners Chan
Sau Wah and Fu Yan Fun in the amount of P4,000.00.
Here is petitioner's posture. They enjoyed their stay in the
Philippines upon a bond. Now they come to court and say that as the
prescribed form of this bond was not expressly approved by the
Secretary of Justice in accordance with Section 3 of Commonwealth
Act 613, which reads —
"Sec. 3. . . . He [Commissioner of Immigration] shall, subject
to the approval of the Department Head, such rules and regulations
and prescribe such forms of bond, reports, and other papers, and
shall issue from time to time such instruction, not inconsistent
with law, as he shall deem best calculated to carry out the
provisions of the immigration laws . . ." that bond is void.
Reasons there are which prevent us from giving our imprimatur to
this argument.
The provision requiring official approval of a bond is merely
directory. "Irregularity or entire failure in this respect does not affect
the validity of the bond." 16The reason for the rule is found in 9 C.J., p.
26 (footnote), which reads:
"(a) Reason for rule. — 'Statutes requiring bonds to be
approved by certain officials are not for the purpose of protecting
the obligors in the bond, but are aimed to protect the public, to
insure their solvency, and to create evidence of an unimpeachable
character of the fact of their execution. When they are executed for
a legal purpose, before a proper tribunal, and are in fact accepted
and approved by the officer or body, whose duty it was to approve
them, it could serve no useful purpose of the law to hold them
invalid, to release all the obligation thereon, and to defeat every
purpose of its execution, simply because the fact of approval was
not indorsed precisely as had been directed by the Legislature.'
American Book Co., vs. Wells, 83 SW 622, 627, 26 Kyl 1159."
(emphasis supplied)
And another. This bond was accepted by the government. It has
been there. The form of the bond here used is of long continued usage.
If the government did not question the form of the bond at all, then we
must assume that it counted with the Secretary's approval. For the
presumption is that official duty has been legally performed.
Surely enough, equitable considerations will stop petitioners from
pleading invalidity of the bond. They offered that bond to enable them
to enter and stay in this country. They enjoyed benefits therefrom.
They cannot, "in law and good conscience, be allowed to reap the
fruits" of that bond, and then jettison the same. They are "precluded
from attacking the validity" of such bond. 17
Actually, to petitioners the bond was good while they sought
entry into the Philippines; they offered it as security for the
undertaking that they "will actually depart from the Philippines" when
their term of stay expires. Now that the bond is being confiscated
because they overstayed, they make an about-face and say that such
bond is null and void. They shall not profit from this inconsistent
position. Their bond should be confiscated.
Conformably to the foregoing, the judgment under review is
hereby modified as follows:
(1) The portion thereof which reads:
"(a) Granting this petition for Mandamus and Prohibition with
respect to petitioner CHAN SAU WAH, who is hereby declared a
citizen of the Philippines; ordering the respondent to cancel her
Alien Certificate of Registration and other immigration papers,
upon the payment of proper dues; and declaring the preliminary
injunction with respect to her permanent, prohibiting the
respondent, his representatives or subordinates from arresting
and/or deporting said petitioner;"
is hereby reversed; and, in consequence —
The petition for mandamus and prohibition with respect to
petitioner Chan Sau Wah is hereby denied; and the judgment declaring
her a citizen of the Philippines, directing respondent to cancel her
Alien Certificate of Registration and other immigration papers, and
declaring the preliminary injunction with respect to her permanent,
are all hereby set aside; and
(2) In all other respects, the decision appealed from is hereby
affirmed.
No costs. So ordered.
||| (Morano v. Vivo, G.R. No. L-22196, [June 30, 1967], 126 PHIL 928-943)
[G.R. No. 82544. June 28, 1988.]

IN THE MATTER OF THE PETITION FOR HABEAS CORPUS OF:


ANDREW HARVEY, JOHN SHERMAN and ADRIAAN VAN DEL
ELSHOUT, petitioners, vs.HONORABLE COMMISSIONER
MIRIAM DEFENSOR SANTIAGO, COMMISSION ON
IMMIGRATION AND DEPORTATION, respondent.

DECISION

MELENCIO-HERRERA,J : p

A petition for Habeas Corpus.


Petitioners Andrew Harvey and John Sherman, 52 and 72 years,
respectively, are both American nationals residing at Pagsanjan,
Laguna, while Adriaan Van Den Elshout, 58 years old, is a Dutch citizen
also residing at Pagsanjan, Laguna.
The case stems from the apprehension of petitioners on 27
February 1988 from their respective residences by agents of the
Commission on Immigration and Deportation (CID) by virtue of Mission
Orders issued by respondent Commissioner Miriam Defensor Santiago
of the CID. Petitioners are presently detained at the CID Detention
Center.
Petitioners were among the twenty-two (22) suspected alien
pedophiles who were apprehended after three months of close
surveillance by CID agents in Pagsanjan, Laguna. Two (2) days after
apprehension, or on 29 February 1988, seventeen (17) of the twenty-
two (22) arrested aliens opted for self-deportation and have left the
country. One was released for lack of evidence; another was charged
not for being a pedophile but for working without a valid working visa.
Thus, of the original twenty two (22),only the three petitioners have
chosen to face deportation.
Seized during petitioners' apprehension were rolls of photo
negatives and photos of the suspected child prostitutes shown in
salacious poses as well as boys and girls engaged in the sex act.
There were also posters and other literature advertising the child
prostitutes.
The "Operation Report" on Andrew Harvey and Richard Sherman
dated 29 February 1988 stated:
xxx xxx xxx
"ANDREW MARK HARVEY was found together with two young
boys.
"RICHARD SHERMAN was found with two naked boys inside
his room."
In respect of Van Den Elshout, the "After Mission Report," dated
27 February 1988 read in part:
"Noted:
There were two (2) children ages 14 & 16 which subject
readily accepted having been in his care and live-in for quite
sometime."
On 4 March 1988, deportation proceedings were instituted
against petitioners for being undesirable aliens under Section 69 of
the Revised Administrative Code (Deportation Case No. 88-13).The
"Charge Sheet" read inter alia:
"Wherefore, this Office charges the respondents for
deportation, as undesirable aliens, in that: they, being pedophiles,
are inimical to public morals, public health and public safety as
provided in Section 69 of the Revised Administrative Code."
On 7 March 1988, Warrants of Arrest were issued by respondent
against petitioners for violation of Sections 37, 45 and 46 of
the Immigration Act and Section 69 of the Revised Administrative
Code. On the same date, the Board of Special Inquiry III commenced
trial against petitioners.
On 14 March 1988, petitioners filed an Urgent Petition for Release
Under Bond alleging that their health was being seriously affected by
their continuous detention. Upon recommendation of the Board of
Commissioners for their provisional release, respondent ordered the
CID doctor to examine petitioners, who certified that petitioners were
healthy.
On 22 March 1988, petitioners filed a Petition for Bail which,
however, respondent denied considering the certification by the CID
physician that petitioners were healthy. To avoid congestion,
respondent ordered petitioners' transfer to the CID detention cell at
Fort Bonifacio, but the transfer was deferred pending trial due to the
difficulty of transporting them to and from the CID where trial was on-
going.
On 4 April 1988 petitioner Andrew Harvey filed a
Manifestation/Motion stating that he had "finally agreed to a self-
deportation" and praying that he be "provisionally released for at least
15 days and placed under the custody of Atty. Asinas before he
voluntarily departs the country." On 7 April 1988, the Board of Special
Inquiry — III allowed provisional release of five (5) days only under
certain conditions. However, it appears that on the same date that the
aforesaid Manifestation/Motion was filed, Harvey and his co-petitioners
had already filed the present petition.
On 4 April 1988, as heretofore stated, petitioners availed of this
Petition for a Writ of Habeas Corpus. A Return of the Writ was filed by
the Solicitor General and the Court heard the case on oral argument
on 20 April 1988. A Traverse to the Writ was presented by petitioners to
which a Reply was filed by the Solicitor General.LLjur

Petitioners question the validity of their detention on the


following grounds:
1) There is no provision in the Philippine Immigration Act of
1940 nor under Section 69 of the Revised Administrative Code, which
legally clothes the Commissioner with any authority to arrest and
detain petitioners pending determination of the existence of a
probable cause leading to an administrative investigation.
2) Respondent violated Section 2, Article III of the 1987
Constitution prohibiting unreasonable searches and seizures since the
CID agents were not clothed with valid Warrants of arrest, search and
seizure as required by the said provision.
3) Mere confidential information made to the CID agents and their
suspicion of the activities of petitioners that they are pedophiles,
coupled with their association with other suspected pedophiles, are
not valid legal grounds for their arrest and detention unless they are
caught in the act. They further allege that being a pedophile is not
punishable by any Philippine Law nor is it a crime to be a pedophile.
We reject petitioners' contentions and uphold respondent's
official acts ably defended by the Solicitor General.
There can be no question that the right against unreasonable
searches and seizures guaranteed by Article III, Section 2 of the 1987
Constitution, is available to all persons, including aliens, whether
accused of crime or not (Moncado vs. People's Court,80 Phil. 1
[1948].One of the constitutional requirements of a valid search
warrant or warrant of arrest is that it must be based upon probable
cause. Probable cause has been defined as referring to "such facts
and circumstances antecedent to the issuance of the warrant that in
themselves are sufficient to induce a cautious man to rely on them
and act in pursuance thereof" ( People vs. Syjuco,64 Phil. 667
[1937];Alvarez vs. CFI,64 Phil. 33 [1937]).
The 1985 Rules on Criminal Procedure also provide that an arrest
without a warrant may be effected by a peace officer or even a private
person (1) when such person has committed, actually committing, or
is attempting to commit an offense in his presence; and (2) when an
offense has, in fact, been committed and he has personal knowledge
of facts indicating that the person to be arrested has committed it
(Rule 113, Section 5).
prcd

In this case, the arrest of petitioners was based on probable


cause determined after close surveillance for three (3) months during
which period their activities were monitored. The existence of
probable cause justified the arrest and the seizure of the photo
negatives, photographs and posters without warrant (See Papa vs.
Mago,L-27360, February 28, 1968, 22 SCRA 857; People vs. Court of
First Instance of Rizal,L-41686, November 17, 1980, 101 SCRA 86, cited
in CRUZ, Constitutional Law, 1987 ed.,p. 143).Those articles were
seized as an incident to a lawful arrest and, are therefore, admissible
in evidence (Section 12, Rule 126, 1985 Rules on Criminal Procedure).
But even assuming arguendo that the arrest of petitioners was
not valid at its inception, the records show that formal deportation
charges have been filed against them, as undesirable aliens, on 4
March 1988. Warrants of arrest were issued against them on 7 March
1988 "for violation of Section 37, 45 and 46 of theImmigration Act and
Section 69 of the Administrative Code." A hearing is presently being
conducted by a Board of Special Inquiry. The restraint against their
persons, therefore, has become legal. The Writ has served its purpose.
The process of the law is being followed ( Cruz vs. Montoya,L-39823,
February 25, 1975, 62 SCRA 543)."Where a person's detention was
later made by virtue of a judicial order in relation to criminal cases
subsequently filed against the detainee, his petition for habeas
corpus becomes moot and academic" (Beltran vs. Garcia,L-49014,
April 30, 1979, 89 SCRA 717)."It is a fundamental rule that a writ
of habeas corpus will not be granted when the confinement is or has
become legal, although such confinement was illegal at the beginning"
(Matsura vs. Director of Prisons,77 Phil. 1050 [1947]).
That petitioners were not "caught in the act" does not make their
arrest illegal. Petitioners were found with young boys in their
respective rooms, the ones with John Sherman being naked. Under
those circumstances the CID agents had reasonable grounds to
believe that petitioners had committed "pedophilia" defined as
"psycho-sexual perversion involving children" (Kraft-Ebbing
Psychopatia Sexualis, p. 555; "Paraphilia or unusual sexual activity in
which children are the preferred sexual object" (Webster's Third New
International Dictionary, 1971 ed.,p. 1665) [Solicitor General's Return
of the Writ, on p. 10].While not a crime under the Revised Penal Code,
it is behavior offensive to public morals and violative of the declared
policy of the State to promote and protect the physical, moral,
spiritual, and social well-being of our youth (Article II, Section 13, 1987
Constitution).
At any rate, the filing by petitioners of a petition to be released
on bail should be considered as a waiver of any irregularity attending
their arrest and estops them from questioning its validity ( Callanta v.
Villanueva,L-24646 & L-24674, June 20, 1977, 77 SCRA 377; Bagcal vs.
Villaraza,L-61770, January 31, 1983, 120 SCRA 525).
The deportation charges instituted by respondent Commissioner
are in accordance with Section 37(a) of the Philippine Immigration Act
of 1940, in relation to Section 69 of the Revised Administrative
Code. Section 37(a) provides in part:
"(a) The following aliens shall be arrested upon the warrant
of the Commissioner of Immigration and Deportation or any other
officer designated by him for the purpose and deported upon the
warrant of the Commissioner of Immigration and Deportation after
a determination by the Board of Commissioners of the existence of
the ground for deportation as charged against the alien;
xxx xxx xxx
The foregoing provision should be construed in its entirety in
view of the summary and indivisible nature of a deportation
proceeding, otherwise, the very purpose of deportation proceedings
would be defeated.
Section 37(a) is not constitutionally proscribed ( Morano vs. Vivo,
L-22196, June 30, 1967, 20 SCRA 562). The specific constraints in both
the 1935 1 and 1987 2Constitutions, which are substantially identical,
contemplate prosecutions essentially criminal in nature. Deportation
proceedings, on the other hand, are administrative in character. An
order of deportation is never construed as a punishment. It is
preventive, not a penal process. It need not be conducted strictly in
accordance with ordinary Court proceedings.
"It is of course well-settled that deportation proceedings do
not constitute a criminal action. The order of deportation is not a
punishment, (Mahler vs. Eby, 264 U.S.,32),it being merely the return
to his country of an alien who has broken the conditions upon
which he could continue to reside within our borders (U.S. vs. De
los Santos, 33 Phil.,397).The deportation proceedings are
administrative in character, (Kessler vs. Stracker, 307 U.S.,22)
summary in nature, and need not be conducted strictly in
accordance with the ordinary court proceedings (Murdock vs.
Clark, 53 F. [2d],155).It is essential, however, that the warrant of
arrest shall give the alien sufficient information about the charges
against him, relating the facts relied upon. (U.S. vs. Uhl, 211 F.,628.)
It is also essential that he be given a fair hearing with the
assistance of counsel, if he so desires, before unprejudiced
investigators (Strench vs. Pedaris, 55 F. [2d],597; Ex parte Jew You
On, 16 F. [2d],153).However, all the strict rules of evidence
governing judicial controversies do not need to be observed; only
such as are fundamental and essential, like the right of cross-
examination. (U.S. vs. Hughes, 104 F. [2d],14; Murdock vs. Clark, 53
F. [2d],155.) Hearsay evidence may even be admitted, provided the
alien is given the opportunity to explain or rebut it (Morrell vs.
Baker, 270 F.,577; Sercerchi vs. Ward, 27 F. Supp.,437)." (Lao Tang
Bun vs. Fabre, 81 Phil. 682 [1948]).
The ruling in Vivo vs. Montesa (G. R. No. 24576, July 29, 1968, 24
SCRA 155) that "the issuance of warrants of arrest by the
Commissioner of Immigration, solely for purposes of investigation and
before a final order of deportation is issued, conflicts with paragraph
3, Section 1 of Article III of the Constitution" (referring to the1935
Constitution) 3 is not invocable herein. Respondent Commissioner's
Warrant of Arrest issued on 7 March 1988 did not order petitioners to
appear and show cause why they should not be deported. They were
issued specifically "for violation of Sections 37, 45 and 46 of
the Immigration Act and Section 69 of the Revised Administrative
Code." Before that, deportation proceedings had been commenced
against them as undesirable aliens on 4 March 1988 and the arrest
was a step preliminary to their possible deportation. cdrep

"Section 37 of the Immigration Law, which empowers the


Commissioner of Immigration to issue warrants for the arrest of
overstaying aliens is constitutional. The arrest is a step
preliminary to the deportation of the aliens who had violated the
condition of their stay in this country." (Morano vs. Vivo, L-22196,
June 30, 1967, 20 SCRA 562).
To rule otherwise would be to render the authority given the
Commissioner nugatory to the detriment of the State.
"The pertinent provision of Commonwealth Act No. 613, as
amended, which gives authority to the Commissioner of
Immigration to order the arrest of an alien temporary visitor
preparatory to his deportation for failure to put up new bonds
required for the stay, is not unconstitutional.
xxx xxx xxx
". . . Such a step is necessary to enable the Commissioner to
prepare the ground for his deportation under Section
37[a]of Commonwealth Act 613. A contrary interpretation would
render such power nugatory to the detriment of the State." (Ng Hua
To vs. Galang, G.R. No. 10140, February 29, 1964, 10 SCRA 411).
"The requirement of probable cause, to be determined by a
Judge, does not extend to deportation proceedings." (Morano vs.
Vivo, supra, citing Tiu Chun Hai vs. Commissioner, infra).There
need be no "truncated" recourse to both judicial and
administrative warrants in a single deportation proceedings.
The foregoing does not deviate from the ruling in Qua Chee Gan
vs. Deportation Board (G. R. No. 10280, September 30, 1963, 9 SCRA 27
[1963]) reiterated in Vivo vs. Montesa, supra, that "under the express
terms of our Constitution (the 1935 Constitution), it is therefore even
doubtful whether the arrest of an individual may be ordered by any
authority other than a judge if the purpose is merely to determine the
existence of a probable cause, leading to an administrative
investigation." For, as heretofore stated, probable cause had already
been shown to exist before the warrants of arrest were issued.
What is essential is that there should be a specific charge
against the alien intended to be arrested and deported, that a fair
hearing be conducted (Section 37[c]) with the assistance of counsel, if
desired, and that the charge be substantiated by competent evidence.
Thus, Section 69 of the Revised Administrative Code explicitly
provides:LibLex

"Sec. 69. Deportation of subject of foreign power. A subject of


a foreign power residing in the Philippines shall not be deported,
expelled, or excluded from said Islands or repatriated to his own
country by the President of the Philippines except upon prior
investigation, conducted by said Executive or his authorized agent,
of the ground upon which such action is contemplated. In such a
case the person concerned shall be informed of the charge or
charges against him and he shall be allowed not less than 3 days
for the preparation of his defense. He shall also have the right to
be heard by himself or counsel, to produce witnesses in his own
behalf, and to cross-examine the opposing witnesses."
The denial by respondent Commissioner of petitioners' release on
bail, also challenged by them, was in order because in deportation
proceedings, the right to bail is not a matter of right but a matter of
discretion on the part of the Commissioner of Immigration and
Deportation. Thus, Section 37(e) of the Philippine Immigration Act of
1940 provides that "any alien under arrest in a deportation proceeding
may be released under bond or under such other conditions as may be
imposed by the Commissioner of Immigration." The use of the word
"may" in said provision indicates that the grant of bail is merely
permissive and not mandatory on the part of the Commissioner. The
exercise of the power is wholly discretionary ( Ong Hee Sang vs.
Commissioner of Immigration, L-9700, February 28, 1962, 4 SCRA 442).
"Neither the Constitution nor Section 69 of the Revised Administrative
Code guarantees the right of aliens facing deportation to provisional
liberty on bail." (Tiu Chun Hai, et al vs. Deportation Board,104 Phil. 949
[1958]).As deportation proceedings do not partake of the nature of a
criminal action, the constitutional guarantee to bail may not be
invoked by aliens in said proceedings (Ong Hee Sang vs.
Commissioner of Immigration,supra).
Every sovereign power has the inherent power to exclude aliens
from its territory upon such grounds as it may deem proper for its self-
preservation or public interest ( Lao Tan Bun vs. Fabre,81 Phil. 682
[1948]).The power to deport aliens is an act of State, an act done by or
under the authority of the sovereign power ( In re McCulloch Dick,38
Phil. 41[1918]).It is a police measure against undesirable aliens whose
continued presence in the country is found to be injurious to the public
good and the domestic tranquility of the people ( Forbes vs. Chuoco
Tiaco, et al.,16 Phil. 534 [1910]).Particularly so in this case where the
State has expressly committed itself to defend the right of children to
assistance and special protection from all forms of neglect, abuse,
cruelty, exploitation, and other conditions prejudicial to their
development (Article XV, Section 3[2]).Respondent Commissioner of
Immigration and Deportation, in instituting deportation proceedings
against petitioners, acted in the interests of the State.
WHEREFORE, the Petition is dismissed and the Writ of Habeas
Corpus is hereby denied.
SO ORDERED.
(In re: Harvey v. Santiago, G.R. No. 82544, [June 28, 1988], 245 PHIL
|||

809-821)
[G.R. No. 81510. March 14, 1990.]

HORTENCIA SALAZAR, petitioner, vs. HON. TOMAS D.


ACHACOSO, in his capacity as Administrator of the Philippine
Overseas Employment Administration, and FERDIE
MARQUEZ, respondents.

Gutierrez & Alo Law Offices for petitioner.

SYLLABUS

1. CONSTITUTIONAL LAW; WARRANTS OF SEARCH AND ARREST;


MAY BE ISSUED ONLY BY A JUDGE; EXCEPTION. — Under the new
Constitution, which states: . . . no search warrant or warrant of arrest
shall issue except upon probable cause to be determined personally by
the judge after examination under oath or affirmation of the
complainant and the witnesses he may produce, and particularly
describing the place to be searched and the persons or things to be
seized. It is only judges, and no other, who may issue warrants of
arrest and search. The exception is in cases of deportation of illegal
and undesirable aliens, whom the President or the Commissioner of
Immigration may order arrested, following a final order of deportation,
for the purpose of deportation.
2. ID.;ID.;SECRETARY OF LABOR; NO LONGER AUTHORIZED TO
ISSUE WARRANT. — The Secretary of Labor, not being a judge, may no
longer issue search or arrest warrants. Hence, the authorities must go
through the judicial process. To that extent, we declare Article 38,
paragraph (c),of the Labor Code, unconstitutional and of no force and
effect.
3. ID.;ID.;IDENTIFY CLEARLY THE THINGS TO BE SEIZED. — We
have held that a warrant must identify clearly the things to be seized,
otherwise, it is null and void, thus: ...Another factor which makes the
search warrants under consideration constitutionally objectionable is
that they are in the nature of general warrants. ...In Stanford v. State
of Texas, the search warrant which authorized the search for `books,
records, pamphlets, cards, receipts, lists, memoranda, pictures,
recordings and other written instruments concerning the Communist
Parties of Texas, and the operations of the Community Party in Texas,"
was declared void by the U.S. Supreme Court for being too general. In
like manner, directions to 'seize any evidence in connection with the
violation of SDC 13-3703 or otherwise" have been held too general, and
that portion of a search warrant which authorized the seizure of any
"paraphernalia which could be used to violate Sec. 54-197 of the
Connecticut General Statutes (the statute dealing with the crime of
conspiracy)" was held to be a general warrant, and therefore invalid.
The description of the articles sought to be seized under the search
warrants in question cannot be characterized differently. ...

DECISION

SARMIENTO,J : p

This concerns the validity of the power of the Secretary of Labor


to issue warrants of arrest and seizure under Article 38 of the Labor
Code, prohibiting illegal recruitment. LibLex

The facts are as follows:


xxx xxx xxx
1. On October 21, 1987, Rosalie Tesoro of 177 Tupaz Street,
Leveriza, Pasay City, in a sworn statement filed with the Philippine
Overseas Employment Administration (POEA for brevity) charged
petitioner Hortencia Salazar, viz.:
"04.  T: Ano ba ang dahilan at ikaw ngayon ay narito at
nagbibigay ng salaysay?
    S: Upang ireklamo sa dahilan ang aking PECC Card ay ayaw
ibigay sa akin ng dati kong manager. — Horty Salazar — 615
R.O. Santos, Mandaluyong, Mla.
 05.  T: Kailan at saan naganap ang ginawang panloloko sa iyo
ng tao/mga taong inireklamo mo?
 S: Sa bahay ni Horty Salazar.
 06.  T: Paano naman naganap ang pangyayari?
           S: Pagkagaling ko sa Japan ipinatawag niya ako. Kinuha
ang PECC Card ko at sinabing hahanapan ako ng Booking sa
Japan. Mag-9 months na ako sa Phils. ay hindi pa niya ako
napa-alis. So lumipat ako ng ibang company pero ayaw niyang
ibigay and PECC Card ko.
2. On November 3, 1987, public respondent Atty. Ferdinand Marquez
to whom said complaint was assigned, sent to the petitioner the
following telegram:
"YOU ARE HEREBY DIRECTED TO APPEAR BEFORE FERDIE
MARQUEZ POEA ANTI ILLEGAL RECRUITMENT UNIT 6TH
FLR. POEA BLDG. EDSA COR. ORTIGAS AVE. MANDALUYONG
MM ON NOVEMBER 6, 1987 AT 10 AM RE CASE FILED
AGAINST YOU. FAIL NOT UNDER PENALTY OF LAW."
4. On the same day, having ascertained that the petitioner had no
license to operate a recruitment agency, public respondent
Administrator Tomas D. Achacoso issued his challenged CLOSURE
AND SEIZURE ORDER NO. 1205 which reads:
"HORTY SALAZAR
No. 615 R.O. Santos St.
Mandaluyong, Metro Manila
Pursuant to the powers vested in me under Presidential
Decree No. 1920 and Executive Order No. 1022, I hereby order
the CLOSURE of your recruitment agency being operated at
No. 615 R.O. Santos St., Mandaluyong, Metro Manila and the
seizure of the documents and paraphernalia being used or
intended to be used as the means of committing illegal
recruitment, it having verified that you have —
(1) No valid license or authority from the Department of Labor
and Employment to recruit and deploy workers for
overseas employment;
(2) Committed/are committing acts prohibited under Article
34 of the New Labor Code in relation to Article 38 of the
same code.
This ORDER is without prejudice to your criminal prosecution
under existing laws.
Done in the City of Manila, this 3rd day of November, 1987."
5. On January 26, 1988 POEA Director on Licensing and Regulation
Atty. Estelita B. Espiritu issued an office order designating
respondents Atty. Marquez, Atty. Jovencio Abara and Atty. Ernesto
Vistro as members of a team tasked to implement Closure and
Seizure Order No. 1205. Doing so, the group assisted by
Mandaluyong policemen and mediamen Lito Castillo of the
People's Journal and Ernie Baluyot of News Today proceeded to
the residence of the petitioner at 615 R.O. Santos St.,Mandaluyong,
Metro Manila. There it was found that petitioner was operating
Hannalie Dance Studio. Before entering the place, the team served
said Closure and Seizure order on a certain Mrs. Flora Salazar who
voluntarily allowed them entry into the premises. Mrs. Flora
Salazar informed the team that Hannalie Dance Studio was
accredited with Moreman Development (Phil.).However, when
required to show credentials, she was unable to produce any.
Inside the studio, the team chanced upon twelve talent performers
— practicing a dance number and saw about twenty more waiting
outside. The team confiscated assorted costumes which were duly
receipted for by Mrs. Asuncion Maguelan and witnessed by Mrs.
Flora Salazar.
cdphil

6. On January 28, 1988, petitioner filed with POEA the following


letter:
"Gentlemen:
On behalf of Ms. Horty Salazar of 615 R.O. Santos,
Mandaluyong, Metro Manila, we respectfully request that the
personal properties seized at her residence last January 26,
1988 be immediately returned on the ground that said seizure
was contrary to law and against the will of the owner thereof.
Among our reasons are the following:
1. Our client has not been given any prior notice or hearing,
hence the Closure and Seizure Order No. 1205 dated
November 3, 1987 violates "due process of law"
guaranteed under Sec. 1, Art. III, of the Philippine
Constitution.
2. Your acts also violate Sec. 2, Art. III of the Philippine
Constitution which guarantees right of the people "to
be secure in their persons, houses, papers, and effects
against unreasonable searches and seizures of
whatever nature and for any purpose."
3. The premises invaded by your Mr. Ferdie Marquez and five
(5) others (including 2 policemen) are the private
residence of the Salazar family, and the entry, search as
well as the seizure of the personal properties belonging
to our client were without her consent and were done
with unreasonable force and intimidation, together with
grave abuse of the color of authority, and constitute
robbery and violation of domicile under Arts. 293 and
128 of the Revised Penal Code.
Unless said personal properties worth around TEN
THOUSAND PESOS (P10,000.00) in all (and which were
already due for shipment to Japan) are returned within
twenty-four (24) hours from your receipt hereof, we shall feel
free to take all legal action, civil and criminal, to protect our
client's interests.
We trust that you will give due attention to these important
matters."
7. On February 2, 1988, before POEA could answer the letter,
petitioner filed the instant petition; on even date, POEA filed a
criminal complaint against her with the Pasig Provincial Fiscal,
docketed as IS-88-836. 1
On February 2, 1988, the petitioner filed this suit for prohibition.
Although the acts sought to be barred are already fait
accompli,thereby making prohibition too late, we consider the petition
as one for certiorari in view of the grave public interest involved.
The Court finds that a lone issue confronts it: May the Philippine
Overseas Employment Administration (or the Secretary of Labor)
validly issue warrants of search and seizure (or arrest) under Article
38 of the Labor Code? It is also an issue squarely raised by the
petitioner for the Court's resolution.
Under the new Constitution, which states:
. . . no search warrant or warrant of arrest shall issue except upon
probable cause to be determined personally by the judge after
examination under oath or affirmation of the complainant and the
witnesses he may produce, and particularly describing the place to
be searched and the persons or things to be seized. 2
it is only a judge who may issue warrants of search and arrest. 3 In one
case, it was declared that mayors may not exercise this power:
xxx xxx xxx
But it must be emphasized here and now that what has just been
described is the state of the law as it was in September, 1985. The
law has since been altered. No longer does the mayor have at this
time the power to conduct preliminary investigations, much less
issue orders of arrest. Section 143 of the Local Government Code,
conferring this power on the mayor has been abrogated,
rendered functus officio by the 1987 Constitution which took effect
on February 2, 1987; the date of its ratification by the Filipino
people. Section 2, Article III of the 1987 Constitution pertinently
provides that "no search warrant or warrant of arrest shall issue
except upon probable cause to be determined personally by the
judge after examination under oath or affirmation of the
complainant and the witnesses he may produce, and particularly
describing the place to be searched and the person or things to be
seized." The constitutional proscription has thereby been
manifested that thenceforth, the function of determining probable
cause and issuing, on the basis thereof, warrants of arrest or
search warrants; may be validly exercised only by judges, this
being evidenced by the elimination in the present Constitution of
the phrase, "such other responsible officer as may be authorized
by law" found in the counterpart provision of said 1973
Constitution, who, aside from judges, might conduct preliminary
investigations and issue warrants of arrest or search warrants. 4
Neither may it be done by a mere prosecuting body:
We agree that the Presidential Anti-Dollar Salting Task Force
exercises, or was meant to exercise, prosecutorial powers, and on
that ground, it cannot be said to be a neutral and detached "judge"
to determine the existence of probable cause for purposes of
arrest or search. Unlike a magistraté, a prosecutor is naturally
interested in the success of his case. Although his office "is to see
that justice is done and not necessarily to secure the conviction of
the person accused," he stands, invariably, as the accused's
adversary and his accuser. To permit him to issue search warrants
and indeed, warrants of arrest, is to make him both judge and jury
in his own right, when he is neither. That makes, to our mind and to
that extent, Presidential Decree No. 1936 as
amended by Presidential Decree No. 2002, unconstitutional. 5
Section 38, paragraph (c), of the Labor Code, as now written, was
entered as an amendment by Presidential Decrees Nos. 1920 and 2018
of the late President Ferdinand Marcos, to Presidential Decree No.
1693, in the exercise of his legislative powers under Amendment No. 6
of the 1973 Constitution. Under the latter, the then Minister of Labor
merely exercised recommendatory powers: prLL

(c) The Minister of Labor or his duly authorized representative shall


have the power to recommend the arrest and detention of any
person engaged in illegal recruitment. 6
On May 1, 1984, Mr. Marcos promulgated Presidential Decree No.
1920, with the avowed purpose of giving more teeth to the campaign
against illegal recruitment. The Decree gave the Minister of Labor
arrest and closure powers:
(b) The Minister of Labor and Employment shall have the power to
cause the arrest and detention of such non-licensee or non-holder
of authority if after proper investigation it is determined that his
activities constitute a danger to national security and public order
or will lead to further exploitation of job-seekers. The Minister shall
order the closure of companies, establishment and entities found
to be engaged in the recruitment of workers for overseas
employment, without having been licensed or authorized to do so. 7
On January 26, 1986, he, Mr. Marcos, promulgated Presidential
Decree No. 2018, giving the Labor Minister search and seizure powers
as well:
(c) The Minister of Labor and Employment or his duly authorized
representatives shall have the power to cause the arrest and
detention of such non-licensee or non-holder of authority if after
investigation it is determined that his activities constitute a
danger to national security and public order or will lead to further
exploitation of job-seekers. The Minister shall order the search of
the office or premises and seizure of documents, paraphernalia,
properties and other implements used in illegal recruitment
activities and the closure of companies, establishment and entities
found to be engaged in the recruitment of workers for overseas
employment, without having been licensed or authorized to do so. 8
The above has now been etched as Article 38, paragraph (c) of
the Labor Code.
The decrees in question, it is well to note, stand as the dying
vestiges of authoritarian rule in its twilight moments.
We reiterate that the Secretary of Labor, not being a judge, may
no longer issue search or arrest warrants. Hence, the authorities must
go through the judicial process. To that extent, we declare Article 38,
paragraph (c),of the Labor Code, unconstitutional and of no force and
effect.
The Solicitor General's reliance on the case of Morano v. Vivo 9 is
not well-taken. Vivo involved a deportation case, governed by Section
69 of the defunct Revised Administrative Code and by Section 37 of
the Immigration Law. We have ruled that in deportation cases, an
arrest (of an undesirable alien) ordered by the President or his duly
authorized representatives, in order to carry out a final decision of
deportation is valid. 10 It is valid, however, because of the recognized
supremacy of the Executive in matters involving foreign affairs. We
have held: 11
xxx xxx xxx
The State has the inherent power to deport undesirable aliens
(Chuoco Tiaco vs. Forbes,228 U.S. 549, 57 L. Ed. 960, 40 Phil. 1122,
1125).That power may be exercised by the Chief Executive "when
he deems such action necessary for the peace and domestic
tranquility of the nation." Justice Johnson's opinion is that when
the Chief Executive finds that there are aliens whose continued
presence in the country is injurious to the public interest, "he may,
even in the absence of express law, deport them".( Forbes vs.
Chuoco Tiaco and Crossfield, 16 Phil. 534, 568, 569; In re
McCulloch Dick, 38 Phil. 41).cdll

"The right of a country to expel or deport aliens because their


continued presence is detrimental to public welfare is absolute
and unqualified" (Tiu Chun Hai and Go Tam vs. Commissioner of
Immigration and the Director of NBI, 104 Phil. 949, 956). 12
The power of the President to order the arrest of aliens for
deportation is, obviously, exceptional. It (the power to order arrests)
can not be made to extend to other cases, like the one at bar. Under
the Constitution, it is the sole domain of the courts.
Moreover, the search and seizure order in question, assuming, ex
gratia argumenti, that it was validly issued, is clearly in the nature of a
general warrant:
Pursuant to the powers vested in me under Presidential Decree No.
1920 and Executive Order No. 1022, I hereby order the CLOSURE of
your recruitment agency being operated at No. 615 R.O. Santos St.,
Mandaluyong, Metro Manila and the seizure of the documents and
paraphernalia being used or intended to be used as the means of
committing illegal recruitment, it having verified that you have —
(1) No valid license or authority from the Department of Labor
and Employment to recruit and deploy workers for
overseas employment;
(2) Committed/are committing acts prohibited under Article
34 of the New Labor Code in relation to Article 38 of the
same code.
This ORDER is without prejudice to your criminal prosecution
under existing laws. 13
We have held that a warrant must identify clearly the things to be
seized, otherwise, it is null and void, thus:
xxx xxx xxx
Another factor which makes the search warrants under
consideration constitutionally objectionable is that they are in the
nature of general warrants. The search warrants describe the
articles sought to be seized in this wise:
"1) All printing equipment, paraphernalia, paper, ink, photo
equipment, typewriters, cabinets, tables,
communications/recording equipment, tape recorders,
dictaphone and the like used an/or connected in the printing
of the 'WE FORUM' newspaper and any and all
documents/communications, letters and facsimile of prints
related to the 'WE FORUM' newspaper.
2) Subversive documents, pamphlets, leaflets, books, and
other publications to promote the objectives and purposes of
the subversive organizations known as Movement for Free
Philippines, Light-a-Fire Movement and April 6 Movement; and
3) Motor vehicles used in the distribution/circulation of the
'WE FORUM' and other subversive materials and propaganda,
more particularly,
1) Toyota-Corolla, colored yellow with Plate No. NKA
892;
2) DATSUN pick-up colored white with Plate No. NKV
969;
3) A delivery truck with Plate No. NBS 542;
4) TOYOTA-TAMARAW, colored white with Plate No. PBP
665; and
5) TOYOTA Hi-Lux, pick-up truck with Plate No. NGV 472
with marking 'Bagong Silang.'"
In Stanford v. State of Texas ,the search warrant which authorized
the search for 'books, records, pamphlets, cards, receipts, lists,
memoranda, pictures, recordings and other written instruments
concerning the Communist Parties of Texas, and the operations of
the Community Party in Texas,' was declared void by the U.S.
Supreme Court for being too general. In like manner, directions to
'seize any evidence in connection with the violation of SDC 13-
3703 or otherwise" have been held too general, and that portion of
a search warrant which authorized the seizure of any
"paraphernalia which could be used to violate Sec. 54-197 of the
Connecticut General Statutes (the statute dealing with the crime
of conspiracy)" was held to be a general warrant, and therefore
invalid. The description of the articles sought to be seized under
the search warrants in question cannot be characterized
differently.
LLpr

In the Stanford case, the U.S. Supreme court calls to mind a


notable chapter in English history: the era of disaccord between
the Tudor Government and the English Press, when "Officers of the
Crown were given roving commissions to search where they
pleased in order to suppress and destroy the literature of dissent
both Catholic and Puritan." Reference herein to such historical
episode would not be relevant for it is not the policy of our
government to suppress any newspaper or publication that speaks
with "the voice of non-conformity" but poses no clear and
imminent danger to state security. 14
For the guidance of the bench and the bar, we reaffirm the
following principles:
1. Under Article III, Section 2, of the 1987 Constitution, it is only
judges, and no other, who may issue warrants of arrest and search;
2. The exception is in cases of deportation of illegal and
undesirable aliens, whom the President or the Commissioner of
Immigration may order arrested, following a final order of deportation,
for the purpose of deportation.
WHEREFORE, the petition is GRANTED. Article 38, paragraph (c)
of the Labor Code is declared UNCONSTITUTIONAL and null and void.
The respondents are ORDERED to return all materials seized as a
result of the implementation of Search and Seizure Order No. 1205.cdrep

No costs.
SO ORDERED.
(Salazar v. Achacoso, G.R. No. 81510, [March 14, 1990], 262 PHIL 160-
|||

171)

[G.R. No. 45358. January 29, 1937.]

NARCISO ALVAREZ, petitioner, vs. THE COURT OF FIRST


INSTANCE OF TAYABAS and THE ANTI-USURY
BOARD, respondents.

Godofredo Reyes for petitioner.


Adolfo N. Feliciano for respondent Anti-Usury Board.
No appearance for other respondent.

SYLLABUS

1. CRIMINAL PROCEDURE; SEARCH WARRANT; DEFINITION. — A


search warrant is an order in writing, issued in the name of the People
of the Philippine Islands, signed by a judge or a justice of the peace,
and directed to a peace officer, commanding him to search for
personal property and bring it before the court (section 95, General
Orders, No. 58, as amended by section 6 of Act No. 2886).
2. CONSTITUTIONAL LAW; SEARCHES AND SEIZURES. — Of all
the rights of a citizen, few are of greater importance or more essential
to his peace and happiness than the right of personal security, and
that involves the exemption of his private affairs, books, and papers
from the inspection and scrutiny of others (In re Pacific Railway
Commission, 32 Fed., 241; Interstate Commerce Commn. vs. Brimson,
38 Law. ed., 1047; Boyd vs. U.S., 29 Law. ed., 746; Carroll vs. U. S., 69
Law. ed., 543, 549). While the power to search and seize is necessary
to the public welfare, still it must be exercised and the law enforced
without transgressing the constitutional rights of citizens, for the
enforcement of no statute is of sufficient importance to justify
indifference to the basic principles of government (People vs. Elias,
147 N. E., 472).
3. ID.; ID. — As the protection of the citizen and the maintenance
of his constitutional rights is one of the highest duties and privileges
of the court, these constitutional guaranties should be given a liberal
construction or a strict construction in favor of the individual, to
prevent stealthy encroachment upon, or gradual depreciation of, the
rights secured by them (State vs. Custer County, 198 Pac., 362; State
vs. McDaniel, 231 Pac., 965; 237 Pac., 373). Since the proceeding is a
drastic one, it is the general rule that statutes authorizing searches
and seizures or search warrants must be strictly construed (Rose vs.
St. Clair, 28 Fed. [2d], 189; Leonard vs. U. S., 6 Fed. [2d], 353; Perry vs.
U. S., 14 Fed. [2d], 88; Cofer vs. State, 118 So., 613).
4. ID.; ID.; OATH. — In its broadest sense, an oath includes any
form of attestation by which a party signifies that he is bound in
conscience to perform an act faithfully and truthfully; and it is
sometimes defined as an outward pledge given by the person taking it
that his attestation or promise is made under an immediate sense of
his responsibility to God (Bouvier's Law Dictionary; State vs. Jackson,
137 N. W., 1034; In re Sage, 24 Oh. Cir. Ct. [N. S.], 7; Pumphrey vs.
State, 122 N. W., 19; Priest vs. State, 6 N. W., 468; State vs. Jones, 154
Pac., 378; Atwood vs. State, 111 So., 865). The oath required must refer
to the truth of the facts within the personal knowledge of the
petitioner or his witnesses, because the purpose thereof is to
convince the committing magistrate, not the individual making the
affidavit and seeking the issuance of the warrant, of the existence of
probable cause (U. S. vs. Tureaud, 20 Fed., 621; U. S. vs. Michalski, 265
Fed., 839; U. S. vs. Pitotto, 267 Fed., 603; U. S. vs. Lai Chew, 298 Fed.,
652.) The true test of sufficiency of an affidavit to warrant issuance of
a search warrant is whether it has been drawn in such a manner that
perjury could be charged thereon and affiant be held liable for
damages caused (State vs. Roosevelt County 20th Jud. Dis. Ct., 244
Pac. 280; State vs. Quartier, 236 Pac., 746).
5. ID.; UNREASONABLE SEARCH AND SEIZURE. — Unreasonable
searches and seizures are a menace against which the constitutional
guaranties afford full protection. The term "unreasonable search and
seizure" is not defined in the Constitution or in General Orders, No. 58,
and it is said to have no fixed, absolute or unchangeable meaning,
although the term has been defined in general language. All illegal
searches and seizures are unreasonable while lawful ones are
reasonable. What constitute a reasonable or unreasonable search or
seizure in any particular case is purely a judicial question,
determinable from a consideration of the circumstances involved,
including the purpose of the search, the presence or absence of
probable cause, the manner in which the search and seizure was
made, the place or thing searched, and the character of the articles
procured (Go-Bart Importing Co., vs. U.S., 75 Law. ed., 374; Peru vs. U.
S., 4 Fed. [2d], 881; U.S., vs. Vatune, 229 Fed., 497; Agnello vs. U. S., 70
Law. ed., 145; Lambert vs. U. S. 282 Fed., 413; U. S. vs. Bateman, 278
Fed., 231; Mason vs. Rollins, 16 Fed. Cas. [No. 9252], 2 Biss., 99).
6. ID.; ID.; NECESSITY OF TAKING THE AFFIDAVITS OF THE
WITNESSES. — Neither the Constitution nor General Orders, No 58
provides it of imperative necessity to take the depositions of the
witnesses to be presented by the applicant or complainant in addition
to the affidavit of the latter. The purpose of both in requiring the
presentation of depositions is nothing else than to satisfy the
committing magistrate of the existence of probable cause. Therefore,
if the affidavit of the applicant or complainant is sufficient, the judge
may dispense with that of other witnesses. Inasmuch as the affidavit
of the agent in this case was insufficient because his knowledge of
the facts was not personal but merely hearsay, it was the duty of the
judge to require the affidavit of one or more witnesses for the purpose
of determining the existence of probable cause to warrant the
issuance of the search warrant. When the affidavit of the applicant or
complainant contains sufficient if the judge is satisfied that there
exists probable cause; when the applicants knowledge of the facts is
mere hearsay, the affidavit of one or more witnesses having personal
knowledge of the facts is necessary. We conclude, therefore, that the
warrant issued is likewise illegal because it was based only on the
affidavit of the agent who had no personal knowledge of the facts.
7. CRIMINAL PROCEDURE; SEARCH WARRANT; SERVICE AT
NIGHT. — Section 101 of General Orders, No. 58 authorizes that the
search be made at night when it is positively asserted in the affidavit
that the property is on the person or in the place ordered to be
searched. As we have declared the affidavit insufficient and the
warrant issued exclusively upon it illegal, our conclusion is that the
contention is equally well founded and that the search could not
legally be made at night.
8. ID.; ID.; DESCRIPTION OF ARTICLES. — Section 1, paragraph (3)
of Article III of the Constitution, and section 97 of General Orders, No.
58 provide that the affidavit to be presented, which shall serve as the
basis for determining whether probable cause exists and whether the
warrant should be issued, must contain a particular description of the
place to be searched and the person or thing to be seized. These
provisions are mandatory and must be strictly complied with (Munch
vs. U. S., 24 Fed. [2d], 518; U. S. vs. Boyd, 1 Fed. [2d], 1019; U. S. vs.
Carlson, 292 Fed., 463; U. S. vs. Borkowski, 268 Fed., 408; In re Tri-
State Coal & Coke Co., 253 Fed., 605; People vs. Mayen, 188 Cal., 237;
People vs. Kahn, 256 Ill. A., 415); but where, by the nature of the goods
to be seized, their description must be rather general, it is not required
that a technical description be given, as this would mean that no
warrant could issue (People vs. Rubio, 57 Phil., 384; People vs.
Kahn, supra).
9. ID.; ID.; ID. — The only description of the articles given in the
affidavit presented to the judge was as follows: "that there are being
kept in said premises books, documents, receipts, lists, chits and
other papers used by him in connection with his activities as money-
lender, charging a usurious rate of interest, in violation of the law."
Taking into consideration the nature of the articles so described, it is
clear that no other more adequate and detailed description could be
given, particularly because it is difficult to give a particular
description of the contents thereof. The description so made
substantially complies with the legal provisions because the officer of
the law who executed the warrant was thereby placed in a position
enabling him to identify the articles in question, which he did.
10. CONSTITUTIONAL LAW; SEARCHES AND SEIZURES; SEIZURE
OF BOOKS AND DOCUMENTS TO BE USED AS EVIDENCE IN CRIMINAL
PROCEEDINGS AGAINST THE OWNER OR POSSESSOR THEREOF. — At
the hearing of the incidents of the case raised before the court, it
clearly appeared that the books and documents had really been seized
to enable the Anti-Usury Board to conduct an investigation and later
use all or some of the articles in question as evidence against the
petitioner in the criminal cases that may be filed against him. The
seizure of books and documents by means of a search warrant, for the
purpose of using them as evidence in a criminal case against the
person in whose possession they were found, is unconstitutional
because it makes the warrant unreasonable, and it is equivalent to a
violation of the constitutional provision prohibiting the compulsion of
an accused to testify against himself (Uy Kheytin vs. Villareal, 42 Phil.,
886; Brady vs. U. S., 266 U. S., 620; Temperani vs. U. S., 299 Fed., 365;
U. S. vs. Madden, 297 Fed., 679; Boyd vs. U. S., 116 U. S., 616; Carroll vs.
U. S., 267 U. S., 132). Therefore, it appearing that at least nineteen of
the documents in question were seized for the purpose of using them
as evidence against the petitioner in the criminal proceeding or
proceedings for violation of the Anti-Usury Law, which it is attempted
to institute against him, we hold that the search warrant issued is
illegal and that the documents should be returned to him.
11. ID.; ID.; WAIVER OF THE CONSTITUTIONAL GUARANTEES. —
The Anti- Usury Board insinuates in its answer that the petitioner
cannot now question the validity of the search warrant or the
proceedings had subsequent to the issuance thereof, because he has
waived his constitutional rights in proposing a compromise whereby
he agreed to pay a fine of P200 for the purpose of evading the criminal
proceeding or proceedings. We are of the opinion that there was no
such waiver, first, because the petitioner has emphatically denied the
offer of compromise and, second, because if there was a compromise
it referred not to the search warrant and the incidents thereof but to
the institution of criminal proceedings for violation of the Anti-Usury
Law. The waiver would have been a good defense for the respondents
had the petitioner voluntarily consented to the search and seizure of
the articles in question, but such was not the case because the
petitioner protested from the beginning and stated his protest in
writing in the insufficient inventory furnished him by the agents.
12. CIVIL PROCEDURE; MANDAMUS; WHEN IT LIES. — Section 222
of the Code of Civil Procedure in fact provides that mandamus will not
issue when there is another plain, speedy and adequate remedy in the
ordinary course of law. We are of the opinion, however, that an appeal
from said orders would not in this case be a plain, speedy and
adequate remedy for the petitioner because a long time would have to
elapse before he recovers possession of the documents and before the
rights, for which he has been unlawfully deprived, are restored to him
(Fajardo vs. Llorente, 6 Phil., 426; Manotoc vs. McMicking and Trinidad,
10 Phil., 119; Cruz Herrera de Lukban vs. McMicking, 14 Phil., 641;
Lamb vs. Phipps, 22 Phil., 456).

DECISION

IMPERIAL, J :p
The petitioner asks that the warrant of June 3, 1936, issued by
the Court of First Instance of Tayabas, ordering the search of his house
and the seizure, at any time of the day or night, of certain accounting
books, documents and papers belonging to him in his residence
situated in Infanta, Province of Tayabas, as well as the order of a later
date, authorizing the agents of the Anti-Usury Board to retain the
articles seized, be declared illegal and set aside, and prays that all the
articles in question be returned to him.
On the date above-mentioned, the chief of the secret service of
the Anti-Usury Board, of the Department of Justice, presented to
Judge Eduardo Gutierrez David then presiding over the Court of First
Instance of Tayabas, an affidavit alleging that according to reliable
information, the petitioner kept in his house in Infanta, Tayabas, books,
documents, receipts, lists, chits and other papers used by him in
connection with his activities as a money-lender, charging usurious
rates of interest in violation of the law. In his oath at the end of the
affidavit, the chief of the secret service stated that his answers to the
questions were correct to the best of his knowledge and belief. He did
not swear to the truth of his statements upon his own knowledge of
the facts but upon the information received by him from a reliable
person. Upon the affidavit in question the judge, on said date, issued
the warrant which is the subject matter of the petition, ordering the
search of the petitioner's house at any time of the day or night, the
seizure of the books and documents above-mentioned and the
immediate delivery thereof to him to be disposed of in accordance
with the law. With said warrant, several agents of the Anti-Usury Board
entered the petitioner's store and residence at seven o'clock on the
night of June 4, 1936, and seized and took possession of the following
articles: internal revenue licenses for the years 1933 to 1936, one
ledger, two journals, two cashbooks, nine order books, four notebooks,
four check stubs, two memorandums, three bankbooks, two contracts,
four stubs, forty-eight stubs of purchases of copra, two inventories,
two bundles of bills of lading, one bundle of credit receipts, one bundle
of stubs of purchases of copra, two packages of correspondence, one
receipt book belonging to Luis Fernandez, fourteen bundles of invoices
and other papers, many documents and loan contracts with security
and promissory notes, 504 chits, promissory notes and stubs of used
checks of the Hongkong & Shanghai Banking Corporation. The search
for and seizure of said articles were made with the opposition of the
petitioner who stated his protest below the inventories on the ground
that the agents seized even the originals of the documents. As the
articles had not been brought immediately to the judge who issued the
search warrant, the petitioner, through his attorney, filed a motion on
June 8, 1936, praying that the agent Emilio L. Siongco, or any other
agent, be ordered immediately to deposit all the seized articles in the
office of the clerk of court and that said agent be declared guilty of
contempt for having disobeyed the order of the court. On said date the
court issued an order directing Emilio L. Siongco to deposit all the
articles seized within twenty-four hours from the receipt of notice
thereof and giving him a period of five (5) days within which to show
cause why he should not be punished for contempt of court. On June
10th, Attorney Arsenio Rodriguez, representing the Anti-Usury Board,
filed a motion praying that the order of the 8th of said month be set
aside and that the Anti-Usury Board be authorized to retain the
articles seized for a period of thirty (30) days for the necessary
investigation. The attorney for the petitioner, on June 20th, filed
another motion alleging that, notwithstanding the order of the 8th of
said month, the officials of the Anti-Usury Board had failed to deposit
the articles seized by them and praying that a search warrant be
issued, that the sheriff be ordered to take all the articles into his
custody and deposit them in the clerk's office, and that the officials of
the Anti-Usury Board be punished for contempt o court. Said attorney,
on June 24th, filed an ex parte petition alleging that while agent
Emilio L. Siongco had deposited some documents and papers in the
office of the clerk of court, he had so far failed to file an inventory duly
verified by oath of all the documents seized by him, to return the
search warrant together with the affidavit presented in support
thereof, or to present the report of the proceedings taken by him; and
prayed that said agent be directed to file the documents in question
immediately. On the 25th of said month the court issued an order
requiring agent Emilio L. Siongco forthwith to file the search warrant
and the affidavit in the court, together with the proceedings taken by
him, and to present an inventory duly verified by oath of all the articles
seized. On July 2d of said year, the attorney for the petitioner filed
another petition alleging that the search warrant issued was illegal
and that it had not yet been returned to date together with the
proceedings taken in connection therewith, and praying that said
warrant be cancelled, that an order be issued directing the return of
all the articles seized to the petitioner, that the agent who seized them
be declared guilty of contempt of court, and that charges be filed
against him for abuse of authority. On September 10, 1936, the court
issued an order holding: that the search warrant was obtained and
issued in accordance with the law, that it had been duly complied with
and, consequently, should not be cancelled, and that agent Emilio L.
Siongco did not commit any contempt of court and must, therefore, be
exonerated, and ordering the chief of the Anti-Usury Board in Manila to
show cause, if any, within the unextendible period of two (2) days from
the date of notice of said order, why all the articles seized appearing
in the inventory, Exhibit 1, should not be returned to the petitioner. The
assistant chief of the Anti-Usury Board of the Department of Justice
filed a motion praying, for the reasons stated therein, that the articles
seized be ordered retained for the purpose of conducting an
investigation of the violation of the Anti-Usury Law committed by the
petitioner. In view of the opposition of the attorney for the petitioner,
the court, on September 25th, issued an order requiring the Anti-Usury
Board to specify the time needed by it to examine the documents and
papers seized and which of them should be retained, granting it a
period of five (5) days for said purpose. On the 30th of said month the
assistant chief of the Anti-Usury Board filed a motion praying that he
be granted ten (10) days to comply with the order of September 25th
and that the clerk of court be ordered to return to him all the
documents and papers together with the inventory thereof. The court,
in an order of October 2d of said year, granted him the additional
period of ten (10) days and ordered the clerk of court to send him a
copy of the inventory. On October 10th, said official again filed another
motion alleging that he needed sixty (60) days to examine the
documents and papers seized, which are designated on pages 1 to 4 of
the inventory by Nos. 5, 10, 16, 23, 25, 26, 27, 30, 31 , 34, 36, 37, 38, 39,
40, 41, 42, 43 and 45, and praying that he be granted said period of
sixty (60) days. In an order of October 16th, the court granted him the
period of sixty (60) days to investigate said nineteen (19) documents.
The petitioner alleges, and it is not denied by the respondents, that
these nineteen (19) documents continue in the possession of the
court, the rest having been returned to said petitioner.
I. A search warrant is an order in writing, issued in the name of
the People of the Philippine Islands, signed by a judge or a justice of
the peace, and directed to a peace officer, commanding him to search
for personal property and bring it before the court (section 95, General
Orders, No. 58, as amended by section 6 of Act No. 2886). Of all the
rights of a citizen, few are of greater importance or more essential to
his peace and happiness than the right of personal security, and that
involves the exemption of his private affairs, books, and papers from
the inspection and scrutiny of others (In re Pacific Railway
Commission, 32 Fed., 241; Interstate Commerce Commn. vs. Brimson,
38 Law. ed., 1047; Boyd vs. U. S., 29 Law. ed., 746; Carroll vs. U. S., 69
Law. ed., 543, 549). While the power to search and seize is necessary
to the public welfare, still it must be exercised and the law enforced
without transgressing the constitutional rights of citizens, for the
enforcement of no statute is of sufficient importance to justify
indifference to the basic principles of government (People vs. Elias,
147 N. E., 472).
II. As the protection of the citizen and the maintenance of his
constitutional rights is one of the highest duties and privileges of the
court, these constitutional guaranties should be given a liberal
construction or a strict construction in favor of the individual, to
prevent stealthy encroachment upon, or gradual depreciation of, the
rights secured by them (State vs. Custer County, 198 Pac., 362; State
vs. McDaniel, 231 Pac., 965; 237 Pac., 373). Since the proceeding is a
drastic one, it is the general rule that statutes authorizing searches
and seizures or search warrants must be strictly construed (Rose vs.
St. Clair, 28 Fed. [2d], 189; Leonard vs. U. S., 6 Fed. [2d], 353; Perry vs.
U. S., 14 Fed. [2d], 88; Cofer vs. State, 118 So., 613).

III. The petitioner claims that the search warrant issued by the
court is illegal because it has been based upon the affidavit of agent
Mariano G. Almeda in whose oath he declared that he had no personal
knowledge of the facts which were to serve as a basis for the issuance
of the warrant but that he had knowledge thereof through mere
information secured from a person whom he considered reliable. To
the question "What are your reasons for applying for this search
warrant", appearing in the affidavit, the agent answered: "It has been
reported to me by a person whom I consider to be reliable that there
are being kept in said premises, books, documents, receipts, lists,
chits, and other papers used by him in connection with his activities
as a money- lender, charging a usurious rate of interest, in violation of
the law" and in attesting the truth of his statements contained in the
affidavit, the said agent stated that he found them to be correct and
true to the best of his knowledge and belief.
Section 1, paragraph 3, of Article III of the Constitution, relative
to the bill of rights, provides that "The right of the people to be secure
in their persons, houses, papers, and effects against unreasonable
searches and seizures shall not be violated, and no warrants shall
issue but upon probable cause, to be determined by the judge after
examination under oath or affirmation of the complainant and the
witnesses he may produce, and particularly describing the place to be
searched, and the persons or things to be seized." Section 97 of
General Orders, No. 58 provides that "A search warrant shall not issue
except for probable cause and upon application supported by oath
particularly describing the place to be searched and the person or
thing to be seized." It will be noted that both provisions require that
there be not only probable cause before the issuance of a search
warrant but that the search warrant must be based upon an
application supported by oath of the applicant and the witnesses he
may produce. In its broadest sense, an oath includes any form of
attestation by which a party signifies that he is bound in conscience
to perform an act faithfully and truthfully; and it is sometimes defined
as an outward pledge given by the person taking it that his attestation
or promise is made under an immediate sense of his responsibility to
God (Bouvier's Law Dictionary; State vs. Jackson, 137 N. W., 1034; In
re Sage, 24 Oh. Cir. Ct. [N. S.], 7; Pumphrey vs. State, 122 N. W., 19;
Priest vs. State, 6 N. W., 468; State vs. Jones, 154 Pac., 378; Atwood
vs. State, 111 So., 865). The oath required must refer to the truth of the
facts within the personal knowledge of the petitioner or his witnesses,
because the purpose thereof is to convince the committing
magistrate, not the individual making the affidavit and seeking the
issuance of the warrant, of the existence of probable cause (U. S. vs.
Tureaud, 20 Fed., 621; U. S. vs. Michalski, 265 Fed., 839; U. S. vs.
Pitotto, 267 Fed., 603; U. S. vs. Lai Chew, 298 Fed., 652). The true test
of sufficiency of an affidavit to warrant issuance of a search warrant is
whether it has been drawn in such a manner that perjury could be
charged thereon and affiant be held liable for damages caused (State
vs. Roosevelt County 20th Jud. Dis. Ct., 244 Pac., 280; State vs.
Quartier, 236 Pac., 746).
It will likewise be noted that section 1, paragraph 3, of Article III
of the Constitution prohibits unreasonable searches and seizures.
Unreasonable searches and seizures are a menace against which the
constitutional guaranties afford full protection. The term
"unreasonable search and seizure" is not defined in the Constitution or
in General Orders, No. 58, and it is said to have no fixed, absolute or
unchangeable meaning, although the term has been defined in general
language. All illegal searches and seizures are unreasonable while
lawful ones are reasonable. What constitutes a reasonable or
unreasonable search or seizure in any particular case is purely a
judicial question, determinable from a consideration of the
circumstances involved, including the purpose of the search, the
presence or absence of probable cause, the manner in which the
search and seizure was made, the place or thing searched, and the
character of the articles procured (Go-Bart Importing Co. vs. U. S., 75
Law. ed., 374; Peru vs. U. S., 4 Fed., [2d], 881; U. S. vs. Vatune, 292 Fed.,
497; Agnello vs. U. S., 70 Law. ed., 145; Lambert vs. U. S., 282 Fed., 413;
U. S. vs. Bateman, 278 Fed., 231; Mason vs. Rollins, 16 Fed. Cas. [No.
9252], 2 Biss., 99).
In view of the foregoing and under the above-cited authorities, it
appears that the affidavit, which served as the exclusive basis of the
search warrant, is insufficient and fatally defective by reason of the
manner in which the oath was made, and therefore, it is hereby held
that the search warrant in question and the subsequent seizure of the
books, documents and other papers are illegal and do not in any way
warrant the deprivation to which the petitioner was subjected.
IV. Another ground alleged by the petitioner in asking that the
search warrant be declared illegal and cancelled is that it was not
supported by other affidavits aside from that made by the applicant. In
other words, it is contended that the search warrant cannot be issued
unless it be supported by affidavits made by the applicant and the
witnesses to be presented necessarily by him. Section 1, paragraph 3,
of Article III of the Constitution provides that no warrants shall issue
but upon probable cause, to be determined by the judge after
examination under oath or affirmation of the complainant and the
witnesses he may produce. Section 98 of General Orders, No. 58
provides that the judge or justice must, before issuing the warrant,
examine under oath the complainant and any witnesses he may
produce and take their depositions in writing. It is the practice in this
jurisdiction to attach the affidavit of at least the applicant or
complainant to the application. It is admitted that the judge who
issued the search warrant in this case, relied exclusively upon the
affidavit made by agent Mariano G. Almeda and that he did not require
nor take the deposition of any other witness. Neither the Constitution
nor General Orders, No. 58 provides that it is of imperative necessity
to take the depositions of the witnesses to be presented by the
applicant or complainant in addition to the affidavit of the latter. The
purpose of both in requiring the presentation of depositions is nothing
more than to satisfy the committing magistrate of the existence of
probable cause. Therefore, if the affidavit of the applicant or
complainant is sufficient, the judge may dispense with that of other
witnesses. Inasmuch as the affidavit of the agent in this case was
insufficient because his knowledge of the facts was not personal but
merely hearsay, it is the duty of the judge to require the affidavit of
one or more witnesses for the purpose of determining the existence of
probable cause to warrant the issuance of the search warrant. When
the affidavit of the applicant or complainant contains sufficient facts
within his personal and direct knowledge, it is sufficient if the judge is
satisfied that there exists probable cause; when the applicant's
knowledge of the facts is mere hearsay, the affidavit of one or more
witnesses having a personal knowledge of the facts is necessary. We
conclude, therefore, that the warrant issued is likewise illegal because
it was based only on the affidavit of the agent who had no personal
knowledge of the facts.
V. The petitioner alleged as another ground for the declaration of
the illegality of the search warrant and the cancellation thereof, the
fact that it authorized its execution at night. Section 101 of General
Orders, No. 58 authorizes that the search be made at night when it is
positively asserted in the affidavit that the property is on the person
or in the place ordered to be searched. As we have declared the
affidavit insufficient and the warrant issued exclusively upon it illegal,
our conclusion is that the contention is equally well founded and that
the search could not legally be made at night.
VI. One of the grounds alleged by the petitioner in support of his
contention that the warrant was issued illegally is the lack of an
adequate description of the books and documents to be seized.
Section 1, paragraph 3, of Article III of the Constitution, and section 97
of General Orders, No. 58 provide that the affidavit to be presented,
which shall serve as the basis for determining whether probable cause
exists and whether the warrant should be issued, must contain a
particular description of the place to be searched and the person or
thing to be seized. These provisions are mandatory and must be
strictly complied with (Munch vs. U. S., 24 Fed. [2d], 518; U. S. vs. Boyd,
1 Fed. [2d], 1019; U. S. vs. Carlson, 292 Fed., 463; U. S. vs. Borkowski,
268 Fed., 408; In re Tri-State Coal & Coke Co., 253 Fed., 605; People vs.
Mayen, 188 Cal., 237; People vs. Kahn, 256 Ill. App., 415); but where, by
the nature of the goods to be seized, their description must be rather
general, it is not required that a technical description be given, as this
would mean that no warrant could issue (People vs. Rubio, 57 Phil.,
284; People vs. Kahn, supra). The only description of the articles given
in the affidavit presented to the judge was as follows: "that there are
being kept in said premises books, documents, receipts, lists, chits
and other papers used by him in connection with his activities as
money-lender, charging a usurious rate of interest, in violation of the
law." Taking into consideration the nature of the articles so described,
it is clear that no other more adequate and detailed description could
have been given, particularly because it is difficult to give a particular
description of the contents thereof. The description so made
substantially complies with the legal provisions because the officer of
the law who executed the warrant was thereby placed in a position
enabling him to identify the articles, which he did.
VII. The last ground alleged by the petitioner, in support of his
claim that the search warrant was obtained illegally, is that the
articles were seized in order that the Anti-Usury Board might provide
itself with evidence to be used by it in the criminal case or cases
which might be filed against him for violation of the Anti-Usury Law. At
the hearing of the incidents of the case raised before the court, it
clearly appeared that the books and documents had really been seized
to enable the Anti-Usury Board to conduct an investigation and later
use all or some of the articles in question as evidence against the
petitioner in the criminal cases that may be filed against him. The
seizure of books and documents by means of a search warrant, for the
purpose of using them as evidence in a criminal case against the
person in whose possession they were found, is unconstitutional
because it makes the warrant unreasonable, and it is equivalent to a
violation of the constitutional provision prohibiting the compulsion of
an accused to testify against himself (Uy Kheytin vs. Villareal , 42
Phil., 886; Brady vs. U. S., 266 U. S., 620; Temperani vs. U. S., 299 Fed.,
365; U. S. vs. Madden, 297 Fed., 679; Boyd vs. U. S., 116 U. S., 616;
Carroll vs. U. S., 267 U. S., 132). Therefore, it appearing that at least
nineteen of the documents in question were seized for the purpose of
using them as evidence against the petitioner in the criminal
proceeding or proceedings for violation of the Anti-Usury Law, which it
is attempted to institute against him, we hold that the search warrant
issued is illegal and that the documents should be returned to him.
The Anti-Usury Board insinuates in its answer that the petitioner
cannot now question the validity of the search warrant or the
proceedings had subsequent to the issuance thereof, because he has
waived his constitutional rights in proposing a compromise whereby
he agreed to pay a fine of P200 for the purpose of evading the criminal
proceeding or proceedings. We are of the opinion that there was no
such waiver, first, because the petitioner has emphatically denied the
offer of compromise and, second, because if there was a compromise
it referred not to the search warrant and the incidents thereof but to
the institution of criminal proceedings for violation of the Anti- Usury
Law. The waiver would have been a good defense for the respondents
had the petitioner voluntarily consented to the search and seizure of
the articles in question, but such was not the case because the
petitioner protested from the beginning and stated his protest in
writing in the insufficient inventory furnished him by the agents.
Said board alleges as another defense that the remedy sought by
the petitioner does not lie because he can appeal from the orders
which prejudiced him and are the subject matter of his petition.
Section 222 of the Code of Civil Procedure in fact provides that
mandamus will not issue when there is another plain, speedy and
adequate remedy in the ordinary course of law. We are of the opinion,
however, that an appeal from said orders would not in this case be a
plain, speedy and adequate remedy for the petitioner because a long
time would have to elapse before he recovers possession of the
documents and before the rights, of which he has been unlawfully
deprived, are restored to him (Fajardo vs. Llorente, 6 Phil., 426;
Manotoc vs. McMicking and Trinidad, 10 Phil., 119; Cruz Herrera de
Lukban vs. McMicking, 14 Phil., 641; Lamb vs. Phipps, 22 Phil., 456).
Summarizing the foregoing conclusions, we hold:
1. That the provisions of the Constitution and General Orders, No.
58, relative to search and seizure, should be given a liberal
construction in favor of the individual in order to maintain the
constitutional guaranties whole and in their full force;
2. That since the provisions in question are drastic in their form
and fundamentally restrict the enjoyment of the ownership, possession
and use of the personal property of the individual, they should be
strictly construed;
3. That the search and seizure made are illegal for the following
reasons: (a) Because the warrant was based solely upon the affidavit
of the petitioner who had no personal knowledge of the facts
necessary to determine the existence or non-existence of probable
cause, and (b) because the warrant was issued for the sole purpose of
seizing evidence which would later be used in the criminal
proceedings that might be instituted against the petitioner, for
violation of the Anti- Usury Law;
4. That as the warrant had been issued unreasonably, and as it
does not appear positively in the affidavit that the articles were in the
possession of the petitioner and in the place indicated, neither could
the search and seizure be made at night;
5. That although it is not mandatory to present affidavits of
witnesses to corroborate the applicant or complainant in cases where
the latter has personal knowledge of the facts, when the applicant's or
complainant's knowledge of the facts is merely hearsay, it is the duty
of the judge to require affidavits of other witnesses so that he may
determine whether probable cause exists;
6. That a detailed description of the person and place to be
searched and the articles to be seized is necessary, but where, by the
nature of the articles to be seized, their description must be rather
general, it is not required that a technical description be given, as this
would mean that no warrant could issue;
7. That the petitioner did not waive his constitutional rights
because the offer of compromise or settlement attributed to him, does
not mean, if so made, that he voluntarily tolerated the search and
seizure; and
8. That an appeal from the orders questioned by the petitioner, if
taken by him, would not be an effective, speedy or adequate remedy in
the ordinary course of law, and, consequently, the petition for
mandamus filed by him lies.
For the foregoing considerations, the search warrant and the
seizure of June 3, 1936, and the orders of the respondent court
authorizing the retention of the books and documents, are declared
illegal and are set aside, and it is ordered that the judge presiding over
the Court of First Instance of Tayabas direct the immediate return to
the petitioner of the nineteen (19) documents designated on pages 1
to 4 of the inventory by Nos. 5, 10, 16, 23, 25, 26, 27, 30, 31, 34, 36, 37,
38, 39, 40, 41, 42, 43 and 45, without special pronouncement as to
costs. So ordered.
(Alvarez v. Court of First Instance of Tayabas, G.R. No. 45358, [January
|||

29, 1937], 64 PHIL 33-51)

[G.R. No. 50720. March 26, 1984.]

SORIANO MATA, petitioner, vs. HON. JOSEPHINE K. BAYONA,


in her capacity as Presiding Judge of the City Court of Ormoc,
BERNARDO GOLES and REYNALDO MAYOTE, respondents.

Valeriano R. Ocubillo for petitioner.


The Solicitor General for respondents.

SYLLABUS

1. CONSTITUTIONAL LAW; BILL OF RIGHTS; RIGHT AGAINST


UNLAWFUL SEARCH AND SEIZURE; REQUISITES FOR ISSUANCE OF
SEARCH WARRANT. — Under the Constitution "no search warrant shall
issue but upon probable cause to be determined by the Judge or such
other responsible officer as may be authorized by law after
examination under oath or affirmation of the complainant and the
witnesses he may produce". More emphatic and detailed is the
implementing rule of the constitutional injunction, Section 4 of Rule
126 which provides that the judge must before issuing the warrant
personally examine on oath or affirmation the complainant and any
witnesses he may produce and take their depositions in writing, and
attach them to the record, in addition to any affidavits presented to
him.
2. ID.; ID.; ID.; ID.; INSUFFICIENCY OF AFFIDAVITS OF
COMPLAINANT AND HIS WITNESSES IN THE CASE AT BAR. — Before
issuing a search warrant, the examining Judge has to take depositions
in writing of the complainant and the witnesses he may produce and
to attach them to the record. Such written deposition is necessary in
order that the Judge may be able to properly determine the existence
or non-existence of the probable cause, and to hold liable for perjury
the person giving it if it will be found later that his declarations are
false. Mere affidavits of the complainant and his witnesses are thus
not sufficient.
3. ID.; ID.; ID.; ID.; NO "DEPOSITION IN WRITING" ATTACHED TO
RECORDS OF CASE IN CASE AT BAR. — The judge's insistence that she
examined the complainants under oath has become dubious by
petitioner's claim that at the particular time when he examined all the
relevant papers connected with the issuance of the questioned search
warrant, after he demanded the same from the lower court since they
were not attached to the records, he did not find any certification at
the back of the joint affidavit of the complainants. Before he filed his
motion to quash the search warrant and for the return of the articles
seized, he was furnished, upon his request, certified true copies of the
said affidavits by the Clerk of Court but which certified true copies do
not bear any certification at the back. Petitioner likewise claims that
his xerox copy of the said joint affidavit obtained at the outset of this
case does not show also the certification of respondent judge. This
doubt becomes more confirmed by respondent Judge's own admission,
while insisting that she did examine thoroughly the applicants, that
"she did not take the deposition of Mayote and Goles because to have
done so would be to hold a judicial proceeding which will be open and
public", such that, according to her, the persons subject of the
intended raid will just disappear and move his illegal operations
somewhere else. Could it be that the certification was made belatedly
to cure the defect of the warrant? Be that as it may, there was no
"deposition in writing" attached to the records of the case in palpable
disregard of the statutory prohibition heretofore quoted.
4. ID.; ID.; ID.; ID.; DEPOSITIONS, HOW TAKEN. — The searching
questions propounded to the applicants of the search warrant and his
witnesses must depend to a large extent upon the discretion of the
Judge just as long as the answers establish a reasonable ground to
believe the commission of a specific offense and that the applicant is
one authorized by law, and said answers particularly describe with
certainty the place to be searched and the persons or things to be
seized. The examination or investigation which must be under oath
may not be in public. It may even be held in the secrecy of his
chambers. Far more important is that the examination or investigation
is not merely routinary but one that is thorough and elicit the required
information. To repeat, it must be under oath and must be in writing.
5. ID.; ID.; ID.; ID.; MUST BE STRICTLY COMPLIED WITH; CASE AT
BAR. — Nothing can justify the issuance of the search warrant but the
fulfillment of the legal requisites. Thus, in issuing a search warrant the
Judge must strictly comply with the requirements of the Constitution
and the statutory provisions. In the case at bar, the search warrant is
tainted with illegality by the failure of the Judge to conform with
essential requisites of taking the depositions in writing and attaching
them to record, rendering the search warrant invalid.
6. ID.; ID.; ID.; ALTHOUGH ILLEGAL, THINGS SEIZED CANNOT BE
RETURNED; CASE AT BAR. — While the search warrant is illegal, the
return of the things seized cannot be ordered. In Castro vs. Pabalan
(70 SCRA 478), it was held that the illegality of the search warrant
does not call for the return of the things seized, the possession of
which is prohibited.

DECISION

DE CASTRO, J : p

The validity of the search warrant issued by respondent Judge


(not reappointed) is challenged by petitioner for its alleged failure to
comply with the requisites of the Constitution and the Rules of Court.
Specifically, the contention is that the search warrant issued by
respondent Judge was based merely on the application for search
warrant and a joint affidavit of private respondents which were
wrongfully it is alleged subscribed, and sworn to before the Clerk of
Court of respondent Judge. Furthermore, there was allegedly a failure
on the part of respondent Judge to attach the necessary papers
pertinent to the issuance of the search warrant to the records of
Criminal Case No. 4298-CC wherein petitioner is accused under PD
810, as amended by PD 1306, the information against him alleging that
Soriano Mata offered, took and arranged bets on the Jai Alai game by
"selling illegal tickets known as 'Masiao tickets' without any authority
from the Philippine Jai Alai & Amusement Corporation or from the
government authorities concerned." 1
Petitioner claims that during the hearing of the case, he
discovered that nowhere from the records of the said case could be
found the search warrant and other pertinent papers connected to the
issuance of the same, so that he had to inquire from the City Fiscal its
whereabouts, and to which inquiry respondent Judge replied, "it is
with the court". The Judge then handed the records to the Fiscal who
attached them to the records. prcd

This led petitioner to file a motion to quash and annul the search
warrant and for the return of the articles seized, citing and invoking,
among others, Section 4 of Rule 126 of the Revised Rules of Court. The
motion was denied by respondent Judge on March 1, 1979, stating that
the court has made a thorough investigation and examination under
oath of Bernardo U. Goles and Reynaldo T. Mayote, members of the
Intelligence Section of 352nd PC Co./Police District II INP; that in fact
the court made a certification to that effect; and that the fact that
documents relating to the search warrant were not attached
immediately to the record of the criminal case is of no moment,
considering that the rule does not specify when these documents are
to be attached to the records. 2 Petitioner's motion for reconsideration
of the aforesaid order having been denied, he came to this Court, with
the instant petition, praying, among others, that this Court declare the
search warrant to be invalid and all the articles confiscated under
such warrant as inadmissible as evidence in the case, or in any
proceedings on the matter.
We hold that the search warrant is tainted with illegality for being
violative of the Constitution and the Rules of Court.
Under the Constitution "no search warrant shall issue but upon
probable cause to be determined by the Judge or such other
responsible officer as may be authorized by law after examination
under oath or affirmation of the complainant and the witnesses he
may produce". More emphatic and detailed is the implementing rule of
the constitutional injunction, Section 4 of Rule 126 which provides that
the judge must before issuing the warrant personally examine on oath
or affirmation the complainant and any witnesses he may produce and
take their depositions in writing, and attach them to the record, in
addition to any affidavits presented to him.
Mere affidavits of the complainant and his witnesses are thus not
sufficient. The examining Judge has to take depositions in writing of
the complainant and the witnesses he may produce and to attach
them to the record. Such written deposition is necessary in order that
the Judge may be able to properly determine the existence or non-
existence of the probable cause, to hold liable for perjury the person
giving it if it will be found later that his declarations are false.
We, therefore, hold that the search warrant is tainted with
illegality by the failure of the Judge to conform with the essential
requisites of taking the depositions in writing and attaching them to
the record, rendering the search warrant invalid. cdll

The judge's insistence that she examined the complainants under


oath has become dubious by petitioner's claim that at the particular
time when he examined all the relevant papers connected with the
issuance of the questioned search warrant, after he demanded the
same from the lower court since they were not attached to the
records, he did not find any certification at the back of the joint
affidavit of the complainants. As stated earlier, before he filed his
motion to quash the search warrant and for the return of the articles
seized, he was furnished, upon his request, certified true copies of the
said affidavits by the Clerk of Court but which certified true copies do
not bear any certification at the back. Petitioner likewise claims that
his xerox copy of the said joint affidavit obtained at the outset of this
case does not show also the certification of respondent judge. This
doubt becomes more confirmed by respondent Judge's own admission,
while insisting that she did examine thoroughly the applicants, that
"she did not take the deposition of Mayote and Goles because to have
done so would be to hold a judicial proceeding which will be open and
public", 3 such that, according to her, the persons subject of the
intended raid will just disappear and move his illegal operations
somewhere else.
Could it be that the certification was made belatedly to cure the
defect of the warrant? Be that as it may, there was no "deposition in
writing" attached to the records of the case in palpable disregard of
the statutory prohibition heretofore quoted.
Respondent Judge impresses this Court that the urgency to stop
the illegal gambling that lures every man, woman and child, and even
the lowliest laborer who could hardly make both ends meet justifies
her action. She claims that in order to abate the proliferation of this
illegal "masiao" lottery, she thought it more prudent not to conduct
the taking of deposition which is done usually and publicly in the court
room.
Two points must be made clear. The term "depositions" is
sometimes used in a broad sense to describe any written statement
verified by oath; but in its more technical and appropriate sense the
meaning of the word is limited to written testimony of a witness given
in the course of a judicial proceeding in advance of the trial or hearing
upon oral examination. 4 A deposition is the testimony of a witness,
put or taken in writing, under oath or affirmation before a
commissioner, examiner or other judicial officer, in answer to
interlocutory and cross interlocutory, and usually subscribed by the
witnesses. 5 The searching questions propounded to the applicants of
the search warrant and his witnesses must depend to a large extent
upon the discretion of the Judge just as long as the answers establish
a reasonable ground to believe the commission of a specific offense
and that the applicant is one authorized by law, and said answers
particularly describe with certainty the place to be searched and the
persons or things to be seized. The examination or investigation which
must be under oath may not be in public. It may even be held in the
secrecy of his chambers. Far more important is that the examination
or investigation is not merely routinary but one that is thorough and
elicit the required information. To repeat, it must be under oath and
must be in writing. LexLib

The other point is that nothing can justify the issuance of the
search warrant but the fulfillment of the legal requisites. It might be
well to point out what has been said in Asian Surety & Insurance Co.,
Inc. vs. Herrera:
"It has been said that of all the rights of a citizen, few are of
greater importance or more essential to his peace and happiness
than the right of personal security, and that involves the exemption
of his private affairs, books, and papers from inspection and
scrutiny of others. While the power to search and seize is
necessary to the public welfare, still it must be exercised and the
law enforced without transgressing the constitutional rights of the
citizens, for the enforcement of no statute is of sufficient
importance to justify indifference to the basic principles of
government." 6

Thus, in issuing a search warrant the Judge must strictly comply


with the requirements of the Constitution and the statutory provisions.
A liberal construction should be given in favor of the individual to
prevent stealthy encroachment upon, or gradual depreciation of the
rights secured by the Constitution. 7 No presumption of regularity are
to be invoked in aid of the process when an officer undertakes to
justify it. 8
While We hold that the search warrant is illegal, the return of the
things seized cannot be ordered. In Castro vs. Pabalan, 9 it was held
that the illegality of the search warrant does not call for the return of
the things seized, the possession of which is prohibited.
WHEREFORE, the writ of certiorari is granted and the order of
March 1, 1979 denying the motion to annul the search warrant as well
as the order of March 21, 1979 denying the motion for reconsideration
are hereby reversed, the search warrant, being declared herein as
illegal. Notwithstanding such illegality, the things seized under such
warrant, such as stock of "masiao" tickets; "masiao" issue tickets; bet
money; control pad or "masiao" numbers; stamping pad with rubber
stamp marked Ormoc City Jai-Alai," cannot be returned as sought by
petitioner. No costs.
SO ORDERED.
||| (Mata v. Bayona, G.R. No. 50720, [March 26, 1984], 213 PHIL 348-355)
[G.R. No. 109633. July 20, 1994.]

THE PEOPLE OF THE PHILIPPINES, plaintiff-


appellee, vs. NORMANDO DEL ROSARIO Y LOPEZ, accused-
appellant.

SYLLABUS

1. REMEDIAL LAW; EVIDENCE; IN CASE AT BAR; FAILURE TO


PRESENT POSEUR-BUYER FATAL TO PROSECUTION'S CASE. — Foremost
among the inadequacies of the prosecution is its failure to call to the
witness stand PO1 Verando Luna, the alleged poseur-buyer. There is,
thus, a total absence of evidence to establish the purported sale of
shabu by accused-appellant to Venerando Luna, the supposed poseur-
buyer. The omission to present the poseur-buyer casts serious doubts
that an illegal sale of a dangerous drug actually took place. The trial
court gave much weight to the testimonies of the police members of the
buy-bust operation. However, the prosecution did not present as witness
the supposed poseur-buyer. Such omission casts serious doubt on
appellant's guilt because without the testimony of the poseur-buyer,
there is not convincing evidence to show that appellant sold marijuana.
The testimonies of the rest of the buy-bust operation are hearsay in view
of the fact that the poseur-buyer was never presented at the trial. There
was even no testimony that when the accused-appellant handed the stuff
to the poseur-buyer that the latter in turn handed the marked money. The
failure of the prosecution to present the alleged buyer of the marijuana
was a fatal flaw in the case against the accused.( People vs. Fulgarillas,
212 SCRA 76, 80 [1992]) The testimony of prosecution witness PO3
Rogelio Francisco that Veneracion Luna, the alleged poseur-buyer,
bought shabu from accused-appellant was derived solely from what Luna
supposedly told him (pp. 19-20, tsn., December 11, 1991) and, therefore,
is patently hearsay evidence, without any evidentiary weight whatsoever.
Likewise, the statements of prosecution witnesses Policemen Reynaldo
de la Cruz, Raymundo Untiveros, and Eduardo Novera, Jr. as to the
alleged sale of shabu are hearsay, without weight, as all of them were
not present during the alleged sale.
2. ID.; ID.; CREDIBILITY OF WITNESSES; IN CASE AT BAR,
PROSECUTION'S VERSION OF "BUY-BUST" OPERATION HIGHLY
INCREDIBLE. — According to the version of the prosecution, during the
alleged buy-bust operation, accused-appellant handed over to Veneracion
Luna, the alleged poseur-buyer, a quantity of shabu, and Luna in turn paid
accused-appellant a marked 100 bill and then returned to the police
station and informed the raiding team that he had already bought the
shabu from accused-appellant. Thereupon, the raiding team proceeded to
the house of accused-appellant to implement the search warrant. The
version of the prosecution is highly incredible. The record is devoid of
any reason why the police officers did not make any attempt to arrest
accused-appellant at the time he allegedly sold the shabu to Veneracion
Luna who was accompanied by another police officer. That was the
opportune moment to arrest accused-appellant. The version foisted by
the prosecution upon this Court is contrary to human experience in the
ordinary course of human conduct. The usual procedure in a buy-bust
operation is for the police officers to arrest the pusher of drugs at the
very moment he hands over the dangerous drug to the poseur-buyer. That
is the every reason why such a police operation is called a "buy-bust"
operation. The police poseur-buyer "buys" dangerous drugs from the
pusher and "bust" (arrests) him the moment the pusher hands over the
drug to the police officer.
3. ID.; ID.; WEIGHT AND SUFFICIENCY OF EVIDENCE; IN CASE AT
BAR, PRESUMPTION OF INNOCENCE IN FAVOR OF ACCUSED NOT
OVERTHROWN. — The manner the police officers conducted the
subsequent and much-delayed search is highly irregular. Upon barging
into the residence of accused-appellant, the police officers found him
lying down and they immediately arrested and detained him in the living
room while they searched the other parts of the house. Although they
fetched two persons to witness the search, the witnesses were called in
only after the policemen had already entered accused-appellant's
residence (pp. 22-23, tsn, December 11, 1991), and, therefore, the
policemen had more than ample time to plant the shabu. Corollary to the
constitutional precept that, in all criminal prosecutions, the accused
shall be presumed innocent until the contrary is proved (Sec. 14(2),
Article III, Constitution of the Republic of the Philippines) is the rule that
in order to convict an accused the circumstances of the case must
exclude all and each and every hypothesis consistent with his innocence
(People vs. Tanchoco; 76 Phil. 463 [1946]; People vs. Constante, 12 SCRA
653 [1964]; People vs. Jara, 144 SCRA 516 [1986]). The facts of the case
do not rule out the hypothesis that accused-appellant is innocent.
4. ID.; CRIMINAL PROCEDURE; JUDGMENT; ACCUSED CANNOT BE
CONVICTED OF CRIME NOT CHARGED IN THE INFORMATION. — Accused-
appellant cannot be convicted of possession of the shabu contained in a
canister and allegedly seized at his house, for the charge against him
was for selling shabu with the information alleging that the "accused,
without legal authority did Hydrocholoride." Sale is totally different from
possession. Article 1458 of the Civil Code defines sale as a contract
whereby "one of the contracting parties obligates himself to transfer the
ownership of and to deliver a determinate thing, and the other to pay
therefor a price certain in money or its equivalent", while "possession is
the holding of a thing or the enjoyment of a right" as defined by Article
523 of the Civil Code. Accused-appellant cannot be convicted of a crime
which is not charged in the information for to do so would deny him the
due process of law (People vs. Despavellador, 2 SCRA 205 [1961];People
vs. Mori, 55 SCRA 382 [1974]).
5. CONSTITUTIONAL LAW; BILL OF RIGHTS; RIGHT AGAINST
UNREASONABLE SEARCHES AND SEIZURES; EXCLUSION IN EVIDENCE
OF ILLEGALLY SEIZED ARTICLES. — The search warrant implemented by
the raiding party authorized only the search and seizure of ".. the
described quantity of Methamphetamine Hydrochloride commonly known
as shabu and its paraphernalia" (Exh. O, p. 50, original record). Thus, the
raiding party was authorized to seize only shabu and paraphernalia for
the use thereof and no other. A search warrant is not a sweeping
authority empowering a raiding party to undertake a fishing expedition to
seize and confiscate any and all kinds of evidence or articles relating to
a crime. The Constitution itself (Section 2, Article III) and the Rules of
Court (Section 3, Rule 126) specifically mandate that the search warrant
must particularly describe the things to be seized. Thus, the search
warrant was no authority for the police officers to seize the firearm
which was not mentioned, much less described with particularly, in the
search warrant. Neither may it be maintained that the gun was seized in
the course of an arrest, for as earlier observed, accused-appellant's
arrest was far from regular and legal. Said firearm, having been illegally
seized, the same is not admissible in evidence ( Stonehill vs. Diokno, 20
SCRA 383 [1967]). The Constitution expressly ordains the exclusion in
evidence of illegally seized articles. Any evidence obtained in violation of
this or the preceding section shall be inadmissible for any purpose in any
proceeding.(Section 3 [2], Article III, Constitution of the Republic of the
Philippines).With the exclusion in evidence of the illegally seized firearm,
there is, therefore, a total absence of evidence to support the charge of
illegal possession of firearm, against accused-appellant. The same way
may be said of the charge of illegal possession of ammunition.

DECISION

MELO, J :p
Normando del Rosario was charged before Branch 17 of the
Regional Trial Court of the Fourth Judicial Region stationed in Cavite City
with Illegal Possession of Firearm and Ammunitions in Criminal Case No.
236-91 and Illegal Sale of Regulated Drugs in Criminal Case No. 237-91,
under two informations reading, respectively, as follows:
Criminal Case No. 236-91
That on or about September 4, 1991, in the City of Cavite,
Republic of the Philippines and within the jurisdiction of this
Honorable Court, the above-named accused, without legal
authority, did, then and there, willfully, unlawfully, feloniously and
knowingly have in his possession and control a homemade (paltik)
caliber 22 revolver with three (3) live ammunition.cdrep

Contrary to law.
Criminal Case No. 237-91
That on or about September 4, 1991, in the City of Cavite,
Republic of the Philippines and within the jurisdiction of this
Honorable Court, the above-named accused, without legal
authority, did, then and there, willfully, unlawfully, feloniously and
knowingly sell to a poseur buyer and aluminum foil containing
Methamphetamine Hydrochloride also known as "Shabu", a
regulated drug.
Contrary to law.
(pp. 20-21, Rollo.)
Upon arraignment, accused-appellant pleaded not guilty to both
charges, and after joint trial of the two cases, the court a quo rendered a
decision, the dispositive portion of which reads:
WHEREFORE, in view of the foregoing, the Court finds the
accused Normando del Rosario y Lopez guilty beyond reasonable
doubt in the above-entitled cases and he is hereby sentenced to
undergo imprisonment: in Crim. Case No. 236-91 for Violation
of P.D. 1866 of Seventeen (17) years, Four (4) months and One (1)
day of reclusion temporal, as minimum to Twenty (20) years of
reclusion temporal, as maximum and in Crim. Case No. 237-91 for a
Violation of Section 15, Article III ofRepublic Act 6425, as amended
of life imprisonment and to pay a fine of P30,000.00, without
subsidiary imprisonment in case of insolvency and to pay the costs
in both cases.
The shabu, the One Hundred Peso bill and other
paraphernalia are hereby ordered confiscated in favor of the
government.
(pp. 28-29, Rollo.)
From said decision, the instant appeal has been interposed.
The prosecution's version of the case, as set forth in appellee's
brief, is as follows:
Upon application of SPO3 Raymundo Untiveros of the
Philippine National Police (PNP) of Cavite City, Regional Trial Court
Judge Arturo de Guia issued in the morning of September 4, 1991 a
search warrant (Exh. T, p. 50, Rec. Crim. Case No. 237-91)
authorizing the search and seizure of an "undetermined quantity of
Methamphetamine Hydrocholoride commonly known as shabu and
its paraphernalias" in the premises of appellant's house located at
828 R. Basa St., San Roque, Cavite City. However, the search
warrant was not implemented immediately due to the lack of
police personnel to form the raiding team (pp. 4, 7, tsn., Feb. 4,
1992).cdrep

At about 9 o'clock in the evening of that day, a raiding team


was finally organized. SPO3 Untiveros headed the raiding team
with PO3 Rogelio Francisco, SPO1 Eduardo Novero, SPO3 Reynaldo
de la Cruz, PO1 Carlito Barbuco, PO3 Onrubio and SPO2 Villegas as
members (pp. 5, 10, tsn., Feb. 4, 1992; p. 7, tsn., Dec. 11, 1991).
In the final briefing of the raiding team at the police station,
it was agreed upon that PO1 Venerando Luna will buy shabu from
appellant and after his return from appellant's house, the raiding
team will implement the search warrant (p. 10, tsn., Feb. 4, 1992; p.
17-18, tsn., Dec. 11, 1991). A marked money consisting of a P100 bill
bearing serial no. PQ 329406 (Exh. P, p. 51, Rec.) was given by the
Station Commander to PO1 Luna and entered in the police logbook
(p. 12, Feb. 4, 1992). PO1 Luna with a companion proceeded to
appellant's house to implement the search warrant. Barangay
Capt. Maigue, Norma del Rosario and appellant witnessed the
search at appellant's house (p. 10, tsn., Dec. 11, 1991). SPO3 de la
Cruz and PO3 Francisco found a black canister containing shabu,
an aluminum foil, plastik .22 caliber (Exh. O) atop the TV set, three
used ammunition in a cup and three wallets (Exhs. Q, R, S), one
containing the marked money (Exh. P; pp. 11-12, tsn., Dec. 11,
1991). SPO1 Novero found inside a show box aluminum foils,
napkins and a burner (p. 9, tsn., March 11, 1992). SPO3 de la Cruz
turned over the wallet containing the marked money to PO3
Onrubio (p. 8, tsn., Jan. 7, 1992). The seized items were
photographed thereat by Fred Agana and then turned over to PO3
Onrubio (pp. 8, 32, tsn., Jan. 7, 1992). SPO3 Untiveros issued
receipts (Exhs. V, V-1, pp. 53-54, Rc.) for the seized items with
Barangay Capt. Maigue and appellant's sister Norma as signing
witnesses. He also made a return (Exh. U, p. 52, Rec.) of the seized
items to the court (pp. 11-155, tsn., Feb. 18, 1992).
At police station, the seized items were taped and initialed by
SPO3 de la Cruz (p. 33, tsn., Jan. 7, 1992). The next day, SPO4
Pilapil, through PO1 Barbuco, forwarded to NBI Forensic Chemist
Mary Ann Aranas for laboratory analysis the aluminum foil (Exhs. A,
J, pp. 37, 46, Rec.) containing suspected shabu bought by PO1 Luna
from appellant in the buy-bus operation as well as the aluminum
foils (Exhs. G, K, pp. 43, 47, Rec.) containing suspected marijuana
which were confiscated by virtue of the search warrant.
The findings of NBI Forensic Chemist Aranas disclosed that
all the specimen submitted to her for laboratory analysis by SPO1
Pilapil, thru PO1 Barbuco, gave positive results for
Methamphetamine Hydrocholoride (pp. 2-9, tsn., Dec. 3, 1991; Exh.
B, C, H, I, pp. 38, 39, 44, 45, Rec.).
(pp. 102-105, Rollo.)
Carefully evaluation the evidence on record, we believe that the
prosecution has failed to prove the guilt of accused-appellant. Much is to
be desired in the manner the police authorities effected the arrest of
accused-appellant and the same observation may be made with regard to
the way the prosecution conducted its case. Cdpr

Foremost among the inadequacies of the prosecution is its failure to


call to the witness stand PO1 Verando Luna, the alleged poseur-buyer.
There is, thus, a total absence of evidence to establish the purported
sale of shabu by accused-appellant to Venerando Luna, the supposed
poseur-buyer. The omission to present the poseur-buyer casts serious
doubts that an illegal sale of a dangerous drug actually took place.
The trial court gave much weight to the testimonies of the
police members of the buy-bust operation. However, the
prosecution did not present as witness the supposed poseur-buyer.
Such omission casts serious doubt on appellant's guilt because
without the testimony of the poseur-buyer, there is not convincing
evidence to show that appellant sold marijuana. The testimonies of
the rest of the buy-bust operation are hearsay in view of the fact
that the poseur-buyer was never presented at the trial. There was
even no testimony that when the accused-appellant handed the
stuff to the poseur-buyer that the latter in turn handed the marked
money. The failure of the prosecution to present the alleged buyer
of the marijuana was a fatal flaw in the case against the accused.
(People vs. Fulgarillas, 212 SCRA 76, 80 [1992])
The testimony of prosecution witness PO3 Rogelio Francisco that
Veneracion Luna, the alleged poseur-buyer, bought shabu from accused-
appellant was derived solely from what Luna supposedly told him (pp. 19-
20, tsn., December 11, 1991) and, therefore, is patently hearsay evidence,
without any evidentiary weight whatsoever. Likewise, the statements of
prosecution witnesses Policemen Reynaldo de la Cruz, Raymundo
Untiveros, and Eduardo Novera, Jr. as to the alleged sale of shabu are
hearsay, without weight, as all of them were not present during the
alleged sale.
According to the version of the prosecution, during the alleged buy-
bust operation, accused-appellant handed over to Veneracion Luna, the
alleged poseur-buyer, a quantity of shabu, and Luna in turn paid accused-
appellant a marked 100 bill and then returned to the police station and
informed the raiding team that he had already bought the shabu from
accused-appellant. Thereupon, the raiding team proceeded to the house
of accused-appellant to implement the search warrant. The version of the
prosecution is highly incredible. The record is devoid of any reason why
the police officers did not make any attempt to arrest accused-appellant
at the time he allegedly sold the shabu to Veneracion Luna who was
accompanied by another police officer. That was the opportune moment
to arrest accused-appellant. The version foisted by the prosecution upon
this Court is contrary to human experience in the ordinary course of
human conduct. The usual procedure in a buy-bust operation is for the
police officers to arrest the pusher of drugs at the very moment he hands
over the dangerous drug to the poseur-buyer. That is the every reason
why such a police operation is called a "buy-bust" operation. The police
poseur-buyer "buys" dangerous drugs from the pusher and "bust"
(arrests) him the moment the pusher hands over the drug to the police
officer.
We thus entertain serious doubts that the shabu contained in a
small canister was actually seized or confiscated at the residence of
accused-appellant. in consequence, the manner the police officers
conducted the subsequent and much-delayed search is highly irregular.
Upon barging into the residence of accused-appellant, the police officers
found him lying down and they immediately arrested and detained him in
the living room while they searched the other parts of the house.
Although they fetched two persons to witness the search, the witnesses
were called in only after the policemen had already entered accused-
appellant's residence (pp. 22-23, tsn, December 11, 1991), and, therefore,
the policemen had more than ample time to plant the shabu. Corollary to
the constitutional precept that, in all criminal prosecutions, the accused
shall be presumed innocent until the contrary is proved (Sec. 14(2),
Article III, Constitution of the Republic of the Philippines) is the rule that
in order to convict an accused the circumstances of the case must
exclude all and each and every hypothesis consistent with his innocence
(People vs. Tanchoco; 76 Phil. 463 [1946]; People vs. Constante, 12 SCRA
653 [1964]; People vs. Jara, 144 SCRA 516 [1986]). The facts of the case
do not rule out the hypothesis that accused-appellant is innocent.
At any rate, accused-appellant cannot be convicted of possession of
the shabu contained in a canister and allegedly seized at his house, for
the charge against him was for selling shabu with the information
alleging that the "accused, without legal authority did . . . sell to a poseur
buyer an aluminum foil containing Methamphetamine
Hydrocholoride . . .". Sale is totally different from possession. Article
1458 of the Civil Code defines sale as a contract whereby "one of the
contracting parties obligates himself to transfer the ownership of and to
deliver a determinate thing, and the other to pay therefor a price certain
in money or its equivalent", while "possession is the holding of a thing or
the enjoyment of a right" as defined by Article 523 of the Civil Code.
Accused-appellant cannot be convicted of a crime which is not charged
in the information for to do so would deny him the due process of law
(People vs. Despavellador, 2 SCRA 205 [1961]; People vs. Mori, 55 SCRA
382 [1974]).LLpr

Neither can accused-appellant be convicted of illegal possession of


firearm and ammunition. The search warrant implemented by the raiding
party authorized only the search and seizure of ". . . the described
quantity of Methamphetamine Hydrochloride commonly known as shabu
and its paraphernalia" (Exh. O, p. 50, original record). Thus, the raiding
party was authorized to seize only shabu and paraphernalia for the use
thereof and no other. A search warrant is not a sweeping authority
empowering a raiding party to undertake a fishing expedition to seize and
confiscate any and all kinds of evidence or articles relating to a crime.
The Constitution itself (Section 2, Article III) and the Rules of Court
(Section 3, Rule 126) specifically mandate that the search warrant must
particularly describe the things to be seized. Thus, the search warrant
was no authority for the police officers to seize the firearm which was
not mentioned, much less described with particularly, in the search
warrant. Neither may it be maintained that the gun was seized in the
course of an arrest, for as earlier observed, accused-appellant's arrest
was far from regular and legal. Said firearm, having been illegally seized,
the same is not admissible in evidence (Stonehill vs. Diokno, 20 SCRA
383 [1967]). The Constitution expressly ordains the exclusion in evidence
of illegally seized articles.
Any evidence obtained in violation of this or the preceding
section shall be inadmissible for any purpose in any proceeding.
(Section 3 [2], Article III, Constitution of the Republic of the
Philippines).
With the exclusion in evidence of the illegally seized firearm, there
is, therefore, a total absence of evidence to support the charge of illegal
possession of firearm, against accused-appellant. Cdpr

The same way may be said of the charge of illegal possession of


ammunition.
WHEREFORE, the decision appealed from is hereby REVERSED and
accused-appellant is hereby ACQUITTED in Criminal Case No. 236-91 and
Criminal Case No. 237-91.
The immediate release of accused-appellant is hereby ordered
unless there exists of pending valid cause against him. Cdpr

The shabu, the marked P100 bill, firearm, and ammunition are
hereby ordered confiscated in favor of the government.
SO ORDERED.
(People v. Del Rosario y Lopez, G.R. No. 109633, [July 20, 1994], 304
|||

PHIL 418-428)

[G.R. No. 95847-48. March 10, 1993.]

PEOPLE OF THE PHILIPPINES, plaintiff-appellee, vs. GABRIEL


GERENTE y BULLO, accused-appellant.

The Solicitor General for plaintiff-appellee.


Public Attorney's Office for accused-appellant.

SYLLABUS

1. REMEDIAL LAW; CRIMINAL PROCEDURE; ARREST WITHOUT WARRANT;


LAWFUL WHEN ARRESTING OFFICER HAS PERSONAL KNOWLEDGE THAT
THE PERSON TO BE ARRESTED HAS COMMITTED THE CRIME; CASE AT
BAR. — The policemen arrested Gerente only some three (3) hours after
Gerente and his companions had killed Blace. They saw Blace dead in
the hospital and when they inspected the scene of the crime, they found
the instruments of death: a piece of wood and a concrete hollow block
which the killers had used to bludgeon him to death. The eye-witness,
Edna Edwina Reyes, reported the happening to the policemen and
pinpointed her neighbor, Gerente, as one of the killers. Under those
circumstances, since the policemen had personal knowledge of the
violent death of Blace and of facts indicating that Gerente and two
others had killed him, they could lawfully arrest Gerente without a
warrant. If they had postponed his arrest until they could obtain a
warrant, he would have fled the law as his two companions did.
2. ID.; ID.; SEARCH AND SEIZURE; VALID EVEN WITHOUT A WARRANT
WHEN MADE AS AN INCIDENT TO LAWFUL ARREST; RATIONALE. — The
search conducted on Gerente's person was likewise lawful because it
was made as an incident to a valid arrest. This is in accordance with
Section 12, Rule 126 of the Revised Rules of Court which provides:
"Section 12. Search incident to lawful arrest. — A person lawfully
arrested may be searched for dangerous weapons or anything which may
be used as proof of the commission of an offense, without a search
warrant." The frisk and search of appellant's person upon his arrest was
a permissible precautionary measure of arresting officers to protect
themselves, for the person who is about to be arrested may be armed
and might attack them unless he is first disarmed. In Adams vs.Williams,
47 U.S. 143, cited in Justice Isagani A. Cruz's Constitutional Law, 1991
Edition, p. 150, it was ruled that "the individual being arrested may be
frisked for concealed weapons that may be used against the arresting
officer and all unlawful articles found his person, or within his immediate
control may be seized."
3. CRIMINAL LAW; CONSPIRACY; LIABILITY OF CONSPIRATORS; RULE;
CASE AT BAR. — There is no merit in appellant's allegation that the trial
court erred in convicting him of having conspired and cooperated with
Fredo and Totoy Echigoren to kill Blace despite the testimony of Dr.
Valentin Bernales that the fracture on the back of the victim's skull could
have been inflicted by one person only. what Dr. Bernales stated was a
mere possibility that only one person dropped the concrete hollow block
on the head of the victim, smashing it. That circumstance, even if true,
does not absolve the other two co-conspirators in the murder of Blace for
when there is a conspiracy to commit a crime, the act of one conspirator
is the act of all. The conspiracy was proven by the eyewitness-testimony
of Edna Edwina Reyes, that she overheard the appellant and his
companions conspire to kill Blace, that acting in concert, they attacked
their victim with a piece of wood and a hollow block and caused his
death. "When there is no evidence indicating that the principal witness
for the prosecution was moved by improper motive, the presumption is
that he was not so moved and his testimony is entitled to full faith and
credit" (People vs. Belibet, 199 SCRA 587, 588). Hence, the trial court did
not err in giving full credit to Edna Reyes' testimony.
4. ID.; CIVIL INDEMNITY FOR DEATH; INCREASED TO P50,000.00. — The
Solicitor General correctly pointed out in the appellee's brief that the
award of P30,000.00 as civil indemnity for the death of Clarito Blace
should be increased to P50,000.00 in accordance with our ruling in
People vs. Sison, 189 SCRA 643.

DECISION
GRIÑO-AQUINO, J : p

This is an appeal from the decision of the Regional Trial Court of


Valenzuela, Metro Manila, Branch 172, which found the appellant guilty
of Violation of Section 8 of Republic Act 6425 (Dangerous Drugs Act of
1972) and sentenced him to suffer the penalty of imprisonment for a
term of twelve (12) years and one (1) day, as minimum, to twenty (20)
years, as maximum; and also found him guilty of Murder for which
crime he was sentenced to suffer the penalty of reclusion perpetua.
The dispositive portion of the appealed decision reads:
"WHEREFORE, in view of the foregoing the Court finds the
accused Gabriel Gerente in Criminal Case No. 10255-V-90 guilty
beyond reasonable doubt of Violation of Section 8 of R.A. 6425 and
hereby sentences him to suffer the penalty of imprisonment of
twelve years and one day as minimum to twenty years as
maximum, and a fine of twelve thousand, without subsidiary
imprisonment in case of insolvency, and to pay the costs.
"In Criminal Case No. 10256-V-90, the Court finds the accused
Gabriel Gerente guilty beyond reasonable doubt of the crime of
Murder, and there by (sic) no aggravating circumstances nor
mitigating circumstances, is hereby sentenced to suffer the
penalty of reclusion perpetua; to indemnify the heirs of the victim
in the sum of P30,000.00, and in the amount of P17,609.00 as
funeral expenses, without subsidiary imprisonment in case of
insolvency, and to pay the costs. The accused Gabriel Gerente
shall be credited with the full term of his preventive
imprisonment." (p. 25, Rollo.)
Appellant Gabriel Gerente y Bullo was charged with Violation of
Section 8, Art. II of R.A. 6425, which was docketed as Criminal Case
No. 10255-V-90 of the Regional Trial Court of Valenzuela, Metro Manila.
The Information reads:
"That on or about the 30th day of April, 1990, in the
municipality of Valenzuela, Metro Manila, Philippines, and within
the jurisdiction of this Honorable Court, the above-named accused,
without justification, did then and there wilfully, unlawfully and
feloniously have in his possession and control dried flowering tops
wrapped in foil with markings and place in a transparent plastic
bag which are considered prohibited drugs." (p. 2, Rollo.)
The same accused, together with Totoy and Fredo Echigoren who
are both at large, was charged with Murder in Criminal Case No.
10256-V-90 in an information of the same date and signed by the same
Assistant Provincial Prosecutor, as follows: cdphil

"That on or about the 30th day of April, 1990, in the


municipality of Valenzuela, Metro Manila, Philippines, and within
the jurisdiction of this Honorable Court, the above-named accused
together with two (2) others who are still at large and against
whom the preliminary investigation has not yet been terminated by
the Office of the Provincial Prosecutor of Bulacan, conspiring,
confederating together and mutually helping one another, armed
with a piece of wood and hallow (sic) block and with intent to kill
one Clarito B. Blace, did then and there wilfully, unlawfully and
feloniously, with evident premeditation and treachery, attack,
assault and hit with the said piece of wood and hollow block the
said Clarito B. Blace, hitting the latter on the different parts of his
body, thereby inflicting serious physical injuries which directly
caused the death of the said victim." (p. 3, Rollo.)
Edna Edwina Reyes testified that at about 7:00 a.m. of April 30,
1990, appellant Gabriel Gerente, together with Fredo Echigoren and
Totoy Echigoren, started drinking liquor and smoking marijuana in the
house of the appellant which is about six (6) meters away from the
house of the prosecution witness who was in her house on that day.
She overheard the three men talking about their intention to kill
Clarito Blace. She testified that she heard Fredo Echigoren saying,
"Gabriel, papatayin natin si Clarito Blace," and Totoy Echigoren
allegedly seconded Fredo's suggestion saying: "Papatayin natin 'yan
mamaya." Appellant allegedly agreed: "Sigue, papatayin natin
mamaya." (pp. 3-4, tsn, August 24, 1990.)
Fredo and Totoy Echigoren and Gerente carried out their plan to
kill Clarito Blace at about 2:00 p.m. of the same day. The prosecution
witness, Edna Edwina Reyes, testified that she witnessed the killing.
Fredo Echigoren struck the first blow against Clarito Blace, followed
by Totoy Echigoren and Gabriel Gerente who hit him twice with a piece
of wood in the head and when he fell, Totoy Echigoren dropped a
hollow block on the victim's head. Thereafter, the three men dragged
Blace to a place behind the house of Gerente.
At about 4:00 p.m. of the same day, Patrolman Jaime Urrutia of
the Valenzuela Police Station received a report from the Palo Police
Detachment about a mauling incident. He went to the Valenzuela
District Hospital where the victim was brought. He was informed by
the hospital officials that the victim died on arrival. The cause of
death was massive fracture of the skull caused by a hard and heavy
object. Right away, Patrolman Urrutia, together with Police Corporal
Romeo Lima and Patrolman Alex Umali, proceeded to Paseo de Blas
where the mauling incident took place. There they found a piece of
wood with blood stains, a hollow block and two roaches of marijuana.
They were informed by the prosecution witness, Edna Edwina Reyes,
that she saw the killing and she pointed to Gabriel Gerente as one of
the three men who killed Clarito.
The policemen proceeded to the house of the appellant who was
then sleeping. They told him to come out of the house and they
introduced themselves as policemen. Patrolman Urrutia frisked
appellant and found a coin purse in his pocket which contained dried
leaves wrapped in cigarette foil. The dried leaves were sent to the
National Bureau of Investigation for examination. The Forensic
Chemist found them to be marijuana. LexLib

Only the appellant, Gabriel Gerente, was apprehended by the


police. The other suspects, Fredo and Totoy Echigoren, are still at
large.

On May 2, 1990, two separate informations were filed by


Assistant Provincial Prosecutor Benjamin Caraig against him for
Violation of Section 8, Art. II, of R.A. 6425, and for Murder.
When arraigned on May 16, 1990, the appellant pleaded not guilty
to both charges. A joint trial of the two cases was held. On September
24, 1990, the trial court rendered a decision convicting him of
Violation of Section 8 of R.A. 6425 and of Murder.
In this appeal of the appellant, the following errors are ascribed
to the trial court:
1. the court a quo gravely erred in admitting the marijuana leaves
adduced in evidence by the prosecution; and
2. the court a quo gravely erred in convicting the accused-
appellant of the crimes charged despite the absence of evidence
required to prove his guilt beyond reasonable doubt.

The appellant contends that the trial court erred in admitting the
marijuana leaves as evidence in violation of his constitutional right
not to be subjected to illegal search and seizure, for the dried
marijuana leaves were seized from him in the course of a warrantless
arrest by the police officers. We do not agree.
The search of appellant's person and the seizure of the marijuana
leaves in his possession were valid because they were incident to a
lawful warrantless arrest.
Paragraphs (a) and (b), Section 5, Rule 113 of the Revised Rules of
Court provide:
'SECTION 5. Arrest without warrant; when lawful . — A peace
officer or a private person may, without a warrant, arrest a person:
"(a) When, in his presence, the person to be arrested has
committed, is actually committing, or is attempting to commit an
offense;"
"(b) When an offense has in fact just been committed, and he
has personal knowledge of facts indicating that the person to be
arrested has committed it; . . .'
The policemen arrested Gerente only some three (3) hours after
Gerente and his companions had killed Blace. They saw Blace dead in
the hospital and when they inspected the scene of the crime, they
found the instruments of death: a piece of wood and a concrete hollow
block which the killers had used to bludgeon him to death. The eye-
witness, Edna Edwina Reyes, reported the happening to the policemen
and pinpointed her neighbor, Gerente, as one of the killers. Under
those circumstances, since the policemen had personal knowledge of
the violent death of Blace and of facts indicating that Gerente and two
others had killed him, they could lawfully arrest Gerente without a
warrant. If they had postponed his arrest until they could obtain a
warrant, he would have fled the law as his two companions did. prLL

In Umil vs. Ramos, 187 SCRA 311, the arrest of the accused
without a warrant was effected one (1) day after he had shot to death
two Capcom soldiers. The arrest was held lawful by this Court upon
the rationale stated by us in People vs. Malasugui, 63 Phil. 221, 228,
thus:
"To hold that no criminal can, in any case, be arrested and
searched for the evidence and tokens of his crime without a
warrant, would be to leave society, to a large extent, at the mercy
of the shrewdest, the most expert, and the most depraved of
criminals, facilitating their escape in many instances."
The search conducted on Gerente's person was likewise lawful
because it was made as an incident to a valid arrest. This is in
accordance with Section 12, Rule 126 of the Revised Rules of Court
which provides:
"SECTION 12. Search incident to lawful arrest. — A person
lawfully arrested may be searched for dangerous weapons or
anything which may be used as proof of the commission of an
offense, without a search warrant."
The frisk and search of appellant's person upon his arrest was a
permissible precautionary measure of arresting officers to protect
themselves, for the person who is about to be arrested may be armed
and might attack them unless he is first disarmed. In Adams vs.
Williams, 47 U.S. 143, cited in Justice Isagani A. Cruz's Constitutional
Law, 1991 Edition, p. 150, it was ruled that "the individual being
arrested may be frisked for concealed weapons that may be used
against the arresting officer and all unlawful articles found in his
person, or within his immediate control may be seized."
There is no merit in appellant's allegation that the trial court
erred in convicting him of having conspired and cooperated with Fredo
and Totoy Echigoren to kill Blace despite the testimony of Dr. Valentin
Bernales that the fracture on the back of the victim's skull could have
been inflicted by one person only.
What Dr. Bernales stated was a mere possibility that only one
person dropped the concrete hollow block on the head of the victim,
smashing it. That circumstance, even if true, does not absolve the
other two co-conspirators in the murder of Blace for when there is a
conspiracy to commit a crime, the act of one conspirator is the act of
all. The conspiracy was proven by the eyewitness-testimony of Edna
Edwina Reyes, that she overheard the appellant and his companions
conspire to kill Blace, that acting in concert, they attacked their
victim with a piece of wood and a hollow block and caused his death.
"When there is no evidence indicating that the principal witness for
the prosecution was moved by improper motive, the presumption is
that he was not so moved and his testimony is entitled to full faith and
credit" (People vs. Belibet, 199 SCRA 587, 588). Hence, the trial court
did not err in giving full credit to Edna Reyes' testimony.
prcd

Appellant's failure to escape (because he was very drunk) is no


indicium of his innocence.
The Solicitor General correctly pointed out in the appellee's brief
that the award of P30,000.00 as civil indemnity for the death of Clarito
Blace should be increased to P50,000.00 in accordance with our ruling
in People vs. Sison, 189 SCRA 643.
WHEREFORE, the appealed decision is hereby AFFIRMED, with
modification of the civil indemnity awarded to the heirs of the victim,
Clarito Blace, which is hereby increased to P50,000.00.
SO ORDERED.
||| (People v. Gerente y Bullo, G.R. No. 95847-48, [March 10, 1993])
[G.R. No. 81567. July 9, 1990.]

IN THE MATTER OF THE PETITION FOR HABEAS CORPUS OF


ROBERTO UMIL, ROLANDO DURAL and RENATO VILLANUEVA.
MANOLITA O. UMIL, and NICANOR P. DURAL, FELICITAS V.
SESE, petitioners, vs. FIDEL V. RAMOS, MAJ. GEN. RENATO DE
VILLA, BRIG. GEN. RAMON MONTANO, BRIG. GEN. ALEXANDER
AGUIRRE, respondents.

[G.R. Nos. 84581-82. July 9, 1990.]

AMELIA ROQUE and WILFREDO


BUENAOBRA, petitioners, vs. GEN. RENATO DE VILLA and GEN.
RAMON MONTANO, respondents.

[G.R. Nos. 84583-84. July 9, 1990.]

IN THE MATTER OF THE PETITION FOR HABEAS CORPUS OF


ATTY. DOMINGO T. ANONUEVO and RAMON CASIPLE. DOMINGO T.
ANONUEVO and RAMON CASIPLE, petitioners, vs. HON. FIDEL V.
RAMOS, GEN. RENATO S. DE VILLA, COL. EVARISTO CARINO, LT.
COL. REX D. PIAD, T/ SGT. CONRADO DE TORRES, S/SGT. ARNOLD
DURIAN, and Commanding Officer, PC-INP Detention Center,
Camp Crame, Quezon City, respondents.

[G.R. No. 83162. July 9, 1990.]

IN THE MATTER OF THE APPLICATION FOR HABEAS CORPUS OF


VICKY A. OCAYA AND DANNY RIVERA. VIRGILIO A.
OCAYA, petitioner, vs. BRIG. GEN. ALEXANDER AGUIRRE,
COL., HERCULES CATALUNA, COL. NESTOR
MARIANO, respondents.

[G.R. No. 85727. July 9, 1990.]

IN THE MATTER OF APPLICATION FOR HABEAS CORPUS OF:


DEOGRACIAS ESPIRITU, petitioner, vs. BRIG. GEN. ALFREDO S.
LIM, COL. RICARDO REYES,respondents.

[G.R. No. 86332. July 9, 1990.]


IN THE MATTER OF THE PETITION FOR HABEAS CORPUS OF
NARCISO B. NAZARENO, ALFREDO NAZARENO, petitioner, vs. THE
STATION COMMANDER OF THE MUNTINGLUPA POLICE STATION,
Muntinglupa, Metro Manila, P/SGT. JACINTO MEDINA, P/SGT.
ELADIO TAGLE, P/SGT. LEVI SOLEDAD, and P/SGT. MAURO
AROJADO, respondents.

Efren H. Mercado for petitioners in G.R. No. 81567.


Ricardo C. Valmonte for petitioners in G.R. Nos. 84581-82.
Ramon S. Esguerra Barbara Anne C. Migallos and Agripino G. Morga for
petitioners in G.R. Nos. 84583-84.
Efren H. Mercado for petitioner in G.R. No. 83162.
Banzuela, Flores, Miralles, Raneses, Sy, Taquio & Associates for
petitioner in G.R. No. 85727.
Josefina G. Campbell-Castillo for petitioners in G.R. No. 86332.
The Solicitor General for the respondents.

DECISION

PER CURIAM : p

These are eight (8) petitions for habeas corpus filed before the Court,
which have been consolidated because of the similarity of issues raised,
praying for the issuance of the writ of habeas corpus, ordering the
respective respondents to produce the bodies of the persons named
therein and to explain why they should not be set at liberty without
further delay.
In their respective Returns, the respondents uniformly assert that the
privilege of the writ of habeas corpus is not available to the petitioners
as they have been legally arrested and are detained by virtue of valid
informations filed in court against them. LexLib

The petitioners counter that their detention is unlawful as their arrests


were made without warrant and, that no preliminary investigation was
first conducted, so that the informations filed against them are null and
void.
The Court has carefully reviewed the contentions of the parties in their
respective pleadings, and it finds that the persons detained have not
been illegally arrested nor arbitrarily deprived of their constitutional
right to liberty, and that the circumstances attending these cases do not
warrant their release on habeas corpus.
The arrest of a person without a warrant of arrest or previous complaint
is recognized in law. The occasions or instances when such an arrest
may be effected are clearly spelled out in Section 5, Rule 113 of the
Rules of Court, as amended, which provides:
"Sec. 5. Arrest without warrant; when lawful . — A peace officer or
a private person may, without a warrant, arrest a person:
(a) When, in his presence, the person to be arrested has
committed, is actually committing, or is attempting to commit en
offense;
(b) When an offense has in fact just been committed, and he has
personal knowledge of facts indicating that the person to be
arrested has committed it; and
(c) When the person to be arrested is a prisoner who has escaped
from a penal establishment or place where he is serving final
judgment or temporarily confined while his case is pending, or has
escaped while being transferred from one confinement to another.
In cases falling under paragraphs (a) and (b) hereof, the person
arrested without a warrant shall be forthwith delivered to the
nearest police station or jail, and he shall be proceeded against in
accordance with Rule 112, Section 7."
An arrest without a warrant of arrest, under Section 5 paragraphs (a) and
(b) of Rule 113 of the Rules of Court, as amended, is justified when the
person arrested is caught in flagranti delicto, viz., in the act of
committing an offense; or when an offense has just been committed and
the person making the arrest has personal knowledge of the facts
indicating that the person arrested has committed it. The rationale
behind lawful arrests, without warrant, was stated by this Court in the
case ofPeople vs. Kagui Malasugui 1 thus:
"To hold that no criminal can, in any case, be arrested and
searched for the evidence and tokens of his crime without a
warrant, would be to leave society, to a large extent, at the mercy
of the shrewdest, the most expert, and the most depraved of
criminals, facilitating their escape in many instances."
The record of the instant cases would show that the persons in whose
behalf these petitions for habeas corpus have been filed, had freshly
committed or were actually committing an offense, when apprehended,
so that their arrests without a warrant were clearly justified, and that
they are, further, detained by virtue of valid informations filed against
them in court.
A brief narration of the facts and events surrounding each of the eight (8)
petitions is in order.
I
In G.R. No. 81567 (Umil vs. Ramos), the record shows that, on 1 February
1988, the Regional Intelligence Operations Unit of the Capital Command
(RIOU-CAPCOM) received confidential information about a member of the
NPA Sparrow Unit (liquidation squad) being treated for a gunshot wound
at the St. Agnes Hospital in Roosevelt Avenue, Quezon City. Upon
verification, it was found that the wounded person, who was listed in the
hospital records as Ronnie Javelon, is actually Rolando Dural, a member
of the NPA liquidation squad, responsible for the killing of two (2)
CAPCOM soldiers the day before, or on 31 January 1988, in Macanining
Street, Bagong Barrio, Caloocan City. In view of this verification, Rolando
Dural was transferred to the Regional Medical Services of the CAPCOM,
for security reasons. While confined thereat, or on 4 February 1988,
Rolando Dural was positively identified by eyewitnesses as the gunman
who went on top of the hood of the CAPCOM mobile patrol car, and fired
at the two (2) CAPCOM soldiers seated inside the car identified as T/Sgt.
Carlos Pabon and CIC Renato Manligot.
As a consequence of this positive identification, Rolando Dural was
referred to the Caloocan City Fiscal who conducted an inquest and
thereafter filed with the Regional Trial Court of Caloocan City an
information charging Rolando Dural alias Ronnie Javelon with the crime
of "Double Murder with Assault Upon Agents of Persons in Authority." The
case was docketed therein as Criminal Case No. C-30112 and no bail was
recommended. On 15 February 1988, the information was amended to
include, as defendant, Bernardo Itucal, Jr. who, at the filing of the
original information, was still unidentified.
cdphil

Meanwhile, on 6 February 1988, a petition for habeas corpus was filed


with this Court on behalf of Roberto Umil, Rolando Dural, and Renato
Villanueva. The Court issued the writ of habeas corpus on 9 February
1988 and the respondents filed a Return of the Writ on 12 February 1988.
Thereafter, the parties were heard on 15 February 1988.
On 26 February 1988, however, Roberto Umil and Renato
Villanueva posted bail before the Regional Trial Court of Pasay City
where charges for violation of the Anti-Subversion Act had been filed
against them, and they were accordingly released. The petition for
habeas corpus, insofar as Umil and Villanueva are concerned, is now
moot and academic and is accordingly dismissed, since the writ of
habeas corpus does not lie in favor of an accused in a criminal case who
has been released on bail. 2
As to Rolando Dural,it clearly appears that he was not arrested while in
the act of shooting the two (2) CAPCOM soldiers aforementioned. Nor
was he arrested just after the commission of the said offense for his
arrest came a day after the said shooting incident. Seemingly, his arrest
without warrant is unjustified.
However, Rolando Dural was arrested for being a member of the New
Peoples Army (NPA), an outlawed subversive organization. Subversion
being a continuing offense,the arrest of Rolando Dural without warrant is
justified as it can be said that he was committing an offense when
arrested. The crimes of rebellion, subversion, conspiracy or proposal to
commit such crimes, and crimes or offenses committed in furtherance
thereof or in connection therewith constitute direct assaults against the
State and are in the nature of continuing crimes. As stated by the Court
in an earlier case:
"From the facts as above-narrated, the claim of the petitioners that
they were initially arrested illegally is, therefore, without basis in
law and in fact. The crimes of insurrection or rebellion, subversion,
conspiracy or proposal to commit such crimes, and other crimes
and offenses committed in the furtherance, on the occasion
thereof, or incident thereto, or in connection therewith
under Presidential Proclamation No. 2045, are all in the nature of
continuing offenses which set them apart from the common
offenses, aside from their essentially involving a massive
conspiracy of nationwide magnitude. Clearly then, the arrest of the
herein detainees was well within the bounds of the law and
existing jurisprudence in our jurisdiction.

2. The arrest of persons involved in the rebellion whether as its


fighting armed elements, or for committing non-violent acts but in
furtherance of the rebellion, is more an act of capturing them in
the course of an armed conflict, to quell the rebellion, than for the
purpose of immediately prosecuting them in court for a statutory
offense. The arrest, therefore, need not follow the usual procedure
in the prosecution of offenses which requires the determination by
a judge of the existence of probable cause before the issuance of a
judicial warrant of arrest and the granting of bail if the offense is
bailable. Obviously, the absence of a judicial warrant is no legal
impediment to arresting or capturing persons committing overt
acts of violence against government forces, or any other milder
acts but equally in pursuance of the rebellious movement. The
arrest or capture is thus impelled by the exigencies of the
situation that involves the very survival of society and its
government and duly constituted authorities. If killing and other
acts of violence against the rebels find justification in the
exigencies of armed hostilities which is of the essence of waging a
rebellion or insurrection, most assuredly so in case of invasion,
merely seizing their persons and detaining them while any of these
contingencies continue cannot be less justified. . . ." 3
The record, moreover, shows that the criminal case filed against Rolando
Dural and Bernardo Itucal, Jr. for "Double Murder, etc." was tried in the
court below and at the conclusion thereof, or on 17 August 1988, Rolando
Dural and Bernardo Itucal, Jr. were found guilty of the charge and
sentenced accordingly. Rolando Dural is now serving the sentence
imposed upon him by the trial court. Thus, the writ of habeas corpus is
no longer available to him. For, as held in the early case of U.S. vs.
Wilson: 4
"In this case, whatever may be said about the manner of his arrest,
the fact remains that the defendant was actually in court in the
custody of the law on March 29, when a complaint sufficient in
form and substance was read to him. To this he pleaded not guilty.
The trial followed, in which, and in the judgment of guilty
pronounced by the court, we find no error. Whether, if there were
irregularities in bringing him personally before the court, he could
have been released on a writ of habeas corpusor now has a civil
action for damages against the person who arrested him we need
not inquire. It is enough to say that such irregularities are not
sufficient to set aside a valid judgment rendered upon a sufficient
complaint and after a trial free from error."
II
In G.R. Nos. 84581-82 (Roque vs. De Villa), the arrest of Amelia
Roque and Wilfredo Buenaobra, without warrant, is also justified. When
apprehended at the house of Renato Constantino in Marikina Heights,
Marikina, Metro Manila, Wilfredo Buenaobra admitted that he was an NPA
courier and he had with him letters to Renato Constantino and other
members of the rebel group. Amelia Roque, upon the other hand, was a
member of the National United Front Commission, in charge of finance,
and admitted ownership of subversive documents found in the house of
her sister in Caloocan City. She was also in possession of ammunition
and a fragmentation grenade for which she had no permit or authority to
possess.LLpr

The record of these two (2) cases shows that on 27 June 1988, one
Rogelio Ramos y Ibanes, a member of the NPA, who had surrendered to
the military authorities, told military agents about the operations of the
Communist Party of the Philippines (CPP) and the New Peoples Army
(NPA) in Metro Manila. He identified some of his former comrades as "Ka
Mong", a staff member of the Communications and Transportation
Bureau; "Ka Nelia" a staff member in charge of finance; "Ka Miller", an
NPA courier from Sorsogon and Lopez, Quezon; "Ka Ted", and "Ka Totoy".
He also pointed to a certain house occupied by Renato Constantino
located in the Villaluz Compound, Molave St., Marikina Heights, Marikina,
Metro Manila, which is used as a safehouse of the National United Front
Commission (NUFC) of the CPP-NPA.
In view of these revelations, the Constantino house was placed under
military surveillance and on 12 August 1988, pursuant to a search
warrant issued by Judge Eutropio Migrino of the Regional Trial Court of
Pasig, a search of the house was conducted at about 5:00 o'clock in the
afternoon, by a combined team of the Criminal Investigation Service,
National Capital District (CIS-NCD) and the Constabulary Security Group
(CSG). In the course of the search, the following articles were found and
taken under proper receipt:
a) One (1 ) Colt M1 6A1 long rifle with defaced serial number;
b) One (1) Cal. .380 ACT/9mm Model PPK 8 SN: 260577 &
2605778;
c) Two (2) fragmentation hand grenades;
d) Fifty-six (56) live ammunition for Cal. 5.56mm;
e) Five (5) live ammunition for Cal. .380;
f) One (1) ICOM VHF FM Radio Transceiver SN: 14903
g) One (1) Regulated power supply 220V AC;
h) One (1) Antennae (adjustable);
i) One (1 ) Speaker with cord ALEXAR;
j) Voluminous Subversive documents.
When confronted, Renato Constantino could not produce any permit or
authority to possess the firearms, ammunition, radio and other
communications equipment. Hence, he was brought to the CIS
Headquarters for investigation. When questioned, he refused to give a
written statement, although he admitted that he was a staff member of
the executive committee of the NUFC and a ranking member of the
International Department of the Communist Party of the Philippines
(CPP).
At about 8:00 o'clock in the evening of the same day (12 August 1988),
Wilfredo Buenaobra arrived at the house of Renato Constantino in the
Villaluz Compound. When accosted, he readily admitted to the military
agents that he is a regular member of the CPP/NPA and that he went to
the place to deliver letters to "Ka Mong", referring to Renato
Constantino, and other members of the rebel group. On further
questioning, he also admitted that he is known as "Ka Miller" and that he
was from Barangay San Pedro, Lopez, Quezon. Among the items taken
from him were the following:
(1) Handwritten letter addressed to "Ka Bing & Co. from A & Co."
dated August 11, 1988;
(2) Handwritten letter addressed to "ROD from VIC (Schell datre)"
dated August 11, 1988;
(3) Handwritten letter addressed to "Suzie" from "Vic", dated
August 11, 1988.
Also found in Buenaobra's possession was a piece of paper containing a
written but jumbled telephone number of Florida M. Roque, sister
of Amelia Roque alias "Ka Nelia", at 69 Geronimo St., Caloocan City.
Acting on the lead provided as to the whereabouts of Amelia Roque, the
military agents went to the given address the next day (13 August 1988).
They arrived at the place at about 11:00 o'clock in the morning. After
identifying themselves as military agents and after seeking permission
to search the place, which was granted, the military agents conducted a
search in the presence of the occupants of the house and the barangay
captain of the place, one Jesus D. Olba.
The military agents found the place to be another safehouse of the
NUFC/CPP. They found ledgers, journals, vouchers, bank deposit books,
folders, computer diskettes, and subversive documents as well as live
ammunition for a .38 SPL Winchester, 11 rounds of live ammunition for a
cal. .45, 19 rounds of live ammunition for an M16 Rifle, and a
fragmentation grenade. As a result, Amelia Roque and the other
occupants of the house were brought to the PC-CIS Headquarters at
Camp Crame, Quezon City, for investigation. Amelia Roque admitted to
the investigators that the voluminous documents belonged to her and
that the other occupants of the house had no knowledge of them. As a
result, the said other occupants of the house were released from
custody.
On 15 August 1988, Amelia Roque was brought to the Caloocan City
Fiscal for inquest after which an information charging her with violation
of PD 1866 was filed with the Regional Trial Court of Caloocan City. The
case is docketed therein as Criminal Case No. C-1196. Another
information for violation of the Anti-Subversion Act was filed against
Amelia Roque before the Metropolitan Trial Court of Caloocan City, which
is docketed therein as Criminal Case No. C-150458.
An information for violation of the Anti-Subversion Act was filed
against Wilfredo Buenaobra before the Metropolitan Trial Court of
Marikina, Metro Manila. The case is docketed therein as Criminal Case
No. 23715. Bail was set at P4,000.00.
On 24 August 1988, a petition for habeas corpus was filed before this
Court on behalf of Amelia Roque and Wilfredo Buenaobra. At the hearing
of the case, however, Wilfredo Buenaobra manifested his desire to stay in
the PC-INP Stockade at Camp Crame, Quezon City. Accordingly, the
petition for habeas corpus filed on his behalf is now moot and academic.
Only the petition of Amelia Roque remains for resolution. LLjur

The contention of respondents that petitioners Roque and Buenaobra are


officers and or members of the National United Front Commission (NUFC)
of the CPP was not controverted or traversed by said petitioners. The
contention must be deemed admitted. 5 As officers and/ or members of
the NUFC-CPP, their arrest, without warrant, was justified for the same
reasons earlier stated vis-a-vis Rolando Dural. The arrest without warrant
of Roque was additionally justified as she was, at the time of
apprehension, in possession of ammunitions without license to possess
them.
III
In G.R. Nos. 84583-84 (Anonuevo vs. Ramos), the arrest of Domingo
Anonuevo and Ramon Casiple, without warrant, is also justified under the
rules. Both are admittedly members of the standing committee of the
NUFC and, when apprehended in the house of Renato Constantino, they
had a bag containing subversive materials, and both carried firearms and
ammunition for which they had no license to possess or carry.
The record of these two (2) cases shows that at about 7:30 o'clock in the
evening of 13 August 1988, Domingo T. Anonuevo and Ramon Casiple
arrived at the house of Renato Constantino at Marikina Heights,
Marikina, which was still under surveillance by military agents. The
military agents noticed bulging objects on their waist lines. When frisked,
the agents found them to be loaded guns. Anonuevo and Casiple were
asked to show their permit or license to possess or carry firearms and
ammunition, but they could not produce any. Hence, they were brought to
PC Headquarters for investigation. Found in their possession were the
following articles:

a) Voluminous subversive documents


b) One (1) Cal. 7.65 MOD 83 2C Pistol SN: 001412 with one (1)
magazine for Cal. 7.65 containing ten (10) live ammunition of same
caliber;
c) One (1) Cal. 7.65 Pietro Barreta SN; A18868 last digit tampered
with one (1) magazine containing five (5) live ammunition of same
caliber.
At the PC Stockade, Domingo Anonuevo was identified as "KaTed", and
Ramon Casiple as "Ka Totoy" of the CPP, by their comrades who had
previously surrendered to the military.
On 15 August 1988, the record of the investigation and other
documentary evidence were forwarded to the Provincial Fiscal at Pasig,
Metro Manila, who conducted an inquest, after which Domingo Anonuevo
and Ramon Casiple were charged with violation of Presidential Decree
No. 1866 before the Regional Trial Court of Pasig, Metro Manila. The
cases are docketed therein as Criminal Cases Nos. 74386 and 74387,
respectively. No bail was recommended.
On 24 August 1988, a petition for habeas corpus was filed with this Court
on behalf of Domingo Anonuevo and Ramon Casiple, alleging that the
said Anonuevo and Casiple were unlawfully arrested without a warrant
and that the informations filed against them are null and void for having
been filed without prior hearing and preliminary investigation. On 30
August 1988, the Court issued the writ of habeas corpus, and after the
respondents had filed a Return of the Writ, the parties were heard.
The petitioners' (Anonuevo and Casiple) claim that they were unlawfully
arrested because there was no previous warrant of arrest, is without
merit. The record shows that Domingo Anonuevo and Ramon Casiple
were carrying unlicensed firearms and ammunition in their person when
they were apprehended.
There is also no merit in the contention that the informations filed
against them are null and void for want of a preliminary investigation.
The filing of an information, without a preliminary investigation having
been first conducted, is sanctioned by the Rules. Sec. 7, Rule 112 of the
Rules of Court, as amended, reads:
"Sec. 7. When accused lawfully arrested without a warrant . —
When a person is lawfully arrested without a warrant for an
offense cognizable by the Regional Trial Court the complaint or
information may be filed by the offended party, peace officer or
fiscal without a preliminary investigation having been first
conducted, on the basis of the affidavit of the offended party or
arresting officer or person.
However, before the filing of such complaint or information, the
person arrested may ask for a preliminary investigation by a proper
officer in accordance with this Rule, but he must sign a waiver of
the provisions of Article 125 of the Revised Penal Code, as
amended, with the assistance of a lawyer and in case of non-
availability of a lawyer, a responsible person of his choice.
Notwithstanding such waiver, he may apply for bail as provided in
the corresponding rule and the investigation must be terminated
within fifteen (15) days from its inception.
If the case has been filed in court without a preliminary
investigation having been first conducted, the accused may within
five (5) days from the time he learns of the filing of the information,
ask for a preliminary investigation with the same right to adduce
evidence in his favor in the manner prescribed in this Rule."
The petitioners Domingo Anonuevo and Ramon Casiple, however, refused
to sign a waiver of the provisions of Article 125 of the Revised Penal
Code, as amended. In the informations filed against them, the prosecutor
made identical certifications, as follows:
"This is to certify that the accused has been charged in
accordance with Sec. 7, Rule 112 of the 1985 Rules on Criminal
Procedure, that no preliminary investigation was conducted
because the accused has not made and signed a waiver of the
provisions of Art. 125 of the Revised Penal Code, as amended; that
based on the evidence presented, there is reasonable ground to
believe that the crime has been committed, and that the accused
is probably guilty thereof."
Nor did petitioners ask for a preliminary investigation after the
informations had been filed against them in court. Petitioners cannot
now claim that they have been deprived of their constitutional right to
due process.
IV
In G.R. No. 83162 (Ocaya vs. Aguirre), the arrest without warrant,
of Vicky Ocaya is justified under the Rules, since she had with her an
unlicensed ammunition when she was arrested. The record of this case
shows that on 12 May 1988, agents of the PC Intelligence and
Investigation of the Rizal PC-INP Command, armed with a search warrant
issued by Judge Eutropio Migrino of the Regional Trial Court of Pasig,
Metro Manila, conducted a search of a house located at Block 19, Phase
II, Marikina Green Heights, Marikina, Metro Manila, believed to be
occupied by Benito Tiamson, head of the CPP-NPA. In the course of the
search, Vicky Ocaya armed in a car driven by Danny Rivera. Subversive
documents and several rounds of ammunition for a .45 cal. pistol were
found in the car of Vicky Ocaya. As a result, Vicky Ocaya and Danny
Rivera were brought to the PC Headquarters for investigation. When
Vicky Ocaya could not produce any permit or authorization to possess
the ammunition, an information charging her with violation of PD
1866 was filed with the Regional Trial Court of Pasig, Metro Manila. The
case is docketed therein as Criminal Case No. 73447. Danny Rivera, on
the other hand, was released from custody.
On 17 May 1988, a petition for habeas corpus was filed, with this Court
on behalf of Vicky Ocaya and Danny Rivera. It was alleged therein that
Vicky Ocaya was illegally arrested and detained, and denied the right to
a preliminary investigation.
It would appear, however, that Vicky Ocaya was arrested in flagranti
delicto so that her arrest without a warrant is justified. No preliminary
investigation was conducted because she was arrested without a
warrant and she refused to waive the provisions of Article 125 of the
Revised Penal Code, pursuant to Sec. 7, Rule 112 of the Rules of Court, as
amended.
V
The petitioners Vicky Ocaya, Domingo Anonuevo, Ramon Casiple, and
Amelia Roque claim that the firearms, ammunition and subversive
documents alleged to have been found in their possession when they
were arrested, did not belong to them, but were "planted" by the military
agents to justify their illegal arrest.
The petitioners, however, have not introduced any evidence to support
their aforesaid claim. On the other hand, no evil motive or ill-will on the
part of the arresting officers that would cause the said arresting officers
in these cases to accuse the petitioners falsely, has been shown.
Besides, the arresting officers in these cases do not appear to be
seekers of glory and bounty hunters for, as counsel for the petitioners
Anonuevo and Casiple say, "there is absolutely nothing in the evidence
submitted during the inquest that petitioners are on the 'AFP Order of
Battle with a reward of P15,000.00 on each on their heads.' " 6 On the
other hand, as pointed out by the Solicitor General, the arrest of the
petitioners is not a product of a witch hunt or a fishing expedition, but
the result of an in-depth surveillance of NPA safehouses pointed to by no
less than former comrades of the petitioners in the rebel movement. LexLib

The Solicitor General, in his Consolidated Memorandum, aptly observes:


". . . To reiterate, the focal point in the case of petitioners Roque,
Buenaobra, Anonuevo and Casiple, was the lawful search and
seizure conducted by the military at the residence of Renato
Constantino at Villaluz Compound, Molave St., Marikina Heights,
Marikina, Metro Manila. The raid at Constantino's residence, was
not a witch hunting or fishing expedition on the part of the military.
It was a result of an in-depth military surveillance coupled with the
leads provided by former members of the underground subversive
organizations. That raid produced positive results. To date, nobody
has disputed the fact that the residence of Constantino when
raided yielded communication equipment, firearms and
ammunitions, as well as subversive documents.
The military agents working on the information provided by
Constantino that other members of his group were coming to his
place, reasonably conducted a 'stake-out' operation whereby some
members of the raiding team were left behind the place. True
enough, barely two hours after the raid and Constantino's arrest,
petitioner Buenaobra arrived at Constantino's residence. He acted
suspiciously and when frisked and searched by the military
authorities, found in his person were letters. They are no ordinary
letters, as even a cursory reading would show. Not only that,
Buenaobra admitted that he is a NPA courier and was there to
deliver the letters to Constantino.
Subsequently, less than twenty four hours after the arrest of
Constantino and Buenaobra, petitioners Anonuevo and Casiple
arrived at Constantino's place. Would it be unreasonable for the
military agents to believe that petitioners Anonuevo and Casiple
are among those expected to visit Constantino's residence
considering that Constantino's information was true, in that
Buenaobra did come to that place? Was it unreasonable under the
circumstances, on the part of the military agents, not to frisk and
search anyone who should visit the residence of Constantino, such
as petitioners Anonuevo and Casiple? Must this Honorable Court
yield to Anonuevo and Casiple's flimsy and bare assertion that they
went to visit Constantino, who was to leave for Saudi Arabia on the
day they were arrested thereat?.
As to petitioner Roque, was it unreasonable for the military
authorities to effect her arrest without warrant considering that it
was Buenaobra who provided the leads on her identity? It cannot
be denied that Buenaobra had connection with Roque. Because the
former has the phone number of the latter. Why the necessity of
jumbling Roque's telephone number as written on a piece of paper
taken from Buenaobra's possession? Petitioners Roque and
Buenaobra have not offered any plausible reason so far.

In all the above incidents, respondents maintain that they acted


reasonably, under the time, place and circumstances of the events
in question, especially considering that at the time of petitioners'
arrest, incriminatory evidence, i.e, firearms, ammunitions and/or
subversive documents were found in their possession.
Petitioners, when arrested, were neither taking their snacks nor
innocently visiting a camp, but were arrested in such time, place
and circumstances, from which one can reasonably conclude that
they were up to a sinister plot, involving utmost secrecy and
comprehensive conspiracy.".
VI
In G.R. No. 85727 (Espiritu vs. Lim), the release on habeas corpus of the
petitioner Deogracias Espiritu, who is detained by virtue of an
Information for Violation of Article 142 of the Revised Penal Code
(Inciting to Sedition) filed with the Regional Trial Court of Manila, is
similarly not warranted.
The record of the case shows that the said petitioner is the General
Secretary of the Pinagkaisahang Samahan ng Tsuper at Operators
Nationwide (PISTON), an association of drivers and operators of public
service vehicles in the Philippines, organized for their mutual aid and
protection.cdll

Petitioner claims that at about 5:00 o'clock in the morning of 23


November 1988, while he was sleeping in his home located at 363
Valencia St., Sta. Mesa, Manila, he was awakened by his sister Maria Paz
Lalic who told him that a group of persons wanted to hire his jeepney.
When he went down to talk to them, he was immediately put under
arrest. When he asked for the warrant of arrest arrest, the men, headed
by Col. Ricardo Reyes, bodily lifted him and placed him in their owner-
type jeepney. He demanded that his sister, Maria Paz Lalic, be allowed to
accompany him, but the men did not accede to his request and hurriedly
sped away.
He was brought to Police Station No. 8 of the Western Police District at
Blumentritt, Manila where he was interrogated and detained. Then, at
about 9:00 o'clock of the same morning, he was brought before the
respondent Lim and, there and then, the said respondent ordered his
arrest and detention. He was thereafter brought to the General
Assignment Section, Investigation Division of the Western Police District
under Police Capt. Cresenciano A. Cabasal where he was detained,
restrained and deprived of his liberty. 7
The respondents claim however, that the detention of the petitioner is
justified in view of the Information filed against him before the Regional
Trial Court of Manila, docketed therein as Criminal Case No. 88-683-85,
charging him with violation of Art. 142 of the Revised Penal Code
(Inciting to Sedition).
The respondents also claim that the petitioner was lawfully arrested
without a judicial warrant of arrest since petitioner when arrested had in
fact just committed an offense in that in the afternoon of 22 November
1988, during a press conference at the National Press Club.
"Deogracias Espiritu through tri-media was heard urging all drivers
and operators to go on nationwide strike on November 23, 1988, to
force the government to give in to their demands to lower the
prices of spare parts, commodities, water and the immediate
release from detention of the president of the PISTON (Pinag-isang
Samahan ng Tsuper Operators Nationwide). Further, we heard
Deogracias Espiritu taking the place of PISTON president Medardo
Roda and also announced the formation of the Alliance Drivers
Association to go on nationwide strike on November 23, 1988." 8
Policemen waited for petitioner outside the National Press Club in order
to investigate him, but he gave the lawmen the slip. 9 He was next seen
at about 5:00 o'clock that afternoon at a gathering of drivers and
sympathizers at the corner of Magsaysay Blvd. and Valencia Street, Sta.
Mesa, Manila where he was heard to say:
"Bukas tuloy ang welga natin, sumagot na ang Cebu at Bicol na
kasali sila, at hindi tayo titigil hanggang hindi binibigay ng
gobyerno ni Cory ang gusto nating pagbaba ng halaga ng spare
parts, bilihin at ang pagpapalaya sa ating pinuno na si Ka
Roda hanggang sa magkagulo na." 10 (emphasis supplied).

The police finally caught up with the petitioner on 23 November 1988. He


was invited for questioning and brought to police headquarters after
which an Information for violation of Art. 142 of the Revised Penal Code
was filed against him before the Regional Trial Court of Manila. 11
Since the arrest of the petitioner without a warrant was in accordance
with the provisions of Rule 113, Sec. 5(b) of the Rules of Court and that
the petitioner is detained by virtue of a valid information filed with the
competent court, he may not be released on habeas corpus. He may,
however be released upon posting bail as recommended. However, we
find the amount of the recommended bail (P60,000.00) excessive and we
reduce it to P10,000.00 only.
VII
In G.R. No. 86332 (Nazareno vs. Station Commander), we also find no
merit in the submission of Narciso Nazareno that he was illegally
arrested and is unlawfully detained. The record of this case shows that
at about 8:30 o'clock in the morning of 14 December 1988, one Romulo
Bunye II was killed by a group of men near the corner of T. Molina and
Mendiola Streets in Alabang, Muntinglupa, Metro Manila. One of the
suspects in the killing was Ramil Regala who was arrested by the police
on 28 December 1988. Upon questioning, Regala pointed to Narciso
Nazareno as one of his companions in the killing of the said Romulo
Bunye II. In view thereof, the police officers, without warrant, picked up
Narciso Nazareno and brought him to the police headquarters for
questioning. Obviously, the evidence of petitioner's guilt is strong
because on 3 January 1989, an information charging Narciso Nazareno,
Ramil Regala, and two (2) others, with the killing of Romulo Bunye II was
filed with the Regional Trial Court of Makati, Metro Manila. The case is
docketed therein as Criminal Case No. 731. cdphil

On 7 January 1989, Narciso Nazareno filed a motion to post bail, but the
motion was denied by the trial court in an order dated 10 January 1989,
even as the motion to post bail, earlier filed by his co-accused, Manuel
Laureaga, was granted by the same trial court.
On 13 January 1989, a petition for habeas corpus was filed with this
Court on behalf of Narciso Nazareno and on 13 January 1989, the Court
issued the writ of habeas corpus, returnable to the Presiding Judge of
the Regional Trial Court of Biñan, Laguna, Branch 24, ordering said court
to hear the case on 30 January 1989 and thereafter resolve the petition.
At the conclusion of the hearing, or on 1 February 1989, the Presiding
Judge of the Regional Trial Court of Biñan, Laguna issued a resolution
denying the petition for habeas corpus, it appearing that the said Narciso
Nazareno is in the custody of the respondents by reason of an
information filed against him with the Regional Trial Court of Makati,
Metro Manila which had taken cognizance of said case and had, in fact,
denied the motion for bail filed by said Narciso Nazareno (presumably
because of the strength of the evidence against him).
The findings of the Presiding Judge of the Regional Trial Court of Biñan,
Laguna are based upon the facts and the law. Consequently, we will not
disturb the same. Evidently, the arrest of Nazareno was effected by the
police without warrant pursuant to Sec. 5 (b), Rule 113, Rules of Court
after he was positively implicated by his co-accused Ramil Regala in the
killing of Romulo Bunye II; and after investigation by the police
authorities. As held in People vs. Ancheta: 12
"The obligation of an agent of authority to make an arrest by
reason of a crime, does not presuppose as a necessary requisite
for the fulfillment thereof, the indubitable existence of a crime. For
the detention to be perfectly legal, it is sufficient that the agent or
person in authority making the arrest has reasonably sufficient
grounds to believe the existence of an act having the
characteristics of a crime and that the same grounds exist to
believe that the person sought to be detained participated
therein."
VIII
It is to be noted that, in all the petitions here considered, criminal
charges have been filed in the proper courts against the petitioners. The
rule is, that if a person alleged to be restrained of his liberty is in the
custody of an officer under process issued by a court or judge, and that
the court or judge had jurisdiction to issue the process or make the
order, or if such person is charged before any court, the writ of habeas
corpus will not be allowed. Section 4, Rule 102, Rules of Court, as
amended is quite explicit in providing that:
"Sec. 4. When writ is not allowed or discharge authorized . - If it
appears that the person alleged to be restrained of his liberty is in
the custody of an officer under process issued by a court or judge
or by virtue of a judgment or order of a court of record, and that
the court or judge had jurisdiction to issue the process, render the
judgment, or make the order, the writ shall not be allowed; or if the
jurisdiction appears after the writ is allowed, the person shall not
be discharged by reason of any informality or defect in the
process, judgment, or order. Nor shall anything in this rule be held
to authorize the discharge of a person charged with or convicted
of an offense in the Philippines or of a person suffering
imprisonment under lawful judgment." (emphasis supplied)
At this point, we refer to petitioners' plea for the Court to re-examine and,
thereafter, abandon its pronouncement in Ilagan vs. Enrile, 13 that a writ
of habeas corpus is no longer available after an information is filed
against the person detained and a warrant of arrest or an order of
commitment is issued by the court where said information has been
filed. 14 The petitioners claim that the said ruling, which was handed
down during the past dictatorial regime to enforce and strengthen said
regime, has no place under the present democratic dispensation and
collides with the basic, fundamental, and constitutional rights of the
people. Petitioners point out that the said doctrine makes possible the
arrest and detention of innocent persons despite lack of evidence
against them, and, most often, it is only after a petition for habeas
corpus is filed before the court that the military authorities file the
criminal information in the courts of law to be able to hide behind the
protective mantle of the said doctrine. This, petitioners assert, stands as
an obstacle to the freedom and liberty of the people and permits lawless
and arbitrary State action.

We find, however, no compelling reason to abandon the said doctrine. It is


based upon express provision of the Rules of Court and the exigencies
served by the law. The fears expressed by the petitioners are not really
unremediable. As the Court sees it, re-examination or reappraisal, with a
view to its abandonment, of the Ilagan case doctrine is not the answer.
The answer and the better practice would be, not to limit the function of
habeas corpus to a mere inquiry as to whether or not the court which
issued the process, judgment or order of commitment or before whom
the detained person is charged, had jurisdiction or not to issue the
process, judgment or order or to take cognizance of the case, but rather,
as the Court itself states in Morales, Jr. vs. Enrile, 15 "in all petitions for
habeas corpus the court must inquire into every phase and aspect of
petitioner's detention — from the moment petitioner was taken into
custody up to the moment the court passes upon the merits of the
petition;" and "only after such a scrutiny can the court satisfy itself that
the due process clause of our Constitution has in fact been
satisfied." This is exactly what the Court has done in the petitions at bar.
This is what should henceforth be done in all future cases of habeas
corpus. In short, all cases involving deprivation of individual liberty
should be promptly brought to the courts for their immediate scrutiny
and disposition.LLpr

WHEREFORE, the petitions are hereby DISMISSED, except that in G.R. No.
85727 (Espiritu vs. Lim), the bail bond for petitioner's provisional liberty
is hereby ordered reduced from P60,000.00 to P10,000.00. No costs.
SO ORDERED.
(Umil v. Ramos, G.R. No. 81567, 84581-82, 84583-84, 83162, 85727,
|||

86332, [July 9, 1990], 265 PHIL 325-365)

[G.R. No. 93239. March 18, 1991.]

PEOPLE OF THE PHILIPPINES, plaintiff-appellee, vs. EDISON


SUCRO, accused-appellant.

The Solicitor General for plaintiff-appellee.


Fidencio S. Raz accused-appellant.

SYLLABUS

1. REMEDIAL LAW; CRIMINAL PROCEDURE; ARREST WITHOUT A


WARRANT; WHEN LAWFUL. — Section 5, Rule 113 of the Rules on
Criminal Procedure provides for the instances where arrest without
warrant is considered lawful. The rules states: " Arrest without warrant,
when lawful. — A peace officer or private person may, without warrant,
arrest a person: (a) When in his presence, the person to be arrested has
committed, is actually committing, or is attempting to commit an
offense; (b)When an offense has in fact just been committed, and he has
personal knowledge of facts indicating that the person to be arrested
has committed it."
2. ID.; ID.; ID.; AN OFFENSE COMMITTED IN THE PRESENCE OR WITHIN
THE VIEW OF AN OFFICER, CONSTRUED. — An offense is committed in
the presence or within the view of an officer, within the meaning of the
rule authorizing an arrest without a warrant, when the officer sees the
offense, although at a distance, or hears the disturbances created
thereby and proceeds at once to the scene thereof (U.S. v. Fortaleza, 12
Phil. 472 [1909]; and U.S. v. Samonte, 16 Phil. 516 [1910]).
3. ID.; ID.; ID.; PERSONAL KNOWLEDGE OF ACTUAL COMMISSION OF
CRIME. — The court earlier indicated in the case of People v. Bati (G.R.
No. 87429, August 27, 1990) that police officers have personal knowledge
of the actual commission of the crime when it had earlier conducted
surveillance activities of the accused.
4. ID.; ID.; SEARCHES AND SEIZURES, AS A GENERAL RULE MUST BE
SUPPORTED BY A VALID WARRANT; EXCEPTION. — That searches and
seizures must be supported by a valid warrant is not an absolute rule
(Manipon, Jr. v. Sandiganbayan, 143 SCRA 267 [1986]). Among the
exceptions granted by law is a search incidental to a lawful arrest under
Sec. 12, Rule 126 of the Rules on Criminal Procedure, which provides that
a person lawfully arrested may be searched for dangerous weapons or
anything which may be used as proof of the commission of an offense,
without a search warrant. (People v. Castiller, G.R. No. 87783, August 6,
1990)
5. ID.; EVIDENCE; FINDINGS OF FACT OF THE TRIAL COURT, GENERALLY
ENTITLED TO GREAT WEIGHT. — Time and again it has been held that the
findings of the trial court are entitled to great weight and should not be
disturbed on appeal unless it is shown that the trial court had overlooked
certain facts of weight and importance, it being acknowledged that the
court below, having seen and heard the witnesses during the trial, is in a
better position to evaluate their testimonies (People v. Umali, et al., G.R.
No. 84450, February 4, 1991 citing People v. Alvarez, 163 SCRA 745
[1988]; People v. Dorado, 30 SCRA 53 [1969]; and People v. Espejo, 36
SCRA 400 [1970]).
6. ID.; ID.; BURDEN OF PROOF AND PRESUMPTIONS; PRESUMPTION THAT
POLICE OFFICERS PERFORM THEIR DUTIES REGULARLY; APPLIED IN
CASE AT BAR. — There is nothing in the record to suggest that the police
officers were compelled by any motive than to accomplish their mission
to capture a drug pusher in the execution of the crime, the presumption
being that police officers perform their duties regularly in the absence of
any evidence to the contrary (Rule 131, Sec. 3(m), Revised Rules on
Evidence; People v. Castiller, supra citing People v. Natipravat, 145 SCRA
483 [1986]).
7. ID.; ID.; CREDIBILITY; ALIBI; UNAVAILING IN THE FACE OF POSITIVE
IDENTIFICATION. — In contrast to the evidence presented by the
prosecution, accused-appellant's defense is alibi which is unavailing
considering that he was positively identified by Macabante to be the
person from whom he bought marijuana.
8. ID.; ID.; WEIGHT AND SUFFICIENCY; MERE DENIALS CANNOT PREVAIL
OVER POSITIVE IDENTIFICATION. — It is well-settled that mere denials
cannot prevail against the positive identification of the appellant as the
seller of the prohibited substances. (People v. Khan, 161 SCRA 406
[1988]; and People v. Paco, 170 SCRA 681 [1989]).

DECISION

GUTIERREZ, JR., J : p

Edison Sucro was charged with and convicted of violation of Section 4,


Article II of the Dangerous Drugs Act, under an Information which reads:
"That on or about the 21st day of March, 1989, in the evening, in
the Poblacion, Municipality of Kalibo, Province of Aklan, Republic
of the Philippines, and within the jurisdiction of this Honorable
Court, the above-named accused, acting as a pusher or broker in
the business of selling, administering, delivery, giving away to
another and - or distributing prohibited drugs, did then and there
wilfully, unlawfully and feloniously and without authority of law
have in his possession and control nineteen (19) pieces of
marijuana cigarette sticks and four (4) tea bags of dried marijuana
leaves which were confiscated from him by the police authorities
of Kalibo, Aklan, shortly after having sold one tea bag of dried
marijuana leaves to a customer." (Rollo, p. 9)

Upon arraignment, the accused-appellant, assisted by counsel, entered a


plea of "not guilty" to the offense charged. Trial ensued and a judgment
of conviction was rendered, the pertinent portion of which reads:
"WHEREFORE, judgment is rendered finding the accused Edison
Sucro guilty of the sale of prohibited drug under Section 4, Article
II of the Dangerous Drug Act, as amended, and sentencing him to
suffer the penalty of life imprisonment, and pay a fine of P20,000,
and costs. He shall be entitled to full credit in the service of his
sentence with the period for which he has undergone preventive
imprisonment to the date of promulgation of this judgment. All the
items of marijuana confiscated in this case are declared forfeited
in favor of the State." (Rollo, p. 41)

From the foregoing judgment of conviction, accused-appellant interposes


this appeal, assigning the following as errors allegedly committed by the
court a quo, to wit:
I
THE LOWER COURT ERRED IN ADMITTING AS EVIDENCE FOR THE
PROSECUTION EXHIBITS "E"-"E-4", TEA BAGS OF ALLEGED
MARIJUANA, TO BE THE CORPUS DELICTI; FURTHERMORE, THAT
THE SAME WERE TAKEN WITHOUT THE REQUIRED WARRANT OF
SEARCH AND ARREST SINCE THE ACCUSED WAS NOT IN THE ACT
OF COMMITTING ANY OFFENSE AT THE TIME OF HIS ARREST.
II
THE LOWER COURT ERRED IN FINDING THE ACCUSED EDISON
SUCRO GUILTY OF THE SALE OF PROHIBITED DRUGS UNDER
SECTION 4, ARTICLE II, OF THE DANGEROUS DRUGS ACT AND
SENTENCING HIM TO SUFFER A PENALTY OF LIFE IMPRISONMENT
AND TO PAY A FINE OF P20,000.00. (Appellant's Brief, p. 1)

The antecedent facts of the case as summarized by the Solicitor General


are as follows:
"On March 21, 1989, Pat. Roy Fulgencio, a member of the INP,
Kalibo, Aklan, was instructed by P/Lt. Vicente Seraspi, Jr. (Station
Commander of the INP Kalibo, Aklan) to monitor the activities of
appellant Edison Sucro, because of information gathered by
Seraspi that Sucro was selling marijuana. (p. 6, TSN, May 2, 1989).
As planned, at about 5:00 P.M. on said date, Pat. Fulgencio
positioned himself under the house of a certain Arlie Regalado at
C. Quimpo Street. Adjacent to the house of Regalado, about 2
meters away, was a chapel. Thereafter, Pat. Fulgencio saw
appellant enter the chapel, taking something which turned out
later to be marijuana from the compartment of a cart found inside
the chapel, and then return to the street where he handed the
same to a buyer, Aldie Borromeo. After a while appellant went back
to the chapel and again came out with marijuana which he gave to
a group of persons. (pp. 6-8, 15-18, ibid). It was at this instance that
Pat. Fulgencio radioed P/Lt. Seraspi and reported the activity going
on P/Lt. Seraspi instructed Pat. Fulgencio to continue monitoring
developments. At about 6:30 P.M., Pat. Fulgencio again called up
Seraspi to report that a third buyer later identified as Ronnie
Macabante, was transacting with appellant. (pp. 18-19, ibid)
At that point, the team of P/Lt Seraspi proceeded to the area and
while the police officers were at the Youth Hostel at Maagma St.,
Pat. Fulgencio told P/Lt. Seraspi to intercept Macabante and
appellant. P/ Lt. Seraspi and his team caught up with Macabante at
the crossing of Mabini and Maagma Sts. in front of the Aklan
Medical Center. Upon seeing the police, Macabante threw
something to the ground which turned out to be a tea bag of
marijuana. (pp 6-8, TSN, June 19, 1989) When confronted,
Macabante readily admitted that he bought the same from
appellant (Edison Sucro) in front of the chapel. (p. 6, TSN, May 24,
1989) The police team was able to overtake and arrest appellant at
the corner of C. Quimpo and Veterans Sts. The police recovered 19
sticks and 4 teabags of marijuana from the cart inside the chapel
and another teabag from Macabante. The teabags of marijuana
were sent to the PC-INP Crime Laboratory Service, at Camp
Delgado, Iloilo City for analysis. The specimens (Exhibits "G" to "G-
18", Exhibits "E" to "E-4") were all found positive of marijuana. (pp.
4-7, TSN, Sept. 4, 1989)" (Appellee's Brief, pp. 3-6)

As can be seen from the facts, the issue hinges mainly on whether or not
the arrest without warrant of the accused is lawful and consequently,
whether or not the evidence resulting from such arrest is admissible.
We rule in the affirmative.
The accused-appellant contends that his arrest was illegal, being a
violation of his rights granted under Section 2, Artilce III of the 1987
Constitution. He stresses that there was sufficient time for the police
officers to apply for a search and arrest warrants considering that
Fulgencio informed his Station Commander of the activities of the
accused two days before March 21, 1989, the date of his arrest.
This contention is without merit.
Section 5, Rule 113 of the Rules on Criminal Procedure provides for the
instances where arrest without warrant is considered lawful. The rule
states:
"Arrest without warrant, when lawful . — A peace officer or private
person may, without warrant, arrest a person:

(a) When in his presence, the person to be arrested has committed ,


is actually committing, or is attempting to commit an offense;
(b) When an offense has in fact just been committed, and he has
personal knowledge of facts indicating that the person to be
arrested has committed it;" (Emphasis supplied).

An offense is committed in the presence or within the view of an officer,


within the meaning of the rule authorizing an arrest without a warrant,
when the officer sees the offense, although at a distance, or hears the
disturbances created thereby and proceeds at once to the scene thereof
(U.S. v. Fortaleza, 12 Phil. 472 [1909]; and U.S. v. Samonte, 16 Phil. 516
[1910]).
The records show that Fulgencio went to Arlie Regalado's house at C.
Quimpo Street to monitor the activities of the accused who was earlier
reported to be selling marijuana at a chapel two (2) meters away from
Regalado's house.
Fulgencio, within a distance of two meters saw Sucro conduct his
nefarious activity. He saw Sucro talk to some persons, go inside the
chapel, and return to them and exchange some things. These, Sucro did
three times during the time that he was being monitored. Fulgencio
would then relay the on-going transaction to P/Lt. Seraspi.
Anent the second requirement, the fact that Macabante, when
intercepted by the police, was caught throwing the marijuana stick and
when confronted, readily admitted that he bought the same from
accused-appellant clearly indicates that Sucro had just sold the
marijuana stick to Macabante, and therefore, had just committed an
illegal act of which the police officers had personal knowledge, being
members of the team which monitored Sucro's nefarious activity. cdphil

The court earlier indicated in the case of People v. Bati (G.R. No. 87429,
August 27, 1990) that police officers have personal knowledge of the
actual commission of the crime when it had earlier conducted
surveillance activities of the accused. Thus, it stated:
"When Luciano and Caraan reached the place where the alleged
transaction would take place and while positioned at a street
corner, they saw appellant Regalado Bati and Warner Marquez by
the side of the street about forty to fifty meters away from them
(the public officers). They saw Marquez giving something to Bati,
who, thereafter handed a wrapped object to Marquez who then
inserted the object inside the front of his pants infront of his
abdomen while Bati, on his part, placed the thing given to him
inside his pocket. (p. 2)
xxx xxx xxx
. . . Both Patrolman Luciano and Caraan actually witnessed the
same and their testimonies were based on their actual and
personal knowledge of the events that took place leading to
appellant's arrest. They may not have been within hearing
distance, specially since conversation would expectedly be carried
on hushed tones, but they were certainly near enough to observe
the movements of the appellant and the buyer. Moreover, these
prosecution witnesses are all law enforcers and are, therefore,
presumed to have regularly performed their duties in the absence
of proof to the contrary (People v. Bati, supra citing People v.
Agapito, G.R. No. 73786, October 12, 1987)

The accused questions the failure of the police officers to secure a


warrant considering that Fulgencio himself knew of Sucro's activities
even prior to the former s joining the police force. Fulgencio reported
Sucro's activities only three days before the incident.
As the records reveal, Fulgencio and Sucro had known each other since
their childhood years and that after Fulgencio joined the police force, he
told the accused-appellant not to sell drugs in their locality. Hence, it is
possible that because of this friendship, Fulgencio hesitated to report his
childhood friend and merely advised him not to engage in such activity.
However, because of reliable information given by some informants that
selling was going on everyday, he was constrained to report the matter
to the Station Commander.
On the other hand, the failure of the police officers to secure a warrant
stems from the fact that their knowledge acquired from the surveillance
was insufficient to fulfill the requirements for the issuance of a search
warrant. What is paramount is that probable cause existed. Thus, it has
been held in the case of People v. Lo Ho Wing, et al. (G.R. No. 88017,
January 21, 1991):
"In the instant case, it was firmly established from the factual
findings of the trial court that the authorities had reasonable
ground to believe that appellant would attempt to bring in
contraband and transport it within the country. The belief was
based on intelligence reports gathered from surveillance activities
on the suspected syndicate, of which appellant was touted to be a
member. Aside from this, they were also certain as to the expected
date and time of arrival of the accused from China. But such
knowledge was clearly insufficient to enable them to fulfill the
requirements for the issuance of a search warrant. Still and all, the
important thing is that there was probable cause to conduct the
warrantless search, which must still be present in such a case."

As the Solicitor General has pointed out:


"There are several instances when a warrantless search and
seizure can be effected without necessarily being preceded by an
arrest provided the same is effected on the basis of probable
cause (e.g. stop and search without warrant at checkpoints).
Between warrantless searches and seizures at checkpoints and in
the case at bar the latter is more reasonable considering that
unlike in the former, it was effected on the basis of probable cause.
Under the circumstances (monitoring of transactions) there
existed probable cause for the arresting officers, to arrest
appellant who was in fact selling marijuana and to seize the
contraband."
That searches and seizures must be supported by a valid warrant is not
an absolute rule (Manipon, Jr. v. Sandiganbayan, 143 SCRA 267 [1986]).
Among the exceptions granted by law is a search incidental to a lawful
arrest under Sec. 12, Rule 126 of the Rules on Criminal Procedure, which
provides that a person lawfully arrested may be searched for dangerous
weapons or anything which may be used as proof of the commission of
an offense, without a search warrant. ( People v. Castiller, G.R. No. 87783,
August 6, 1990)
The accused-appellant claims that the arrest having been done without
warrant, it follows that the evidence obtained therefrom is inadmissible.
As earlier discussed, there is nothing unlawful about the arrest
considering its compliance with the requirements of a warrantless
arrest. Ergo, the fruits obtained from such lawful arrest are admissible in
evidence. LLpr

Edison Sucro assails the trial court's reliance on the statement of


Macabante whose reason for testifying could be merely to escape
prosecution.
We quote the trial court's finding as to the testimony of Macabante:
"The non-filing of a complaint against him for possession of
marijuana may have been the reason of (sic) his willingness to
testify in court against the accused. But this does not necessarily
taint the evidence that proceeds from his lips. As explained by Lt.
Seraspi, the best sources of information against drug pushers are
usually their customers, especially if as in this case, there is no
other direct evidence of the selling except the testimony of the
buyer. We accept this observation as a realistic appraisal of a
situation in which drug users are, and should be employed by law
enforcement authorities to bolster the drive against pushers who
are the real felons in our society. We have observed the demeanor
of the witness in court, and found him to be straightforward,
unhesitating, and spontaneous in his declarations, so that we are
satisfied as to his intention and disposition to tell the truth" (Rollo,
p. 40)

Time and again it has been held that the findings of the trial court are
entitled to great weight and should not be disturbed on appeal unless it
is shown that the trial court had overlooked certain facts of weight and
importance, it being acknowledged that the court below, having seen and
heard the witnesses during the trial, is in a better position to evaluate
their testimonies (People v. Umali, et al., G.R. No. 84450, February 4, 1991
citing People v. Alvarez, 163 SCRA 745 [1988]; People v. Dorado, 30 SCRA
53 [1969]; and People v. Espejo, 36 SCRA 400 [1970])
Furthermore, the testimony of Macabante was corroborated on material
points by public officers Fulgencio and Seraspi.
There is nothing in the record to suggest that the police officers were
compelled by any motive than to accomplish their mission to capture a
drug pusher in the execution of the crime, the presumption being that
police officers perform their duties regularly in the absence of any
evidence to the contrary (Rule 131, Sec. 3(m), Revised Rules on
Evidence; People v. Castiller, supra citing People v. Natipravat, 145 SCRA
483 [1986])
The prosecution evidence was further bolstered by the findings of the
Forensic Chemist that the items seized were all positive for marijuana.
In contrast to the evidence presented by the prosecution, accused-
appellant's defense is alibi which is unavailing considering that he was
positively identified by Macabante to be the person from whom he
bought marijuana.
Sucro alleges that he could not have committed the crime since he was
with his uncle and cousin distributing handbills for his Auntie's
candidacy. The fact, however, remains that it does not preclude the
possibility that he was present in the vicinity as established by his
admission that he moved a lot and even had the occasion to meet
Macabante on the street.
It is well-settled that mere denials cannot prevail against the positive
identification of the appellant as the seller of the prohibited substances.
(People v. Khan, 161 SCRA 406 [1988]; and People v. Paco, 170 SCRA 681
[1989])
Premises considered, this Court is convinced that appellant Edison
Sucro had indeed committed the offense charged. The trial court's
decision must be upheld.
WHEREFORE, the decision appealed from is hereby AFFIRMED. SO
ORDERED.
||| (People v. Sucro, G.R. No. 93239, [March 18, 1991], 272-A PHIL 362-371)

[G.R. No. 95902. February 4, 1992.]

PEOPLE OF THE PHILIPPINES, plaintiff-appellee, vs. DON


RODRIGUEZA, accused-appellant.

The Solicitor General for plaintiff-appellee.


Public Attorney's Office for accused-appellant.
SYLLABUS

1. CRIMINAL LAW; DANGEROUS DRUGS ACT OF 1972 (RA No. 6425);


ENTRAPMENT; BUY-BUST OPERATION; REQUISITE THEREOF; NOT
COMPLIED WITH IN CASE AT BAR. — A buy-bust operation is a form of
entrapment employed by peace officers to trap and catch a malefactor
in flagrante delicto. Applied to the case at bar, the termin flagrante
delicto requires that the suspected drug dealer must be caught
redhanded in the act of selling marijuana or any prohibited drug to a
person acting or posing as a buyer. In the instant case, however, the
procedure adopted by the NARCOM agents failed to meet this
qualification. Based on the very evidence of the prosecution, after the
alleged consummation of the sale of dried marijuana leaves, CIC Taduran
immediately released appellant Rodrigueza instead of arresting and
taking him into his custody. This act of CIC Taduran,
assuming arguendo that the supposed sale of marijuana did take place,
is decidedly contrary to the natural course of things and inconsistent
with the aforestated purpose of a buy-bust operation. It is rather absurd
on his part to let appellant escape without having been subjected to the
sanctions imposed by law. It is, in fact, a dereliction of duty by an agent
of the law.
2. ID.; ID.; CONFISCATED MARIJUANA LEAVES AND OTHER PROHIBITED
DRUG PARAPHERNALIA CONSTITUTES THE CORPUS DELICTI OF THE
CRIME; PROOF OF THEIR EXISTENCE NECESSARY. — In People
vs. Rubio (142 SCRA 329 [1986]), this Court had the occasion to rule that
the plastic bag and the dried marijuana leaves contained therein
constitutes the corpus delicti of the crime. As such, the existence
thereof must be proved with certainty and conclusiveness. Failure to do
so would be fatal to the cause of the prosecution.
3. CONSTITUTIONAL LAW; BILL OF RIGHTS; RIGHTS OF A PERSON
UNDER CUSTODIAL INVESTIGATION; WAIVER THEREOF MUST BE MADE
WITH ASSISTANCE AND IN THE PRESENCE OF COUNSEL. — The
admissibility of the sworn statement allegedly executed by appellant was
squarely placed in issue and, as correctly pointed out by the defense,
said sworn statement is inadmissible in evidence against appellant. We
have once again to reiterate and emphasize that Article III of the 1987
Constitutionprovides: "Sec. 12 (1). Any person under investigation for the
commission of an offense shall have the right to be informed of his right
to remain silent and to have a competent and independent counsel
preferably of his own choice. If the person cannot afford the services of
counsel, he must be provided with one. These rights cannot be waived
except in writing and in the presence of counsel. . . . (3) Any confession
or admission obtained in violation of this or section 17 hereof shall be
inadmissible in evidence against him." An examination of said sworn
statement shows that appellant was informed of his constitutional right
to remain silent and to be assisted by counsel during custodial
examination. He was also asked if he was waiving his right to be assisted
by counsel and he answered in the affirmative. However, while the rights
of a person under custodial investigation may be waived, such waiver
must be made not only voluntarily, knowingly and intelligently but also in
the presence and with the assistance of counsel (People vs. Olaes, 188
SCRA 91 [1990]; People vs. Hernandez, et al., 162 SCRA 422 [1988]). In
the present case, the waiver made by appellant being without the
assistance of counsel, this omission alone is sufficient to invalidate said
sworn statement (People vs. Nolasco, 163 SCRA 623 [1988]).
4. ID.; ID.; SEARCH AND SEIZURE; SEARCH WARRANT REQUIRED;
EXCEPTIONS; CASE AT BAR. — As provided in the present Constitution, a
search, to be valid, must generally be authorized by a search warrant
duly issued by the proper government authority (Section 2, Article
III, 1987 Constitution). True, in some instances, this Court has allowed
government authorities to conduct searches and seizures even without a
search warrant. Thus, when the owner of the premises waives his right
against such incursion; when the search is incidental to a lawful arrest;
when it is made on vessels and aircraft for violation of customs laws;
when it is made on automobiles for the purpose of preventing violations
of smuggling or immigration laws; when it involves prohibited articles in
plain view; or in cases of inspection of buildings and other premises for
the enforcement of fire, sanitary and building regulations, a search may
be validly made even without a search warrant. In the case at bar,
however, the raid conducted by the NARCOM agents in the house of
Jovencio Rodrigueza was not authorized by any search warrant. It does
not appear, either, that the situation falls under any of the
aforementioned cases. Hence, appellant's right against unreasonable
search and seizure was clearly violated. The NARCOM agents could not
have justified their act by invoking the urgency and necessity of the
situation because the testimonies of the prosecution witnesses reveal
that the place had already been put under surveillance for quite some
time. Had it been their intention to conduct the raid, then they should,
because they easily could, have first secured a search warrant during
that time.
5. REMEDIAL LAW; EVIDENCE; TESTIMONY OF WITNESSES; CREDIBILITY
THEREOF AFFECTED BY MATERIAL INCONSISTENCIES. — It is accepted
that, as a rule, minor inconsistencies in the testimony of a witness will
not affect his credibility. It even enhances such credibility because it
only shows that he has not been rehearsed. However, when the
inconsistencies pertain to material and crucial points, the same detract
from his overall credibility.
6. ID.; ID.; TESTIMONY OF ACCUSED; GIVEN CREDENCE. — We are
constrained to give more credibility to the testimony of appellant
Rodrigueza. While it is true that appellant's defense amounts to an alibi,
and as such is the weakest defense in a criminal prosecution, there are,
nonetheless, some evidentiary aspects pointing to the truth in his
testimony. Firstly, the Joint Affidavit of Arrest corroborates his testimony
that he was not among those who were arrested on the night of July 1,
1987. His co-accused Segovia also testified that appellant Rodrigueza
was not with them when they were apprehended by the NARCOM agents.
Secondly, the apparent motive of the NARCOM agents in prosecuting the
accused was also revealed during the trial of the case. Rebuttal
witnesses Gracita Bahillo, sister of appellant, and Hospicio Segovia,
father of Samuel Segovia, testified that Sgt. Moliñawe, who has since
been reportedly dismissed from the service, asked for P10,000.00 from
each of them in exchange for the liberty of the accused. This allegation
was never refuted by the prosecution.
7. ID.; ID.; PROOF REQUIRED IN CRIMINAL CASES; BEYOND REASONABLE
DOUBT NOT ESTABLISHED IN CASE AT BAR. — The Court has repeatedly
ruled that to sustain the conviction of the accused, the prosecution must
rely on the strength of its own evidence and not on the weakness of the
defense. As clearly shown by the evidence, the prosecution has failed to
establish its cause. It has not overcome the presumption of innocence
accorded to appellant. This being the case, appellant should not be
allowed to suffer for unwarranted and imaginary imputations against him.

DECISION

REGALADO, J : p

On appeal before us is the decision of the Regional Trial Court of Legaspi


City, Branch 10, finding accused-appellant Don Rodrigueza guilty beyond
reasonable doubt of violating Section 4, Article II of the Dangerous Drugs
Act of 1972 (Republic Act No. 6425, as amended) and sentencing him to
suffer the penalty of life imprisonment and to pay a fine of P20,000.00
and costs. 1
However, the Solicitor General, deviating from his conventional stance in
the prosecution of criminal cases, recommends the acquittal of appellant
for the reasons stated in his Manifestation for Acquittal (In Lieu of
Appellee's Brief) filed with the Court. We have reviewed and analyzed the
testimonial and documentary evidence in this case and we find said
recommendation to be well taken. cdrep

The information, dated July 10, 1987, charges Don Rodrigueza and his co-
accused, Samuel Segovia and Antonio Lonceras, with allegedly having in
their custody and possession 100 grams of marijuana leaves and for
selling, in a buy-bust operation, said 100 grams of dried marijuana leaves
for a consideration of P200.00. 2
During the arraignment, all the accused pleaded not guilty to the charge
against them. At the trial, the prosecution and the defense presented
several witnesses after which the court a quo rendered judgment
acquitting Samuel Segovia and Antonio Lonceras but convicting and
penalizing herein appellant as hereinbefore stated.
The following facts are culled from the decision of the trial court and the
evidence presented by the prosecution.
At around 5:00 o'clock in the afternoon of July 1, 1987, CIC Ciriaco
Taduran was in their headquarters at the Office of the Narcotics Regional
Unit at Camp Bagong Ibalon, Legaspi City, together with S/Sgt. Elpidio
Moliñawe, CIC Leonardo B. Galutan and their commanding officer, Major
Crisostomo M. Zeidem, when a confidential informer arrived and told
them that there was an ongoing illegal traffic of prohibited drugs in
Tagas, Daraga, Albay. Major Zeidem formed a team to conduct a buybust
operation, which team was given P200.00 in different denominations to
buy marijuana. These bills were treated with ultraviolet powder at the
Philippine Constabulary Crime Laboratory (PCCL). Sgt. Moliñawe gave the
money to Taduran who acted as the poseur buyer. He was told to look for
a certain Don, the alleged seller of prohibited drugs. Taduran went to
Tagas alone and, while along the road, he met Samuel Segovia. He asked
Segovia where he could find Don and where he could buy marijuana.
Segovia left for a while and when he returned, he was accompanied by a
man who was later on introduced to him as Don, herein appellant. 3

After agreeing on the price of P200.00 for 100 grams of marijuana, Don
halted a passing tricycle driven by Antonio Lonceras. He boarded it and
left Taduran and Segovia. When he came back, Don gave Taduran "a
certain object wrapped in a plastic" which was later identified as
marijuana, and received payment therefor. Thereafter, Taduran returned
to the headquarters and made a report regarding his said purchase of
marijuana. 4
Based on that information, Major Zeidem ordered a team to conduct an
operation to apprehend the suspects. In the evening of the same date,
CIC Galutan and S/Sgt. Moliñawe proceeded to Regidor Street, Daraga,
Albay and arrested appellant, Antonio Lonceras and Samuel Segovia. The
constables were not, however, armed with a warrant of arrest when they
apprehended the three accused. The arrestees were brought to the
headquarters for investigation. 5
Thereafter, agents of the Narcotics Command (NARCOM) conducted a
raid in the house of Jovencio Rodrigueza, father of appellant. Taduran did
not go with them. During the raid, they were able to confiscate dried
marijuana leaves and a plastic syringe, among others. The search,
however, was not authorized by any search warrant. 6
The next day, July 2, 1987, Jovencio Rodrigueza was released from
detention but appellant was detained. An affidavit, allegedly taken from
and executed by him, was sworn to by him before the assistant city
prosecutor. Appellant had no counsel when his sworn statement was
taken during that custodial investigation. The arrestees were also
examined by personnel of the PCCL and were found positive for
ultraviolet powder. 7
The three accused presented different versions of their alleged
participations.
Samuel Segovia testified that he was in their house in the evening of July
1, 1987 listening to the radio. Later, he ate his merienda and then went
out to buy cigarettes from the store. While he was at the store, a jeep
stopped behind him. Several armed men alighted therefrom and ordered
him to get inside the jeep. He refused but he was forced to board the
vehicle. He was even hit by the butt of a gun. 8
He was thereafter brought to Camp Bagong Ibalon where he was
investigated and was repeatedly asked regarding the whereabouts of
Rodrigueza. He was manhandled by the NARCOM agents and was
detained while inside the camp. He was then made to hold a P10.00 bill
treated with ultraviolet powder. When he was taken to the PCCL and
examined, he was found positive of the ultraviolet powder. He was also
made to sign some papers but he did not know what they were all
about. 9
Appellant, on the other hand, testified that on said date he was in the
house of his aunt in San Roque, Legaspi City. He stayed there overnight
and did not leave the place until the next day when his brother arrived
and told him that their father was taken by some military men the
preceding night. Appellant went to Camp Bagong Ibalon and arrived there
at around 8:00 o'clock in the morning of July 2, 1987. When he arrived, he
was asked if he knew anything about the marijuana incident, to which
question he answered in the negative. Like Segovia, he was made to hold
a P10.00 bill and was brought to the crime laboratory for examination.
From that time on, he was not allowed to go home and was detained
inside the camp. He was also tortured in order to make him admit his
complicity in the alleged sale of marijuana. 1 0
In the assignment of errors in his brief, appellant contends that the trial
court erred in (1) admitting in evidence the sworn statement of appellant
which was obtained in violation of his constitutional rights; (2) convicting
appellant of the crime charged despite the fact that the 100 grams of
dried marijuana leaves allegedly bought from him were not properly
identified; (3) convicting appellant of the crime charged despite the fact
that the evidence for the prosecution is weak and not convincing; and (4)
finding appellant guilty beyond reasonable doubt of selling or at least
acting as broker in the sale of the 100 grams of marijuana to CIC Taduran
late in the afternoon of July 1, 1987, despite the failure of the
prosecution to prove his guilt beyond reasonable doubt. 11
We rule for the appellant and approve the recommendation for his
acquittal. In disposing of this case, however, we feel that the issues
raised by appellant should properly be discussed seriatim.
1. A buy-bust operation is a form of entrapment employed by peace
officers to trap and catch a malefactor in flagrante delicto. 12 Applied to
the case at bar, the term in flagrante delicto requires that the suspected
drug dealer must be caught redhanded in the act of selling marijuana or
any prohibited drug to a person acting or posing as a buyer.
In the instant case, however, the procedure adopted by the NARCOM
agents failed to meet this qualification. Based on the very evidence of
the prosecution, after the alleged consummation of the sale of dried
marijuana leaves, CIC Taduran immediately released appellant
Rodrigueza instead of arresting and taking him into his custody. This act
of CIC Taduran, assuming arguendo that the supposed sale of marijuana
did take place, is decidedly contrary to the natural course of things and
inconsistent with the aforestated purpose of a buy-bust operation. It is
rather absurd on his part to let appellant escape without having been
subjected to the sanctions imposed by law. It is, in fact, a dereliction of
duty by an agent of the law.
2. The admissibility of the sworn statement allegedly executed by
appellant was squarely placed in issue and, as correctly pointed out by
the defense, said sworn statement is inadmissible in evidence against
appellant.
We have once again to reiterate and emphasize that Article III of
the 1987 Constitution provides:
"Sec. 12 (1). Any person under investigation for the commission of
an offense shall have the right to be informed of his right to remain
silent and to have a competent and independent counsel
preferably of his own choice. If the person cannot afford the
services of counsel, he must be provided with one. These rights
cannot be waived except in writing and in the presence of counsel.
xxx xxx xxx
(3) Any confession or admission obtained in violation of this or
section 17 hereof shall be inadmissible in evidence against him."
An examination of said sworn statement shows that appellant was
informed of his constitutional right to remain silent and to be assisted by
counsel during custodial examination. He was also asked if he was
waiving his right to be assisted by counsel and he answered in the
affirmative. However, while the rights of a person under custodial
investigation may be waived, such waiver must be made not only
voluntarily, knowingly and intelligently but also in the presence and with
the assistance of counsel. 13 In the present case, the waiver made by
appellant being without the assistance of counsel, this omission alone is
sufficient to invalidate said sworn statement. 14
3. Corollary to this, we take cognizance of the error of the trial court in
admitting in evidence against appellant the articles allegedly
confiscated during the raid conducted in the house of Jovencio
Rodrigueza. cdll

As provided in the present Constitution, a search, to be valid, must


generally be authorized by a search warrant duly issued by the proper
government authority. 15 True, in some instances, this Court has allowed
government authorities to conduct searches and seizures even without a
search warrant. Thus, when the owner of the premises waives his right
against such incursion; 16 when the search is incidental to a lawful
arrest; 17 when it is made on vessels and aircraft for violation of customs
laws; 18 when it is made on automobiles for the purpose of preventing
violations of smuggling or immigration laws; 19 when it involves
prohibited articles in plain view; 20or in cases of inspection of buildings
and other premises for the enforcement of fire, sanitary and building
regulations, 21 a search may be validly made even without a search
warrant.
In the case at bar, however, the raid conducted by the NARCOM agents in
the house of Jovencio Rodrigueza was not authorized by any search
warrant. It does not appear, either, that the situation falls under any of
the aforementioned cases. Hence, appellant's right against unreasonable
search and seizure was clearly violated. The NARCOM agents could not
have justified their act by invoking the urgency and necessity of the
situation because the testimonies of the prosecution witnesses reveal
that the place had already been put under surveillance for quite some
time. Had it been their intention to conduct the raid, then they should,
because they easily could, have first secured a search warrant during
that time.
4. The Court further notes the confusion and ambiguity in the
identification of the confiscated marijuana leaves and other prohibited
drug paraphernalia presented as evidence against appellant.
CIC Taduran, who acted as the poseur buyer, testified that appellant sold
him 100 grams of dried marijuana leaves wrapped in a plastic bag.
Surprisingly, and no plausible explanation has been advanced therefor,
what were submitted to and examined by the PCCL and thereafter
utilized as evidence against the appellant were the following items:
"One (1) red and white colored plastic bag containing the
following:
Exh. 'A' — Thirty (30) grams of suspected dried marijuana fruiting
tops contained inside a transparent plastic bag.
Exh. 'B' — Fifty (50) grams of suspected dried marijuana leaves and
seeds contained inside a white colored plastic labelled
'Robertson'.
Exh. 'C' — Four (4) aluminum foils each containing suspected dried
marijuana fruiting tops having a total weight of seven grams then
further wrapped with a piece of aluminum foil.

Exh. 'D' — Five (5) small transparent plastic bags each containing
suspected dried marijuana fruiting tops having a total weight of
seventeen grams.
Exh. 'E' — One plastic syringe." 22

Evidently, these prohibited articles were among those confiscated during


the so-called follow-up raid in the house of Jovencio Rodrigueza. The
unanswered question then arises as to the identity of the marijuana
leaves that became the basis of appellant's conviction. 23 In People vs.
Rubio, 24 this Court had the occasion to rule that the plastic bag and the
dried marijuana leaves contained therein constitutes the corpus
delicti of the crime. As such, the existence thereof must be proved with
certainty and conclusiveness. Failure to do so would be fatal to the
cause of the prosecution. cdphil

5. It is accepted that, as a rule, minor inconsistencies in the testimony of


a witness will not affect his credibility. It even enhances such credibility
because it only shows that he has not been rehearsed. 25 However, when
the inconsistencies pertain to material and crucial points, the same
detract from his overall credibility.
The exception, rather than the rule, applies in the case at bar. As
correctly pointed out by the Solicitor General, the testimonies of the
prosecution witnesses are tainted with serious flaws and material
inconsistencies rendering the same incredible. 26
CIC Taduran, in his testimony, said that they had already been conducting
surveillance of the place where the buy-bust operation was to take place.
It turned out, however, that he did not even know the exact place and the
identity of the person from whom he was to buy marijuana leaves. Thus:
"FISCAL TOLOSA:
Q What place in Tagas were you able to go (to)?
WITNESS.
A I am not actually familiar in (sic) that place, in Tagas, although
we occasionally passed there.
Q Now, upon your arrival in Tagas, what did you do that afternoon?
A I waited for the suspect because previously, we have already
been conducted (sic) surveylance (sic) in the vicinity.
Q Upon arrival in Tagas, were you able to see the suspect?
A By the road, sir.
Q Who was the first person did you see (sic) when you arrived at
Tagas?
A The first person whom I saw is Samuel Segovia.
Q Were you able to talk with this Samuel Segovia?
A According to him, we could get some." 27

The same findings go for the testimony of witness Galutan. In his direct
examination, he declared that they arrested the three accused all at the
same time on the fateful night of July 1, 1987. But, in his cross-
examination and as corroborated by the Joint Affidavit of
Arrest 28 submitted by him and Moliñawe, it appeared that Lonceras and
Segovia were arrested on different times and that appellant Don
Rodrigueza was not among those who were arrested. Instead, it was
Jovencio Rodrigueza, Don's father, who was picked up at a much later
time.
With said inconsistencies in sharp focus, we are constrained to give
more credibility to the testimony of appellant Rodrigueza. While it is true
that appellant's defense amounts to an alibi, and as such is the weakest
defense in a criminal prosecution, there are, nonetheless, some
evidentiary aspects pointing to the truth in his testimony. Firstly, the
Joint Affidavit of Arrest corroborates his testimony that he was not
among those who were arrested on the night of July 1, 1987. His co-
accused Segovia also testified that appellant Rodrigueza was not with
them when they were apprehended by the NARCOM agents.
Secondly, the apparent motive of the NARCOM agents in prosecuting the
accused was also revealed during the trial of the case. Rebuttal
witnesses Gracita Bahillo, sister of appellant, and Hospicio Segovia,
father of Samuel Segovia, testified that Sgt. Moliñawe, who has since
been reportedly dismissed from the service, asked for P10,000.00 from
each of them in exchange for the liberty of the accused. 29 This allegation
was never refuted by the prosecution. Hence, the rule laid down by this
Court that the statements of prosecution witnesses are entitled to full
faith and credit 30 has no application in the case at bar.
Finally, the Court has repeatedly ruled that to sustain the conviction of
the accused, the prosecution must rely on the strength of its own
evidence and not on the weakness of the defense. 31 As clearly shown by
the evidence, the prosecution has failed to establish its cause. It has not
overcome the presumption of innocence accorded to appellant. This
being the case, appellant should not be allowed to suffer for unwarranted
and imaginary imputations against him.
WHEREFORE, the judgment of conviction of the court below is hereby
REVERSED and SET ASIDE and accused-appellant Don Rodrigueza is
hereby ACQUITTED of the crime charged. It is hereby ordered that he be
immediately released from custody unless he is otherwise detained for
some other lawful cause.
SO ORDERED.
(People v. Rodrigueza, G.R. No. 95902, [February 4, 1992], 282 PHIL 829-
|||

842)
[G.R. No. 101837. February 11, 1992.]

ROLITO GO y TAMBUNTING, petitioner, vs. THE COURT OF


APPEALS; THE HON. BENJAMIN V. PELAYO, Presiding Judge,
Branch 168, Regional Trial Court, NCJR Pasig, M.M.; and
PEOPLE OF THE PHILIPPINES, respondents.

SYLLABUS

1. REMEDIAL LAW; CRIMINAL PROCEDURE; WARRANTLESS ARREST


UNDER SEC. 5, RULE 113, NOT APPLICABLE IN CASE AT BAR. —We do not
believe that the warrantless "arrest" or detention of petitioner in the
instant case falls within the terms of Section 5 of Rule 113 of the 1985
Rules on Criminal Procedure which provides as follows: "Sec. 5. Arrest
without warrant; when lawful. — A peace officer or a private person may,
without a warrant, arrest a person: (a) When, in his presence, the person
to be arrested has committed, is actually committing, or is attempting to
commit an offense; (b) When an offense has in fact just been committed,
and he has personal knowledge of facts indicating that the person to be
arrested has committed it; and (c) When the person to be arrested is a
prisoner who has escaped from a penal establishment or place where he
is serving final judgment or temporarily confined while his case is
pending, or has escaped while being transferred from one confinement to
another. In cases falling under paragraphs (a) and (b) hereof, the person
arrested without a warrant shall be forthwith delivered to the nearest
police station or jail, and he shall be proceeded against in accordance
with Rule 112, Section 7." Petitioner's "arrest" took place six (6) days
after the shooting of Maguan. The arresting officers obviously were not
present, within the meaning of Section 5(a), at the time petitioner had
allegedly shot Maguan. Neither could the "arrest" effected six (6) days
after the shooting be reasonably regarded as effected "when [the
shooting had] in fact just been committed" within the meaning of Section
5 (b). Moreover, none of the "arresting" officers had any "personal
knowledge" of facts indicating that petitioner was the gunman who had
shot Maguan. The information upon which the police acted had been
derived from statements made by alleged eyewitnesses to the shooting --
one stated that petitioner was the gunman; another was able to take
down the alleged gunman's car's plate number which turned out to be
registered in petitioner's wife's name. That information did not, however,
constitute "personal knowledge." It is thus clear to the Court that there
was no lawful warrantless arrest of petitioner within the meaning of
Section 5 of Rule 113.
2. ID.; ID.; ID.; ACCUSED ENTITLED TO A PRELIMINARY INVESTIGATION
WITHOUT ANY CONDITIONS. — Petitioner was not arrested at all. When
he walked into the San Juan Police Station, accompanied by two (2)
lawyers, he in fact placed himself at the disposal of the police
authorities. He did not state that he was "surrendering" himself, in all
probability to avoid the implication he was admitting that he had slain
Eldon Maguan or that he was otherwise guilty of a crime. When the police
filed a complaint for frustrated homicide with the Prosecutor, the latter
should have immediately scheduled a preliminary investigation to
determine whether there was probable cause for charging petitioner in
court for the killing of Eldon Maguan. Instead, as noted earlier, the
Prosecutor proceeded under the erroneous supposition that Section 7 of
Rule 112 was applicable and required petitioner to waive the provisions
of Article 125 of the Revised Penal Code as a condition for carrying out a
preliminary investigation. This was substantive error, for petitioner was
entitled to a preliminary investigation and that right should have been
accorded him without any conditions. Moreover, since petitioner had not
been arrested, with or without a warrant, he was also entitled to be
released forthwith subject only to his appearing at the preliminary
investigation.
3. ID.; ID.; ID.; RIGHT TO PRELIMINARY INVESTIGATION; NO WAIVER
THEREOF MADE IN CASE AT BAR. — Turning to the second issue of
whether or not petitioner had waived his right to preliminary
investigation, we note that petitioner had from the very beginning
demanded that a preliminary investigation be conducted. As earlier
pointed out, on the same day that the information for murder was filed
with the Regional Trial Court, petitioner filed with the Prosecutor an
omnibus motion for immediate release and preliminary investigation. The
Solicitor General contends that that omnibus motion should have been
filed with the trial court and not with the Prosecutor, and that petitioner
should accordingly be held to have waived his right to preliminary
investigation. We do not believe that waiver of petitioner's statutory right
to preliminary investigation may be predicated on such a slim basis. The
preliminary investigation was to be conducted by the Prosecutor, not by
the Regional Trial Court. It is true that at the time of filing of petitioner's
omnibus motion, the information for murder had already been filed with
the Regional Trial Court: it is not clear from the record whether petitioner
was aware of this fact at the time his omnibus motion was actually filed
with the Prosecutor. Nonetheless, since petitioner in his omnibus motion
was asking for preliminary investigation and not for a re-investigation
and since the Prosecutor himself did file with the trial court, on the 5th
day after filing the information for murder, a motion for leave to conduct
preliminary investigation (attaching to his motion a copy of petitioner's
omnibus motion), we conclude that petitioner's omnibus motion was in
effect filed with the trial court. What was crystal clear was that
petitioner did ask for a preliminary investigation on the very day that the
information was filed without such preliminary investigation, and that the
trial court was five (5) days later apprised of the desire of the petitioner
for such preliminary investigation. Finally, the trial court did in fact grant
the Prosecutor's prayer for leave to conduct preliminary investigation.
Thus, even on the (mistaken) supposition apparently made by the
Prosecutor that Section 7 of Rule 112 of the Revised Court was
applicable, the 5-day reglementary period in Section 7, Rule 112 must be
held to have been substantially complied with. We believe and so hold
that petitioner did not waive his right to a preliminary investigation. The
rule is that the right to preliminary investigation is waived when the
accused fails to invoke it before or at the time of entering a plea at
arraignment. In the instant case, petitioner Go had vigorously insisted on
his right to preliminary investigation before his arraignment. We do not
believe that by posting bail, petitioner had waived his right to preliminary
investigation. Petitioner Go asked for release on recognizance or on bail
and for preliminary investigation in one omnibus motion. He had thus
claimed his right to preliminary investigation before respondent Judge
approved the cash bond posted by petitioner and ordered his release on
12 July 1991. Accordingly, we cannot reasonably imply waiver of
preliminary investigation on the part of petitioner. In fact, when the
Prosecutor filed a motion in court asking for leave to conduct preliminary
investigation, he clearly if impliedly recognized that petitioner's claim to
preliminary investigation was a legitimate one.
4. ID.; ID.; ID.; ID.; A SUBSTANTIVE RIGHT AND A COMPONENT PART OF
DUE PROCESS. — While the right to a preliminary investigation is
statutory rather than constitutional in its fundament, since it has in fact
been established by statute, it is a component part of due process in
criminal justice. The right to have a preliminary investigation conducted
before being bound over to trial for a criminal offense and hence formally
at risk of incarceration or some other penalty, is not a mere formal or
technical right; it is a substantive right. The accused in a criminal trial is
inevitably exposed to prolonged anxiety, aggravation, humiliation, not to
speak of expense; the right to an opportunity to avoid a process painful
to any one save, perhaps, to hardened criminals, is a valuable right. To
deny petitioner's claim to a preliminary investigation would be to deprive
him of the full measure of his right to due process.
5. ID.; ID.; FAILURE TO ACCORD PRELIMINARY INVESTIGATION DOES NOT
IMPAIR VALIDITY OF INFORMATION FILED. — Contrary to petitioner's
contention the failure to accord preliminary investigation, while
constituting a denial of the appropriate and full measure of the statutory
process of criminal justice, did not impair the validity of the information
for murder nor affect the jurisdiction of the trial court.
6. ID.; ID.; ACCUSED ENTITLED TO PRELIMINARY INVESTIGATION EVEN
THOUGH TRIAL ON THE MERITS HAS ALREADY BEGAN. — We consider
that petitioner remains entitled to a preliminary investigation although
trial on the merits has already began. Trial on the merits should be
suspended or held in abeyance and a preliminary investigation forthwith
accorded to petitioner. The constitutional point is that petitioner
was not accorded what he was entitled to by way of procedural due
process. Petitioner was forced to undergo arraignment and literally
pushed to trial without preliminary investigation, with extraordinary
haste, to the applause from the audience that filled the courtroom. If he
submitted to arraignment and trial, petitioner did so "kicking and
screaming," in a manner of speaking. During the proceedings held before
the trial court on 23 August 1991, the date set for arraignment of
petitioner, and just before arraignment, counsel made very clear
petitioner's vigorous protest and objection to the arraignment precisely
because of the denial of preliminary investigation.
7. ID.; ID.; ACCUSED ENTITLED TO BE RELEASED ON BAIL AS A MATTER
OF RIGHT. — In respect of the matter of bail, petitioner remains entitled
to be released on bail as a matter of right. Should the evidence already
of record concerning petitioner's guilt be, in the reasonable belief of the
Prosecutor, strong, the Prosecutor may move in the trial court for
cancellation of petitioner's bail. It would then be up to the trial court,
after a careful and objective assessment of the evidence on record, to
grant or deny the motion for cancellation of bail. It must also be recalled
that the Prosecutor had actually agreed that petitioner was entitled to
bail. This was equivalent to an acknowledgment on the part of the
Prosecutor that the evidence of guilt then in his hands was not strong.
Accordingly, we consider that the 17 July 1991 order of respondent
Judge recalling his own order granting bail and requiring petitioner to
surrender himself within forty-eight (48) hours from notice, was plainly
arbitrary considering that no evidence at all — and certainly
no new or additional evidence — had been submitted to respondent
Judge that could have justified the recall of his order issued just five (5)
days before.

8. ID.; ID.; RELIANCE ON THE CASE OF UMIL v. RAMOS, MISPLACED;


OFFENSE COMMITTED NOT CONSIDERED A "CONTINUING CRIME." — The
reliance of both petitioner and the Solicitor General upon Umil v.
Ramos (G.R. No. 81567, promulgated 3 October 1991) is, in the
circumstances of this case, misplaced. In Umil v. Ramos, by an eight-to-
six vote, the Court sustained the legality of the warrantless arrests of
petitioners made from one (1) to fourteen (14) days after the actual
commission of the offenses, upon the ground that such offenses
constituted "continuing crimes." Those offenses were subversion,
membership in an outlawed organization like the New Peoples Army, etc.
In the instant case, the offense for which petitioner was arrested was
murder, an offense which was obviously commenced and completed at
one definite location in time and space. No one had pretended that the
fatal shooting of Maguan was a "continuing crime."
9. CONSTITUTIONAL LAW; STATE SHOULD REAFFIRM ITS OBLIGATION TO
RESPECT THE RIGHTS AND LIBERTIES OF ITS CONSTITUENTS; TO
ACCORD AN ACCUSED HIS RIGHT TO A PRELIMINARY INVESTIGATION
AND TO BAIL IN CASE AT BAR, NOT AN IDLE CEREMONY. — To reach any
other conclusion here, that is, to hold that petitioner's rights to a
preliminary investigation and to bail were effectively obliterated by
evidence subsequently admitted into the record would be to legitimize
the deprivation of due process and to permit the Government to benefit
from its own wrong or culpable omission and effectively to dilute
important rights of accused persons well-nigh to the vanishing point. It
may be that to require the State to accord petitioner his rights to a
preliminary investigation and to bail at this point, couldturn out
ultimately to be largely a ceremonial exercise. But the Court is not
compelled to speculate. And, in any case, it would not be idle ceremony;
rather it would be a celebration by the State of the rights and liberties of
its own people and a re-affirmation of its obligation and determination to
respect those rights and liberties.
CRUZ, J., concurring:
1. REMEDIAL LAW; CRIMINAL PROCEDURE; STATUTORY RIGHTS OF
ACCUSED TO PROCEDURAL DUE PROCESS VITIATED IN CASE AT BAR. —
Petitioner had from the start demanded a preliminary investigation and
that his counsel has reluctantly participated in the trial only because the
court threatened to replace him with a counselde oficio if he did not.
Under these circumstances, I am convinced that there was no waiver.
The petitioner was virtually compelled to go to trial. Such compulsion
and the unjustified denial of a clear statutory right of the petitioner
vitiated the proceedings as violative of procedural due process.
GUTIERREZ, JR., J., concurring:
1. JUDICIAL ETHICS; COURTS; IMPORTANCE OF FOLLOWING THE RULES
EMPHASIZED. — The need for a trial court to follow the Rules and to be
fair, impartial, and persistent in getting the true facts of a case is present
in all cases but it is particularly important if the accused is indigent;
more so, if he is one of those unfortunates who seem to spend more time
behind bars than outside. Unlike the accused in this case who enjoys the
assistance of competent counsel, a poor defendant convicted by wide
and unfavorable media coverage may be presumed guilty before trial and
be unable to defend himself properly. Hence, the importance of the court
always following the Rules.
GRIÑO-AQUINO, J., dissenting:
1. REMEDIAL LAW; CRIMINAL PROCEDURE; PRELIMINARY
INVESTIGATION, NO LONGER NEEDED; RETURN OF CASE TO THE
PROSECUTOR, SUPEREROGATORY. — I do not believe that there is still
need to conduct a preliminary investigation the sole purpose of which
would be to ascertain if there is sufficient ground to believe that a crime
was committed (which the petitioner does not dispute) and that he (the
petitioner) is probably guilty thereof (which the prosecutor, by filing the
information against him, presumably believed to be so). In the present
stage of the presentation of the prosecution's evidence, to return the
case to the Prosecutor to conduct a preliminary investigation under Rule
112 of the 1985 Rules on Criminal Procedure would be supererogatory.
2. ID.; ID.; RIGHT TO PRELIMINARY INVESTIGATION, NOT A
CONSTITUTIONAL RIGHT. — It should be remembered that as important
as is the right of the accused to a preliminary investigation, it is not a
constitutional right. Its absence is not a ground to quash the information
(Doromal vs. Sandiganbayan, 177 SCRA 354). It does not affect the
court's jurisdiction, nor impair the validity of the information
(Rodis vs. Sandiganbayan, 166 SCRA 618), nor constitute an infringement
of the right of the accused to confront witnesses (Bustos vs. Lucero, 81
Phil. 640).
3. ID.; ID.; HEARING OF APPLICATION FOR BAIL; SHOULD NOT BE
SUSPENDED AND SHOULD NOT BE SUBORDINATED TO THE
PRELIMINARY INVESTIGATION OF THE CHARGE. — The court's hearing of
the application for bail should not be subordinated to the preliminary
investigation of the charge. The hearing should not be suspended, but
should be allowed to proceed for it will accomplish a double purpose. The
parties will have an opportunity to show not only: (a) whether or not
there is probable cause to believe that the petitioner killed Eldon
Maguan, but more importantly (b) whether or not the evidence of his guilt
is strong. The judge's determination that the evidence of his guilt is
strong would naturally foreclose the need for a preliminary investigation
to ascertain the probability of his guilt. The bail hearing may not be
suspended because upon the filing of an application for bail by one
accused of a capital offense, "the judge is under a legal obligation to
receive evidence with the view of determining whether evidence of guilt
is so strong as to warrant denial of bond."
4. ID.; ID.; ABOLITION OF DEATH PENALTY DID NOT MAKE THE RIGHT TO
BAIL ABSOLUTE; ACCUSED MAY NOT BE RELEASED PENDING HEARING
OF PETITION FOR BAIL. — The abolition of the death penalty did not
make the right to bail absolute, for persons charged with offenses
punishable by reclusion perpetua, when evidence of guilt is strong, are
not bailable (Sec. 3, Art. III, 1987 Constitution). In People vs.
Dacudao, 170 SCRA 489, we called down the trial court for having
granted the motion for bail in a murder case without any hearing and
without giving the prosecution an opportunity to comment or file
objections thereto. Similarly, this Court held inPeople vs. Bocar, 27 SCRA
512: ". . . due process also demands that in the matter of bail the
prosecution should be afforded full opportunity to present proof of the
guilt of the accused. Thus, if it were true that the prosecution in this
case was deprived of the right to present its evidence against the bail
petition, or that the order granting such petition was issued upon
incomplete evidence, then the issuance of the order would really
constitute abuse of discretion that would call for the remedy of
certiorari." The petitioner may not be released pending the hearing of his
petition for bail for it would be incongruous to grant bail to one who is
not in the custody of the law (Feliciano vs. Pasicolan, 2 SCRA 888).
5. ID.; ID.; TERM "ARREST," CONSTRUED. — Arrest is the taking of a
person into custody in order that he may be bound to answer for the
commission of an offense (Sec. 1, Rule 113, Rules of Court). An arrest is
made by an actual restraint of the person to be arrested, or by his
submission to the custody of the person making the arrest (Sec. 2, Rule
113, Rules of Court). When Go walked into the San Juan Police Station on
July 8, 1991, and placed himself at the disposal of the police authorities
who clamped him in jail after he was identified by an eyewitness as the
person who shot Maguan, he was actually and effectively arrested. His
filing of a petition to be released on bail was a waiver of any irregularity
attending his arrest and estops him from questioning its validity
(Callanta vs. Villanueva, 77 SCRA 377; Bagcal vs.Villaraza, 120 SCRA
525).

DECISION

FELICIANO, J :p

According to the findings of the San Juan Police in their Investigation


Report, 1 on 2 July 1991, Eldon Maguan was driving his car along Wilson
St., San Juan, Metro Manila, heading towards P. Guevarra St. Petitioner
entered Wilson St., where it is a one-way street and started travelling in
the opposite or "wrong" direction. At the corner of Wilson and J. Abad
Santos Sts., petitioner's and Maguan's cars nearly bumped each other.
Petitioner alighted from his car, walked over and shot Maguan inside his
car. Petitioner then boarded his car and left the scene. A security guard
at a nearby restaurant was able to take down petitioner's car plate
number. The police arrived shortly thereafter at the scene of the shooting
and there retrieved an empty shell and one round of live ammunition for a
9mm caliber pistol. Verification at the Land Transportation Office showed
that the car was registered to one Elsa Ang Go.
The following day, the police returned to the scene of the shooting to find
out where the suspect had come from; they were informed that petitioner
had dined at Cravings Bake Shop shortly before the shooting. The police
obtained a facsimile or impression of the credit card used by petitioner
from the cashier of the bake shop. The security guard of the bake shop
was shown a picture of petitioner and he positively identified him as the
same person who had shot Maguan. Having established that the
assailant was probably the petitioner, the police launched a manhunt for
petitioner.
On 8 July 1991, petitioner presented himself before the San Juan Police
Station to verify news reports that he was being hunted by the police; he
was accompanied by two (2) lawyers. The police forthwith detained him.
An eyewitness to the shooting, who was at the police station at that
time, positively identified petitioner as the gunman. That same day, the
police promptly filed a complaint for frustrated homicide 2 against
petitioner with the Office of the Provincial Prosecutor of Rizal. First
Assistant Provincial Prosecutor Dennis Villa Ignacio ("Prosecutor")
informed petitioner, in the Presence of his lawyers, that he could avail
himself of his right to preliminary investigation but that he must first sign
a waiver of the provisions of Article 125 of the Revised Penal Code.
Petitioner refused to execute any such waiver.
On 9 July 1991, while the complaint was still with the Prosecutor, and
before an information could be filed in court, the victim, Eldon Maguan,
died of his gunshot wound(s). prcd

Accordingly, on 11 July 1991, the Prosecutor, instead of filing an


information for frustrated homicide, filed an information for
murder 3 before the Regional Trial Court. No bail was recommended. At
the bottom of the information, the Prosecutor certified that no
preliminary investigation had been conducted because the accused did
not execute and sign a waiver of the provisions of Article 125 of the
Revised Penal Code.
In the afternoon of the same day, 11 July 1991, counsel for petitioner
filed with the prosecutor an omnibus motion for immediate release and
proper preliminary investigation, 4 alleging that the warrantless arrest of
petitioner was unlawful and that no preliminary investigation had been
conducted before the information was filed. Petitioner also prayed that
he be released on recognizance or on bail. Provincial Prosecutor Mauro
Castro, acting on the omnibus motion, wrote on the last page of the
motion itself that he interposed no objection to petitioner being granted
provisional liberty on a cash bond of P100,000.00.
On 12 July 1991, petitioner filed an urgent ex-parte motion for special
raffle 5 in order to expedite action on the Prosecutor's bail
recommendation. The case was raffled to the sala of respondent Judge,
who, on the same date, approved the cash bond 6 posted by petitioner
and ordered his release. 7 Petitioner was in fact released that same day.
On 16 July 1991, the Prosecutor filed with the Regional Trial Court a
motion for leave to conduct preliminary investigation 8 and prayed that in
the meantime all proceedings in the court be suspended. He stated that
petitioner had filed before the Office of the Provincial Prosecutor of Rizal
an omnibus motion for immediate release and preliminary investigation,
which motion had been granted by Provincial Prosecutor Mauro Castro,
who also agreed to recommend cash bail of P100,000.00. The Prosecutor
attached to the motion for leave a copy of petitioner's omnibus motion of
11 July 1991.
Also on 16 July 1991, the trial court issued an Order 9 granting leave to
conduct preliminary investigation and cancelling the arraignment set for
15 August 1991 until after the prosecution shall have concluded its
preliminary investigation.
On 17 July 1991, however, respondent Judge motu proprio issued an
Order, 10 embodying the following: (1) the 12 July 1991 Order which
granted bail was recalled; petitioner was given 48 hours from receipt of
the Order to surrender himself; (2) the 16 July 1991 Order which granted
leave to the prosecutor to conduct preliminary investigation was recalled
and cancelled; (3) petitioner's omnibus motion for immediate release and
preliminary investigation dated 11 July 1991 was treated as a petition for
bail and set for hearing on 23 July 1991.
On 19 July 1991, petitioner filed a petition for certiorari, prohibition and
mandamus before the Supreme Court assailing the 17 July 1991 Order,
contending that the information was null and void because no
preliminary investigation had been previously conducted, in violation of
his right to due process. Petitioner also moved for suspension of all
proceedings in the case pending resolution by the Supreme Court of his
petition; this motion was, however, denied by respondent Judge. LLphil

On 23 July 1991, petitioner surrendered to the police.


By a Resolution dated 24 July 1991, this Court remanded the petition
for certiorari, prohibition and mandamus to the Court of Appeals.
On 16 August 1991, respondent Judge issued an order in open court
setting the arraignment of petitioner on 23 August 1991.
On 19 August 1991, petitioner filed with the Court of Appeals a motion to
restrain his arraignment.
On 23 August 1991, respondent judge issued a Commitment Order
directing the Provincial Warden of Rizal to admit petitioner into his
custody at the Rizal Provincial Jail. On the same date, petitioner was
arraigned. In view, however, of his refusal to enter a plea, the trial court
entered for him a plea of not guilty. The trial court then set the criminal
case for continuous hearings on 19, 24 and 26 September; on 2, 3, 11 and
17 October; and on 7, 8, 14, 15, 21 and 22 November 1991. 11
On 27 August 1991, petitioner filed a petition for habeas corpus 12 in the
Court of Appeals. He alleged that in view of public respondents' failure to
join issues in the petition for certiorari earlier filed by him, after the
lapse of more than a month, thus prolonging his detention, he was
entitled to be released on habeas corpus.
On 30 August 1991, the Court of Appeals issued the writ of habeas
corpus. 13 The petition for certiorari, prohibition and mandamus, on the
one hand, and the petition for habeas corpus, upon the other, were
subsequently consolidated in the Court of Appeals.
The Court of Appeals, on 2 September 1991, issued a resolution denying
petitioner's motion to restrain his arraignment on the ground that motion
had become moot and academic.
On 19 September 1991, trial of the criminal case commenced and the
prosecution presented its first witness.
On 23 September 1991, the Court of Appeals rendered a consolidated
decision 14 dismissing the two (2) petitions, on the following grounds:
a. Petitioner's warrantless arrest was valid because the offense for
which he was arrested and charged had been "freshly committed."
His identity had been established through investigation. At the
time he showed up at the police station, there had been an existing
manhunt for him. During the confrontation at the San Juan Police
Station, one witness positively identified petitioner as the culprit.
b. Petitioner's act of posting bail constituted waiver of any
irregularity attending his arrest. He waived his right to preliminary
investigation by not invoking it properly and seasonably under the
Rules.
c. The trial court did not abuse its discretion when it issued the 17
July 1991 Order because the trial court had the inherent power to
amend and control its processes so as to make them conformable
to law and justice.
d. Since there was a valid information for murder against petitioner
and a valid commitment order (issued by the trial judge after
petitioner surrendered to the authorities whereby petitioner was
given to the custody of the Provincial Warden), the petition
for habeas corpus could not be granted.
On 3 October 1991, the prosecution presented three (3) more witnesses
at the trial. Counsel for petitioner also filed a "Withdrawal of
Appearance" 15 with the trial court, with petitioner's conformity.
On 4 October 1991, the present petition for Review on Certiorari was
filed. On 14 October 1991, the Court issued a Resolution directing
respondent Judge to held in abeyance the hearing of the criminal case
below until further orders from this Court.
In this Petition for Review, two (2) principal issues need to be addressed:
first, whether or not a lawful warrantless arrest had been effected by the
San Juan Police in respect of petitioner Go; and second, whether
petitioner had effectively waived his right to preliminary investigation.
We consider these issues seriatim. LLphil

In respect of the first issue, the Solicitor General argues that under the
facts of the case, petitioner had been validly arrested without warrant.
Since petitioner's identity as the gunman who had shot Eldon Maguan on
2 July 1991 had been sufficiently established by police work, petitioner
was validly arrested six (6) days later at the San Juan Police Station. The
Solicitor General invokes Nazareno v. Station Commander, etc., et
al., 16 one of the seven (7) cases consolidated with In the Matter of the
Petition for Habeas Corpus of Roberto Umil, etc. v. Ramos et al., 17 where
a majority of the Court upheld a warrantless arrest as valid although
effected fourteen (14) days after the killing in connection with which
Nazareno had been arrested. Accordingly, in the view of the Solicitor
General, the provisions of Section 7, Rule 112 of the Rules of Court were
applicable and because petitioner had declined to waive the provisions
of Article 125 of the Revised Penal Code, the Prosecutor was legally
justified in filing the information for murder even without preliminary
investigation.
On the other hand, petitioner argues that he was not lawfully arrested
without warrant because he went to the police station six (6) days after
the shooting which he had allegedly perpetrated. Thus, petitioner argues,
the crime had not been "just committed" at the time that he was
arrested. Moreover, none of the police officers who arrested him had
been an eyewitness to the shooting of Maguan and accordingly none had
the "personal knowledge" required for the lawfulness of a warrantless
arrest. Since there had been no lawful warrantless arrest, Section 7, Rule
112 of the Rules of Court which establishes the only exception to the
right to preliminary investigation, could not apply in respect of petitioner.
The reliance of both petitioner and the Solicitor General upon Umil v.
Ramos is, in the circumstances of this case, misplaced. In Umil v. Ramos,
by an eight-to-six vote, the Court sustained the legality of the
warrantless arrests of petitioners made from one (1) to fourteen (14)
days after the actual commission of the offenses, upon the ground that
such offenses constituted "continuing crimes." Those offenses were
subversion, membership in an outlawed organization like the New
Peoples Army, etc. In the instant case, the offense for which petitioner
was arrested was murder, an offense which was obviously commenced
and completed at one definite location in time and space. No one had
pretended that the fatal shooting of Maguan was a "continuing crime."
Secondly, we do not believe that the warrantless "arrest" or detention of
petitioner in the instant case falls within the terms of Section 5 of Rule
113 of the 1985 Rules on Criminal Procedure which provides as follows:

"Sec. 5. Arrest without warrant; when lawful. — A peace officer or


a private person may, without a warrant, arrest a person:
(a) When, in his presence, the person to be created has committed,
is actually committing, or is attempting to commit an offense;
(b) When an offense has in fact just been committed, and he has
personal knowledge of facts indicating that the person to be
arrested has committed it; and
(c) When the person to be arrested is a prisoner who has escaped
from a penal establishment or place where he is serving final
judgment or temporarily confined while his case is pending, or has
escaped while being transferred from one confinement to another.
In cases falling under paragraphs (a) and (b) hereof, the person
arrested without a warrant shall be forthwith delivered to the
nearest police station or jail, and he shall be proceeded against in
accordance with Rule 112, Section 7."
Petitioner's "arrest" took place six (6) days after the shooting of
Maguan. The "arresting" officers obviously were not present, within
the meaning of Section 5(a), at the time petitioner had allegedly shot
Maguan. Neither could the "arrest" effected six (6) days after the
shooting be reasonably regarded as effected "when [the shooting had]
in fact just been committed" within the meaning of Section 5 (b).
Moreover, none of the "arresting" officers had any "personal
knowledge" of facts indicating that petitioner was the gunman who
had shot Maguan. The information upon which the police acted had
been derived from statements made by alleged eyewitnesses to the
shooting — one stated that petitioner was the gunman; another was
able to take down the alleged gunman's car's plate number which
turned out to be registered in petitioner's wife's name. That
information did not, however, constitute "personal knowledge." 18
It is thus clear to the Court that there was no lawful warrantless arrest
of petitioner within the meaning of Section 5 of Rule 113. It is clear too
that Section 7 of Rule 112, which provides:
"Sec. 7. When accused lawfully arrested without warrant. — When
a person is lawfully arrested without a warrant for an offense
cognizable by the Regional Trial Court the complaint or information
may be filed by the offended party, peace officer or fiscal without a
preliminary investigation having been first conducted, on the basis
of the affidavit of the offended party or arresting office or person.
However, before the filing of such complaint or information, the
person arrested may ask for a preliminary investigation by a proper
officer in accordance with this Rule, but he must sign a waiver of
the provisions of Article 125 of the Revised Penal Code, as
amended, with the assistance of a lawyer and in case of non-
availability of a lawyer, a responsible person of his
choice. Notwithstanding such waiver, he may apply for bail as
provided in the corresponding rule and the investigation must be
terminated within fifteen (15) days from its inception.Cdpr

If the case has been filed in court without a preliminary


investigation having been first conducted, the accused may within
five (5) days from the time he learns of the filing of the
information, ask for a preliminary investigation with the same right
to adduce evidence in his favor in the manner prescribed in this
Rule." (Underscoring supplied).
is also not applicable. Indeed, petitioner was not arrested at all. When
he walked into the San Juan Police Station, accompanied by two (2)
lawyers, he in fact placed himself at the disposal of the police
authorities. He did not state that he was "surrendering" himself, in all
probability to avoid the implication he was admitting that he had slain
Eldon Maguan or that he was otherwise guilty of a crime. When the
police filed a complaint for frustrated homicide with the Prosecutor,
the latter should have immediately scheduled a preliminary
investigation to determine whether there was probable cause for
charging petitioner in court for the killing of Eldon Maguan. Instead, as
noted earlier, the Prosecutor proceeded under the erroneous
supposition that Section 7 of Rule 112 was applicable and required
petitioner to waive the provisions of Article 125 of the Revised Penal
Code as a condition for carrying out a preliminary investigation. This
was substantive error, for petitioner was entitled to a preliminary
investigation and that right should have been accorded him without
any conditions. Moreover, since petitioner had not been arrested; with
or without a warrant, he was also entitled to be released forthwith
subject only to his appearing at the preliminary investigation.
Turning to the second issue of whether or not petitioner had waived his
right to preliminary investigation, we note that petitioner had from the
very beginning demanded that a preliminary investigation be conducted.
As earlier pointed out, on the same day that the information for murder
was filed with the Regional Trial Court. Petitioner filed with the
prosecutor an omnibus motion for immediate release and preliminary
investigation. The Solicitor General contends that omnibus motion should
have been filed with the trial court and not with the Prosecutor, and that
petitioner should accordingly be held to have waived his right to
preliminary investigation. We do not believe that waiver of petitioner's
statutory right to preliminary investigation may be predicated on such a
slim basis. The preliminary investigation was to be conducted by the
Prosecutor, not by the Regional Trial Court. It is true that at the time of
filing of petitioner's omnibus motion, the information for murder had
already been filed with the Regional Trial Court; it is not clear from the
record whether petitioner was aware of this fact at the time his omnibus
motion was actually filed with the Prosecutor. In Crespo v. Mogul, 19 this
Court held:
"The preliminary investigation conducted by the fiscal for the
purpose of determining whether a prima facie case exists
warranting the prosecution of the accused is terminated upon the
filing of the information in the proper court. In turn, as above
stated, the filing of said information sets in motion the criminal
action against the accused in Court. Should the fiscal find it
proper to conduct a reinvestigation of the case, at such stage, the
permission of the Court must be secured. After such
reinvestigation the finding and recommendations of the fiscal
should be submitted to the Court for appropriate action. While it is
true that the fiscal has the quasi judicial discretion to determine
whether or not a criminal case should be filed in court or not, once
the case had already been brought to Court whatever disposition
the fiscal may feel should be proper in the case thereafter should
be addressed for the consideration of the Court. The only
qualification is that the action of the Court must not impair the
substantial rights of the accused, or the right of the People to due
process of law.
xxx xxx xxx
The rule therefore in this jurisdiction is that once a complaint or
information is filed in Court any disposition of the case [such] as
its dismissal or the conviction or acquittal of the accused rests in
the sound discretion of the Court. Although the fiscal retains the
direction and control of the prosecution of criminal cases even
while the case is already in Court he cannot impose his opinion on
the trial court. The Court is the best and sole judge on what to do
with the case before it. . . ." 20 (Citations omitted, underscoring
supplied).
Nonetheless, since petitioner in his omnibus motion was asking for
preliminary investigation and not for a reinvestigation (Crespo v.
Mogul involved a re-investigation), and since the Prosecutor himself
did file with the trial court, on the 5th day after filing the information
for murder, a motion for leave to conduct preliminary investigation
(attaching to his motion a copy of petitioner's omnibus motion), we
conclude that petitioner's omnibus motion was in effect filed with the
trial court. What was crystal clear was that petitioner did ask for a
preliminary investigation on the very day that the information was
filed without such preliminary investigation, and that the trial court
was five (5) days later apprised of the desire of the petitioner for such
preliminary investigation. Finally, the trial court did in factgrant the
Prosecutor's prayer for leave to conduct preliminary investigation.
Thus, even on the (mistaken) supposition apparently made by the
Prosecutor that Section 7 of Rule 112 of the Revised Court was
applicable, the 5-day reglementary period in Section 7, Rule 112 must
be held to have been substantially complied with. LexLib

We believe and so hold that petitioner did not waive his right to a
preliminary investigation. While that right is statutory rather than
constitutional in its fundament, since it has in fact been established by
statute, it is a component part of due process in criminal justice. 20 The
right to have a preliminary investigation conducted before being bound
over to trial for a criminal offense and hence formally at risk of
incarceration or some other penalty, is not a mere formal or technical
right; it is asubstantive right. The accused in a criminal trial is inevitably
exposed to prolonged anxiety, aggravation, humiliation, not to speak of
expense; the right to an opportunity to avoid a process painful to any one
save, perhaps, to hardened criminals, is a valuable right. To deny
petitioner's claim to a preliminary investigation would be to deprive him
of the full measure of his right to due process.
The question may be raised whether petitioner still retains his right to a
preliminary investigation in the instant case considering that he was
already arraigned on 23 August 1991. The rule is that the right to
preliminary investigation is waived when the accused fails to invoke
it before or at the time of entering a plea at arraignment. 21In the instant
case, petitioner Go had vigorously insisted on his right to preliminary
investigation before his arraignment. At the time of his arraignment,
petitioner was already before the Court of Appeals on certiorari,
prohibition and mandamus precisely asking for a preliminary
investigation before being forced to stand trial.

Again, in the circumstances of this case, we do not believe that by


posting bail, petitioner had waived his right to preliminary investigation.
In People v. Selfaison, 22 we did hold that appellants there had waived
their right to preliminary investigation because immediately after their
arrest, they filed bail and proceeded to trial " without previously claiming
that they did not have the benefit of a preliminary investigation ." 23 In the
instant case, petitioner Go asked for release on recognizance or on bail
and for preliminary investigation in one omnibus motion. He had thus
claimed his right to preliminary investigation before respondent Judge
approved the cash bond posted by petitioner and ordered his release on
12 July 1991. Accordingly, we cannot reasonably imply waiver of
preliminary investigation on the part of petitioner. In fact, when the
Prosecutor filed a motion in court asking for leave to conduct preliminary
investigation, he clearly if impliedly recognized that petitioner's claim to
preliminary investigation was a legitimate one.
We would clarify, however, that contrary to petitioner's contention the
failure to accord preliminary investigation, while constituting a denial of
the appropriate and full measure of the statutory process of criminal
justice, did not impair the validity of the information for murder nor affect
the jurisdiction of the trial court. 24
It must also be recalled that the Prosecutor had actually agreed that
petitioner was entitled to bail. This was equivalent to an
acknowledgment on the part of the prosecutor that the evidence of guilt
then in his hands was not strong. Accordingly, we consider that the 17
July 1991 order of respondent Judge recalling his own order granting bail
and requiring petitioner to surrender himself within forty-eight (48) hours
from notice, was plainly arbitrary considering that no evidence at all —
and certainly no new or additional evidence — had been submitted to
respondent Judge that could have justified the recall of his order issued
just five (5) days before. It follows that petitioner was entitled to be
released on bail as a matter of right.
The final question which the Court must face is this: how does the fact
that, in the instant case, trial on the merits has already commenced, the
Prosecutor having already presented four (4) witnesses, impact upon,
firstly, petitioner's right to a preliminary investigation and secondly,
petitioner's right to be released on bail? Does he continue to be entitled
to have a preliminary investigation conducted in respect of the charge
against him? Does petitioner remain entitled to be released on bail?
Turning first to the matter of preliminary investigation, we consider that
petitioner remains entitled to a preliminary investigation although trial
on the merits has already began. Trial on the merits should be suspended
or held in abeyance and a preliminary investigation forthwith accorded to
petitioner. 25 It is true that the prosecutor might, in view of the evidence
that he may at this time have on hand, conclude that probable cause
exists; upon the other hand, the Prosecutor conceivably could reach the
conclusion that the evidence on hand does not warrant a finding of
probable cause. In any event, the constitutional point is that petitioner
was notaccorded what he was entitled to by way of procedural due
process. 26 Petitioner was forced to undergo arraignment and literally
pushed to trial without preliminary investigation, with extraordinary
haste, to the applause from the audience that filled the courtroom. If he
submitted to arraignment and trial, petitioner did so "kicking and
screaming," in a manner of speaking. During the proceedings held before
the trial court on 23 August 1991, the date set for arraignment of
petitioner, and just before arraignment, counsel made very clear
petitioner's vigorous protest and objection to the arraignment precisely
because of the denial of preliminary investigation. 27 So energetic and
determined were petitioner's counsel's protest and objection that an
obviously angered court and prosecutor dared him to withdraw or
walkout, promising to replace him with counsel de oficio. During the trial,
just before the prosecution called its first witness, petitioner through
counsel once again reiterated his objection to going to trial without
preliminary investigation; petitioner's counsel made or record his
"continuing objection." 28 Petitioner had promptly gone to the appellate
court on certiorari and prohibition to challenge the lawfulness of the
procedure he was being forced to undergo and the lawfulness of his
detention. 29 If he did not walkout on the trial, and if he cross-examined
the Prosecution's witnesses, it was because he was extremely loath to
be represented by counsel de oficioselected by the trial judge, and to run
the risk of being held to have waived also his right to use what is
frequently the only test of truth in the judicial process.
In respect of the matter of bail, we similarly believe and so hold that
petitioner remains entitled to be released on bail as a matter of right.
Should the evidence already of record concerning petitioner's guilt be, in
the reasonable belief of the Prosecutor, strong, the Prosecutor may move
in the trial court for cancellation of petitioner's bail. It would then be up
to the trial court, after a careful and objective assessment of the
evidence on record, to grant or deny the motion for cancellation of bail.
To reach any other conclusion here, that is, to hold that petitioner's
rights to a preliminary investigation and to bail were effectively
obliterated by evidence subsequently admitted into the record would be
to legitimize the deprivation of due process and to permit the
Government to benefit from its own wrong or culpable omission and
effectively to dilute important rights of accused persons well-nigh to the
vanishing point. It may be that to require the State to accord petitioner
his rights to a preliminary investigation and to bail at this
point, could turn out ultimately to be largely a ceremonial exercise. But
the Court is not compelled to speculate. And, in any case, it would not
be idle ceremony; rather it would be a celebration by the State of the
rights and liberties of its own people and a re-affirmation of its obligation
and determination to respect those rights and liberties.
ACCORDINGLY, the Court Resolved to GRANT the Petition for Review on
Certiorari. The Order of the trial court dated 17 July 1991 is hereby SET
ASIDE and NULLIFIED, and the Decision of the Court of Appeals dated 23
September 1991 hereby REVERSED.
The Office of the Provincial Prosecutor is hereby ORDERED to conduct
forthwith a preliminary investigation of the charge of murder against
petitioner Go, and to complete such preliminary investigation within a
period of fifteen (15) days from commencement thereof. The trial on the
merits of the criminal case in the Regional Trial Court shall be
SUSPENDED to await the conclusion of the preliminary investigation.
Meantime, petitioner is hereby ORDERED released forthwith upon posting
of a cash bail bond of One Hundred Thousand Pesos (P100,000.00). This
release shall be without prejudice to any lawful order that the trial court
may issue, should the Office of the Provincial Prosecutor move for
cancellation of bail at the conclusion of the preliminary investigation.
No pronouncement as to costs. This Decision is immediately executory.
SO ORDERED.
(Go y Tambunting v. Court of Appeals, G.R. No. 101837, [February 11,
|||

1992], 283 PHIL 24-58)


[G.R. No. 89139. August 2, 1990.]

ROMEO POSADAS y ZAMORA, petitioner, vs. THE HONORABLE


COURT OF APPEALS and THE PEOPLE OF THE
PHILIPPINES, respondents.

Rudy G. Agravante for petitioner.

SYLLABUS

1. REMEDIAL LAW; REVISED RULES ON CRIMINAL PROCEDURE; ARREST


WITHOUT WARRANT; SEC. 5, RULE 113 THEREOF. — From the foregoing
provision of law it is clear that an arrest without a warrant may be
effected by a peace officer or private person, among others, when in his
presence the person to be arrested has committed, is actually
committing, or is attempting to commit an offense; or when an offense
has in fact just been committed, and he has personal knowledge of the
facts indicating that the person arrested has committed it.
2. CONSTITUTIONAL LAW; BILL OF RIGHTS; WARRANTLESS SEARCH AND
SEIZURE; NOT INCIDENTAL TO A LAWFUL ARREST IN THE CASE AT BAR.
— The Solicitor General, in justifying the warrantless search and seizure
of the buri bag then carried by the petitioner, argued that when the two
policemen approached the petitioner, he was actually committing or had
just committed the offense of illegal possession of firearms and
ammunitions in the presence of the police officers and consequently the
search and seizure of the contraband was incidental to the lawful arrest
in accordance with Section 12, Rule 126 of the 1985 Rules on Criminal
Procedure. We disagree. At the time the peace officers in this case
identified themselves and apprehended the petitioner as he attempted to
flee they did not know that he had committed, or was actually
committing the offense of illegal possession of firearms and
ammunitions. They just suspected that he was hiding something in the
buri bag. They did now know what its contents were. The said
circumstances did not justify an arrest without a warrant.
3. ID.; ID.; ID.; CAN BE VALIDLY EFFECTED WITHOUT BEING PRECEDED
BY AN ARREST; CASE AT BAR. — However, there are many instances
where a warrant and seizure can be effected without necessarily being
preceded by an arrest, foremost of which is the "stop and search"
without a search warrant at military or police checkpoints, the
constitutionality or validity of which has been upheld by this Court
in Valmonte vs. de Villa. As between a warrantless search and seizure
conducted at military or police checkpoints and the search thereat in the
case at bar, there is no question that, indeed, the latter is more
reasonable considering that unlike in the former, it was effected on the
basis of a probable cause. The probable cause is that when the petitioner
acted suspiciously and attempted to flee with the buri bag there was a
probable cause that he was concealing something illegal in the bag and
it was the right and duty of the police officers to inspect the same. It is
too much indeed to require the police officers to search the bag in the
possession of the petitioner only after they shall have obtained a search
warrant for the purpose. Such an exercise may prove to be useless, futile
and much too late.

DECISION

GANCAYCO, J : p

The validity of a warrantless search on the person of petitioner is put


into issue in this case.
On October 16, 1986 at about 10:00 o'clock in the morning Pat. Ursicio
Ungab and Pat. Umbra Umpar, both members of the Integrated National
Police (INP) of the Davao Metrodiscom assigned with the Intelligence
Task Force, were conducting a surveillance along Magallanes Street,
Davao City. While they were within the premises of the Rizal Memorial
Colleges they spotted petitioner carrying a "buri" bag and they noticed
him to be acting suspiciously.
They approached the petitioner and identified themselves as members of
the INP. Petitioner attempted to flee but his attempt to get away was
thwarted by the two notwithstanding his resistance.
They then checked the "buri" bag of the petitioner where they found one
(1) caliber .38 Smith & Wesson revolver with Serial No. 770196 1 two (2)
rounds of live ammunition for a .38 caliber gun, a smoke (tear gas)
2

grenade a and two (2) live ammunitions for a .22 caliber gun. They
3 4

brought the petitioner to the police station for further investigation. In


the course of the same, the petitioner was asked to show the necessary
license or authority to possess firearms and ammunitions found in his
possession but he failed to do so. He was then taken to the Davao
Metrodiscom office and the prohibited articles recovered from him were
indorsed to M/Sgt. Didoy, the officer then on duty. He was prosecuted for
illegal possession of firearms and ammunitions in the Regional Trial
Court of Davao City wherein after a plea of not guilty and trial on the
merits a decision was rendered on October 8, 1987 finding petitioner
guilty of the offense charged as follows:
"WHEREFORE, in view of all the foregoing, this Court finds the
accused guilty beyond reasonable doubt of the offense charged.
It appearing that the accused was below eighteen (18) years old at
the time of the commission of the offense (Art. 68, par. 2), he is
hereby sentenced to an indeterminate penalty ranging from TEN
(10) YEARS and ONE (1) DAY of prision mayor to TWELVE (12) Years,
FIVE (5) months and Eleven (11) days of Reclusion Temporal, and to
pay the costs.
The firearm, ammunitions and smoke grenade are forfeited in favor
of the government and the Branch Clerk of Court is hereby directed
to turn over said items to the Chief, Davao Metrodiscom, Davao
City." 5

Not satisfied therewith the petitioner interposed an appeal to the Court


of Appeals wherein in due course a decision was rendered on February
23, 1989 affirming in toto the appealed decision with costs against the
petitioner. 6
Hence, the herein petition for review, the main thrust of which is that
there being no lawful arrest or search and seizure, the items which were
confiscated from the possession of the petitioner are inadmissible in
evidence against him. LexLib

The Solicitor General, in justifying the warrantless search of the buri bag
then carried by the petitioner, argues that under Section 12, Rule 126 of
the Rules of Court a person lawfully arrested may be searched for
dangerous weapons or anything used as proof of a commission of an
offense without a search warrant. It is further alleged that the arrest
without a warrant of the petitioner was lawful under the circumstances.
Section 5, Rule 113 of the 1985 Rules on Criminal Procedure provides as
follows:
"SEC. 5. Arrest without warrant; when lawful. — A peace officer or
a private person may, without a warrant, arrest a person:
(a) When in his presence, the person to be arrested has committed,
is actually committing, or is attempting to commit an offense;
(b) When an offense has in fact just been committed, and he has
personal knowledge of facts indicating that the person to be
arrested has committed it; and
(c) When the person to be arrested is a prisoner who has escaped
from a penal establishment or place where he is serving final
judgment or temporarily confined while his case is pending, or has
escaped while being transferred from one confinement to another.
In cases falling under paragraphs (a) and (b) hereof, the person
arrested without a warrant shall be forthwith delivered to the
nearest police station or jail, and he shall be proceeded against in
accordance with Rule 112, Section 7. (6a, 17a)"

From the foregoing provision of law it is clear that an arrest without a


warrant may be effected by a peace officer or private person, among
others, when in his presence the person to be arrested has committed, is
actually committing, or is attempting to commit an offense; or when an
offense has in fact just been committed, and he has personal knowledge
of the facts indicating that the person arrested has committed it.
The Solicitor General argues that when the two policemen approached
the petitioner, he was actually committing or had just committed the
offense of illegal possession of firearms and ammunitions in the
presence of the police officers and consequently the search and seizure
of the contraband was incidental to the lawful arrest in accordance
with Section 12, Rule 126 of the 1985 Rules on Criminal Procedure. We
disagree.
At the time the peace officers in this case identified themselves and
apprehended the petitioner as he attempted to flee they did not know
that he had committed, or was actually committing the offense of illegal
possession of firearms and ammunitions. They just suspected that he
was hiding something in the buri bag. They did now know what its
contents were. The said circumstances did not justify an arrest without a
warrant.llcd

However, there are many instances where a warrant and seizure can be
effected without necessarily being preceded by an arrest, foremost of
which is the "stop and search" without a search warrant at military or
police checkpoints, the constitutionality or validity of which has been
upheld by this Court in Valmonte vs. de Villa, 7 as follows:
"Petitioner Valmonte's general allegation to the effect that he had
been stopped and searched without a search warrant by the
military manning the checkpoints, without more, i.e., without
stating the details of the incidents which amount to a violation of
his right against unlawful search and seizure, is not sufficient to
enable the Court to determine whether there was a violation of
Valmonte's right against unlawful search and seizure. Not all
searches and seizures are prohibited. Those which are reasonable
are not forbidden. A reasonable search is not to be determined by
any fixed formula but is to be resolved according to the facts of
each case.
Where, for example, the officer merely draws aside the curtain of a
vacant vehicle which is parked on the public fair grounds, or simply
looks into a vehicle or flashes a light therein, these do not
constitute unreasonable search.
The setting up of the questioned checkpoints in Valenzuela (and
probably in other areas) may be considered as a security measure
to enable the NCRDC to pursue its mission of establishing effective
territorial defense and maintaining peace and order for the benefit
of the public. Checkpoints may also be regarded as measures to
thwart plots to destabilize the government in the interest of public
security. In this connection, the Court may take judicial notice of
the shift to urban centers and their suburbs of the insurgency
movement, so clearly reflected in the increased killings in cities of
police and military men by NPA "sparrow units," not to mention the
abundance of unlicensed firearms and the alarming rise in
lawlessness and violence in such urban centers, not all of which
are reported in media, most likely brought about by deteriorating
economic conditions — which all sum up to what one can rightly
consider, at the very least, as abnormal times. Between the
inherent right of the state to protect its existence and promote
public welfare and an individual's right against a warrantless
search which is however reasonably conducted, the former should
prevail.

True, the manning of checkpoints by the military is susceptible of


abuse by the men in uniform in the same manner that all
governmental power is susceptible of abuse. But, at the cost of
occasional inconvenience, discomfort and even irritation to the
citizen, the checkpoints during these abnormal times, when
conducted within reasonable limits, are part of the price we pay
for an orderly society and a peaceful community. (Emphasis
supplied)."

Thus, as between a warrantless search and seizure conducted at military


or police checkpoints and the search thereat in the case at bar, there is
no question that, indeed, the latter is more reasonable considering that
unlike in the former, it was effected on the basis of a probable cause. The
probable cause is that when the petitioner acted suspiciously and
attempted to flee with the buri bag there was a probable cause that he
was concealing something illegal in the bag and it was the right and duty
of the police officers to inspect the same.
It is too much indeed to require the police officers to search the bag in
the possession of the petitioner only after they shall have obtained a
search warrant for the purpose. Such an exercise may prove to be
useless, futile and much too late.
In People vs. CFI of Rizal, 8 this Court held as follows:
". . . In the ordinary cases where warrant is indispensably
necessary, the mechanics prescribed by the Constitution and
reiterated in the Rules of Court must be followed and satisfied. But
We need not argue that there are exceptions. Thus in the
extraordinary events where warrant is not necessary to effect a
valid search or seizure, or when the latter cannot be performed
except without warrant, what constitutes a reasonable or
unreasonable search or seizure becomes purely a judicial
question, determinable from the uniqueness of the circumstances
involved, including the purpose of the search or seizure, the
presence or absence of probable cause, the manner in which the
search and seizure was made, the place or thing searched and the
character of the articles procured."

The Court reproduces with approval the following disquisition of the


Solicitor General:cdphil

"The assailed search and seizure may still be justified as akin to a


"stop and frisk" situation whose object is either to determine the
identity of a suspicious individual or to maintain the status
quo momentarily while the police officer seeks to obtain more
information. This is illustrated in the case of Terry vs. Ohio, 392
U.S. 1 (1968). In this case, two men repeatedly walked past a store
window and returned to a spot where they apparently conferred
with a third man. This aroused the suspicion of a police officer. To
the experienced officer, the behavior of the men indicated that
they were sizing up the store for an armed robbery. When the
police officer approached the men and asked them for their names,
they mumbled a reply. Whereupon, the officer grabbed one of them,
spun him around and frisked him. Finding a concealed weapon in
one, he did the same to the other two and found another weapon.
In the prosecution for the offense of carrying a concealed weapon,
the defense of illegal search and seizure was put up. The United
States Supreme Court held that "a police officer may in
appropriate circumstances and in an appropriate manner approach
a person for the purpose of investigating possible criminal
behavior even though there is no probable cause to make an
arrest." In such a situation, it is reasonable for an officer rather
than simply to shrug his shoulder and allow a crime to occur, to
stop a suspicious individual briefly in order to determine his
identity or maintain the status quo while obtaining more
information. . . .
Clearly, the search in the case at bar can be sustained under the
exceptions heretofore discussed, and hence, the constitutional
guarantee against unreasonable searches and seizures has not
been violated." 9

WHEREFORE, the petition is DENIED with costs against petitioner.


SO ORDERED.
(Posadas y Zamora v. Court of Appeals, G.R. No. 89139, [August 2, 1990],
|||

266 PHIL 306-313)

[G.R. No. 87059. June 22, 1992.]


THE PEOPLE OF THE PHILIPPINES, plaintiff-
appellee, vs. ROGELIO MENGOTE Y TEJAS, accused-
appellant.

The Solicitor General for plaintiff-appellee.


Violeta C. Drilon counsel de oficio for accused-appellant.

SYLLABUS

1. CONSTITUTIONAL LAW; BILL OF RIGHTS; RIGHT AGAINST ILLEGAL


SEARCH AND SEIZURE; EVIDENCE OBTAINED IN VIOLATION THEREOF;
EFFECT; CASE AT BAR. — It is submitted in the Appellant's Brief that the
revolver should not have been admitted in evidence because of its illegal
seizure, no warrant therefor having been previously obtained. Neither
could it have been seized as an incident of a lawful arrest because the
arrest of Mengote was itself unlawful, having been also effected without
a warrant. The defense also contends that the testimony regarding the
alleged robbery in Danganan's house was irrelevant and should also have
been disregarded by the trial court. There is no question that evidence
obtained as a result of an illegal search or seizure is inadmissible in any
proceeding for any purpose. That is the absolute prohibition of Article III,
Section 3(2), of the Constitution. This is the celebrated exclusionary rule
based on the justification given by Judge Learned Hand that "only in
case the prosecution which itself controls the seizing officials, knows
that it cannot profit by their wrong will the wrong be repressed."
2. REMEDIAL LAW; CRIMINAL PROCEDURE; ARREST WITHOUT WARRANT;
WHEN LAWFUL; REQUISITES; NOT ESTABLISHED IN CASE AT BAR. — The
Solicitor General, while conceding the rule, maintains that it is not
applicable in the case at bar. His reason is that the arrest and search of
Mengote and the seizure of the revolver from him were lawful under Rule
113, Section 5, of the Rules of Court. We have carefully examined the
wording of this rule and cannot see how we can agree with the
prosecution. Par. (c) of Section 5 is obviously inapplicable as Mengote
was not an escapee from a penal institution when he was arrested. We
therefore confine ourselves to determining the lawfulness of his arrest
under either Par. (a) or Par. (b) of this section. Par. (a) requires that the
person be arrested (1) after he has committed or while he is actually
committing or is at least attempting to commit an offense, (2) in the
presence of the arresting officer. These requirements have not been
established in the case at bar. At the time of the arrest in question, the
accused-appellant was merely "looking from side to side" and "holding
his abdomen," according to the arresting officers themselves. There was
apparently no offense that had just been committed or was being
actually committed or at least being attempted by Mengote in their
presence. Par. (b) is no less applicable because its no less stringent
requirements have also not been satisfied. The prosecution has not
shown that at the time of Mengote's arrest an offense had in fact just
been committed and that the arresting officers had personal
knowledge of facts indicating that Mengote had committed it. All they
had was hearsay information from the telephone caller, and about a
crime that had yet to be committed.

DECISION

CRUZ, J :
p

Accused-appellant Rogelio Mengote was convicted of illegal possession


of firearms on the strength mainly of the stolen pistol found on his
person at the moment of his warrantless arrest. In this appeal, he pleads
that the weapon was not admissible as evidence against him because it
had been illegally seized and was therefore the fruit of the poisonous
tree. The Government disagrees. It insists that the revolver was validly
received in evidence by the trial judge because its seizure was incidental
to an arrest that was doubtless lawful even if admittedly without
warrant.
The incident occurred shortly before noon of August 8, 1987, after the
Western Police District received a telephone call from an informer that
there were three suspicious-looking persons at the corner of Juan Luna
and North Bay Boulevard in Tondo, Manila. A surveillance team of
plainclothesmen was forthwith dispatched to the place. As later narrated
at the trial by Patrolmen Rolando Mercado and Alberto Juan, 1 they there
saw two men "looking from side to side," one of whom was holding his
abdomen. They approached these persons and identified themselves as
policemen, whereupon the two tried to run away but were unable to
escape because the other lawmen had surrounded them. The suspects
were then searched. One of them, who turned out to be the accused-
appellant, was found with a .38 caliber Smith and Wesson revolver with
six live bullets in the chamber. His companion, later identified as Nicanor
Morellos, had a fan knife secreted in his front right pants pocket. The
weapons were taken from them. Mengote and Morellos were then turned
over to police headquarters for investigation by the Intelligence
Division.
LLpr

On August 11, 1987, the following information was filed against the
accused-appellant before the Regional Trial Court of Manila:
The undersigned accuses ROGELIO MENGOTE y TEJAS of a
violation of Presidential Decree No. 1866, committed as follows:
That on or about August 8, 1987, in the City of Manila, Philippines,
the said accused did then and there wilfully, unlawfully and
knowingly have in his possession and under his custody and
control a firearm, to wit:
one (1) cal. 38 "S & W" bearing
Serial No. 8720-T.
without first having secured the necessary license or permit
therefor from the proper authorities.
Besides the police officers, one other witness presented by the
prosecution was Rigoberto Danganan, who identified the subject weapon
as among the articles stolen from him during the robbery in his house in
Malabon on June 13, 1987. He pointed to Mengote as one of the robbers.
He had duly reported the robbery to the police, indicating the articles
stolen from him, including the revolver. 2 For his part, Mengote made no
effort to prove that he owned the firearm or that he was licensed to
possess it and claimed instead that the weapon had been "planted" on
him at the time of his arrest. 3
The gun, together with the live bullets and its holster, were offered as
Exhibits A, B and C and admitted over the objection of the defense. As
previously stated, the weapon was the principal evidence that led to
Mengote's conviction for violation of P.D. 1866. He was sentenced
to reclusion perpetua. 4
It is submitted in the Appellant's Brief that the revolver should not have
been admitted in evidence because of its illegal seizure, no warrant
therefor having been previously obtained. Neither could it have been
seized as an incident of a lawful arrest because the arrest of Mengote
was itself unlawful, having been also effected without a warrant. The
defense also contends that the testimony regarding the alleged robbery
in Danganan's house was irrelevant and should also have been
disregarded by the trial court.
The following are the pertinent provision of the Bill of Rights:
Sec. 2. The right of the people to be secure in their persons,
houses, papers, and effects against unreasonable searches and
seizures of whatever nature and for any purpose shall be
inviolable, and no search warrant or warrant of arrest shall issue
except upon probable cause to be determined personally by the
judge after examination under oath or affirmation of the
complainant and the witnesses he may produce, and particularly
describing the place to be searched and the persons or things to
be seized.
Sec. 3 (1). The privacy of communication and correspondence shall
be inviolable except upon lawful order of the court, or when public
safety or order requires otherwise as prescribed by law.
(2) Any evidence obtained in violation of this or the preceding
section shall be inadmissible for any purpose in any proceeding.
There is no question that evidence obtained as a result of an illegal
search or seizure is inadmissible in any proceeding for any purpose. That
is the absolute prohibition of Article III, Section 3(2), of the Constitution.
This is the celebrated exclusionary rule based on the justification given
by Judge Learned Hand that "only in case the prosecution, which itself
controls the seizing officials, knows that it cannot profit by their wrong
will the wrong be repressed."
The Solicitor General, while conceding the rule, maintains that it is not
applicable in the case at bar. His reason is that the arrest and search of
Mengote and the seizure of the revolver from him were lawful under Rule
113, Section 5, of the Rules of Court reading as follows:
Sec. 5. Arrest without warrant; when lawful. — A peace officer or
private person may without a warrant, arrest a person: Cdpr

(a) When, in his presence, the person to be arrested has


committed, is actually committing, or is attempting to commit an
offense;
(b) When an offense has in fact just been committed, and he has
personal knowledge of facts indicating that the person to be
arrested has committed it; and
(c) When the person to be arrested is a prisoner who has escaped
from a penal establishment or place where he is serving final
judgment or temporarily confined while his case is pending, or has
escaped while being transferred from one confinement to another.
In cases falling under paragraphs (a) and (b) hereof, the person
arrested without a warrant shall be forthwith delivered to the
nearest police station or jail, and he shall be proceeded against in
accordance with Rule 112, Section 7.
We have carefully examined the wording of this rule and cannot see how
we can agree with the prosecution.
Par. (c) of Section 5 is obviously inapplicable as Mengote was not an
escapee from a penal institution when he was arrested. We therefore
confine ourselves to determining the lawfulness of his arrest under
either Par. (a) or Par. (b) of this section.
Par. (a) requires that the person be arrested (1) after he has committed
or while he is actually committing or is at least attempting to commit an
offense, (2) in the presence of the arresting officer.
These requirements have not been established in the case at bar. At the
time of the arrest in question, the accused-appellant was merely "looking
from side to side" and "holding his abdomen," according to the arresting
officers themselves. There was apparently no offense that had just been
committed or was being actually committed or at least being attempted
by Mengote in their presence.
The Solicitor General submits that the actual existence of an offense
was not necessary as long as Mengote's acts "created a reasonable
suspicion on the part of the arresting officers and induced in them the
belief that an offense had been committed and that the accused-
appellant had committed it." The question is, What offense? What offense
could possibly have been suggested by a person "looking from side to
side" and "holding his abdomen" and in a place not exactly forsaken?
These are certainly not sinister acts. And the setting of the arrest made
them less so, if at all. It might have been different if Mengote had been
apprehended at an ungodly hour and in a place where he had no reason
to be, like a darkened alley at 3 o'clock in the morning. But he was
arrested at 11:30 in the morning and in a crowded street shortly after
alighting from a passenger jeep with his companion. He was not skulking
in the shadows but walking in the clear light of day. There was nothing
clandestine about his being on that street at that busy hour in the blaze
of the noonday sun.
On the other hand, there could have been a number of reasons, all of
them innocent, why his eyes were darting from side to side and he was
holding his abdomen. If they excited suspicion in the minds of the
arresting officers, as the prosecution suggests, it has nevertheless not
been shown what their suspicion was all about. In fact, the policemen
themselves testified that they were dispatched to that place only
because of the telephone call from the informer that there were
"suspicious-looking" persons in that vicinity who were about to commit a
robbery at North Bay Boulevard. The caller did not explain why he
thought the men looked suspicious nor did he elaborate on the impending
crime.LLpr

In the recent case of People v. Malmstedt, 5 the Court sustained the


warrantless arrest of the accused because there was a bulge in his waist
that excited the suspicion of the arresting officer and, upon inspection,
turned out to be a pouch containing hashish. In People v. Claudio, 6 the
accused boarded a bus and placed the buri bag she was carrying behind
the seat of the arresting officer while she herself sat in the seat before
him. His suspicion aroused, he surreptitiously examined the bag, which
he found to contain marijuana. He then and there made the warrantless
arrest and seizure that we subsequently upheld on the ground that
probable cause had been sufficiently established.
The case before us is different because there was nothing to support the
arresting officers' suspicion other than Mengote's darting eyes and his
hand on his abdomen. By no stretch of the imagination could it have been
inferred from these acts that an offense had just been committed, or was
actually being committed, or was at least being attempted in their
presence.
This case is similar to People v. Aminnudin, 7 where the Court held that
the warrantless arrest of the accused was unconstitutional. This was
effected while he was coming down a vessel, to all appearances no less
innocent than the other disembarking passengers. He had not committed
nor was he actually committing or attempting to commit an offense in
the presence of the arresting officers. He was not even acting
suspiciously. In short, there was no probable cause that, as the
prosecution incorrectly suggested, dispensed with the constitutional
requirement of a warrant.
Par. (b) is no less applicable because its no less stringent requirements
have also not been satisfied. The prosecution has not shown that at the
time of Mengote's arrest an offense had in fact just been committed and
that the arresting officers had personal knowledge of facts indicating
that Mengote had committed it. All they had was hearsay information
from the telephone caller, and about a crime that had yet to be
committed.
The truth is that they did not know then what offense, if at all, had been
committed and neither were they aware of the participation therein of
the accused-appellant. It was only later, after Danganan had appeared at
the police headquarters, that they learned of the robbery in his house
and of Mengote's supposed involvement therein.8 As for the illegal
possession or the firearm found on Mengote's person, the policemen
discovered this only after he had been searched and the investigation
conducted later revealed that he was not its owners nor was he licensed
to possess it.
Before these events, the peace officers had no knowledge even of
Mengote' identity, let alone the fact (or suspicion) that he was unlawfully
carrying a firearm or that he was involved in the robbery of Danganan's
house.
In the landmark case of People v. Burgos, 9 this Court declared:
Under Section 6(a) of Rule 113, the officer arresting a person who
has just committed, is committing, or is about to commit an
offense must have personal knowledgeof the fact. The offense
must also be committed in is presence or within his view. (Sayo v.
Chief of Police, 80 Phil. 859). (Emphasis supplied)
xxx xxx xxx
In arrests without a warrant under Section 6(b), however, it is not
enough that there is reasonable ground to believe that the person
to be arrested has committed a crime. A crime must in fact or
actually have been committed first. That a crime has actually been
committed is an essential precondition. It is not enough to suspect
that a crime may have been committed. The fact of the
commission of the offense must be undisputed. The test of
reasonable ground applies only to the identity of the perpetrator.
(Emphasis supplied)
This doctrine was affirmed in Alih v. Castro, 10 thus:
If the arrest was made under Rule 113, Section 5, of the Rules of
Court in connection with a crime about to be committed, being
committed, or just committed, what was that crime? There is no
allegation in the record of such a justification. Parenthetically, it
may be observed that under the Revised Rule 113, Section 5(b), the
officer making the arrest must have personal knowledge of the
ground therefor as stressed in the recent case of People v. Burgos.
(Emphasis supplied)
It would be a sad day, indeed, if any person could be summarily arrested
and searched just because he is holding his abdomen, even if it be
possibly because of a stomachache, or if a peace officer could clamp
handcuffs on any person with a shifty look on suspicion that he may have
committed a criminal act or is actually committing or attempting it. This
simply cannot be done in a free society. This is not a police state where
order is exalted over liberty or, worse, personal malice on the part of the
arresting officer may be justified in the name of security.
There is no need to discuss the other issues raised by the accused-
appellant as the ruling we here make is sufficient to sustain his
exoneration. Without the evidence of the firearm taken from him at the
time of his illegal arrest, the prosecution has lost its most important
exhibit and must therefore fail. The testimonial evidence against
Mengote (which is based on the said firearm) is not sufficient to prove
his guilt beyond reasonable doubt of the crime imputed to him.
We commend Atty. Violeta Calvo-Drilon for her able and spirited defense
of the accused-appellant not only in the brief but also in the reply brief,
which she did not have to file but did so just the same to stress the
constitutional rights of her client. The fact that she was acting only as a
counsel de oficio with no expectation of material reward makes her
representation even more commendable.
The Court feels that if the peace officers had been more mindful of the
provisions of the Bill of Rights, the prosecution of the accused-appellant
might have succeeded. As it happened, they allowed their over-
zealousness to get the better of them, resulting in their disregard of the
requirements of a valid search and seizure that rendered inadmissible
the vital evidence they had invalidly seized.
LLpr

This should be a lesson to other peace officers. Their impulsiveness may


be the very cause of the acquittal of persons who deserve to be
convicted, escaping the clutches of the law because, ironically enough,
it has not been observed by those who are supposed to enforce it.
WHEREFORE, the appealed decision is REVERSED and SET ASIDE. The
accused-appellant is ACQUITTED and ordered released immediately
unless he is validly detained for other offenses. No costs.
SO ORDERED.
(People v. Mengote y Tejas, G.R. No. 87059, [June 22, 1992], 285 PHIL
|||

642-651)

[G.R. No. 123595. December 12, 1997.]


SAMMY MALACAT y MANDAR, petitioner, vs. COURT OF
APPEALS, and PEOPLE OF THE PHILIPPINES, respondents.

Brillantes, Navarro, Jumamil, Arcilla, Escolin and Martinez Law


Offices for petitioner.

SYNOPSIS

In an information filed before the Regional Trial Court (RTC) of


Manila, petitioner was charged with violating Section 3 of Presidential
Decree No. 1866 for keeping, possessing and/or acquiring a hand
grenade, without first securing the necessary license and permit from
the proper authorities. On arraignment, petitioner, assisted by counsel de
officio, entered a plea of not guilty. After trial on the merits, the court a
quo found petitioner guilty of the crime of illegal possession of
explosives under the said law and sentenced him to suffer the penalty of
not less than seventeen years, four months and one day of reclusion
temporal as minimum and not more than thirty years of reclusion
perpetua, as maximum. Petitioner filed a notice of appeal indicating that
he was appealing to the Supreme Court. However, the record of the case
was forwarded to the Court of Appeals. In its decision, the Court of
Appeals affirmed the trial court's decision. Unable to accept conviction,
petitioner filed the instant petition alleging that the respondent court
erred in affirming the findings of the trial court that the warrantless
arrest of petitioner was valid and legal.
The Supreme Court finds the petition impressed with merit. For
purposes of determining appellate jurisdiction in criminal cases, the
maximum of the penalty, and not the minimum, is taken into account.
Since the maximum of the penalty is reclusion perpetua, the appeal
therefrom should have been to the Court and not the Court of Appeals.
Hence, the challenged decision immediately fall in jurisdictional grounds.
Additionally, the Court is convinced that the prosecution failed to
establish petitioner's guilt with moral certainty. First, serious doubts
surrounds the story of police office Yu that a grenade was found in and
seized from petitioner's possession. Notably, Yu did not identify in court
the grenade he allegedly seized. Second, if indeed petitioner had a
grenade with him and that two days earlier he was with the group about
to detonate an explosive at Plaza Miranda, it was then unnatural and
against common experience that petitioner simply stood in Plaza
Miranda in proximity to the police officers. Lastly, even assuming that
petitioner admitted possession of the grenade during his custodial
investigation police officer Serapio, such admission is inadmissible in
evidence for it was taken in palpable violation of Section 12(1) and (3) of
Article III of the Constitution. Verily, the search conducted on petitioner
could not have been one incidental to a lawful arrest. In view thereof, the
challenged decision of the Court of Appeals is set aside for lack of
jurisdiction and on ground of reasonable doubt.

SYLLABUS

1. REMEDIAL LAW; CRIMINAL PROCEDURE; APPEAL TO THE


SUPREME COURT; FOR PURPOSES OF DETERMINING APPELLATE
JURISDICTION OF THE SUPREME COURT IN CRIMINAL CASES, THE
MAXIMUM OF THE PENALTY IMPOSABLE BY LAW IS TAKEN INTO
ACCOUNT AND NOT THE MINIMUM. — For purposes of determining
appellate jurisdiction in criminal cases, the maximum of the penalty, and
not the minimum, is taken into account. Since the maximum of the
penalty is reclusion perpetua, the appeal therefrom should have been to
Supreme Court, and not the Court of Appeals, pursuant to Section 9(3) of
the Judiciary Reorganization Act of 1980 (B.P. Blg. 129), in relation to
Section 17 of the Judiciary Act of 1948, Section 5(2) of Article VIII of the
Constitution and Section 3(c) of Rule 122 of the Rules of Court. The term
life imprisonment as used in Section 9 of B.P. Blg. 129, the Judiciary Act
of 1948, and Section 3 of Rule 122 must be deemed to include reclusion
perpetua in view of Section 5(2) of Article VIII of the Constitution.
2. ID.; ID.; RIGHTS OF THE ACCUSED; PETITIONER'S ADMISSION OF
POSSESSION OF THE GRENADE DURING CUSTODIAL INVESTIGATION,
WITHOUT THE ASSISTANCE OF COUNSEL, INADMISSIBLE IN EVIDENCE.
— Even assuming that petitioner admitted possession of the grenade
during his custodial investigation by police officer Serapio, such
admission was inadmissible in evidence for it was taken in palpable
violation of Section 12(1) and (3) of Article III of the Constitution. Serapio
conducted the custodial investigation on petitioner the day following his
arrest. No lawyer was present and Serapio could not have requested a
lawyer to assist petitioner as no PAO lawyer was then available. Thus,
even if petitioner consented to the investigation and waived his rights to
remain silent and to counsel, the waiver was invalid as it was not in
writing, neither was it executed in the presence of counsel.
3. ID.; ID.; WARRANTLESS ARREST; LACK OF PERSONAL
KNOWLEDGE ON THE PART OF THE ARRESTING OFFICER OR AN OVERT
PHYSICAL ACT ON THE PART OF THE ACCUSED, INDICATING THAT THE
CRIME HAD JUST BEEN COMMITTED, OR WAS GOING TO BE COMMITTED,
MAKES THE SEARCH CONDUCTED ON THE ACCUSED NOT ONE
INCIDENTAL TO A LAWFUL ARREST; CASE AT BAR. — In a search
incidental to a lawful arrest, as the precedent arrest determines the
validity of the incidental search, the legality of the arrest is questioned in
a large majority of these cases, e.g., whether an arrest was merely used
as a pretext for conducting a search. In this instance, the law requires
that there first be a lawful arrest before a search can be made — the
process cannot be reversed. At bottom, assuming a valid arrest, the
arresting officer may search the person of the arrestee and the area
within which the latter may reach for a weapon or for evidence to
destroy, and seize any money or property found which was used in the
commission of the crime, or the fruit of the crime, or that which may be
used as evidence, or which might furnish the arrestee with the means of
escaping or committing violence. Here, there could have been no valid in
flagrante delicto or hot pursuit arrest preceding the search in light of the
lack of personal knowledge on the part of Yu, the arresting officer, or an
overt physical act, on the part of petitioner, indicating that a crime had
just been committed, was being committed or was going to be
committed. Having thus shown the invalidity of the warrantless arrest in
this case, plainly, the search conducted on petitioner could not have
been one incidental to a lawful arrest.
4. ID.; ID.; ID.; "STOP AND FRISK" AS A "LIMITED PROTECTIVE
SEARCH OF OUTER CLOTHING FOR WEAPONS"; JUSTIFICATION FOR AND
ALLOWABLE SCOPE THEREOF. — We now proceed to the justification for
and allowable scope of a "stop-and-frisk" as a "limited protective search
of outer clothing for weapons," as laid down in Terry, thus: We merely
hold today that where a police officer observes unusual conduct which
leads him reasonably to conclude in light of his experience that criminal
activity may be afoot and that the persons with whom he is dealing may
be armed and presently dangerous, where in the course of investigating
this behavior he identifies himself as a policeman and makes reasonable
inquiries, and where nothing in the initial stages of the encounter serves
to dispel his reasonable fear for his own or others' safety, he is entitled
for the protection of himself and others in the area to conduct a carefully
limited search of the outer clothing of such persons in an attempt to
discover weapons which might be used to assault him. Such a search is a
reasonable search under the Fourth Amendment . . . Other notable points
of Terry are that while probable cause is not required to conduct a "stop
and frisk," it nevertheless holds that mere suspicion or a hunch will not
validate a "stop and frisk." A genuine reason must exist, in light of the
police officer's experience and surrounding conditions, to warrant the
belief that the person detained has weapons concealed about him.
Finally, a "stop-and-frisk" serves a two-fold interest: the general interest
of effective crime prevention and detection, which underlies the
recognition that a police officer may, under appropriate circumstances
and in an appropriate manner, approach a person for purposes of
investigating possible criminal behavior even without probable cause;
and (2) the more pressing interest of safety and self-preservation which
permit the police officer to take steps to assure himself that the person
with whom he deals is not armed with a deadly weapon that could
unexpectedly and fatally be used against the police officer.
PANGANIBAN, J., separate opinion:
1. REMEDIAL LAW; CRIMINAL PROCEDURE; WARRANTLESS
ARRESTS AND SEARCHES; PROBABLE CAUSE, NEEDED FOR THEIR
VALIDITY. — After reviewing previous decisions on valid warrantless
arrests and searches, the Court underscored in sum that there was need
for facts providing probable cause, such as "the distinct odor of
marijuana, reports about drug transporting or positive identification by
informers, suspicious behavior, attempt to flee, [or] failure to produce
identification papers" to justify warrantless arrests and searches.
Likewise, urgency must attend such arrests and searches, as where
motor vehicles are used and there is great probability that the suspect
would get away before a warrant can be procured. Most important is that
the law enforcers must act immediately on the information received,
suspicions raised or probable cause established, and should effect the
arrests and searches without any delay.
2. ID.; ID.; ID.; ID.; THE STOP-AND-FRISK CONDUCTED BY ORDINARY
POLICEMEN AGAINST THE ACCUSED ON THE BASIS THAT HIS EYES
WERE MOVING VERY FAST AND THERE IS NO INDICATION THAT HE IS
HIDING EXPLOSIVE PARAPHERNALIA, IS ILLEGAL; CASE AT BAR. — As
in Manalili, lawmen were on surveillance in response to information that
a criminal activity could be in the offing at a specified place. The stark
difference, however, is that in Manalili, the reported activity involved drug
use and the lawmen belonged to the anti-narcotics group, while in the
instant case, the police on patrol were ordinary law enforcers on the
lookout for possible bombers. In the former, the law enforcers concerned
may be presumed to possess special knowledge and skill to detect the
physical features exhibited by a current drug user. Thus, when these
specially trained enforcers saw Manalili with reddish eyes and walking in
a wobbly manner characteristic of a person "high" on drugs per their
experience, and in a known hangout of drug users, there was sufficient
genuine reason to stop and frisk the suspect. It is well to emphasize that
under different circumstances such as where the policemen are not
specially-trained, and in common places where people ordinarily
converge, the same features displayed by a person will not normally
justify a warrantless arrest or search on him. The case before us
presents such a situation. The policemen merely observed that Malacat's
eyes were moving very fast. They did not notice any bulges or packets
about the bodies of these men indicating that they might be hiding
explosive paraphernalia. From their outward look, nothing suggested that
they were at the time armed and dangerous. Hence, there was no
justification for a stop-and-frisk.
3. ID.; ID.; ID.; ID.; ID.; DOCTRINE LAID DOWN IN PEOPLE VS.
MENGOTE, SQUARELY APPLICABLE IN CASE AT BAR. — Bolstering the
invalidity of the arrest and search of Malacat is People vs. Mengote,
another classic on the right against unreasonable searches and seizures.
Upon receiving a telephone call shortly before noon from an informer
that there were suspicious looking persons at a certain street corner in
Tondo, Manila, the Western Police District dispatched a surveillance team
to said place. There they saw two men "looking from side to side" with
one "holding his abdomen." The police approached them and identified
themselves, whereupon the two tried to flee but failed as other lawmen
surrounded them. The suspects were searched, and recovered from
Mengote was a fully loaded pistol; from his companion, a fan knife. The
Court ruled that the situation was not one calling for a lawful
warrantless search and arrest. As the Court, through Mr. Justice Isagani
A. Cruz, succinctly put it: "What offense could possibly have been
suggested by a person 'looking from side to side and 'holding his
abdomen' and in a place not exactly forsaken? Under our rule in Mengote,
petitioner's dubious act of moving his eyes swiftly from side to side can
in no way justify a stop-and-frisk. To convict a person on the basis only of
his queer behavior and to sentence him to practically a lifetime in prison
would simply be unfathomable. Nothing can be more wrong, unjust and
inhuman.

DECISION

DAVIDE, JR., J :
p

In an Information 1 filed on 30 August 1990, in Criminal Case No. 90-


86748 before the Regional Trial Court (RTC) of Manila, Branch 5,
petitioner Sammy Malacat y Mandar was charged with violating Section 3
of Presidential Decree No. 1866, 2 as follows: LLjur

That on or about August 27, 1990, in the City of Manila,


Philippines, the said accused did then and there willfully,
unlawfully and knowingly keep, possess and/or acquire a hand
grenade, without first securing the necessary license and/or
permit therefor from the proper authorities.
At arraignment 3 on 9 October 1990, petitioner, assisted by
counsel de officio, entered a plea of not guilty.
At pre-trial on 11 March 1991, petitioner admitted the existence of
Exhibits "A," "A-1," and "A-2," 4 while the prosecution admitted that the
police authorities were not armed with a search warrant nor warrant of
arrest at the time they arrested petitioner. 5
At trial on the merits, the prosecution presented the following
police officers as its witnesses: Rodolfo Yu, the arresting officer;
Josefino C. Serapio, the investigating officer; and Orlando Ramilo, who
examined the grenade.
Rodolfo Yu of the Western Police District, Metropolitan Police Force
of the Integrated National Police, Police Station No. 3, Quiapo, Manila,
testified that on 27 August 1990, at about 6:30 p.m., in response to bomb
threats reported seven days earlier, he was on foot patrol with three
other police officers (all of them in uniform) along Quezon Boulevard,
Quiapo, Manila, near the Mercury Drug store at Plaza Miranda. They
chanced upon two groups of Muslim-looking men, with each group,
comprised of three to four men, posted at opposite sides of the corner of
Quezon Boulevard near the Mercury Drug Store. These men were acting
suspiciously with "[t]their eyes . . . moving very fast." 6
Yu and his companions positioned themselves at strategic points
and observed both groups for about thirty minutes. The police officers
then approached one group of men, who then fled in different directions.
As the policemen gave chase, Yu caught up with and apprehended
petitioner. Upon searching petitioner, Yu found a fragmentation grenade
tucked inside petitioner's "front waist line." 7 Yu's companion, police
officer Rogelio Malibiran, apprehended Abdul Casan from whom a .38
caliber revolver was recovered. Petitioner and Casan were then brought
to Police Station No. 3 where Yu placed an "X" mark at the bottom of the
grenade and thereafter gave it to his commander. 8
On cross-examination, Yu declared that they conducted the foot
patrol due to a report that a group of Muslims was going to explode a
grenade somewhere in the vicinity of Plaza Miranda. Yu recognized
petitioner as the previous Saturday, 25 August 1990, likewise at Plaza
Miranda, Yu saw petitioner and 2 others attempt to detonate a grenade.
The attempt was aborted when Yu and other policemen chased petitioner
and his companions; however, the former were unable to catch any of the
latter. Yu further admitted that petitioner and Casan were merely
standing on the corner of Quezon Boulevard when Yu saw them on 27
August 1990. Although they were not creating a commotion, since they
were supposedly acting suspiciously, Yu and his companions approached
them. Yu did not issue any receipt for the grenade he allegedly recovered
from petitioner. 9
Josefino C. Serapio declared that at about 9:00 a.m. of 28 August
1990, petitioner and a certain Abdul Casan were brought in by Sgt.
Saquilla 10 for investigation. Forthwith, Serapio conducted the inquest of
the two suspects, informing them of their rights to remain silent and to
be assisted by competent and independent counsel. Despite Serapio's
advice, petitioner and Casan manifested their willingness to answer
questions even without the assistance of a lawyer. Serapio then took
petitioner's uncounselled confession (Exh. "E"), there being no PAO
lawyer available, wherein petitioner admitted possession of the grenade.
Thereafter, Serapio prepared the affidavit of arrest and booking sheet of
petitioner and Casan. Later, Serapio turned over the grenade to the
Intelligence and Special Action Division (ISAD) of the Explosive
Ordinance Disposal Unit for examination. 11
On cross-examination, Serapio admitted that he took petitioner's
confession knowing it was inadmissible in evidence. 12
Orlando Ramilo, a member of the Bomb Disposal Unit, whose
principal duties included, among other things, the examination of
explosive devices, testified that on 22 March 1991, he received a request
dated 19 March 1991 from Lt. Eduardo Cabrera and PO Diosdado Diotoy
for examination of a grenade. Ramilo then affixed an orange tag on the
subject grenade detailing his name, the date and time he received the
specimen. During the preliminary examination of the grenade, he "found
that [the] major components consisting of [a] high filler and fuse
assembly [were] all present," and concluded that the grenade was "[l]ive
and capable of exploding." On even date, he issued a certification stating
his findings, a copy of which he forwarded to Diotoy on 11 August 1991. 13
Petitioner was the lone defense witness. He declared that he
arrived in Manila on 22 July 1990 and resided at the Muslim Center in
Quiapo, Manila. At around 6:30 in the evening of 27 August 1990, he went
to Plaza Miranda to catch a breath of fresh air. Shortly after, several
policemen arrived and ordered all males to stand aside. The policemen
searched petitioner and two other men, but found nothing in their
possession. However, he was arrested with two others, brought to and
detained at Precinct No. 3, where he was accused of having shot a police
officer. The officer showed the gunshot wounds he allegedly sustained
and shouted at petitioner "[i]to ang tama mo sa akin." This officer then
inserted the muzzle of his gun into petitioner's mouth and said, "[y]ou are
the one who shot me."
Petitioner denied the charges and explained that he only recently
arrived in Manila. However, several other police officers mauled him,
hitting him with benches and guns. Petitioner was once again searched,
but nothing was found on him. He saw the grenade only in court when it
was presented. 14
The trial court ruled that the warrantless search and seizure of
petitioner was akin to a "stop and frisk," where a "warrant and seizure
can be effected without necessarily being preceded by an arrest" and
"whose object is either to maintain the status quo momentarily while the
police officer seeks to obtain more information." 15 Probable cause was
not required as it was not certain that a crime had been committed,
however, the situation called for an investigation, hence to require
probable cause would have been "premature." 16 The RTC emphasized
that Yu and his companions were "[c]onfronted with an emergency, in
which the delay necessary to obtain a warrant, threatens the destruction
of evidence" 17 and the officers "[h]ad to act in haste," as petitioner and
his companions were acting suspiciously, considering the time, place
and "reported cases of bombing." Further, petitioner's group suddenly ran
away in different directions as they saw the arresting officers approach,
thus "[i]t is reasonable for an officer to conduct a limited search, the
purpose of which is not necessarily to discover evidence of a crime, but
to allow the officer to pursue his investigation without fear of
violence." 18
The trial court then ruled that the seizure of the grenade from
petitioner was incidental to a lawful arrest, and since petitioner "[l]ater
voluntarily admitted such fact to the police investigator for the purpose
of bombing the Mercury Drug Store," concluded that sufficient evidence
existed to establish petitioner's guilt beyond reasonable doubt.
In its decision 19 dated 10 February 1994 but promulgated on 15
February 1994, the trial court thus found petitioner guilty of the crime of
illegal possession of explosives under Section 3 of P.D. No. 1866, and
sentenced him to suffer:
[T]he penalty of not less than SEVENTEEN (17) YEARS, FOUR
(4) MONTHS AND ONE (1) DAY OF RECLUSION TEMPORAL, as
minimum, and not more than THIRTY (30) YEARS OF RECLUSION
PERPETUA, as maximum.
On 18 February 1994, petitioner filed a notice of appeal 20 indicating
that he was appealing to this Court. However, the record of the case was
forwarded to the Court of Appeals which docketed it as CA-G.R. CR No.
15988 and issued a notice to file briefs. 21
In his Appellant's Brief 22 filed with the Court of Appeals, petitioner
asserted that:
1. THE LOWER COURT ERRED IN HOLDING THAT THE SEARCH
UPON THE PERSON OF ACCUSED-APPELLANT AND THE
SEIZURE OF THE ALLEGED HANDGRENADE FROM HIM "WAS
AN APPROPRIATE INCIDENT TO HIS ARREST."
2. THE LOWER COURT ERRED IN ADMITTING AS EVIDENCE
AGAINST ACCUSED-APPELLANT THE HANDGRENADE
ALLEGEDLY SEIZED FROM HIM AS IT WAS A PRODUCT OF AN
UNREASONABLE AND ILLEGAL SEARCH.
In sum, petitioner argued that the warrantless arrest was invalid
due to absence of any of the conditions provided for in Section 5 of Rule
113 of the Rules of Court, citing People vs. Mengote. 23 As such, the
search was illegal, and the hand grenade seized, inadmissible in
evidence.
In its Brief for the Appellee, the Office of the Solicitor General
agreed with the trial court and prayed that its decision be affirmed in
toto. 24
In its decision of 24 January 1996, 25 the Court of Appeals affirmed
the trial court, noting, first, that petitioner abandoned his original theory
before the court a quothat the grenade was "planted" by the police
officers; and second, the factual finding of the trial court that the
grenade was seized from petitioner's possession was not raised as an
issue. Further, respondent court focused on the admissibility in evidence
of Exhibit "D," the hand grenade seized from petitioner. Meeting the issue
squarely, the Court of Appeals ruled that the arrest was lawful on the
ground that there was probable cause for the arrest as petitioner was
"attempting to commit an offense," thus:
We are at a loss to understand how a man, who was in
possession of a live grenade and in the company of other
suspicious character[s] with unlicensed firearm[s] lurking in Plaza
Miranda at a time when political tension ha[d] been enkindling a
series of terroristic activities, [can] claim that he was not
attempting to commit an offense. We need not mention that Plaza
Miranda is historically notorious for being a favorite bomb site
especially during times of political upheaval. As the mere
possession of an unlicensed grenade is by itself an offense,
Malacat's posture is simply too preposterous to inspire belief.
In so doing, the Court of Appeals took into account petitioner's
failure to rebut the testimony of the prosecution witnesses that they
received intelligence reports of a bomb threat at Plaza Miranda; the fact
that PO Yu chased petitioner two days prior to the latter's arrest, or on
27 August 1990; and that petitioner and his companions acted
suspiciously, the "accumulation" of which was more than sufficient to
convince a reasonable man that an offense was about to be committed.
Moreover, the Court of Appeals observed:
The police officers in such a volatile situation would be guilty
of gross negligence and dereliction of duty, not to mention of gross
incompetence, if they [would] first wait for Malacat to hurl the
grenade, and kill several innocent persons while maiming
numerous others, before arriving at what would then be an assured
but moot conclusion that there was indeed probable cause for an
arrest. We are in agreement with the lower court in saying that the
probable cause in such a situation should not be the kind of proof
necessary to convict, but rather the practical considerations of
everyday life on which a reasonable and prudent mind, and not
legal technicians, will ordinarily act.
Finally, the Court of Appeals held that the rule laid down in People
v. Mengote, 26 which petitioner relied upon, was inapplicable in light of
"[c]rucial differences," to wit:
[In Mengote] the police officers never received any
intelligence report that someone [at] the corner of a busy street
[would] be in possession of a prohibited article. Here the police
officers were responding to a [sic] public clamor to put a check on
the series of terroristic bombings in the Metropolis, and, after
receiving intelligence reports about a bomb threat aimed at the
vicinity of the historically notorious Plaza Miranda, they conducted
foot patrols for about seven days to observe suspicious movements
in the area. Furthermore, in Mengote, the police officers [had] no
personal knowledge that the person arrested has committed, is
actually committing, or is attempting to commit an offense. Here,
PO3 Yu [had] personal knowledge of the fact that he chased
Malacat in Plaza Miranda two days before he finally succeeded in
apprehending him.
Unable to accept his conviction, petitioner forthwith filed the
instant petition and assigns the following errors:
1. THE RESPONDENT COURT ERRED IN AFFIRMING THE FINDING
OF THE TRIAL COURT THAT THE WARRANTLESS ARREST OF
PETITIONER WAS VALID AND LEGAL.
2. THE RESPONDENT COURT ERRED IN HOLDING THAT THE
RULING IN PEOPLE VS. MENGOTE DOES NOT FIND
APPLICATION IN THE INSTANT CASE.
In support thereof, petitioner merely restates his arguments below
regarding the validity of the warrantless arrest and search, then
disagrees with the finding of the Court of Appeals that he was
"attempting to commit a crime," as the evidence for the prosecution
merely disclosed that he was "standing at the corner of Plaza Miranda
and Quezon Boulevard" with his eyes "moving very fast" and "looking at
every person that come (sic) nearer (sic) to them." Finally, petitioner
points out the factual similarities between his case and that of People
v. Mengote to demonstrate that the Court of Appeals miscomprehended
the latter.
In its Comment, the Office of the Solicitor General prays that we
affirm the challenged decision.
For being impressed with merit, we resolved to give due course to
the petition.
The challenged decision must immediately fall on jurisdictional
grounds. To repeat, the penalty imposed by the trial court was:
[N]ot less than SEVENTEEN (17) YEARS, FOUR (4) MONTHS
AND ONE (1) DAY OF RECLUSION TEMPORAL, as minimum, and not
more than THIRTY (30) YEARS OFRECLUSION PERPETUA, as
maximum.
The penalty provided by Section 3 of P.D. No. 1866 upon any person
who shall unlawfully possess grenades is reclusion temporal in its
maximum period toreclusion perpetua.
For purposes of determining appellate jurisdiction in criminal cases,
the maximum of the penalty, and not the minimum, is taken into account.
Since the maximum of the penalty is reclusion perpetua, the appeal
therefrom should have been to us, and not the Court of Appeals, pursuant
to Section 9(3) of the Judiciary Reorganization Act of 1980 (B.P. Blg.
129), 27 in relation to Section 17 of the Judiciary Act of 1948, 28 Section
5(2) of Article VIII of the Constitution 29 and Section 3(c) of Rule 122 of
the Rules of Court. 30 The term "life imprisonment" as used in Section 9
of B.P. Blg. 129, the Judiciary Act of 1948, and Section 3 of Rule 122 must
be deemed to include reclusion perpetua in view of Section 5(2) of Article
VIII of the Constitution.
Petitioner's Notice of Appeal indicated that he was appealing from
the trial court's decision to this Court, yet the trial court transmitted the
record to the Court of Appeals and the latter proceeded to resolve the
appeal.
We then set aside the decision of the Court of Appeals for having
been rendered without jurisdiction, and consider the appeal as having
been directly brought to us, with the petition for review as petitioner's
Brief for the Appellant, the comment thereon by the Office of the Solicitor
General as the Brief for the Appellee and the memoranda of the parties
as their Supplemental Briefs.
Deliberating on the foregoing pleadings, we find ourselves
convinced that the prosecution failed to establish petitioner's guilt with
moral certainty.
First, serious doubt surrounds the story of police officer Yu that a
grenade was found in and seized from petitioner's possession. Notably,
Yu did not identify, in court, the grenade he allegedly seized. According
to him, he turned it over to his commander after putting an "X" mark at
its bottom; however, the commander was not presented to corroborate
this claim. On the other hand, the grenade presented in court and
identified by police officer Ramilo referred to what the latter received
from Lt. Eduardo Cabrera and police officer Diotoy not immediately after
petitioner's arrest, but nearly seven (7) months later or on 19 March
1991; further, there was no evidence whatsoever that what Ramilo
received was the very same grenade seized from petitioner. In his
testimony, Yu never declared that the grenade passed on to Ramilo was
the grenade the former confiscated from petitioner. Yu did not, and was
not made to, identify the grenade examined by Ramilo, and the latter did
not claim that the grenade he examined was that seized from petitioner.
Plainly, the law enforcement authorities failed to safeguard and preserve
the chain of evidence so crucial in cases such as these.
Second, if indeed petitioner had a grenade with him, and that two
days earlier he was with a group about to detonate an explosive at Plaza
Miranda, and Yu and his fellow officers chased, but failed to arrest them,
then considering that Yu and his three fellow officers were in uniform and
therefore easily cognizable as police officers, it was then unnatural and
against common experience that petitioner simply stood there in
proximity to the police officers. Note that Yu observed petitioner for
thirty minutes and must have been close enough to petitioner in order to
discern petitioner's eyes "moving very fast."
Finally, even assuming that petitioner admitted possession of the
grenade during his custodial investigation by police officer Serapio, such
admission was inadmissible in evidence for it was taken in palpable
violation of Section 12(1) and (3) of Article III of the Constitution, which
provide as follows:
SEC. 12 (1). Any person under investigation for the
commission of an offense shall have the right to be informed of his
right to remain silent and to have competent and independent
counsel preferably of his own choice. If the person cannot afford
the services of counsel, he must be provided with one. These
rights cannot be waived except in writing and in the presence of
counsel.
xxx xxx xxx
(3) Any confession or admission obtained in violation of this
or Section 17 hereof shall be inadmissible in evidence against him.
Serapio conducted the custodial investigation on petitioner the day
following his arrest. No lawyer was present and Serapio could not have
requested a lawyer to assist petitioner as no PAO lawyer was then
available. Thus, even if petitioner consented to the investigation and
waived his rights to remain silent and to counsel, the waiver was invalid
as it was not in writing, neither was it executed in the presence of
counsel.
Even granting ex gratia that petitioner was in possession of a
grenade, the arrest and search of petitioner were invalid, as will be
discussed below.
The general rule as regards arrests, searches and seizures is that a
warrant is needed in order to validly effect the same. 31 The
Constitutional prohibition against unreasonable arrests, searches and
seizures refers to those effected without a validly issued
warrant, 32 subject to certain exceptions. As regards valid warrantless
arrests, these are found in Section 5, Rule 113 of the Rules of Court,
which reads, in part:
Sec. 5. Arrest, without warrant; when lawful. — A peace
officer or a private person may, without a warrant, arrest a person:
(a) When, in his presence, the person to be arrested has
committed, is actually committing, or is attempting to
commit an offense;
(b) When an offense has in fact just been committed, and he
has personal knowledge of facts indicating that the
person to be arrested has committed it; and
(c) When the person to be arrested is a prisoner who has
escaped . . .
A warrantless arrest under the circumstances contemplated under
Section 5(a) has been denominated as one " in flagrante delicto," while
that under Section 5(b) has been described as a "hot pursuit" arrest.
Turning to valid warrantless searches, they are limited to the
following: (1) customs searches; (2) search of moving vehicles; (3)
seizure of evidence in plain view; (4) consent searches; 33 (5) a search
incidental to a lawful arrest; 34 and (6) a "stop and frisk." 35
In the instant petition, the trial court validated the warrantless
search as a "stop and frisk" with "the seizure of the grenade from the
accused [as] an appropriate incident to his arrest," hence necessitating
a brief discussion on the nature of these exceptions to the warrant
requirement.
At the outset, we note that the trial court confused the concepts of
a "stop-and-frisk" and of a search incidental to a lawful arrest. These two
types of warrantless searches differ in terms of the requisite quantum of
proof before they may be validly effected and in their allowable scope.
In a search incidental to a lawful arrest, as the precedent arrest
determines the validity of the incidental search, the legality of the arrest
is questioned in a large majority of these cases, e.g., whether an arrest
was merely used as a pretext for conducting a search. 36 In this instance,
the law requires that there first be a lawful arrest before a search can be
made — the process cannot be reversed. 37 At bottom, assuming a valid
arrest, the arresting officer may search the person of the arrestee and
the area within which the latter may reach for a weapon or for evidence
to destroy, and seize any money or property found which was used in the
commission of the crime, or the fruit of the crime, or that which may be
used as evidence, or which might furnish the arrestee with the means of
escaping or committing violence. 38
Here, there could have been no valid in flagrante delicto or hot
pursuit arrest preceding the search in light of the lack of personal
knowledge on the part of Yu, the arresting officer, or an overt physical
act, on the part of petitioner, indicating that a crime had just been
committed, was being committed or was going to be committed.
Having thus shown the invalidity of the warrantless arrest in this
case, plainly, the search conducted on petitioner could not have been
one incidental to a lawful arrest.
We now proceed to the justification for and allowable scope of a
"stop-and-frisk" as a "limited protective search of outer clothing for
weapons," as laid down inTerry; thus:
We merely hold today that where a police officer observes
unusual conduct which leads him reasonably to conclude in light of
his experience that criminal activity may be afoot and that the
persons with whom he is dealing may be armed and presently
dangerous, where in the course of investigating this behavior he
identifies himself as a policeman and makes reasonable inquiries,
and where nothing in the initial stages of the encounter serves to
dispel his reasonable fear for his own or others' safety, he is
entitled for the protection of himself and others in the area to
conduct a carefully limited search of the outer clothing of such
persons in an attempt to discover weapons which might be used to
assault him. Such a search is a reasonable search under the Fourth
Amendment . . . 39
Other notable points of Terry are that while probable cause is not
required to conduct a "stop and frisk," 40 it nevertheless holds that mere
suspicion or a hunch will not validate a "stop and frisk." A genuine
reason must exist, in light of the police officer's experience and
surrounding conditions, to warrant the belief that the person detained
has weapons concealed about him. 41 Finally, a "stop-and-frisk" serves a
two-fold interest: (1) the general interest of effective crime prevention
and detection, which underlies the recognition that a police officer may,
under appropriate circumstances and in an appropriate manner,
approach a person for purposes of investigating possible criminal
behavior even without probable cause; and (2) the more pressing interest
of safety and self-preservation which permit the police officer to take
steps to assure himself that the person with whom he deals is not armed
with a deadly weapon that could unexpectedly and fatally be used
against the police officer.
Here, there are at least three (3) reasons why the "stop-and-frisk"
was invalid:
First, we harbor grave doubts as to Yu's claim that petitioner was a
member of the group which attempted to bomb Plaza Miranda two days
earlier. This claim is neither supported by any police report or record nor
corroborated by any other police officer who allegedly chased that group.
Aside from impairing Yu's credibility as a witness, this likewise
diminishes the probability that a genuine reason existed so as to arrest
and search petitioner. If only to further tarnish the credibility of Yu's
testimony, contrary to his claim that petitioner and his companions had
to be chased before being apprehended, the affidavit of arrest (Exh. "A")
expressly declares otherwise, i.e., upon arrival of five (5) other police
officers, petitioner and his companions were "immediately collared."
Second, there was nothing in petitioner's behavior or conduct which
could have reasonably elicited even mere suspicion other than that his
eyes were "moving very fast" — an observation which leaves us
incredulous since Yu and his teammates were nowhere near petitioner
and it was already 6:30 p.m., thus presumably dusk. Petitioner and his
companions were merely standing at the corner and were not creating
any commotion or trouble, as Yu explicitly declared on cross-
examination: cdrep

Q And what were they doing?


A They were merely standing.
Q You are sure of that?
A Yes, sir.
Q And when you saw them standing, there were nothing or they did
not create any commotion?
A None, sir.
Q Neither did you see them create commotion?
A None, sir. 42
Third, there was at all no ground, probable or otherwise, to believe
that petitioner was armed with a deadly weapon. None was visible to Yu,
for as he admitted, the alleged grenade was "discovered" " inside the
front waistline" of petitioner, and from all indications as to the distance
between Yu and petitioner, any telltale bulge, assuming that petitioner
was indeed hiding a grenade, could not have been visible to Yu. In fact,
as noted by the trial court:
When the policemen approached the accused and his companions,
they were not yet aware that a handgrenade was tucked inside his
waistline. They did not see any bulging object in [sic] his person. 43
What is unequivocal then in this case are blatant violations of
petitioner's rights solemnly guaranteed in Sections 2 and 12(1) of Article
III of the Constitution.
WHEREFORE, the challenged decision of the Seventeenth Division of
the Court of Appeals in CA-G.R. CR No. 15988 is SET ASIDE for lack of
jurisdiction on the part of said Court and, on ground of reasonable doubt,
the decision of 10 February 1994 of Branch 5 of the Regional Trial Court
of Manila is REVERSED and petitioner SAMMY MALACAT y MANDAR is
hereby ACQUITTED and ORDERED immediately released from detention,
unless his further detention is justified for any other lawful cause.
SO ORDERED.
(Malacat y Mandar v. Court of Appeals, G.R. No. 123595, [December 12,
|||

1997], 347 PHIL 462-492)


[G.R. No. 74869. July 6, 1988.]

PEOPLE OF THE PHILIPPINES, plaintiff-appellee, vs. IDEL


AMINNUDIN y AHNI, defendant-appellant.

The Solicitor General, for plaintiff-appellee.


Herminio T. Llariza counsel de-officio, for defendant-appellant.

SYLLABUS

1. CONSTITUTIONAL LAW; BILL OF RIGHTS; RIGHTS AGAINST


UNREASONABLE SEARCHES AND SEIZURES; WARRANTLESS ARREST
AND SEIZURE BASED ON AN INFORMER'S TIP, AT A TIME WHEN
ACCUSED WAS NOT COMMITTING A CRIME, ILLEGAL; EVIDENCE
OBTAINED, INADMISSIBLE. — Where it is not disputed that the PC
officers had no warrant when they arrested Aminnudin while he was
descending the gangplank of the M/V Wilcon 9 and seized the bag he
was carrying, and that their only justification was the tip they had
earlier received from a reliable and regular informer who reported to
them that Aminnudin was arriving in Iloilo by boat with marijuana, the
search was not an incident of a lawful arrest because there was no
warrant of arrest and warrantless arrest did not come under the
exceptions allowed by the Rules of Court. Hence, the warrantless
search was also illegal and the evidence obtained was inadmissible.
2. ID.; ID.; ID.; ID.; NO URGENCY COULD BE INVOKED IN PRESENT
CASE TO DISPENSE WITH OBTENTION OF ARREST AND SEARCH
WARRANT. — The present case presented no such urgency. From the
conflicting declarations of the PC witnesses, it is clear that they had
at least two days within which they could have obtained a warrant to
arrest and search Aminnudin who was coming to Iloilo on the M/V
Wilcon 9. His name was known. The vehicle was identified. The date of
its arrival was certain. And from the information they had received,
they could have persuaded a judge that there was probable cause,
indeed, to justify the issuance of a warrant. Yet they did nothing. No
effort was made to comply with the law. The Bill of Rights was ignored
altogether because the PC lieutenant who was the head of the
arresting team, had determined on his own authority that a "search
warrant was not necessary."
3. ID.; ID.; ID.; ID.; ACCUSED IN CASE AT BAR WAS NOT
COMMITTING A CRIME WHEN HE WAS ARRESTED. — In the case at bar,
the accused-appellant was not, at the moment of his arrest,
committing a crime nor was it shown that he was about to do so or
that he had just done so. What he was doing was descending the
gangplank of the M/V Wilcon 9 and there was no outward indication
that called for his arrest. To all appearances, he was like any of the
other passengers innocently disembarking from the vessel. It was only
when the informer pointed to him as the carrier of the marijuana that
he suddenly became suspect and so subject to apprehension. It was
the furtive finger that triggered his arrest. The identification by the
informer was the probable cause as determined by the officers (and
not a judge) that authorized them to pounce upon Aminnudin and
immediately arrest him.
AQUINO, J., Dissenting:
CONSTITUTIONAL LAW; BILL OF RIGHTS; RIGHT AGAINST
UNREASONABLE SEARCHES AND SEIZURES; ARREST AT TIME OF
COMMISSION OF CRIME IS LAWFUL; SEARCH LIKEWISE LAWFUL.— I
hold that the accused was caught in flagrante, for he was carrying
marijuana leaves in his bag at the moment of his arrest. He was not
"innocently disembarking from the vessel." The unauthorized
transportation of marijuana (Indian hemp), which is a prohibited drug,
is a crime. (Sec. 4, Rep. Act No. 6425). Since he was committing a
crime, his arrest could be lawfully effected without a warrant (Sec. 6-
a, Rule 113, Rules of Court), and the search of his bag (which yielded
the marijuana leaves) without a search warrant was also lawful (Sec.
12, Rule 126, Rules of Court).

DECISION

CRUZ, J :
p

The accused-appellant claimed his business was selling watches


but he was nonetheless arrested, tried and found guilty of illegally
transporting marijuana. The trial court, disbelieving him, held it was
high time to put him away and sentenced him to life imprisonment plus
a fine of P20,000.00. 1
Idel Aminnudin was arrested on June 25, 1984, shortly after
disembarking from the M/V Wilcon 9 at about 8:30 in the evening, in
Iloilo City. The PC officers who were in fact waiting for him simply
accosted him, inspected his bag and finding what looked liked
marijuana leaves took him to their headquarters for investigation. The
two bundles of suspect articles were confiscated from him and later
taken to the NBI laboratory for examination. When they were verified
as marijuana leaves, an information for violation of the Dangerous
Drugs Act was filed against him. 2 Later, the information was amended
to include Farida Ali y Hassen, who had also been arrested with him
that same evening and likewise investigated. 3 Both were arraigned
and pleaded not guilty. 4 Subsequently, the fiscal filed a motion to
dismiss the charge against Ali on the basis of a sworn statement of
the arresting officers absolving her after a "thorough
investigation." 5 The motion was granted, and trial proceeded only
against the accused-appellant, who was eventually convicted. 6
According to the prosecution, the PC officers had earlier received
a tip from one of their informers that the accused-appellant was on
board a vessel bound for Iloilo City and was carrying marijuana. 7 He
was identified by name. 8 Acting on this tip, they waited for him in the
evening of June 25, 1984, and approached him as he descended from
the gangplank after the informer had pointed to him. 9 They detained
him and inspected the bag he was carrying. It was found to contain
three kilos of what were later analyzed as marijuana leaves by an NBI
forensic examiner, 10 who testified that she conducted microscopic,
chemical and chromatographic tests on them. On the basis of this
finding, the corresponding charge was then filed against Aminnudin.
In his defense, Aminnudin disclaimed the marijuana, averring that
all he had in his bag was his clothing consisting of a jacket, two shirts
and two pairs of pants.11 He alleged that he was arbitrarily arrested
and immediately handcuffed. His bag was confiscated without a
search warrant. At the PC headquarters, he was manhandled to force
him to admit he was carrying the marijuana, the investigator hitting
him with a piece of wood in the chest and arms even as he parried the
blows while he was still handcuffed. 12 He insisted he did not even
know what marijuana looked like and that his business was selling
watches and sometimes cigarettes. 13 He also argued that the
marijuana he was alleged to have been carrying was not properly
identified and could have been any of several bundles kept in the stock
room of the PC headquarters. 14
The trial court was unconvinced, noting from its own examination
of the accused that he claimed to have come to Iloilo City to sell
watches but carried only two watches at the time, traveling from Jolo
for that purpose and spending P107.00 for fare, not to mention his
other expenses. 15 Aminnudin testified that he kept the two watches in
a secret pocket below his belt but, strangely, they were not discovered
when he was bodily searched by the arresting officers nor were they
damaged as a result of his manhandling. 1 6 He also said he sold one of
the watches for P400.00 and gave away the other, although the
watches belonged not to him but to his cousin, 17 to a friend whose full
name he said did not even know. 18 The trial court also rejected his
allegations of maltreatment, observing that he had not sufficiently
proved the injuries sustained by him. 19
There is no justification to reverse these factual findings,
considering that it was the trial judge who had immediate access to
the testimony of the witnesses and had the opportunity to weigh their
credibility on the stand. Nuances of tone or voice, meaningful pauses
and hesitation, flush of face and dart of eyes, which may reveal the
truth or expose the lie, are not described in the impersonal record. But
the trial judge sees all of this, discovering for himself the truant fact
amidst the falsities.
The only exception we may make in this case is the trial court's
conclusion that the accused-appellant was not really beaten up
because he did not complain about it later nor did he submit to a
medical examination. That is hardly fair or realistic. It is possible
Aminnudin never had that opportunity as he was at that time under
detention by the PC authorities and in fact has never been set free
since he was arrested in 1984 and up to the present. No bail has been
allowed for his release.
There is one point that deserves closer examination, however,
and it is Aminnudin's claim that he was arrested and searched without
warrant, making the marijuana allegedly found in his possession
inadmissible in evidence against him under the Bill of Rights. The
decision did not even discuss this point. For his part, the Solicitor
General dismissed this after an all-too-short argument that the arrest
of Aminnudin was valid because it came under Rule 113, Section 6(b)
of the Rules of Court on warrantless arrests. This made the search
also valid as incidental to a lawful arrest.
It is not disputed, and in fact it is admitted by the PC officers who
testified for the prosecution, that they had no warrant when they
arrested Aminnudin and seized the bag he was carrying. Their only
justification was the tip they had earlier received from a reliable and
regular informer who reported to them that Aminnudin was arriving in
Iloilo by boat with marijuana. Their testimony varies as to the time
they received the tip, one saying it was two days before the
arrest, 20another two weeks 21 and a third "weeks before June
25." 22 On this matter, we may prefer the declaration of the chief of the
arresting team, Lt. Cipriano Querol, Jr., who testified as follow:
"Q You mentioned an intelligence report, you mean with respect to
the coming of Idel Aminnudin on June 25, 1984?
"A Yes, sir.
"Q When did you receive this intelligence report?
"A Two days before June 25, 1984 and it was supported by reliable
sources.
"Q Were you informed of the coming of the Wilcon 9 and the
possible trafficking of marijuana leaves on that date?
"A Yes, sir, two days before June 25, 1984 when we received this
information from that particular informer, prior to June 25,
1984 we have already reports of the particular operation
which was being participated by Idel Aminnudin.
"Q You said you received an intelligence report two days before
June 25, 1984 with respect to the coming of Wilcon 9?
"A Yes, sir.
"Q Did you receive any other report aside from this intelligence
report?
"A Well, I have received also other reports but not pertaining to the
coming of Wilcon 9. For instance, report of illegal gambling
operation.
"COURT:
"Q Previous to that particular information which you said two days
before June 25, 1984, did you also receive any report
regarding the activities of Idel Aminnudin?
"A Previous to June 25, 1984 we received reports on the activities
of Idel Aminnudin.
"Q What were those activities?
"A Purely marijuana trafficking.
"Q From whom did you get that information?
"A It came to my hand which was written in a required sheet of
information, maybe for security reason and we cannot
identify the person.
"Q But you received it from your regular informer?
"A Yes, sir.
"ATTY. LLARIZA:
"Q Previous to June 25, 1984, you were more or less sure that Idel
Aminnudin is coming with drugs?
"A Marijuana, sir.
"Q And this information respecting Idel Aminnudin's coming to
Iloilo with marijuana was received by you many days before
you received the intelligence report in writing?
"A Not a report of the particular coming of Aminnudin but his
activities.
"Q You only knew that he was coming on June 25, 1984 two days
before?
"A Yes, sir.
"Q You mean that before June 23, 1984 you did not know that
Aminnudin was coming?
"A Before June 23, 1984, I, in my capacity, did not know that he
was coming but on June 23, 1984 that was the time when I
received the information that he was coming. Regarding the
reports on his activities, we have reports that he has already
consummated the act of selling and shipping marijuana stuff.
"COURT:
"Q And as a result of that report, you put him under surveillance?
"A Yes, sir.
"Q In the intelligence report, only the name of Idel Aminnudin was
mentioned?
"A Yes, sir.
"Q Are you sure of that?
"A On the 23rd he will be coming with the woman.
"Q So that even before you received the official report on June 23,
1984, you had already gathered information to the effect that
Idel Aminnudin was coming to Iloilo on June 25, 1984?
"A Only on the 23rd of June.
"Q You did not try to secure a search warrant for the seizure or
search of the subject mentioned in your intelligence report?
"A No, more.
"Q Why not?
"A Because we were very very sure that our operation will yield
positive result.
"Q Is that your procedure that whenever it will yield positive result
you do not need a search warrant anymore?
"A Search warrant is not necessary." 23

That last answer is a cavalier pronouncement, especially as it


comes from a mere lieutenant of the PC. The Supreme Court cannot
countenance such a statement. This is still a government of laws and
not of men.
The mandate of the Bill of Rights is clear:
"Sec. 2. The right of the people to be secure in their persons,
houses, papers, and effects against unreasonable searches and
seizures of whatever nature and for any purpose shall be
inviolable, and no search warrant or warrant of arrest shall issue
except upon probable cause to be determined personally by the
judge after examination under oath or affirmation of the
complainant and the witnesses he may produce, and particularly
describing the place to be searched and the persons or things to
be seized."
In the case at bar, there was no warrant of arrest or search
warrant issued by a judge after personal determination by him of the
existence of probable cause. Contrary to the averments of the
government, the accused-appellant was not caught in flagrante nor
was a crime about to be committed or had just been committed to
justify the warrantless arrest allowed under Rule 113 of the Rules of
Court. Even expediency could not be invoked to dispense with the
obtention of the warrant as in the case of Roldan v. Arca, 24 for
example. Here it was held that vessels and aircraft are subject to
warrantless searches and seizures for violation of the customs law
because these vehicles may be quickly moved out of the locality or
jurisdiction before the warrant can be secured.
The present case presented no such urgency. From the
conflicting declarations of the PC witnesses, it is clear that they had
at least two days within which they could have obtained a warrant to
arrest and search Aminnudin who was coming to Iloilo on the M/V
Wilcon 9. His name was known. The vehicle was identified. The date of
its arrival was certain. And from the information they had received,
they could have persuaded a judge that there was probable cause,
indeed, to justify the issuance of a warrant. Yet they did nothing. No
effort was made to comply with the law. The Bill of Rights was ignored
altogether because the PC lieutenant who was the head of the
arresting team, had determined on his own authority that "search
warrant was not necessary."
In the many cases where this Court has sustained the
warrantless arrest of violators of the Dangerous Drugs Act, it has
always been shown that they were caught red-handed, as result of
what are popularly called "buy-bust" operations of the narcotics
agents. 25 Rule 113 was clearly applicable because at the precise time
of arrest the accused was in the act of selling the prohibited drug.
In the case at bar, the accused-appellant was not, at the moment
of his arrest, committing a crime nor was it shown that he was about
to do so or that he had just done so. What he was doing was
descending the gangplank of the M/V Wilcon 9 and there was no
outward indication that called for his arrest. To all appearances, he
was like any of the other passengers innocently disembarking from the
vessel. It was only when the informer pointed to him as the carrier of
the marijuana that he suddenly became suspect and so subject to
apprehension. It was the furtive finger that triggered his arrest. The
identification by the informer was the probable cause as determined
by the officers (and not a judge) that authorized them to pounce upon
Aminnudin and immediately arrest him.
Now that we have succeeded in restoring democracy in our
country after fourteen years of the despised dictatorship, when any
one could be picked up at will, detained without charges and punished
without trial, we will have only ourselves to blame if that kind of
arbitrariness is allowed to return, to once more flaunt its disdain of the
Constitution and the individual liberties its Bill of Rights guarantees.
While this is not to say that the accused-appellant is innocent, for
indeed his very own words suggest that he is lying, that fact alone
does not justify a finding that he is guilty. The constitutional
presumption is that he is innocent, and he will be so declared even if
his defense is weak as long as the prosecution is not strong enough to
convict him.
Without the evidence of the marijuana allegedly seized from
Aminnudin, the case of the prosecution must fall. That evidence
cannot be admitted, and should never have been considered by the
trial court for the simple fact is that the marijuana was seized illegally.
It is the fruit of the poisonous tree, to use Justice Holmes' felicitous
phrase. The search was not an incident of a lawful arrest because
there was no warrant of arrest and the warrantless arrest did not
come under the exceptions allowed by the Rules of Court. Hence, the
warrantless search was also illegal and the evidence obtained thereby
was inadmissible.
The Court strongly supports the campaign of the government
against drug addiction and commends the efforts of our law-
enforcement officers against those who would inflict this malediction
upon our people, especially the susceptible youth. But as demanding
as this campaign may be, it cannot be more so than the compulsions
of the Bill of Rights for the protection of the liberty of every individual
in the realm, including the basest of criminals. The Constitution covers
with the mantle of its protection the innocent and the guilty alike
against any manner of high-handedness from the authorities, however
praiseworthy their intentions.
Those who are supposed to enforce the law are not justified in
disregarding the rights of the individual in the name of order. Order is
too high a price for the loss of liberty. As Justice Holmes, again, said,
"I think it a less evil that some criminal should escape than that the
government should play an ignoble part." It is simply not allowed in the
free society to violate a law to enforce another, especially if the law
violated is the Constitution itself.
We find that with the exclusion of the illegally seized marijuana
as evidence against the accused-appellant, his guilt has not been
proved beyond reasonable doubt and he must therefore be discharged
on the presumption that he is innocent.
ACCORDINGLY, the decision of the trial court is REVERSED and
the accused-appellant is ACQUITTED. It is so ordered.
Narvasa, Gancayco and Medialdea JJ. concur.
(People v. Aminnudin y Ahni, G.R. No. 74869, [July 6, 1988], 246 PHIL
|||

424-435)
[G.R. No. 91107. June 19, 1991.]

THE PEOPLE OF THE PHILIPPINES, plaintiff-


appellee, vs. MIKAEL MALMSTEDT, * defendant-appellant.

The Solicitor General for plaintiff-appellee.


Romulo, Mabanta, Buenaventura, Sayoc & De los Angeles for defendant-
appellant.

DECISION

PADILLA, J :p

In an information dated 15 June 1989, accused-appellant Mikael


Malmstedt (hereinafter referred to as the accused) was charged before
the Regional Trial Court (RTC) of La Trinidad, Benguet, Branch 10, in
Criminal Case No. 89-CR-0663, for violation of Section 4, Art. II
of Republic Act 6425, as amended, otherwise known as the Dangerous
Drugs Act of 1972, as amended. The factual background of the case is as
follows:
Accused Mikael Malmstedt, a Swedish national, entered the Philippines
for the third time in December 1988 as a tourist. He had visited the
country sometime in 1982 and 1985.
In the evening of 7 May 1989, accused left for Baguio City. Upon his
arrival thereat in the morning of the following day, he took a bus to
Sagada and stayed in that place for two (2) days.
At around 7:00 o'clock in the morning of 11 May 1989, accused went to
the Nangonogan bus stop in Sagada to catch the first available trip to
Baguio City. From Baguio City, accused planned to take a late afternoon
trip to Angeles City, then proceed to Manila to catch his flight out of the
country, scheduled on 13 May 1989. From Sagada, accused took a Skyline
bus with body number 8005 and Plate number AVC 902. 1
At about 8:00 o'clock in the morning of that same day (11 May 1989),
Captain Alen Vasco, the Commanding Officer of the First Regional
Command (NARCOM) stationed at Camp Dangwa, ordered his men to set
up a temporary checkpoint at Kilometer 14, Acop, Tublay, Mountain
Province, for the purpose of checking all vehicles coming from the
Cordillera Region. The order to establish a checkpoint in the said area
was prompted by persistent reports that vehicles coming from Sagada
were transporting marijuana and other prohibited drugs. Moreover,
information was received by the Commanding Officer of NARCOM, that
same morning, that a Caucasian coming from Sagada had in his
possession prohibited drugs. 2
The group composed of seven (7) NARCOM officers, in coordination with
Tublay Police Station, set up a checkpoint at the designated area at
about 10:00 o'clock in the morning and inspected all vehicles coming
from the Cordillera Region.
At about 1:30 o'clock in the afternoon, the bus where accused was riding
was stopped. Sgt. Fider and CIC Galutan boarded the bus and announced
that they were members of the NARCOM and that they would conduct an
inspection. The two (2) NARCOM officers started their inspection from
the front going towards the rear of the bus. Accused who was the sole
foreigner riding the bus was seated at the rear thereof.
During the inspection, CIC Galutan noticed a bulge on accused's waist.
Suspecting the bulge on accused's waist to be a gun, the officer asked
for accused's passport and other identification papers. When accused
failed to comply, the officer required him to bring out whatever it was
that was bulging on his waist. The bulging object turned out to be a
pouch bag and when accused opened the same bag, as ordered, the
officer noticed four (4) suspicious-looking objects wrapped in brown
packing tape, prompting the officer to open one of the wrapped objects.
The wrapped objects turned out to contain hashish, a derivative of
marijuana.
Thereafter, accused was invited outside the bus for questioning. But
before he alighted from the bus, accused stopped to get two (2) travelling
bags from the luggage carrier.
Upon stepping out of the bus, the officers got the bags and opened them.
A teddy bear was found in each bag. Feeling the teddy bears, the officer
noticed that there were bulges inside the same which did not feel like
foam stuffing. It was only after the officers had opened the bags that
accused finally presented his passport.
Accused was then brought to the headquarters of the NARCOM at Camp
Dangwa, La Trinidad, Benguet for further investigation. At the
investigation room, the officers opened the teddy bears and they were
found to also contain hashish. Representative samples were taken from
the hashish found among the personal effects of accused and the same
were brought to the PC Crime Laboratory for chemical analysis.
In the chemistry report, it was established that the objects examined
were hashish, a prohibited drug which is a derivative of marijuana. Thus,
an information was filed against accused for violation of the Dangerous
Drugs Act.
During the arraignment, accused entered a plea of "not guilty." For his
defense, he raised the issue of illegal search of his personal effects. He
also claimed that the hashish was planted by the NARCOM officers in his
pouch bag and that the two (2) travelling bags were not owned by him,
but were merely entrusted to him by an Australian couple whom he met
in Sagada. He further claimed that the Australian couple intended to take
the same bus with him but because there were no more seats available
in said bus, they decided to take the next ride and asked accused to take
charge of the bags, and that they would meet each other at the Dangwa
Station.
Likewise, accused alleged that when the NARCOM officers demanded for
his passport and other identification papers, he handed to one of the
officers his pouch bag which was hanging on his neck containing, among
others, his passport, return ticket to Sweden and other papers. The
officer in turn handed it to his companion who brought the bag outside
the bus. When said officer came back, he charged the accused that there
was hashish in the bag. He was told to get off the bus and his picture
was taken with the pouch bag placed around his neck. The trial court did
not give credence to accused's defense. LibLex

The claim of the accused that the hashish was planted by the NARCOM
officers, was belied by his failure to raise such defense at the earliest
opportunity. When accused was investigated at the Provincial Fiscal's
Office, he did not inform the Fiscal or his lawyer that the hashish was
planted by the NARCOM officers in his bag. It was only two (2) months
after said investigation when he told his lawyer about said claim, denying
ownership of the two (2) travelling bags as well as having hashish in his
pouch bag.
In a decision dated 12 October 1989, the trial court found accused guilty
beyond reasonable doubt for violation of the Dangerous Drugs Act,
specifically Section 4, Art. II of RA 6425, as amended. 3 The dispositive
portion of the decision reads as follows:
"WHEREFORE, finding the guilt of the accused Mikael Malmstedt
established beyond reasonable doubt, this Court finds him GUILTY
of violation of Section 4, Article II ofRepublic Act 6425, as
amended, and hereby sentences him to suffer the penalty of life
imprisonment and to pay a fine of Twenty Thousand Pesos
(P20,000.00), with subsidiary imprisonment in case of insolvency
and to pay the costs.
Let the hashish subject of this case be turned over to the First
Narcotics Regional Unit at Camp Bado; Dangwa, La Trinidad,
Benguet for proper disposition under Section 20, Article IV
of Republic Act 425, as amended.
SO ORDERED." 4

Seeking the reversal of the decision of the trial court finding him guilty of
the crime charged, accused argues that the search of his personal
effects was illegal because it was made without a search warrant and,
therefore, the prohibited drugs which were discovered during the illegal
search are not admissible as evidence against him.
The Constitution guarantees the right of the people to be secure in their
persons, houses, papers and effects against unreasonable searches and
seizures. 5 However, where the search is made pursuant to a lawful
arrest, there is no need to obtain a search warrant. A lawful arrest
without a warrant may be made by a peace officer or a private person
under the following circumstances. 6
"SEC. 5. Arrest without warrant; when lawful. — A peace officer or
a private person may, without a warrant, arrest a person:
(a) When, in his presence, the person to be arrested has
committed, is actually committing, or is attempting to commit an
offense;
(b) When an offense has in fact just been committed, and he has
personal knowledge of facts indicating that the person to be
arrested has committed it; and
(c) When the person to be arrested is a prisoner who has escaped
from a penal establishment or place where he is serving final
judgment or temporarily confined while his case is pending, or has
escaped while being transferred from one confinement to another.
In cases falling under paragraphs (a) and (b) hereof, the person
arrested without a warrant shall be forthwith delivered to the
nearest police station or jail, and he shall be proceeded against in
accordance with Rule 112, Section 7. (6a, 17a)."
Accused was searched and arrested while transporting prohibited drugs
(hashish). A crime was actually being committed by the accused and he
was caught in flagrante delicto. Thus, the search made upon his personal
effects falls squarely under paragraph (1) of the foregoing provisions of
law, which allow a warrantless search incident to a lawful arrest. 7
While it is true that the NARCOM officers were not armed with a search
warrant when the search was made over the personal effects of accused,
however, under the circumstances of the case, there was sufficient
probable cause for said officers to believe that accused was then and
there committing a crime. LLphil

Probable cause has been defined as such facts and circumstances which
could lead a reasonable, discreet and prudent man to believe that an
offense has been committed, and that the objects sought in connection
with the offense are in the place sought to be searched. 8 The required
probable cause that will justify a warrantless search and seizure is not
determined by any fixed formula but is resolved according to the facts of
each case. 9

Warrantless search of the personal effects of an accused has been


declared by this Court as valid, because of existence of probable cause,
where the smell of marijuana emanated from a plastic bag owned by the
accused, 10 or where the accused was acting suspiciously, 11 and
attempted to flee. 12
Aside from the persistent reports received by the NARCOM that vehicles
coming from Sagada were transporting marijuana and other prohibited
drugs, their Commanding Officer also received information that a
Caucasian coming from Sagada on that particular day had prohibited
drugs in his possession. Said information was received by the
Commanding Officer of NARCOM the very same morning that accused
came down by bus from Sagada on his way to Baguio City.
When NARCOM received the information, a few hours before the
apprehension of herein accused, that a Caucasian travelling from Sagada
to Baguio City was carrying with him prohibited drugs, there was no time
to obtain a search warrant. In the Tangliben case, 13 the police
authorities conducted a surveillance at the Victory Liner Terminal
located at Bgy. San Nicolas, San Fernando Pampanga, against persons
engaged in the traffic of dangerous drugs, based on information supplied
by some informers. Accused Tangliben who was acting suspiciously and
pointed out by an informer was apprehended and searched by the police
authorities. It was held that when faced with on-the spot information, the
police officers had to act quickly and there was no time to secure a
search warrant.
It must be observed that, at first, the NARCOM officers merely conducted
a routine check of the bus (where accused was riding) and the
passengers therein, and no extensive search was initially made. It was
only when one of the officers noticed a bulge on the waist of accused,
during the course of the inspection, that accused was required to
present his passport. The failure of accused to present his identification
papers, when ordered to do so, only managed to arouse the suspicion of
the officer that accused was trying to hide his identity. For is it not a
regular norm for an innocent man, who has nothing to hide from the
authorities, to readily present his identification papers when required to
do so?
The receipt of information by NARCOM that a Caucasian coming from
Sagada had prohibited drugs in his possession, plus the suspicious
failure of the accused to produce his passport, taken together as a
whole, led the NARCOM officers to reasonably believe that the accused
was trying to hide something illegal from the authorities. From these
circumstances arose a probable cause which justified the warrantless
search that was made on the personal effects of the accused. In other
words, the acts of the NARCOM officers in requiring the accused to open
his pouch bag and in opening one of the wrapped objects found inside
said bag (which was discovered to contain hashish) as well as the two (2)
travelling bags containing two (2) teddy bears with hashish stuffed inside
them, were prompted by accused's own attempt to hide his identity by
refusing to present his passport, and by the information received by the
NARCOM that a Caucasian coming from Sagada had prohibited drugs in
his possession. To deprive the NARCOM agents of the ability and facility
to act accordingly, including, to search even without warrant, in the light
of such circumstances, would be to sanction impotence and
ineffectiveness in law enforcement, to the detriment of society.
WHEREFORE, premises considered, the appealed judgment of conviction
by the trial court is hereby AFFIRMED. Costs against the accused-
appellant.
SO ORDERED.
||| (People v. Malmstedt, G.R. No. 91107, [June 19, 1991], 275 PHIL 447-472)

[G.R. No. 197788. February 29, 2012.]

RODEL LUZ y ONG, petitioner,vs.PEOPLE OF THE


PHILIPPINES, 1 respondent.
DECISION

SERENO, J :p

This is a Petition for Review on Certiorari under Rule 45 seeking to


set aside the Court of Appeals (CA) Decision in CA-G.R. CR No. 32516
dated 18 February 2011 2and Resolution dated 8 July 2011.
Statement of the Facts and of the Case
The facts, as found by the Regional Trial Court (RTC),which
sustained the version of the prosecution, are as follows:
PO2 Emmanuel L. Alteza, who was then assigned at the Sub-
Station 1 of the Naga City Police Station as a traffic enforcer,
substantially testified that on March 10, 2003 at around 3:00
o'clock in the morning, he saw the accused, who was coming from
the direction of Panganiban Drive and going to Diversion Road,
Naga City, driving a motorcycle without a helmet; that this
prompted him to flag down the accused for violating a municipal
ordinance which requires all motorcycle drivers to wear
helmet (sic) while driving said motor vehicle; that he invited the
accused to come inside their sub-station since the place where he
flagged down the accused is almost in front of the said sub-
station; that while he and SPO1 Rayford Brillante were issuing a
citation ticket for violation of municipal ordinance, he noticed that
the accused was uneasy and kept on getting something from his
jacket; that he was alerted and so, he told the accused to take out
the contents of the pocket of his jacket as the latter may have a
weapon inside it; that the accused obliged and slowly put out the
contents of the pocket of his jacket which was a nickel-like tin or
metal container about two (2) to three (3) inches in size, including
two (2) cellphones, one (1) pair of scissors and one (1) Swiss knife;
that upon seeing the said container, he asked the accused to open
it; that after the accused opened the container, he noticed a
cartoon cover and something beneath it; and that upon his
instruction, the accused spilled out the contents of the container
on the table which turned out to be four (4) plastic sachets, the
two (2) of which were empty while the other two (2) contained
suspected shabu.3
Arraigned on 2 July 2003, petitioner, assisted by counsel, entered a
plea of "Not guilty" to the charge of illegal possession of dangerous
drugs. Pretrial was terminated on 24 September 2003, after which, trial
ensued.
During trial, Police Officer 3 (PO3) Emmanuel Alteza and a forensic
chemist testified for the prosecution. On the other hand, petitioner
testified for himself and raised the defense of planting of evidence and
extortion.
In its 19 February 2009 Decision, 4 the RTC convicted petitioner of
illegal possession of dangerous drugs 5 committed on 10 March 2003. It
found the prosecution evidence sufficient to show that he had been
lawfully arrested for a traffic violation and then subjected to a valid
search, which led to the discovery on his person of two plastic sachets
later found to contain shabu.The RTC also found his defense of frame-up
and extortion to be weak, self-serving and unsubstantiated. The
dispositive portion of its Decision held:
WHEREFORE, judgment is hereby rendered, finding accused
RODEL LUZ y ONG GUILTY beyond reasonable doubt for the crime
of violation of Section 11, Article II of Republic Act No. 9165 and
sentencing him to suffer the indeterminate penalty of
imprisonment ranging from twelve (12) years and (1) day, as
minimum, to thirteen (13) years, as maximum, and to pay a fine of
Three Hundred Thousand Pesos (P300,000.00).
The subject shabu is hereby confiscated for turn over to the
Philippine Drug Enforcement Agency for its proper disposition and
destruction in accordance with law.
SO ORDERED. 6

Upon review, the CA affirmed the RTC's Decision.


On 12 September 2011, petitioner filed under Rule 45 the instant
Petition for Review on Certiorari dated 1 September 2011. In a Resolution
dated 12 October 2011, this Court required respondent to file a comment
on the Petition. On 4 January 2012, the latter filed its Comment dated 3
January 2012.
Petitioner raised the following grounds in support of his Petition:
(i) THE SEARCH AND SEIZURE OF THE ALLEGED SUBJECT SHABU
IS INVALID.
(ii) THE PRESUMPTION OF REGULARITY IN THE PERFORMANCE OF
DUTY OF THE POLICE OFFICER CANNOT BE RELIED UPON IN
THIS CASE.
(iii) THE INTEGRITY AND EVIDENTIARY VALUE OF THE ALLEGED
SUBJECT SPECIMEN HAS BEEN COMPROMISED.
(iv) THE GUILT OF THE ACCUSED-PETITIONER WAS NOT PROVEN
BEYOND THE REASONABLE DOUBT (sic). 7
Petitioner claims that there was no lawful search and seizure,
because there was no lawful arrest. He claims that the finding that there
was a lawful arrest was erroneous, since he was not even issued a
citation ticket or charged with violation of the city ordinance. Even
assuming there was a valid arrest, he claims that he had never
consented to the search conducted upon him.
On the other hand, finding that petitioner had been lawfully
arrested, the RTC held thus:
It is beyond dispute that the accused was flagged down and
apprehended in this case by Police Officers Alteza and Brillante for
violation of City Ordinance No. 98-012, an ordinance requiring the
use of crash helmet by motorcycle drivers and riders thereon in the
City of Naga and prescribing penalties for violation thereof. The
accused himself admitted that he was not wearing a helmet at the
time when he was flagged down by the said police officers, albeit
he had a helmet in his possession. Obviously, there is legal basis
on the part of the apprehending officers to flag down and arrest
the accused because the latter was actually committing a crime in
their presence, that is, a violation of City Ordinance No. 98-012. In
other words, the accused, being caught in flagrante
delicto violating the said Ordinance, he could therefore be lawfully
stopped or arrested by the apprehending officers. ....8
We find the Petition to be impressed with merit, but not for the
particular reasons alleged. In criminal cases, an appeal throws the entire
case wide open for review and the reviewing tribunal can correct errors,
though unassigned in the appealed judgment, or even reverse the trial
court's decision based on grounds other than those that the parties
raised as errors. 9
First, there was no valid arrest of petitioner.When he was flagged
down for committing a traffic violation, he was not, ipso facto and solely
for this reason, arrested.
Arrest is the taking of a person into custody in order that he or she
may be bound to answer for the commission of an offense. 10 It is
effected by an actual restraint of the person to be arrested or by that
person's voluntary submission to the custody of the one making the
arrest. Neither the application of actual force, manual touching of the
body, or physical restraint, nor a formal declaration of arrest, is required.
It is enough that there be an intention on the part of one of the parties to
arrest the other, and that there be an intent on the part of the other to
submit, under the belief and impression that submission is necessary. 11
Under R.A. 4136, or the Land Transportation and Traffic Code, the
general procedure for dealing with a traffic violation is not the arrest of
the offender, but the confiscation of the driver's license of the latter:
SECTION 29. Confiscation of Driver's License. — Law
enforcement and peace officers of other agencies duly deputized
by the Director shall, in apprehending a driver for any violation of
this Act or any regulations issued pursuant thereto, or of local
traffic rules and regulations not contrary to any provisions of this
Act, confiscate the license of the driver concerned and issue a
receipt prescribed and issued by the Bureau therefor which shall
authorize the driver to operate a motor vehicle for a period not
exceeding seventy-two hours from the time and date of issue of
said receipt. The period so fixed in the receipt shall not be
extended, and shall become invalid thereafter. Failure of the driver
to settle his case within fifteen days from the date of apprehension
will be a ground for the suspension and/or revocation of his
license.
Similarly, the Philippine National Police (PNP) Operations
Manual 12 provides the following procedure for flagging down vehicles
during the conduct of checkpoints:
SECTION 7. Procedure in Flagging Down or Accosting
Vehicles While in Mobile Car. — This rule is a general concept and
will not apply in hot pursuit operations. The mobile car crew shall
undertake the following, when applicable: ...
m.If it concerns traffic violations, immediately issue a Traffic
Citation Ticket (TCT) or Traffic Violation Report (TVR).Never
indulge in prolonged, unnecessary conversation or argument
with the driver or any of the vehicle's occupants;
At the time that he was waiting for PO3 Alteza to write his citation
ticket, petitioner could not be said to have been "under arrest." There
was no intention on the part of PO3 Alteza to arrest him, deprive him of
his liberty, or take him into custody. Prior to the issuance of the ticket,
the period during which petitioner was at the police station may be
characterized merely as waiting time. In fact, as found by the trial court,
PO3 Alteza himself testified that the only reason they went to the police
sub-station was that petitioner had been flagged down "almost in front"
of that place. Hence, it was only for the sake of convenience that they
were waiting there. There was no intention to take petitioner into
custody.
In Berkemer v. McCarty, 13 the United States (U.S.) Supreme Court
discussed at length whether the roadside questioning of a motorist
detained pursuant to a routine traffic stop should be considered
custodial interrogation. The Court held that, such questioning does not
fall under custodial interrogation, nor can it be considered a formal
arrest, by virtue of the nature of the questioning, the expectations of the
motorist and the officer, and the length of time the procedure is
conducted. It ruled as follows:
It must be acknowledged at the outset that a traffic stop
significantly curtails the "freedom of action" of the driver and the
passengers, if any, of the detained vehicle. Under the law of most
States, it is a crime either to ignore a policeman's signal to stop
one's car or, once having stopped, to drive away without
permission. ...
However, we decline to accord talismanic power to the
phrase in the Miranda opinion emphasized by respondent. Fidelity
to the doctrine announced in Miranda requires that it be enforced
strictly, but only in those types of situations in which the concerns
that powered the decision are implicated. Thus, we must decide
whether a traffic stop exerts upon a detained person pressures
that sufficiently impair his free exercise of his privilege against
self-incrimination to require that he be warned of his constitutional
rights.
Two features of an ordinary traffic stop mitigate the danger
that a person questioned will be induced "to speak where he would
not otherwise do so freely," Miranda v. Arizona,384 U.S.,at
467. First, detention of a motorist pursuant to a traffic stop is
presumptively temporary and brief. The vast majority of roadside
detentions last only a few minutes. A motorist's expectations,
when he sees a policeman's light flashing behind him, are that he
will be obliged to spend a short period of time answering questions
and waiting while the officer checks his license and registration,
that he may then be given a citation, but that in the end he most
likely will be allowed to continue on his way. In this respect,
questioning incident to an ordinary traffic stop is quite different
from stationhouse interrogation, which frequently is prolonged,
and in which the detainee often is aware that questioning will
continue until he provides his interrogators the answers they seek.
See id.,at 451.
Second, circumstances associated with the typical traffic
stop are not such that the motorist feels completely at the mercy
of the police.To be sure, the aura of authority surrounding an
armed, uniformed officer and the knowledge that the officer has
some discretion in deciding whether to issue a citation, in
combination, exert some pressure on the detainee to respond to
questions. But other aspects of the situation substantially offset
these forces. Perhaps most importantly, the typical traffic stop is
public, at least to some degree. ...
In both of these respects, the usual traffic stop is more
analogous to a so-called "Terry stop," see Terry v. Ohio,392 U.S. 1
(1968),than to a formal arrest....The comparatively nonthreatening
character of detentions of this sort explains the absence of any
suggestion in our opinions that Terry stops are subject to the
dictates of Miranda. The similarly noncoercive aspect of ordinary
traffic stops prompts us to hold that persons temporarily detained
pursuant to such stops are not "in custody" for the purposes of
Miranda.
xxx xxx xxx
We are confident that the state of affairs projected by
respondent will not come to pass. It is settled that the safeguards
prescribed by Miranda become applicable as soon as a suspect's
freedom of action is curtailed to a "degree associated with formal
arrest." California v. Beheler,463 U.S. 1121, 1125 (1983) (per
curiam).If a motorist who has been detained pursuant to a traffic
stop thereafter is subjected to treatment that renders him "in
custody" for practical purposes, he will be entitled to the full
panoply of protections prescribed by Miranda. See Oregon v.
Mathiason,429 U.S. 492, 495 (1977) (per curiam).(Emphasis
supplied.)
The U.S. Court in Berkemer thus ruled that, since the motorist
therein was only subjected to modest questions while still at the scene
of the traffic stop, he was not at that moment placed under custody
(such that he should have been apprised of his Miranda rights),and
neither can treatment of this sort be fairly characterized as the
functional equivalent of a formal arrest. Similarly, neither can petitioner
here be considered "under arrest" at the time that his traffic citation was
being made.
It also appears that, according to City Ordinance No. 98-012, which
was violated by petitioner, the failure to wear a crash helmet while riding
a motorcycle is penalized by a fine only. Under the Rules of Court, a
warrant of arrest need not be issued if the information or charge was
filed for an offense penalized by a fine only. It may be stated as a
corollary that neither can a warrantless arrest be made for such an
offense.
This ruling does not imply that there can be no arrest for a traffic
violation. Certainly, when there is an intent on the part of the police
officer to deprive the motorist of liberty, or to take the latter into
custody, the former may be deemed to have arrested the motorist. In this
case, however, the officer's issuance (or intent to issue) a traffic citation
ticket negates the possibility of an arrest for the same violation.
Even if one were to work under the assumption that petitioner was
deemed "arrested" upon being flagged down for a traffic violation and
while awaiting the issuance of his ticket, then the requirements for a
valid arrest were not complied with.
This Court has held that at the time a person is arrested, it shall be
the duty of the arresting officer to inform the latter of the reason for the
arrest and must show that person the warrant of arrest, if any. Persons
shall be informed of their constitutional rights to remain silent and to
counsel, and that any statement they might make could be used against
them. 14 It may also be noted that in this case, these constitutional
requirements were complied with by the police officers
only afterpetitioner had been arrested for illegal possession of
dangerous drugs.
In Berkemer,the U.S. Court also noted that the Miranda warnings
must also be given to a person apprehended due to a traffic violation:
The purposes of the safeguards prescribed by Miranda are to
ensure that the police do not coerce or trick captive suspects into
confessing, to relieve the "inherently compelling pressures"
"generated by the custodial setting itself," "which work to
undermine the individual's will to resist," and as much as possible
to free courts from the task of scrutinizing individual cases to try
to determine, after the fact, whether particular confessions were
voluntary. Those purposes are implicated as much by in-custody
questioning of persons suspected of misdemeanors as they are by
questioning of persons suspected of felonies.
If it were true that petitioner was already deemed "arrested" when
he was flagged down for a traffic violation and while he waiting for his
ticket, then there would have been no need for him to be arrested for a
second time — after the police officers allegedly discovered the drugs —
as he was already in their custody.
Second, there being no valid arrest, the warrantless search that
resulted from it was likewise illegal.
The following are the instances when a warrantless search is
allowed: (i) a warrantless search incidental to a lawful arrest; (ii) search
of evidence in "plain view;" (iii) search of a moving vehicle; (iv)
consented warrantless search; (v) customs search; (vi) a "stop and frisk"
search; and (vii) exigent and emergency circumstances. 15 None of the
above-mentioned instances, especially a search incident to a lawful
arrest, are applicable to this case.
It must be noted that the evidence seized, although alleged to be
inadvertently discovered, was not in "plain view." It was actually
concealed inside a metal container inside petitioner's pocket. Clearly,
the evidence was not immediately apparent. 16
Neither was there a consented warrantless search. Consent to a
search is not to be lightly inferred, but shown by clear and convincing
evidence. 17 It must be voluntary in order to validate an otherwise illegal
search; that is, the consent must be unequivocal, specific, intelligently
given and uncontaminated by any duress or coercion. While the
prosecution claims that petitioner acceded to the instruction of PO3
Alteza, this alleged accession does not suffice to prove valid and
intelligent consent. In fact, the RTC found that petitioner was merely
"told" to take out the contents of his pocket. 18
Whether consent to the search was in fact voluntary is a question of
fact to be determined from the totality of all the circumstances. Relevant
to this determination are the following characteristics of the person
giving consent and the environment in which consent is given: (1) the
age of the defendant; (2) whether the defendant was in a public or a
secluded location; (3) whether the defendant objected to the search or
passively looked on; (4) the education and intelligence of the defendant;
(5) the presence of coercive police procedures; (6) the defendant's belief
that no incriminating evidence would be found; (7) the nature of the
police questioning; (8) the environment in which the questioning took
place; and (9) the possibly vulnerable subjective state of the person
consenting. It is the State that has the burden of proving, by clear and
positive testimony, that the necessary consent was obtained, and was
freely and voluntarily given. 19 In this case, all that was alleged was that
petitioner was alone at the police station at three in the morning,
accompanied by several police officers. These circumstances weigh
heavily against a finding of valid consent to a warrantless search.
Neither does the search qualify under the "stop and frisk" rule.
While the rule normally applies when a police officer observes
suspicious or unusual conduct, which may lead him to believe that a
criminal act may be afoot, the stop and frisk is merely a limited
protective search of outer clothing for weapons. 20
In Knowles v. Iowa, 21 the U.S. Supreme Court held that when a
police officer stops a person for speeding and correspondingly issues a
citation instead of arresting the latter, this procedure does not authorize
the officer to conduct a full search of the car. The Court therein held that
there was no justification for a full-blown search when the officer does
not arrest the motorist. Instead, police officers may only conduct
minimal intrusions, such as ordering the motorist to alight from the car
or doing a patdown:
In Robinson, supra,we noted the two historical rationales for
the "search incident to arrest" exception: (1) the need to disarm
the suspect in order to take him into custody, and (2) the need to
preserve evidence for later use at trial. ...But neither of these
underlying rationales for the search incident to arrest exception is
sufficient to justify the search in the present case.
We have recognized that the first rationale — officer safety —
is "'both legitimate and weighty,'" ...The threat to officer safety
from issuing a traffic citation, however, is a good deal less than in
the case of a custodial arrest. In Robinson, we stated that a
custodial arrest involves "danger to an officer" because of "the
extended exposure which follows the taking of a suspect into
custody and transporting him to the police station." 414 U.S.,at
234-235. We recognized that "[t]he danger to the police officer
flows from the fact of the arrest, and its attendant proximity,
stress, and uncertainty, and not from the grounds for arrest." Id.,at
234, n. 5. A routine traffic stop, on the other hand, is a relatively
brief encounter and "is more analogous to a so-called 'Terry
stop' ...than to a formal arrest." Berkemer v. McCarty,468 U.S. 420,
439 (1984).See also Cupp v. Murphy,412 U.S. 291, 296 (1973)
("Where there is no formal arrest ...a person might well be less
hostile to the police and less likely to take conspicuous, immediate
steps to destroy incriminating evidence").
This is not to say that the concern for officer safety is absent
in the case of a routine traffic stop. It plainly is not. See
Mimms, supra,at 110; Wilson,supra,at 413-414. But while the
concern for officer safety in this context may justify the "minimal"
additional intrusion of ordering a driver and passengers out of the
car, it does not by itself justify the often considerably greater
intrusion attending a full fieldtype search. Even without the search
authority Iowa urges, officers have other, independent bases to
search for weapons and protect themselves from danger. For
example, they may order out of a vehicle both the driver,
Mimms, supra,at 111, and any passengers, Wilson, supra,at 414;
perform a "patdown" of a driver and any passengers upon
reasonable suspicion that they may be armed and dangerous, Terry
v. Ohio,392 U.S. 1 (1968);conduct a "Terry patdown" of the
passenger compartment of a vehicle upon reasonable suspicion
that an occupant is dangerous and may gain immediate control of
a weapon, Michigan v. Long,463 U.S. 1032, 1049 (1983);and even
conduct a full search of the passenger compartment, including any
containers therein, pursuant to a custodial arrest, New York v.
Belton,453 U.S. 454, 460 (1981).
Nor has Iowa shown the second justification for the authority
to search incident to arrest — the need to discover and preserve
evidence. Once Knowles was stopped for speeding and issued a
citation, all the evidence necessary to prosecute that offense had
been obtained. No further evidence of excessive speed was going
to be found either on the person of the offender or in the passenger
compartment of the car. (Emphasis supplied.)
The foregoing considered, petitioner must be acquitted. While he
may have failed to object to the illegality of his arrest at the earliest
opportunity, a waiver of an illegal warrantless arrest does not, however,
mean a waiver of the inadmissibility of evidence seized during the illegal
warrantless arrest. 22
The Constitution guarantees the right of the people to be secure in
their persons, houses, papers and effects against unreasonable searches
and seizures. 23 Any evidence obtained in violation of said right shall be
inadmissible for any purpose in any proceeding. While the power to
search and seize may at times be necessary to the public welfare, still it
must be exercised and the law implemented without contravening the
constitutional rights of citizens, for the enforcement of no statute is of
sufficient importance to justify indifference to the basic principles of
government. 24
The subject items seized during the illegal arrest are
inadmissible. 25 The drugs are the very corpus delicti of the crime of
illegal possession of dangerous drugs. Thus, their inadmissibility
precludes conviction and calls for the acquittal of the accused. 26
WHEREFORE, the Petition is GRANTED. The 18 February 2011
Decision of the Court of Appeals in CA-G.R. CR No. 32516 affirming the
judgment of conviction dated 19 February 2009 of the Regional Trial
Court, 5th Judicial Region, Naga City, Branch 21, in Criminal Case No.
RTC 2003-0087, is hereby REVERSEDand SET ASIDE. nPetitioner Rodel
Luz y Ong is hereby ACQUITTED. The bail bond posted for his provisional
liberty is CANCELLED and RELEASED. SO ORDERED.
(Luz y Ong v. People, G.R. No. 197788, [February 29, 2012], 683 PHIL
|||

399-415)

[G.R. No. 120431. April 1, 1998.]

RODOLFO ESPANO, accused-petitioner,vs.COURT OF


APPEALS and PEOPLE OF THE PHILIPPINES, respondents.

Ceferino Padua Law Office for petitioner.


The Solicitor General for respondents.

SYNOPSIS

This is a petition for review of the decision of the court of Appeals


in CA G.R. CR No. 13976 dated January 16, 1995 which affirmed in
toto the judgment of the Regional Trial Court of Manila, Branch 1,
convicting petitioner Rodolfo Espano for violation of Article II Section 8
of Republic Act No. 6425, as amended, otherwise known as
the Dangerous Drugs Act of 1972. The records of the case reveal that
herein petitioner was caught in possession of and under his custody
twelve plastic cellophane bags weighing 5.5 grams containing crushed
flowering tops, marijuana which is a prohibited drug. In his appeal before
the Supreme Court, petitioner contends that the trial and appellate
courts erred in convicting him because (1) the pieces of evidence seized
were inadmissible; (2) the superiority of his constitutional right to be
presumed innocent over the doctrine of presumption of regularity; (3) he
was denied the constitutional right of confrontation and to compulsory
process; and (4) his conviction was based on evidence which was
irrelevant and not properly identified.CIScaA

The Supreme Court finds that there was no compelling reason to


reverse the decisions of the trial and appellate courts. In this case, the
findings of the trial court that the prosecution witnesses were more
credible than those of the defense must stand. Petitioner failed to show
that Pat. Romeo Pagilagan, in testifying against him, was motivated by
reasons other than his duty to curb drug abuse and had any intent to
falsely impute to him such a serious crime as possession of prohibited
drugs. In the absence of such ill motive, the presumption of regularity in
the performance of his official duty must prevail. Furthermore, the
defense of alibi set up by petitioner deserved scant consideration. He
simply contended that he was in his house sleeping at the time of the
incident. Lastly, the two cellophane bags of marijuana seized were
admissible in evidence because he was caught in flagranti as a result of
a buy-bust operation conducted by police officers. However, as for the
other ten cellophane bags of marijuana found at petitioner's residence,
the same are inadmissible in evidence considering that the said bags
were seized at petitioner's house after his arrest, hence, do not fall under
the exceptions provided under Article III, Section 2 of the 1987
Constitution. In view thereof, the instant petition is denied and the
challenged decision is affirmed with modification as to the penalty.

SYLLABUS

1. REMEDIAL LAW; EVIDENCE; CREDIBILITY OF WITNESSES;


FINDINGS OF TRIAL COURTS ON THE CREDIBILITY OF WITNESSES
DESERVE A HIGH DEGREE OF RESPECT; CASE AT BAR. — It is a well-
settled doctrine that findings of trial courts on the credibility of witness
deserve a high degree of respect. Having observed the deportment of
witnesses during the trial, the trial judge is in a better position to
determine the issue of credibility and, thus, his findings will not be
disturbed during appeal in the absence of any clear and showing that he
had overlooked, misunderstood or misapplied some facts or
circumstances of weight and substance which could have altered the
conviction of the appellants. In this case, the findings of the trial court
that the prosecution witnesses were more credible that those of the
defense must stand. Petitioner failed to show that Pat. Pagilagan, in
testifying against him, was motivated by reasons other than his duty to
curb drug abuse and had any intent to falsely impute to him such a
serious crime as possession of prohibited drugs. In the absence of such
ill motive, the presumption of regularity in the performance of his official
duty must prevail.
2. ID.; ID.; ID.; CLAIM OF FRAME-UP, LIKE ALIBI, IS A DEFENSE THAT
HAS BEEN INVARIABLY VIEWED BY THE COURT WITH DISFAVOR; CASE AT
BAR. — The defense set up by petitioner does not deserve any
consideration. He simply contended that he was in his house sleeping at
the time of the incident. This court has consistently held that alibi is the
weakest of all defenses; and for it to prosper, the accused has the burden
of proving that he was not at the scene of the crime at the time of its
commission and that it was physically impossible for him to be there.
Moreover, the "claim of 'frame-up,' like alibi, is a defense that has been
invariably viewed by the Court with disfavor for it can just as easily be
concocted but difficult to prove, and is a common and standard line of
defense in most prosecutions arising from violations of the Dangerous
Drugs Act." No clear and convincing evidence was presented by
petitioner to prove his defense of alibi.
3. ID.; CRIMINAL PROCEDURE; WARRANTLESS ARREST; THE
MARIJUANA SEIZED FROM PETITIONER'S HOUSE AFTER HIS ARREST IS
INADMISSIBLE IN EVIDENCE; CASE AT BAR. — The 1987
Constitution guarantees freedom against unreasonable searches and
seizures under Article III, Section 2 which provides: "The right of the
people to be secure in their persons, houses, papers and effects against
unreasonable searches and seizures of whatever nature and for any
purpose shall be inviolable, and no search warrant or warrant of arrest
shall issue except upon probable cause to be determined personally by
the judge after examination under oath or affirmation of the complainant
and the witnesses he may produce, and particularly describing the place
to be searched and the persons or things to be seized." An exception to
the said rule is a warrantless search incidental to a lawful arrest of
dangerous weapons or anything which may be used as proof of the
commission of an offense. It may extend beyond the person of the one
arrested to include the premises or surroundings under his immediate
control. In this case, the ten cellophane bags of marijuana seized at
petitioner's house after his arrest at Pandacan and Zamora Streets do
not fall under the said exceptions. . . . The articles seized from petitioner
during his arrest were valid under the doctrine of search made incidental
to a lawful arrest. The warrantless search made in his house, however,
which yielded ten cellophane bags of marijuana became unlawful since
the police officers were not armed with a search warrant at the time.
Moreover, it was beyond the reach and control of petitioner. HTScEI

4. CRIMINAL LAW; REPUBLIC ACT NO. 6425 AS


AMENDED BY REPUBLIC ACT 7659; IF THE QUANTITY OF MARIJUANA
INVOLVED IS LESS THAN 750 GRAMS, THE IMPOSABLE PENALTY
RANGES FROM PRISION CORRECTIONAL TO RECLUSION TEMPORAL;
CASE AT BAR. — This Court finds petitioner Rodolfo Espano guilty beyond
reasonable doubt of violating Article II, Section 8, in relation to Section 2
(e-L)(I) of Republic Act No. 6425, as amended. Under the said provision,
the penalty imposed is six years and one day to twelve years and a fine
ranging from six thousand to twelve thousand pesos. With the passage
of Republic Act No. 7659, which took effect on December 31, 1993, the
imposable penalty shall now depend on the quantity of drugs recovered.
Under the provisions of Republic Act No. 7659, Section 20, and as
interpreted in People v. Simon (234 SCRA 555 [1994]) and People v. Lara,
(236 SCRA 291 [1994]) if the quantity of marijuana involved is less than
750 grams, the imposable penalty ranges from prision
correccional to reclusion temporal. Taking into consideration that
petitioner is not a habitual delinquent, the amendatory provision is
favorable to him and the quantity of marijuana involved is less than 750
grams, the penalty imposed under Republic Act No. 7659 should be
applied.
5. ID.;ID.;PROPER PENALTY THEREOF; CASE AT BAR. — There being
no mitigating nor aggravating circumstances, the imposable penalty shall
be prision correccional in its medium period. Applying the Indeterminate
Sentence Law, the maximum penalty shall be taken from the medium
period of prision correccional,which is two (2) years, four (4) months and
one (1) day to four (4) years and two (2) months, while the minimum shall
be taken from the penalty next lower in degree, which is one (1) month
and one (1) day to six (6) months of arresto mayor. cSDHEC

DECISION

ROMERO, J : p

This is a petition for review of the decision of the Court of Appeals


in CA-G.R. CR No. 13976 dated January 16, 1995, 1 which affirmed in
toto the judgment of the Regional Trial Court of Manila, Branch 1,
convicting petitioner Rodolfo Espano for violation of Article II, Section 8
of Republic Act No. 6425, as amended, otherwise known as the
Dangerous Drugs Act. aisadc

Petitioner was charged under the following information:


"That on or about July 14, 1991, in the City of Manila,
Philippines the said accused, not being authorized by law to
possess or use any prohibited drug, did then and there wilfully,
unlawfully and knowingly have in his possession and under his
custody and control twelve (12) plastic cellophane (bags)
containing crushed flowering tops, marijuana weighing 5.5 grams
which is prohibited drug.
Contrary to law." 2

The evidence for the prosecution, based on the testimony of Pat.


Romeo Pagilagan, shows that on July 14, 1991, at about 12:30 a.m., he
and other police officers, namely, Pat. Wilfredo Aquilino, Simplicio Rivera,
and Erlindo Lumboy of the Western Police District (WPD), Narcotics
Division went to Zamora and Pandacan Streets, Manila to confirm reports
of drug pushing in the area. They saw petitioner selling "something" to
another person. After the alleged buyer left, they approached petitioner,
identified themselves as policemen, and frisked him. The search yielded
two plastic cellophane tea bags of marijuana . When asked if he had
more marijuana, he replied that there was more in his house. The
policemen went to his residence where they found ten more cellophane
tea bags of marijuana. Petitioner was brought to the police headquarters
where he was charged with possession of prohibited drugs. On July 24,
1991, petitioner posted bail 3 and the trial court issued his order of
release on July 29, 1991. 4
Annabelle Alip, forensic chemist of the WPD Criminal Investigation
Laboratory Section, testified that the articles sent to her by Pat. Wilfredo
Aquino regarding the apprehension of a certain Rodolfo Espano for
examination tested positive for marijuana, with total weight of 5.5 grams.
By way of defense, petitioner that on said evening, he was sleeping
in house and was awakened only when the policemen handcuffed him. He
alleged that the policemen were looking for his brother-in-law Lauro, and
when they could not find the latter, he was brought to the police station
for investigation and later indicted for possession of prohibited drugs. His
wife Myrna corroborated his story.
The trial court rejected petitioner's defense as a "mere
afterthought" and found the version of the prosecution "more credible
and trustworthy."
Thus, on August 14, 1992, the trial court rendered a decision,
convicting petitioner of the crime charged, the dispositive portion of
which reads:
"WHEREFORE there being proof beyond reasonable doubt, the
court finds the accused Rodolfo Espano y Valeria guilty of the
crime of violation of Section 8, Article II, in relation to Section 2 (e-
L) (I) of Republic Act No. 6425 as amended by Batas Pambansa Blg.
179, and pursuant to law hereby sentences him to suffer
imprisonment of six (6) years and one (1) day to twelve (12) years
and to pay a fine of P6,000.00 with subsidiary imprisonment in
case of default plus costs.
The marijuana is declared fortified in favor of government and
shall be turned over to the Dangerous Drugs Board without delay.
SO ORDERED." 5

Petitioner appealed the decision to the Court of Appeals. The


appellate court, however, affirmed the decision of the trial court in toto.
Hence, this petition.
Petitioner contends that the trial and appellate courts erred in
convicting him on the basis of the following: (a) the pieces of evidence
seized were inadmissible; (b) the superiority of his constitutional right to
be presumed innocent over the doctrine of presumption of regularity; (c)
he was denied the constitutional right of confrontation and to
compulsory process; and (d) his conviction was based on evidence which
was irrelevant and not properly identified.
After a careful examination of the records of the case, this Court
finds no compelling reason sufficient to reverse the decisions of the trial
and appellate courts.
First, it is a well settled doctrine that findings of trial courts on the
credibility of witnesses deserve a high degree of respect. Having
observed the deportment of witnesses during the trial, the trial judge is
in a better position to determine the issue of credibility and, thus, his
findings will not be disturbed during appeal in the absence of any clear
showing that he had overlooked, misunderstood or misapplied some facts
or circumstances of weight and substance which could have altered the
conviction of the appellants. 6
In this case, the findings of the trial court that the prosecution
witnesses were more credible than those of the defense must stand.
Petitioner failed to show that Pat. Pagilagan, in testifying against him,
was motivated by reasons other than his duty to curb drug abuse and had
any intent to falsely impute to him such a serious crime as possession of
prohibited drugs. In the absence of such ill motive, the presumption of
regularity of his official duty must prevail.
In People v. Velasco, 7 this Court reiterated the doctrine of
presumption of regularity in the performance of official duty which
provides:
"...Appellant failed to establish that Pat. Godoy and the other
members of the buy-bust team are policemen engaged in mulcting
or other unscrupulous activities who where motivated either by the
desire to extort money or exact personal vengeance, or by sheer
whim and caprice, when they entrapped her. And in the absence of
proof of any intent on the part of the police authorities to falsely
impute such a serious crime against appellant, as in this case, the
presumption of regularity in the performance of official duty,
...,must prevail over the self-serving and uncorroborated claim of
appellant that she had been framed." 8
Furthermore, the defense set up by petitioner does not deserve any
consideration. He simply contended that he was in his house sleeping at
the time of the incident. This Court has consistently held that alibi is the
weakest of all defenses; and for it to prosper, the accused has the burden
of proving that he was not at the scene of the crime of its commission
and that it was physically impossible for him to be there. Moreover, the
"claim of a 'frame-up', like alibi, is a defense that has been invariably
viewed by the Court with disfavor for it can just as easily be concocted
but difficult to prove, and is a common and standard line of defense in
most prosecutions arising from violations of the Dangerous Drugs
Act." 9 No clear and convincing evidence was presented by petitioner to
prove his defense of alibi.
Second, petitioner contends that the prosecution's failure to
present the alleged informant in court cast a reasonable doubt which
warrants his acquittal. This is again without merit, since failure of the
prosecution to produce the informant in court is of no moment especially
when he is not even the best witness to establish the fact that a buy-bust
operation had indeed been conducted. In this case, Pat. Pagilagan, one of
the policemen who apprehended petitioner, testified on the actual
incident of July 14, 1991, and identified him as the one they caught in
possession of prohibited drugs. Thus,
"We find that the prosecution had satisfactorily proved its
case against appellants. There is no compelling reason for us to
overturn the finding of the trial court that the testimony of Sgt.
Gamboa, the lone witness for the prosecution, was
straightforward, spontaneous and convincing. The testimony of a
sole witness, if credible and positive and satisfies the court
beyond reasonable doubt, is sufficient to convict." 10
Thus on the basis of Pat. Pagilagan's testimony, the prosecution
was able to prove that petitioner indeed committed the crime charged;
consequently, the finding of conviction was proper.
Lastly, the issue on the admissibility of the marijuana seized should
likewise be ruled upon. Rule 113 Section 5(a) of the Rules of Court
provides:
"A peace officer or a private person may, without a warrant,
arrest a person:
a. when, in his presence, the person to be arrested has
committed, is actually committing, or is attempting to commit an
offense;
xxx xxx xxx."
Petitioner's arrest falls squarely under the aforecited rule. He was
caught in flagranti as a result of a buy-bust operation conducted by
police officers on the basis of information received regarding the illegal
trade of drugs within the area of Zamora and Pandacan Streets, Manila.
The police officer saw petitioner handing over something to an alleged
buyer. After the buyer left, they searched him and discovered two
cellophanes of marijuana. His arrest was, therefore, lawful and the two
cellophane bags of marijuana seized were admissible in evidence, being
the fruits of the crime.
As for the ten cellophane bags of marijuana found at petitioner's
residence, however, the same inadmissible in evidence.
The 1987 Constitution guarantees freedom against unreasonable
searches and seizures under Article III, Section 2 which provides:
"The right of the people to be secure in their persons, houses,
papers and effects against unreasonable searches and seizures of
whatever nature and for any purposes shall be inviolable, and no
search warrant or warrant of arrest shall issue except upon
probable cause to be determined personally by the judge after
examination under oath or affirmation of the complainant and the
witnesses he may produce, and particularly describing the place to
be searched and the persons or things to be seized."
An exception to the said rule is a warrantless search incidental to a
lawful arrest for dangerous weapons or anything which may be used as
proof of the commission of an offense. 11 It may extend beyond the person
of the one arrested to include the premises or surroundings under his
immediate control. In this case, the ten cellophane bags of marijuana
seized at petitioner's house after his arrest at Pandacan and Zamora
Streets do not fall under the said exceptions.
In the case of People v. Lua, 12 this Court held:
"As regards the brick of marijuana found inside the
appellant's house, the trial court correctly ignored it apparently in
view of its inadmissibility. While initially the arrest as well as the
body search was lawful, the warrantless search made inside the
appellant's house became unlawful since the police operatives
were not armed with a search warrant. Such search cannot fall
under "search made incidental to a lawful arrest," the same being
limited to body search and to that point within reach or control of
the person arrested, or that which may furnish him with the means
of committing violence or of escaping. In the case at bar, appellant
was admittedly outside his house when he was arrested. Hence, it
can hardly be said that the inner portion of his house was within
his reach or control."
The articles seized from petitioner during his arrest were valid
under the doctrine of search made incidental to a lawful arrest. The
warrantless search made in his house, however, which yielded ten
cellophane bags of marijuana became unlawful since the police officers
were not armed with a search warrant at the time. Moreover, it was
beyond the reach and control of petitioner.
In sum, this Court finds petitioner Rodolfo Espano guilty beyond
reasonable doubt of violating Article II, Section 8, in relation to Section 2
(e-L) (I) of Republic Act No. 6425, as amended. Under the said provision,
the penalty imposed is six years and one day to twelve years and a fine
ranging from six thousand to twelve thousands pesos. With the passage
of Republic Act No. 7659, with took effect on December 31, 1993, the
imposable penalty shall now depend on the quantity of drugs recovered.
Under the provisions of Republic Act No. 7659, Section 20, and as
interpreted in People v. Simon 13 and People v. Lara, 14 if the quantity of
marijuana involved is less than 750 grams, the imposable penalty ranges
from prision correccional to reclusion temporal. Taking into
consideration that petitioner is not a habitual delinquent, the
amendatory provision is favorable to him and the quantity of marijuana
involved is less than 750 grams, the penalty imposed under Republic Act
No. 7659 should be applied. There being no mitigating nor aggravating
circumstances, the imposable penalty shall be prision correccional in its
medium period. Applying the Indeterminate Sentence Law, the maximum
penalty shall be taken from the medium period of prision
correccional,which is two (2) years, four (4) months and one (1) day to
four (4) years and two (2) months, while the minimum shall be taken from
the penalty next lower in degree, which is one (1) month and one (1) day
six (6) months of arresto mayor.
WHEREFORE, the instant petition is hereby DENIED. The decision of
the Court of Appeals in C.A.-G.R. CR No. 13976 dated January 16, 1995 is
AFFIRMED with the MODIFICATION that petitioner Rodolfo Espano is
sentenced to suffer an indeterminate penalty of TWO (2) months and ONE
(1) day of arresto mayor,as minimum of TWO (2) years, FOUR (4) months
and ONE (1) day of prision correccional,as minimum.
SO ORDERED.
(Espano v. Court of Appeals, G.R. No. 120431, [April 1, 1998], 351 PHIL
|||

798-809)
[G.R. No. 163858. June 28, 2005.]

UNITED LABORATORIES, INC., petitioner, vs. ERNESTO ISIP


and/or SHALIMAR PHILIPPINES and/or OCCUPANTS, Shalimar
Building, No. 1571, Aragon Street, Sta. Cruz,
Manila, respondents.

DECISION
CALLEJO, SR., J :p

Rolando H. Besarra, Special Investigator III of the National Bureau


of Investigation (NBI), filed an application, in the Regional Trial Court
(RTC) of Manila, for the issuance of a search warrant concerning the first
and second floors of the Shalimar Building, located at No. 1571, Aragon
Street (formerly No. 1524, Lacson Avenue, Sta. Cruz, Manila) occupied
and/or used by Shalimar Philippines, owned/operated by Ernesto Isip; and
for the seizure of the following for violation of Section 4(a), in relation to
Section 8, of Republic Act (R.A.) No. 8203:
a. Finished or unfinished products
of UNITED LABORATORIES (UNILAB), particularly REVICON
multivitamins;
b. Sundry items such as tags, labels, boxes, packages, wrappers,
receptacles, advertisements and other paraphernalia used in
the offering for sale, sale and/or distribution of counterfeit
REVICON multivitamins;
c. Sales invoices, delivery receipts, official receipts, ledgers,
journals, purchase orders and all other books of accounts and
documents used in recording the manufacture and/or
importation, distribution and/or sales of counterfeit REVICON
multivitamins. 1
The application was docketed as People v. Ernesto Isip, et al.,
Respondents, Search Warrant Case No. 04-4916 and raffled to Branch 24
of the court. Appended thereto were the following: (1) a sketch 2 showing
the location of the building to be searched; (2) the affidavit 3 of Charlie
Rabe of the Armadillo Protection and Security Agency hired
by United Laboratories, Inc. (UNILAB), who allegedly saw the
manufacture, production and/or distribution of fake drug products such
as Revicon by Shalimar Philippines; (3) the letter-request of UNILAB, the
duly licensed and exclusive manufacturer and/or distributor of Revicon
and Disudrin, for the monitoring of the unauthorized
production/manufacture of the said drugs and, if warranted, for their
seizure; (4) the letter-complaint 4 of UNILAB issued through its Director
of the Security and Safety Group; and (5) the joint affidavit 5 of NBI
Agents Roberto Divinagracia and Rolando Besarra containing the
following allegations:
2. When learned that an Asset was already placed by ARMADILLO
PROTECTIVE AND SECURITY AGENCY named CHARLIE RABE,
who was renting a room since November 2003, at the said
premises located at No. 1571 Aragon St., Sta. Cruz, Manila.
MR. RABE averred that the owner of the premises is a certain
MR. ERNESTO ISIP and that the said premises which is
known as SHALIMAR PHILIPPINES, Shalimar Building, are
being used to manufacture counterfeit UNILAB products,
particularly REVICON multivitamins, which was already
patented by UNILAB since 1985; SacDIE

3. Upon verification of the report, we found out that the said


premises is a six-story structure, with an additional floor as a
penthouse, and colored red-brown. It has a tight security
arrangement wherein non-residents are not allowed to enter
or reconnoiter in the premises;
4. We also learned that its old address is No. 1524 Lacson Avenue,
Sta. Cruz, Manila, and has a new address as 1571 Aragon St.,
Sta. Cruz, Manila; and that the area of counterfeiting
operations are the first and second floors of Shalimar
Building;
5. Since we cannot enter the premises, we instructed the Asset to
take pictures of the area especially the places wherein the
clandestine manufacturing operations were being held. At a
peril to his well-being and security, the Asset was able to
take photographs herein incorporated into this Search
Warrant Application. 6
A representative from UNILAB, Michael Tome, testified during the
hearing on the application for the search warrant. After conducting the
requisite searching questions, the court granted the application and
issued Search Warrant No. 04-4916 dated January 27, 2004, directing any
police officer of the law to conduct a search of the first and second
floors of the Shalimar Building located at No. 1571, Aragon Street, Sta.
Cruz, Manila. The court also directed the police to seize the following
items:
a. Finished or unfinished products
of UNITED LABORATORIES (UNILAB), particularly REVICON
multivitamins;
b. Sundry items such as tags, labels, boxes, packages, wrappers,
receptacles, advertisements and other paraphernalia used in
the offering for sale, sale and/or distribution of counterfeit
REVICON multivitamins;
c. Sales invoices, delivery receipts, official receipts, ledgers,
journals, purchase orders and all other books of accounts and
documents used in recording the manufacture and/or
importation, distribution and/or sales of counterfeit REVICON
multivitamins. 7
The court also ordered the delivery of the seized items before it,
together with a true inventory thereof executed under oath.
The search warrant was implemented at 4:30 p.m. on January 27,
2004 by NBI agents Besarra and Divinagracia, in coordination with
UNILAB employees. No fake Revicon multivitamins were found; instead,
there were sealed boxes at the first and second floors of the Shalimar
Building which, when opened by the NBI agents in the presence of
respondent Isip, contained the following:
QUANTITY/UNIT DESCRIPTION

792 Bottles Disudrin 60 ml.


30 Boxes (100 pieces each) Inoflox 200
mg. 8
NBI Special Investigator Divinagracia submitted an inventory of the
things seized in which he declared that the search of the first and
second floors of the Shalimar Building at No. 1571, Aragon Street, Sta.
Cruz, Manila, the premises described in the warrant, was done in an
orderly and peaceful manner. He also filed a Return of Search
Warrant, 9 alleging that no other articles/items other than those
mentioned in the warrant and inventory sheet were seized. The agent
prayed that of the items seized, ten boxes of Disudrin 60 ml., and at least
one box of Inoflox be turned over to the custody of the Bureau of Food
and Drugs (BFAD) for examination. 10The court issued an order granting
the motion, on the condition that the turn over be made before the court,
in the presence of a representative from the respondents and the
court. 11
The respondents filed an "Urgent Motion to Quash the Search
Warrant or to Suppress Evidence." 12 They contended that the
implementing officers of the NBI conducted their search at the first,
second, third and fourth floors of the building at No. 1524-A, Lacson
Avenue, Sta. Cruz, Manila, where items in "open display" were allegedly
found. They pointed out, however, that such premises was different from
the address described in the search warrant, the first and second floors
of the Shalimar Building located at No. 1571, Aragon Street, Sta. Cruz,
Manila. The respondents, likewise, asserted that the NBI officers seized
Disudrin and Inoflox products which were not included in the list of
properties to be seized in the search warrant. HICATc

UNILAB, in collaboration with the NBI, opposed the motion, insisting


that the search was limited to the first and second floors of the Shalimar
building located at the corner of Aragon Street and Lacson Avenue, Sta.
Cruz, Manila. They averred that, based on the sketch appended to the
search warrant application, Rabe's affidavit, as well as the joint affidavit
of Besarra and Divinagracia, the building where the search was
conducted was located at No. 1571, Aragon Street corner Lacson Avenue,
Sta. Cruz, Manila. They pointed out that No. 1524 Lacson Avenue, Sta.
Cruz, Manila was the old address, and the new address was No. 1571,
Aragon Street, Sta. Cruz, Manila. They maintained that the warrant was
not implemented in any other place. 13
In reply, the respondents insisted that the items seized were
different from those listed in the search warrant. They also claimed that
the seizure took place in the building located at No. 1524-A which was
not depicted in the sketch of the premises which the applicant submitted
to the trial court. 14 In accordance with the ruling of this Court in People
v. Court of Appeals, 15 the respondents served a copy of their pleading on
UNILAB. 16
On March 11, 2004, the trial court issued an Order 17 granting the
motion of the respondents, on the ground that the things seized, namely,
Disudrin and Inoflox, were not those described in the search warrant. On
March 16, 2004, the trial court issued an advisory 18 that the seized
articles could no longer be admitted in evidence against the respondents
in any proceedings, as the search warrant had already been quashed.
UNILAB, through the Ureta Law Office, filed a motion, in
collaboration with the NBI agents, for the reconsideration of the order,
contending that the ground used by the court in quashing the warrant
was not that invoked by the respondents, and that the seizure of the
items was justified by the plain view doctrine. The respondents objected
to the appearance of the counsel of UNILAB, contending that the latter
could not appear for the People of the Philippines. The respondents
moved that the motion for reconsideration of UNILAB be stricken off the
record. Disputing the claims of UNILAB, they insisted that the items
seized were contained in boxes at the time of the seizure at No. 1524-A,
Lacson Avenue corner Aragon Street, Sta. Cruz, Manila, and were not
apparently incriminating on plain view. Moreover, the seized items were
not those described and itemized in the search warrant application, as
well as the warrant issued by the court itself. The respondents
emphasized that the Shalimar Laboratories is authorized to manufacture
galenical preparations of the following products:
Products:
- Povidone Iodine
- Chamomile Oil
- Salicylic Acid 10 g.
- Hydrogen Peroxide 3% Topical Solution
- Aceite de Alcamforado

- Aceite de Manzanilla 19
In a manifestation and opposition, the respondents assailed the
appearance of the counsel of UNILAB, and insisted that it was not
authorized to appear before the court under the Rules of Court, and to
file pleadings. They averred that the BFAD was the authorized
government agency to file an application for a search warrant.
In its counter-manifestation, UNILAB averred that it had the
personality to file the motion for reconsideration because it was the one
which sought the filing of the application for a search warrant; besides, it
was not proscribed by Rule 126 of the Revised Rules of Criminal
Procedure from participating in the proceedings and filing pleadings. The
only parties to the case were the NBI and UNILAB and not the State or
public prosecutor. UNILAB also argued that the offended party, or the
holder of a license to operate, may intervene through counsel
under Section 16 of Rule 110, in relation to Section 7(e), of the Rules of
Criminal Procedure.
UNILAB prayed that an ocular inspection be conducted of the place
searched by the NBI officers. 20 In their rejoinder, the respondents
manifested that an ocular inspection was the option to look forward
to. 21 However, no such ocular inspection of the said premises was
conducted.
In the meantime, the BFAD submitted to the court the result of its
examination of the Disudrin and Inoflox samples which the NBI officers
seized from the Shalimar Building. On its examination of the actual
component of Inoflox, the BFAD declared that the substance failed the
test. 22 The BFAD, likewise, declared that the examined Disudrin syrup
failed the test. 23 The BFAD had earlier issued the following report:

PRODUCT NAME Manufacturer L.N. E.D. FINDINGS

1. Phenylpropanolamin Unilab 21021552 3-06 - Registered, however,


e
(Disudrin) label/physical
appearance
12.5 mg./5mL Syrup does not conform with
the
BFAD approved
label/registered
specifications.

2. Ofloxacin (Inoflox) Unilab 99017407 3-05 - Registered, however,


200 mg. tablet label/physical
appearance
does not conform with
the
BFAD approved
label/registered
specifications. 24
On May 28, 2004, the trial court issued an Order 25 denying the
motion for reconsideration filed by UNILAB. The court declared that:
The Search Warrant is crystal clear: The seizing officers were
only authorized to take possession of "finished or unfinished
products of United Laboratories(UNILAB), particularly REVICON
Multivitamins, and documents evidencing the counterfeit nature of
said products. The Receipt/Inventory of Property Seized pursuant
to the warrant does not, however, include REVICON but other
products. And whether or not these seized products are imitations
of UNILAB items is beside the point. No evidence was shown nor
any was given during the proceedings on the application for search
warrant relative to the seized products. ACTESI

On this score alone, the search suffered from a fatal infirmity


and, hence, cannot be sustained. 26
UNILAB, thus, filed the present petition for review
on certiorari under Rule 45 of the Rules of Court, where the following
issues are raised:
Whether or not the seized 792 bottles of Disudrin 60 ml. and
30 boxes of Inoflox 200 mg. are INADMISSIBLE as evidence against
the respondents because they constitute the "fruit of the
poisonous tree" or, CONVERSELY, whether or not the seizure of the
same counterfeit drugs is justified and lawful under the "plain
view" doctrine and, hence, the same are legally admissible as
evidence against the respondents in any and all actions? 27
The petitioner avers that it was deprived of its right to a day in
court when the trial court quashed the search warrant for a ground
which was not raised by the respondents herein in their motion to quash
the warrant. As such, it argues that the trial court ignored the issue
raised by the respondents. The petitioner insists that by so doing, the
RTC deprived it of its right to due process. The petitioner asserts that the
description in the search warrant of the products to be seized —
"finished or unfinished products of UNILAB" — is sufficient to include
counterfeit drugs within the premises of the respondents not covered by
any license to operate from the BFAD, and/or not authorized or licensed
to manufacture, or repackage drugs produced or manufactured by
UNILAB. Citing the ruling of this Court in Padilla v. Court of
Appeals, 28 the petitioner asserts that the products seized were in plain
view of the officers; hence, may be seized by them. The petitioner posits
that the respondents themselves admitted that the seized articles were
in open display; hence, the said articles were in plain view of the
implementing officers.
In their comment on the petition, the respondents aver that the
petition should have been filed before the Court of Appeals (CA) because
factual questions are raised. They also assert that the petitioner has
no locus standi to file the petition involving the validity and the
implementation of the search warrant. They argue that the petitioner
merely assisted the NBI, the BFAD and the Department of Justice; hence,
it should have impleaded the said government agencies as parties-
petitioners. The petition should have been filed by the Office of the
Solicitor General (OSG) in behalf of the NBI and/or the BFAD,
because under the 1987 Revised Administrative Code, the OSG is
mandated to represent the government and its officers charged in their
official capacity in cases before the Supreme Court. The respondents
further assert that the trial court may consider issues not raised by the
parties if such consideration would aid the court in the just
determination of the case.
The respondents, likewise, maintain that the raiding team slashed
the sealed boxes so fast even before respondent Isip could object. They
argue that the seizure took place at No. 1524-A, Lacson Avenue, Sta.
Cruz, Manila covered by Transfer Certificate of Title (TCT) No. 220778,
and not at No. 1571, Aragon Street, Sta. Cruz, Manila covered by TCT No.
174412 as stated in the search warrant. They assert that the ruling of the
Court in People v. Court of Appeals 29 is applicable in this case. They
conclude that the petitioner failed to prove the factual basis for the
application of the plain view doctrine. 30
In reply, the petitioner asserts that it has standing and is, in fact,
the real party-in-interest to defend the validity of the search warrant
issued by the RTC; after all, it was upon its instance that the application
for a search warrant was filed by the NBI, which the RTC granted. It
asserts that it is not proscribed under R.A. No. 8203 from filing a criminal
complaint against the respondents and requesting the NBI to file an
application for a search warrant. The petitioner points out that the Rules
of Criminal Procedure does not specifically prohibit a private
complainant from defending the validity of a search warrant. Neither is
the participation of a state prosecutor provided in Rule 126 of the said
Rules. After all, the petitioner insists, the proceedings for the application
and issuance of a search warrant is not a criminal action. The petitioner
asserts that the place sought to be searched was sufficiently described
in the warrant for, after all, there is only one building on the two parcels
of land described in two titles where Shalimar Philippines is located, the
place searched by the NBI officers. 31 It also asserts that the building is
located at the corner of Aragon Street and Lacson Avenue, Sta. Cruz,
Manila. 32
The petitioner avers that the plain view doctrine is applicable in this
case because the boxes were found outside the door of the respondents'
laboratory on the garage floor. The boxes aroused the suspicion of the
members of the raiding team — precisely because these were marked
with the distinctive UNILAB logos. The boxes in which the items were
contained were themselves so designated to replicate true and original
UNILAB boxes for the same medicine. Thus, on the left hand corner of
one side of some of the boxes 33 the letters "ABR" under the words "60
ml," appeared to describe the condition/quality of the bottles inside (as it
is with genuine UNILAB box of the true medicine of the same brand). The
petitioner pointed out that "ABR" is the acronym for "amber bottle round"
describing the bottles in which the true and original Disudrin (for
children) is contained.CTAIHc

The petitioner points out that the same boxes also had their own
"license plates" which were instituted as among its internal
control/countermeasures. The license plates indicate that the items
within are, supposedly, "Disudrin." The NBI officers had reasonable
ground to believe that all the boxes have one and the same data
appearing on their supposedly distinctive license plates. The petitioner
insists that although some of the boxes marked with the distinctive
UNILAB logo were, indeed, sealed, the tape or seal was also a copy of
the original because these, too, were marked with the distinctive UNILAB
logo. The petitioner appended to its pleading pictures of the Shalimar
building and the rooms searched showing respondent Isip; 34 the boxes
seized by the police officers containing Disudrin syrup; 35 and the boxes
containing Inoflox and its contents. 36
The issues for resolution are the following: (1) whether the
petitioner is the proper party to file the petition at bench; (2) whether it
was proper for the petitioner to file the present petition in this Court
under Rule 45 of the Rules of Court; and (3) whether the search
conducted by the NBI officers of the first and second floors of the
Shalimar building and the seizure of the sealed boxes which, when
opened, contained Disudrin syrup and Inoflox, were valid.
On the first issue, we agree with the petitioner's contention that a
search warrant proceeding is, in no sense, a criminal action 37 or the
commencement of a prosecution. 38 The proceeding is not one against
any person, but is solely for the discovery and to get possession of
personal property. It is a special and peculiar remedy, drastic in nature,
and made necessary because of public necessity. It resembles in some
respect with what is commonly known as John Doe proceedings. 39While
an application for a search warrant is entitled like a criminal action, it
does not make it such an action.
A search warrant is a legal process which has been likened to a writ
of discovery employed by the State to procure relevant evidence of
crime. 40 It is in the nature of a criminal process, restricted to cases of
public prosecutions. 41 A search warrant is a police weapon, issued under
the police power. A search warrant must issue in the name of the State,
namely, the People of the Philippines. 42
A search warrant has no relation to a civil process. It is not a
process for adjudicating civil rights or maintaining mere private
rights. 43 It concerns the public at large as distinguished from the
ordinary civil action involving the rights of private persons. 44 It may only
be applied for in the furtherance of public prosecution. 45
However, a private individual or a private corporation complaining to
the NBI or to a government agency charged with the enforcement of
special penal laws, such as the BFAD, may appear, participate and file
pleadings in the search warrant proceedings to maintain, inter alia, the
validity of the search warrant issued by the court and the admissibility of
the properties seized in anticipation of a criminal case to be filed; such
private party may do so in collaboration with the NBI or such government
agency. The party may file an opposition to a motion to quash the search
warrant issued by the court, or a motion for the reconsideration of the
court order granting such motion to quash. 46
In this case, UNILAB, in collaboration with the NBI, opposed the
respondents' motion to quash the search warrant. The respondents
served copies of their reply and opposition/comment to UNILAB, through
Modesto Alejandro, Jr. 47 The court a quo allowed the appearance of
UNILAB and accepted the pleadings filed by it and its counsel. CacTIE

The general rule is that the proper party to file a petition in the CA
or Supreme Court to assail any adverse order of the RTC in the search
warrant proceedings is the People of the Philippines, through the OSG.
However, in Columbia Pictures Entertainment, Inc. v. Court of
Appeals, 48 the Court allowed a private corporation (the complainant in
the RTC) to file a petition for certiorari, and considered the petition as
one filed by the OSG. The Court in the said case even held that the
petitioners therein could argue its case in lieu of the OSG:
From the records, it is clear that, as complainants,
petitioners were involved in the proceedings which led to the
issuance of Search Warrant No. 23. In People v. Nano, the Court
declared that while the general rule is that it is only the Solicitor
General who is authorized to bring or defend actions on behalf of
the People or the Republic of the Philippines once the case is
brought before this Court or the Court of Appeals, if there appears
to be grave error committed by the judge or a lack of due process,
the petition will be deemed filed by the private complainants
therein as if it were filed by the Solicitor General. In line with this
ruling, the Court gives this petition due course and will allow
petitioners to argue their case against the questioned order in lieu
of the Solicitor General. 49
The general rule is that a party is mandated to follow the hierarchy
of courts. However, in exceptional cases, the Court, for compelling
reasons or if warranted by the nature of the issues raised, may take
cognizance of petitions filed directly before it. 50 In this case, the Court
has opted to take cognizance of the petition, considering the nature of
the issues raised by the parties.
The Court does not agree with the petitioner's contention that the
issue of whether the Disudrin and Inoflox products were lawfully seized
was never raised in the pleadings of the respondents in the court a quo.
Truly, the respondents failed to raise the issue in their motion to quash
the search warrant; in their reply, however, they averred that the seized
items were not included in the subject warrant and, therefore, were not
lawfully seized by the raiding team. They also averred that the said
articles were not illegal per se, like explosives and shabu, as to justify
their seizure in the course of unlawful search. 51 In their
Opposition/Comment filed on March 15, 2004, the respondents even
alleged the following:
The jurisdiction of this Honorable Court is limited to the
determination of whether there is a legal basis to quash the search
warrant and/or to suppress the seized articles in evidence. Since
the articles allegedly seized during the implementation of the
search warrant — Disudrin and Inoflux products — were not
included in the search warrant, they were, therefore, not lawfully
seized by the raiding team; they are not illegal per se, as it were,
like an arms cache, subversive materials orshabu as to justify their
seizure in the course of a lawful search, or being in plain view or
some such. No need whatever for some public assay.
The NBI manifestation is a glaring admission that it cannot
tell without proper examination or assay that the Disudrin and
Inoflox samples allegedly seized from respondent's place were
counterfeit. All the relevant presumptions are in favor of legality. 52
The Court, therefore, finds no factual basis for the contention of the
petitioner that the respondents never raised in the court a quo the issue
of whether the seizure of the Disudrin and Inoflox products was valid.
In any event, the petitioner filed a motion for the reconsideration of
the March 11, 2004 Order of the court a quo on the following claims:
2.01 The Honorable Court ERRED in ruling on a non-issue or the
issue as to the alleged failure to particularly describe in the
search warrant the items to be seized but upon
which NO challenge was then existing and/or NO controversy
is raised;
2.02 The Honorable Court ERRED in its ruling that "finished or
unfinished products of UNILAB" cannot stand the test of a
particular description for which it then reasons that the
search is, supposedly unreasonable; and,
2.03 The Honorable Court ERRED in finding that the evidence
seized is lawfully inadmissible against respondents. 53
The court a quo considered the motion of the petitioner and the
issue raised by it before finally resolving to deny the same. It cannot thus
be gainsaid that the petitioner was denied its right to due process.
On the validity of the seizure of the sealed boxes and its contents of
Disudrin and Inoflox, the Court, likewise, rejects the contention of the
petitioner.DSAacC

A search warrant, to be valid, must particularly describe the place


to be searched and the things to be seized. The officers of the law are to
seize only those things particularly described in the search warrant. A
search warrant is not a sweeping authority empowering a raiding party to
undertake a fishing expedition to seize and confiscate any and all kinds
of evidence or articles relating to a crime. The search is limited in scope
so as not to be general or explanatory. Nothing is left to the discretion of
the officer executing the warrant. 54
Objects, articles or papers not described in the warrant but on plain
view of the executing officer may be seized by him. However, the seizure
by the officer of objects/articles/papers not described in the warrant
cannot be presumed as plain view. The State must adduce evidence,
testimonial or documentary, to prove the confluence of the essential
requirements for the doctrine to apply, namely: (a) the executing law
enforcement officer has a prior justification for an initial intrusion or
otherwise properly in a position from which he can view a particular
order; (b) the officer must discover incriminating evidence inadvertently;
and (c) it must be immediately apparent to the police that the items they
observe may be evidence of a crime, contraband, or otherwise subject to
seizure. 55
The doctrine is not an exception to the warrant. It merely serves to
supplement the prior justification — whether it be a warrant for another
object, hot pursuit, search as an incident to a lawful arrest or some other
legitimate reason for being present, unconnected with a search directed
against the accused. The doctrine may not be used to extend a general
exploratory search from one object to another until something
incriminating at last emerges. It is a recognition of the fact that when
executing police officers comes across immediately incriminating
evidence not covered by the warrant, they should not be required to
close their eyes to it, regardless of whether it is evidence of the crime
they are investigating or evidence of some other crime. It would be
needless to require the police to obtain another warrant. 56Under the
doctrine, there is no invasion of a legitimate expectation of privacy and
there is no search within the meaning of the Constitution.
The immediate requirement means that the executing officer can, at
the time of discovery of the object or the facts therein available to him,
determine probable cause of the object's incriminating evidence. 57 In
other words, to be immediate, probable cause must be the direct result
of the officer's instantaneous sensory perception of the object. 58 The
object is apparent if the executing officer had probable cause to connect
the object to criminal activity. The incriminating nature of the evidence
becomes apparent in the course of the search, without the benefit of any
unlawful search or seizure. It must be apparent at the moment of
seizure. 59
The requirement of inadvertence, on the other hand, means that the
officer must not have known in advance of the location of the evidence
and intend to seize it.60 Discovery is not anticipated. 61
The immediately apparent test does not require an unduly high
degree of certainty as to the incriminating character of evidence. It
requires merely that the seizure be presumptively reasonable assuming
that there is probable cause to associate the property with criminal
activity; that a nexus exists between a viewed object and criminal
activity. 62

Incriminating means the furnishing of evidence as proof of


circumstances tending to prove the guilt of a person. 63
Indeed, probable cause is a flexible, common sense standard. It
merely requires that the facts available to the officer would warrant a
man of reasonable caution and belief that certain items may be
contrabanded or stolen property or useful as evidence of a crime. It does
not require proof that such belief be correct or more likely than true. A
practical, non-traditional probability that incriminating evidence is
involved is all that is required. The evidence thus collected must be seen
and verified as understood by those experienced in the field of law
enforcement. 64
In this case, Disudrin and/or Inoflox were not listed in the search
warrant issued by the court a quo as among the properties to be seized
by the NBI agents. The warrant specifically authorized the officers only
to seize "counterfeit Revicon multivitamins, finished or unfinished, and
the documents used in recording, manufacture and/or importation,
distribution and/or sale, or the offering for sale, sale and/or distribution of
the said vitamins." The implementing officers failed to find any
counterfeit Revicon multivitamins, and instead seized sealed boxes
which, when opened at the place where they were found, turned out to
contain Inoflox and Disudrin. EcAHDT

It was thus incumbent on the NBI agents and the petitioner to prove
their claim that the items were seized based on the plain view doctrine.
It is not enough to prove that the sealed boxes were in the plain view of
the NBI agents; evidence should have been adduced to prove the
existence of all the essential requirements for the application of the
doctrine during the hearing of the respondents' motion to quash, or at
the very least, during the hearing of the NBI and the petitioner's motion
for reconsideration on April 16, 2004. The immediately apparent aspect,
after all, is central to the plain view exception relied upon by the
petitioner and the NBI. There is no showing that the NBI and the
petitioner even attempted to adduce such evidence. In fact, the
petitioner and the NBI failed to present any of the NBI agents who
executed the warrant, or any of the petitioner's representative who was
present at the time of the enforcement of the warrant to prove that the
enforcing officers discovered the sealed boxes inadvertently, and that
such boxes and their contents were incriminating and immediately
apparent. It must be stressed that only the NBI agent/agents who
enforced the warrant had personal knowledge whether the sealed boxes
and their contents thereof were incriminating and that they were
immediately apparent. 65 There is even no showing that the NBI agents
knew the contents of the sealed boxes before they were opened.
In sum then, the Court finds and so hold that the petitioner and the
NBI failed to prove the essential requirements for the application of the
plain view doctrine.
IN LIGHT OF ALL THE FOREGOING, the petition is DENIED for lack of
merit. The assailed orders of the Regional Trial Court are AFFIRMED.
SO ORDERED.
(United Laboratories Inc. v. Isip, G.R. No. 163858, [June 28, 2005], 500
|||

PHIL 342-364)
[G.R. No. L-27360. February 28, 1968.]

HON. RICARDO G. PAPA, as Chief of Police of Manila, HON.


JUAN PONCE ENRILE, as Commissioner of Customs, PEDRO
PACIS, as Collector of Customs of the Port of Manila, and
MARTIN ALAGAO, as Patrolman of the Manila Police-
Department, petitioners, vs. REMEDIOS MAGO and HON.
HILARION U. JARENCIO, as Presiding Judge of Branch 23,
Court of First Instance of Manila, respondents.

Solicitor General for petitioners.


Juan T . David for respondents.

SYLLABUS

1. CUSTOMS BUREAU; POWERS AND DUTIES OF BUREAU OF


CUSTOMS. — Among others, the Bureau of Customs has the duties,
powers and the jurisdiction to assess and collect all lawful revenues
from imported articles and all other dues, fees, charges, fines and
penalties accruing under the tariff and customs laws; to prevent and
suppress smuggling and other frauds upon the customs; and to
enforce tariff and customs laws.
2. ID.; JURISDICTION; CUSTOMS BUREAU HAS JURISDICTION
OVER IMPORTED GOODS; "IMPORTATION", MEANING OF. — Where the
goods in question were imported from Hongkong as shown in the
statement and receipts of duties collected on informal entry and
where the importation has not been terminated, the imported goods
remain under the jurisdiction of the Bureau of Customs. Importation is
terminated only upon the payment of duties, taxes and other charges
upon the articles, or secured to be paid, at the port of entry and the
legal permit for withdrawal shall have been granted. Payment of the
duties, taxes, fees and other charges must be in full.
3. ID; ID; BUREAU OF CUSTOMS, NOT THE COURT OF FIRST
INSTANCE, HAS JURISDICTION OVER THE CASE WHERE GOODS ARE
UNDER CUSTODY OF SAID BUREAU, EVEN IF NO WARRANT OF
SEIZURE AND DETENTION IS YET ISSUED ON GOODS. — Since the
goods were under the custody and at the disposal of the Bureau of
Customs when the petition for mandamus was filed in the Court of
First Instance, the latter could not exercise jurisdiction over said
goods even if the warrant of seizure and detention of goods for
purposes of seizure and forfeiture proceedings had not yet been
issued by the Collector. It is settled that the Bureau of Customs
acquires exclusive jurisdiction over imported goods for purposes of
enforcing the Customs laws, from the moment the goods are actually
in possession and control of said Bureau even in the absence on any
warrant of seizure or detention.
4. ID.; ID.; SEIZURE OF GOODS BY MPD, DEPUTIZED BY BUREAU
OF CUSTOMS GAVE THE LATTER EXCLUSIVE JURISDICTION OVER
CASE; ISSUANCE OF WARRANT OF SEIZURE BY CUSTOMS BUREAU
AFTER FILLING OF MANDAMUS SUIT IN CFI, DID NOT DIVEST THE
LATTER OF JURISDICTION IT DID NOT ACQUIRE. — Where the Bureau
of Customs, through the Manila Police Department acting under
petitioner police chief Papa who was formally deputized by the
Commissioner of Customs seized the goods on November 4, 1966, the
Bureau from that date acquired jurisdiction over the goods to the
exclusion of the regular courts. The issuance of the warrant of seizure
and detention by the Customs Collector after the filing of the
mandamus suit in the regular court, did not deprive the latter of its
jurisdiction which it never acquired in the first place, as the Bureau of
Customs had already previously acquired jurisdiction on the case to
the exclusion of regular courts for purposes of enforcement of
customs and tariff laws.
5. ID.; ID.; GOODS, EVEN IF BROUGHT OUT OF CUSTOMS AREA,
STILL FALL WITHIN JURISDICTION OF BUREAU OF CUSTOMS;
JURISDICTION OF CUSTOMS BUREAU IS REGAINED. — Even if it be
conceded, arguendo, that after the goods have been brought out of the
customs area, the Bureau of Customs lost jurisdiction over the same,
still when said goods were intercepted at the Agrifina Circle by
members of the MPD acting under directions and orders of petitioner
Papa who had been formally deputized by the Commissioner of
Customs, such jurisdiction was regained by the Bureau of Customs.
Sec. 1206 of the Tariff and Customs Codeimposes upon the Collector of
Customs the duty to hold possession of all imported articles upon
which duties, taxes and other charges have not been paid or secured
to be paid and to dispose of the same according to law.
6. ID.; IMPORTATIONS MADE CONTRARY TO LAW ARE SUBJECT
TO FORFEITURE. — Where from the record, the duties, taxes and other
charges on the imported articles have not been paid in full, such
articles are subject to forfeiture under Section 2530 pars. e and m, (1),
(4) and (5) of the Tariff and Customs Code; for well settled is the rule
that merchandise imported contrary to law is subject to forfeiture and
goods released contrary to law are likewise subject to seizure and
forfeiture.
7. ID.; ID.; SEARCH WARRANT; LAWFUL SEARCH WITHOUT
SEARCH WARRANT CAN BE EFFECTED. — The Tariff and Customs
Code does not require a search warrant for purposes of enforcing
customs and tariff laws. Under Sec. 2203 thereof, persons having
police authority may enter, pass through or search any land, inclosure,
warehouse, store or building not being a dwelling house and also, to
inspect, search and examine any vehicle or aircraft and any trunk,
package, box or envelope or any person on board or stop and search
and examine any vehicle, beast or person suspected of holding or
conveying any dutiable or prohibited article introduced into the
Philippines contrary to law, without mentioning the need of a search
warrant in said cases. Except in the search of a dwelling house,
therefore, persons exercising police authority under the customs law
may effect search and seizure without search warrant in the
enforcement of customs laws.

DECISION

ZALDIVAR, J :p
This is an original action for prohibition and certiorari, with
preliminary injunction, filed by Ricardo Papa, Chief of Police of Manila;
Juan Ponce Enrile, Commissioner of Customs; Pedro Pacis, Collector of
Customs of the Port of Manila; and Martin Alagao, a patrolman of the
Manila Police Department, against Remedios Mago and Hon. Hilarion
Jarencio, Presiding Judge of Branch 23 of the Court of First Instance
of Manila, praying for the annulment of the order issued by respondent
Judge in Civil Case No. 67496 of the Court of First Instance of Manila
under date of March 7, 1967, which authorized the release under bond
of certain goods which were seized and held by petitioners in
connection with the enforcement of the Tariff and Customs Code, but
which were claimed by respondent Remedios Mago, and to prohibit
respondent Judge from further proceeding in any manner whatsoever
in said Civil Case No. 67496. Pending the determination of this case
this Court issued a writ of preliminary injunction restraining the
respondent Judge from executing, enforcing and/or implementing the
questioned order in Civil Case No. 67496 and from proceeding with
said case.
Petitioner Martin Alagao, head of the counter-intelligence unit of
the Manila Police Department, acting upon a reliable information
received on November 3, 1966 to the effect that a certain shipment of
personal effects, allegedly misdeclared and undervalued, would be
released the following day from the customs zone of the port of Manila
and loaded on two trucks, and upon orders of petitioner Ricardo Papa,
Chief of Police of Manila and a duly deputized agent of the Bureau of
Customs, conducted surveillance at gate No. 1 of the customs zone.
When the trucks left gate No. 1 at about 4:30 in the afternoon of
November 4, 1966, elements of the counter-intelligence unit went after
the trucks and intercepted them at the Agrifina Circle, Ermita, Manila.
The load of the two trucks, consisting of nine bales of goods, and the
two trucks, were seized on instructions of the Chief of Police. Upon
investigation, a person claimed ownership of the goods and showed to
the policemen a "Statement and Receipts of Duties Collected on
Informal Entry No. 147-5501", issued by the Bureau of Customs in the
name of a certain Bienvenido Naguit.
Claiming to have been prejudiced by the seizure and detention of
the two trucks and their cargo, Remedios Mago and Valentin B. Lanopa
filed with the Court of First Instance of Manila a petition "for
mandamus with restraining order or preliminary injunction," docketed
as Civil Case No. 67496, alleging, among others, that Remedios Mago
was the owner of the goods seized, having purchased them from the
Sta. Monica Grocery in San Fernando, Pampanga; that she hired the
trucks owned by Valentin B. Lanopa to transport the goods from said
place to her residence at 1657 Laon Laan St., Sampaloc, Manila; that
the goods were seized by members of the Manila Police Department
without search warrant issued by a competent court; that Manila Chief
of Police Ricardo Papa denied the request of counsel for Remedios
Mago that the bales be not opened and the goods contained therein be
not examined; that then Customs Commissioner Jacinto Gavino had
illegally assigned appraisers to examine the goods because the goods
were no longer under the control and supervision of the Commissioner
of Customs; that the goods, even assuming them to have been
misdeclared and undervalued, were not subject to seizure under
Section 2531 of the Tariff and Customs Code because Remedios Mago
had bought them from another person without knowledge that they
were imported illegally; that the bales had not yet been opened,
although Chief of Police Papa had arranged with the Commissioner of
Customs regarding the disposition of the goods, and that unless
restrained their constitutional rights would be violated and they would
truly suffer irreparable injury. Hence Remedios Mago and Valentin
Lanopa prayed for the issuance of a restraining order, ex parte,
enjoining the above-named police and customs authorities, or their
agents, from opening the bales and examining the goods, and a writ of
mandamus for the return of the goods and the trucks, as well as a
judgment for actual, moral and exemplary damages in their favor.
On November 10, 1966, respondent Judge Hilarion Jarencio
issued an order ex parte restraining the respondents in Civil Case No.
67496 — now petitioners in the instant case before this Court — from
opening the nine bales in question, and at the same time set the
hearing of the petition for preliminary injunction on November 16,
1966. However, when the restraining order was received by herein
petitioners, some bales had already been opened by the examiners of
the Bureau of Customs in the presence of officials of the Manila Police
Department, an assistant city fiscal and a representative of herein
respondent Remedios Mago.
Under date of November 15, 1966, Remedios Mago filed an
amended petition in Civil Case No. 67496, including as party
defendants Collector of Customs Pedro Pacis of the Port of Manila and
Lt. Martin Alagao of the Manila Police Department. Herein petitioners
(defendants below) filed, on November 24, 1966, their "Answer with
Opposition to the Issuance of a Writ of Preliminary Injunction", denying
the alleged illegality of the seizure and detention of the goods and the
trucks and of their other actuations, and alleging special and
affirmative defenses, to wit: that the Court of First Instance of Manila
had no jurisdiction to try the case; that the case fell within the
exclusive jurisdiction of the Court of Tax Appeals; that, assuming that
the court had jurisdiction over the case, the petition stated no cause
of action in view of the failure of Remedios Mago to exhaust the
administrative remedies provided for in the Tariff and Customs Code;
that the Bureau of Customs had not lost jurisdiction over the goods
because the full duties and charges thereon had not been paid; that
the members of the Manila Police Department had the power to make
the seizure; that the seizure was not unreasonable; and that the
persons deputized under Section 2203 (c) of the Tariff and Customs
Code could effect searches, seizures and arrests in inland places in
connection with the enforcement of the said Code. In opposing the
issuance of the writ of preliminary injunction, herein petitioners
averred in the court below that the writ could not be granted for the
reason that Remedios Mago was not entitled to the main reliefs she
prayed for; that the release of the goods, which were subject to
seizure proceedings under the Tariff and Customs Code, would deprive
the Bureau of Customs of the authority to forfeit them; and that
Remedios Mago and Valentin Lanopa would not suffer irreparable
injury. Herein petitioners prayed the court below for the lifting of the
restraining order, for the denial of the issuance of the writ of
preliminary injunction, and for the dismissal of the case.
At the hearing on December 9, 1966, the lower court, with the
conformity of the parties, ordered that an inventory of the goods be
made by its clerk of court in the presence of the representatives of the
claimant of the goods, the Bureau of Customs, and the Anti- Smuggling
Center of the Manila Police Department. On December 13, 1966, the
above-named persons filed a "Compliance" itemizing the contents of
the nine bales.
Herein respondent Remedios Mago, on December 23, 1966, filed
an ex parte motion to release the goods, alleging that since the
inventory of the goods seized did not show any article of prohibited
importation, the same should be released as per agreement of the
parties upon her posting of the appropriate bond that may be
determined by the court. Herein petitioners filed their opposition to
the motion, alleging that the court had no jurisdiction to order the
release of the goods in view of the fact that the court had no
jurisdiction over the case, and that most of the goods, as shown in the
inventory, were not declared and were, therefore, subject to forfeiture.
A supplemental opposition was filed by herein petitioners on January
19, 1967, alleging that on January 12, 1967 seizure proceedings against
the goods had been instituted by the Collector of Customs of the Port
of Manila, and the determination of all questions affecting the disposal
of property proceeded against in seizure and forfeiture proceedings
should thereby be left to the Collector of Customs. On January 30,
1967, herein petitioners filed a manifestation that the estimated
duties, taxes and other charges due on the goods amounted to
P95,772.00. On February 2, 1967, herein respondent Remedios Mago
filed an urgent manifestation and reiteration of the motion for the
release under bond of the goods.
On March 7, 1967, the respondent Judge issued an order releasing
the goods to herein respondent Remedios Mago upon her filing of a
bond in the amount of P40,000.00, and on March 13, 1967, said
respondent filed the corresponding bond.
On March 13, 1967, herein petitioner Ricardo Papa, on his own
behalf, filed a motion for reconsideration of the order of the court
releasing the goods under bond, upon the ground that the Manila
Police Department had been directed by the Collector of Customs of
the Port of Manila to hold the goods pending termination of the seizure
proceedings.
Without waiting for the court's action on the motion for
reconsideration, and alleging that they had no plain, speedy and
adequate remedy in the ordinary course of law, herein petitioners filed
the present action for prohibition and certiorari with preliminary
injunction before this Court. In their petition petitioners allege, among
others, that the respondent Judge acted without jurisdiction in
ordering the release to respondent Remedios Mago of the disputed
goods, for the following reasons: (1) the Court of First Instance of
Manila, presided by respondent Judge, had no jurisdiction over the
case; (2) respondent Remedios Mago had no cause of action in Civil
Case No. 67496 of the Court of First Instance of Manila due to her
failure to exhaust all administrative remedies before invoking judicial
intervention; (3) the Government was not estopped by the negligent
and/or illegal acts of its agents in not collecting the correct taxes; and
(4) the bond fixed by respondent Judge for the release of the goods
was grossly insufficient.
In due time, the respondents filed their answer to the petition for
prohibition and certiorari in this case. In their answer, respondents
alleged, among others: (1) that it was within the jurisdiction of the
lower court presided by respondent Judge to hear and decide Civil
Case No. 67496 and to issue the questioned order of March 7, 1967,
because said Civil Case No. 67496 was instituted long before seizure
and identification proceedings against the nine bales of goods in
question were instituted by the Collector of Customs; (2) that
petitioners could no longer go after the goods in question after the
corresponding duties and taxes had been paid and said goods had left
the customs premises and were no longer within the control of the
Bureau of Customs; (3) that respondent Remedios Mago is a purchaser
in good faith of the goods in question so that those goods can not be
the subject of seizure and forfeiture proceedings; (4) that the seizure
of the goods was effected by members of the Manila Police
Department at a place outside the control and jurisdiction of the
Bureau of Customs and effected without any search warrant or a
warrant of seizure and detention; (5) that the warrant of seizure and
detention subsequently issued by the Collector of Customs is illegal
and unconstitutional, it not being issued by a judge; (6) that the
seizing officers have no authority to seize the goods in question
because they are not articles of prohibited importation; (7) that
petitioners are estopped to institute the present action because they
had agreed before the respondent Judge that they would not interpose
any objection to the release of the goods under bond to answer for
whatever duties and taxes the said goods may still be liable; and (8)
that the bond for the release of the goods was sufficient.
The principal issue in the instant case is whether or not, the
respondent Judge had acted with jurisdiction in issuing the order of
March 7, 1967 releasing the goods in question.
The Bureau of Customs has the duties, powers and jurisdiction,
among others, (1) to assess and collect all lawful revenues from
imported articles, and all other dues, fees, charges, fines and
penalties, accruing under the tariff and customs laws; (2) to prevent
and suppress smuggling and other frauds upon the customs; and (3) to
enforce tariff and customs laws. 1 The goods in question were
imported from Hongkong, as shown in the "Statement and Receipts of
Duties Collected on Informal Entry." 2 As long as the importation has
not been terminated the imported goods remain under the jurisdiction
of the Bureau of Customs. Importation is deemed terminated only upon
the payment of the duties, taxes and other charges upon the articles,
or secured to be paid, at the port of entry and the legal permit for
withdrawal shall have been granted. 3 The payment of the duties,
taxes, fees and other charges must be in full. 4
The record shows, by comparing the articles and duties stated in
the aforesaid "Statement and Receipts of Duties Collected on Informal
Entry" with the manifestation of the Office of the Solicitor
General 5 wherein it is stated that the estimated duties, taxes and
other charges on the goods subject of this case amounted to
P95,772.00 as evidenced by the report of the appraiser of the Bureau
of Customs, that the duties, taxes and other charges had not been paid
in full. Furthermore, a comparison of the goods on which duties had
been assessed, as shown in the "Statement and Receipts of Duties
Collected on Informal Entry" and the "compliance" itemizing the
articles found in the bales upon examination and inventory, 6 shows
that the quantity of the goods was underdeclared, presumably to avoid
the payment of duties thereon. For example, Annex B (the statement
and receipts of duties collected) states that there were 40 pieces of
ladies' sweaters, whereas Annex H (the inventory contained in the
"compliance") states that in bale No. 1 alone there were 42 dozens
and 1 piece of ladies' sweaters of assorted colors; in Annex B, only 100
pieces of watch bands were assessed, but in Annex H, there were in
bale No. 2, 209 dozens and 5 pieces of men's metal watch bands
(white) and 120 dozens of men's metal watch bands (gold color), and in
bale No. 7, 320 dozens of men's metal watch bands (gold color); in
Annex B, 20 dozens only of men's handkerchief were declared, but in
Annex H it appears that there were 224 dozens of said goods in bale
No. 2, 120 dozens in bale No. 6, 380 dozens in bale No. 7, 220 dozens in
bale No. 8, and another 200 dozens in bale No. 9. The articles
contained in the nine bales in question, were, therefore, subject to
forfeiture under Section 2530, pars. e and m, (1), (3), (4), and (5) of
the Tariff and Customs Code. And this Court has held that
merchandise, the importation of which is effected contrary to law, is
subject to forfeiture, 7 and that goods released contrary to law are
subject to seizure and forfeiture. 8

Even if it be granted, arguendo, that after the goods in question


had been brought out of the customs area the Bureau of Customs had
lost jurisdiction over the same, nevertheless, when said goods were
intercepted at the Agrifina Circle on November 4, 1966 by members of
the Manila Police Department, acting under directions and orders of
their Chief, Ricardo G. Papa, who had been formally deputized by the
Commissioner of Customs, 9 the Bureau of Customs had regained
jurisdiction and custody of the goods. Section 1206 of the Tariff and
Customs Code imposes upon the Collector of Customs the duty to hold
possession of all imported articles upon which duties, taxes, and other
charges have not been paid or secured to be paid, and to dispose of
the same according to law. The goods in question, therefore, were
under the custody and at the disposal of the Bureau of Customs at the
time the petition for mandamus, docketed as Civil Case No. 67496, was
filed in the Court of First Instance of Manila on November 9, 1966. The
Court of First Instance of Manila, therefore, could not exercise
jurisdiction over said goods even if the warrant of seizure and
detention of the goods for the purposes of the seizure and forfeiture
proceedings had not yet been issued by the Collector of Customs.
The ruling in the case of "Alberto de Joya, et al. v. Hon. Gregorio
Lantin, et al.," G. R. No. L-24037, decided by this Court on April 27,
1967, is squarely applicable to the instant case. In the De Joya case, it
appears that Francindy Commercial of Manila bought from Ernerose
Commercial of Cebu City 90 bales of assorted textiles and rags, valued
at P117,731.00, which had been imported and entered thru the port of
Cebu. Ernerose Commercial shipped the goods to Manila on board an
inter-island vessel. When the goods were about to leave the customs
premises in Manila, on October 6, 1964, the customs authorities held
them for further verification, and upon examination the goods were
found to be different from the declaration in the cargo manifest of the
carrying vessel. Francindy Commercial subsequently demanded from
the customs authorities the release of the goods, asserting that it is a
purchaser in good faith of those goods; that a local purchase was
involved so the Bureau of Customs had no right to examine the goods;
and that the goods came from a coastwise port. On October 26, 1964,
Francindy Commercial filed in the Court of First Instance of Manila a
petition for mandamus against the Commissioner of Customs and the
Collector of Customs of the port of Manila to compel said customs
authorities to release the goods.
Francindy Commercial alleged in its petition for mandamus that
the Bureau of Customs had no jurisdiction over the goods because the
same were not imported to the port of Manila; that it was not liable for
duties and taxes because the transaction was not an original
importation; that the goods were not in the hands of the importer nor
subject to said importer's control, nor were the goods imported
contrary to law with its (Francindy Commercial's) knowledge; and that
the importation had been terminated. On November 12, 1964, the
Collector of Customs of Manila issued a warrant of seizure and
identification against the goods. On December 3, 1964, the
Commissioner of Customs and the Collector of Customs, as
respondents in the mandamus case, filed a motion to dismiss the
petition on the grounds of lack of jurisdiction, lack of cause of action,
and in view of the pending seizure and forfeiture proceedings. The
court of first instance held resolution on the motion to dismiss in
abeyance pending decision on the merits. On December 14, 1964, the
Court of First Instance of Manila issued a writ of preventive and
mandatory injunction, on prayer by Francindy Commercial, upon a bond
of P20,000.00. The Commissioner of Customs and the Collector of
Customs sought the lifting of the preliminary and mandatory
injunction, and the resolution of their motion to dismiss. The Court of
First Instance of Manila, however, on January 12, 1965, ordered them
to comply with the preliminary and mandatory injunction, upon the
filing by Francindy Commercial of an additional bond of P50,000.00.
Said customs authorities thereupon filed with this Court, on January
14, 1965, a petition for certiorari and prohibition with preliminary
injunction. In resolving the question raised in that case, this Court
held:
"This petition raises two related issues: first, has the
Customs bureau jurisdiction to seize the goods and institute
forfeiture proceeding against them? and (2) has the Court of First
Instance jurisdiction to entertain the petition for mandamus to
compel the Customs authorities to release the goods?
"Francindy Commercial contends that since the petition in
the Court of First Instance was filed (on October 26, 1964) ahead of
the issuance of the Customs warrant of seizure and forfeiture (on
November 12, 1964), the Customs bureau should yield to the
jurisdiction of the said court.
"The record shows, however, that the goods in question were
actually seized on October 6, 1964, i.e., before Francindy
Commercial sued in court. The purpose of the seizure by the
Customs bureau was to verify whether or not Customs duties and
taxes were paid for their importation. Hence, on December 23,
1964, Customs released 22 bales thereof, for the same were found
to have been released regularly from the Cebu Port (Petition Annex
'L'). As to goods imported illegally or released irregularly from
Customs custody, these are subject to seizure under Section 2530
m. of the Tariff and Customs Code (RA 1957).
"The Bureau of Customs has jurisdiction and power, among
others, to collect revenues from imported articles, fines and
penalties and suppress smuggling and other frauds on customs;
and to enforce tariff and customs laws (Sec. 602, Republic Act
1957).
"The goods in question are imported articles entered at the
Port of Cebu. Should they be found to have been released
irregularly from Customs custody in Cebu City, they are subject to
seizure and forfeiture, the proceedings for which comes within the
jurisdiction of the Bureau of Customs pursuant to Republic Act
1937.
"Said proceedings should be followed; the owner of the goods
may set up defenses therein (Pacis v. Averia, L-22526, Nov. 20,
1966). From the decision of the Commissioner of Customs appeal
lies to the Court of Tax Appeals, as provided in Sec. 2402
of Republic Act 1937 and Sec. 11 of Republic Act 1125. To permit
recourse to the Court of First Instance in cases of seizure of
imported goods would in effect render ineffective the power of the
Customs authorities under the Tariff Code and deprive the Court of
Tax Appeals of one of its exclusive appellate jurisdictions. As this
Court has ruled in Pacis v. Averia, supra, Republic Acts 1937 and
1125 vest jurisdiction over seizure and forfeiture proceedings
exclusively upon the Bureau of Customs and the Court of Tax
Appeals. Such law being special in nature, while the Judiciary Act
defining the jurisdiction of Courts of First Instance is a general
legislation, not to mention that the former are later enactments,
the Court of First Instance should yield to the jurisdiction of the
Customs authorities."
It is the settled rule, therefore, that the Bureau of Customs
acquires exclusive jurisdiction over imported goods, for the purposes
of enforcement of the customs laws, from the moment the goods are
actually in its possession or control, even if no warrant of seizure or
detention had previously been issued by the Collector of Customs in
connection with seizure and forfeiture proceedings. In the present
case, the Bureau of Customs actually seized the goods in question on
November 4, 1966, and so from that date the Bureau of Customs
acquired jurisdiction over the goods for the purposes of the
enforcement of the tariff and customs laws, to the exclusion of the
regular courts. Much less then would the Court of First Instance of
Manila have jurisdiction over the goods in question after the Collector
of Customs had issued the warrant of seizure and detention on
January 12, 1967. 10 And so, it cannot be said, as respondents contend,
that the issuance of said warrant was only an attempt to divest the
respondent Judge of jurisdiction over the subject matter of the case.
The court presided by respondent Judge did not acquire jurisdiction
over the goods in question when the petition for mandamus was filed
before it, and so there was no need of divesting it of jurisdiction. Not
having acquired jurisdiction over the goods, it follows that the Court of
First Instance of Manila had no jurisdiction to issue the questioned
order of March 7, 1967 releasing said goods.
Respondents also aver that petitioner Martin Alagao, an officer of
the Manila Police Department, could not seize the goods in question
without a search warrant. This contention cannot be sustained. The
Chief of the Manila Police Department, Ricardo G. Papa, having been
deputized in writing by the Commissioner of Customs, could, for the
purposes of the enforcement of the customs and tariff laws, effect
searches, seizures, and arrests, 11 and it was his duty to make seizure,
among others, of any cargo, articles or other movable property when
the same may be subject to forfeiture or liable for any fine imposed
under customs and tariff laws. 12 He could lawfully open and examine
any box, trunk, envelope or other container wherever found when he
had reasonable cause to suspect the presence therein of dutiable
articles introduced into the Philippines contrary to law; and likewise to
stop, search and examine any vehicle, beast or person reasonably
suspected of holding or conveying such article as aforesaid. 13 It
cannot be doubted, therefore, that petitioner Ricardo G. Papa, Chief of
Police of Manila, could lawfully effect the search and seizure of the
goods in question. The Tariff and Customs Code authorizes him to
demand assistance of any police officer to effect said search and
seizure, and the latter has the legal duty to render said
assistance. 14 This was what happened precisely in the case of Lt.
Martin Alagao who, with his unit, made the search and seizure of the
two trucks loaded with the nine bales of goods in question at the
Agrifina Circle. He was given authority by the Chief of Police to make
the interception of the cargo. 15
Petitioner Martin Alagao and his companion policemen had
authority to effect the seizure without any search warrant issued by a
competent court. The Tariff and Customs Code does not require said
warrant in the instant case. The Code authorizes persons having
police authority under Section 2203 of the Tariff and Customs Code to
enter, pass through or search any land, inclosure, warehouse, store or
building, not being a dwelling house; and also to inspect, search and
examine any vessel or aircraft and any trunk, package, box or
envelope or any person on board, or stop and search and examine any
vehicle, beast or person suspected of holding or conveying any
dutiable or prohibited article introduced into the Philippines contrary
to law, without mentioning the need of a search warrant in said
cases. 16 But in the search of a dwelling house, the Code provides that
said "dwelling house may be entered and searched only upon
warrant issued by a judge or justice of the peace . . ." 17 It is our
considered view, therefore, that except in the case of the search of a
dwelling house, persons exercising police authority under the customs
law may effect search and seizure without a search warrant in the
enforcement of customs laws.
Our conclusion finds support in the case of Carroll v. United
States, 39 A.L.R., 790, 799, wherein the court, considering a legal
provision similar to Section 2211 of the Philippine Tariff and Customs
Code, said as follows:
"Thus, contemporaneously with the adoption of the 4th
Amendment, we find in the first Congress, and in the following
second and fourth Congresses, a difference made as to the
necessity for a search warrant between goods subject to
forfeiture, when concealed in a dwelling house or similar place,
and like goods in course of transportation and concealed in a
movable vessel, where they readily could be put out of reach of a
search warrant . . ."
"Again, by the 2d section of the Act of March 3, 1815 (3 Stat.
at L. 231, 232, chap. 94), it was made lawful for customs officers
not only to board and search vessels within their own and
adjoining districts, but also to stop, search, and examine any
vehicle, beast, or person on which or whom they should suspect
there was merchandise which was subject to duty or had been
introduced into the United States in any manner contrary to law,
whether by the person in charge of the vehicle or beast or
otherwise, and if they should find any goods, wares, or
merchandise thereon, which they had probable cause to believe
had been so unlawfully brought into the country, to seize and
secure the same, and the vehicle or beast as well, for trial and
forfeiture. This Act was renewed April 27, 1816 (3 Stat. at L. 315,
chap. 100), for a year and expired. The Act of February 28, 1865,
revived § 2 of the Act of 1815, above described, chap. 67, 13 Stat.
at L. 441. The substance of this section was re-enacted in the 3d
section of the Act of July 18, 1866, chap. 201, 14 Stat. at L. 178,
and was thereafter embodied in the Revised Statutes as § 3061,
Comp. Stat. § 5763, 2 Fed. Stat. Anno. 2d ed. p. 1161. Neither §
3061 nor any of its earlier counterparts has ever been attacked as
unconstitutional. Indeed, that section was referred to and treated
as operative by this court in Von Cotzhausen v. Nazro, 107 U. S.
215, 219, 27 L. ed. 540, 541, 2 Sup. Ct. Rep. 503 . . ."
In the instant case, we note that petitioner Martin Alagao and his
companion policemen did not have to make any search before they
seized the two trucks and their cargo. In their original petition, and
amended petition, in the court below Remedios Mago and Valentin
Lanopa did not even allege that there was a search. 18 All that they
complained of was,
"That while the trucks were on their way, they
were intercepted without any search warrant near the Agrifina
Circle and taken to the Manila Police, where they were detained."
But even if there was a search, there is still authority to the
effect that no search warrant would be needed under the
circumstances obtaining in the instant case. Thus, it has been held
that:
"The guaranty of freedom from unreasonable searches and
seizures is construed as recognizing a necessary difference
between a search of a dwelling house or other structure in respect
of which a search warrant may readily be obtained and a search of
a ship, motorboat, wagon, or automobile for contraband goods,
where it is not practicable to secure a warrant, because the
vehicle can be quickly moved out of the locality or jurisdiction in
which the warrant must be sought." (47 Am. Jur., pp. 513-514,
citing Carroll v. United States, 267 U.S., 132, 69 L. ed., 543, 45 S.
Ct., 280, 39 A.L.R., 790; People v. Case, 320 Mich., 379, 190 N.W.,
389, 27 A.L.R., 686.)
In the case of People v. Case (320 Mich., 379, 190 N.W., 389, 27
A.L.R., 686), the question raised by defendant's counsel was whether
an automobile truck or an automobile could be searched without
search warrant or other process and the goods therein seized used
afterwards as evidence in a trial for violation of the prohibition laws of
the State. Same counsel contended the negative, urging the
constitutional provision forbidding unreasonable searches and
seizures. The Court said:
". . . Neither our state nor the Federal Constitution directly
prohibits search and seizure without a warrant, as is sometimes
asserted. Only 'unreasonable' search and seizure is forbidden. . . .
". . . The question whether a seizure or a search is
unreasonable in the language of the Constitution is a judicial and
not a legislative question; but in determining whether a seizure is
or is not unreasonable, all of the circumstances under which it is
made must be looked to.
"The automobile is a swift and powerful vehicle of recent
development, which has multiplied by quantity production and
taken possession of our highways in battalions, until the slower,
animal- drawn vehicles, with their easily noted individuality, are
rare. Constructed as covered vehicles to standard form in immense
quantities, and with a capacity for speed rivaling express trains,
they furnish for successful commission of crime a disguising
means of silent approach and swift escape unknown in the history
of the world before their advent. The question of their police
control and reasonable search on highways or other public places
is a serious question far deeper and broader than their use in so-
called "bootlegging' or 'rum running,' which is itself is no small
matter. While a possession in the sense of private ownership, they
are but a vehicle constructed for travel and transportation on
highways. Their active use is not in homes or on private premises,
the privacy of which the law especially guards from search and
seizure without process. The baffling extent to which they are
successfully utilized to facilitate commission of crime of all
degrees, from those against morality, chastity, and decency, to
robbery, rape, burglary, and murder, is a matter of common
knowledge. Upon that problem a condition, and not a theory,
confronts proper administration of our criminal laws. Whether
search of and seizure from an automobile upon a highway or other
public place without a search warrant is unreasonable is in its
final analysis to be determined as a judicial question in view of all
the circumstances under which it is made."
Having declared that the seizure by the members of the Manila
Police Department of the goods in question was in accordance with
law and by that seizure the Bureau of Customs had acquired
jurisdiction over the goods for the purposes of the enforcement of the
customs and tariff laws, to the exclusion of the Court of First Instance
of Manila, We have thus resolved the principal and decisive issue in
the present case. We do not consider it necessary, for the purposes of
this decision, to discuss the incidental issues raised by the parties in
their pleadings.
WHEREFORE, judgment is hereby rendered, as follows:
(a) Granting the writ of certiorari and prohibition prayed for by
petitioners;
(b) Declaring null and void, for having been issued without
jurisdiction, the order of respondent Judge Hilarion U. Jarencio, dated
March 7, 1967, in Civil Case No. 67496 of the Court of First Instance of
Manila;
(c) Declaring permanent the preliminary injunction issued by this
Court on March 31, 1967 restraining respondent Judge from executing,
enforcing and/or implementing his order of March 7, 1967 in Civil Case
No. 67496 of the Court of First Instance of Manila, and from proceeding
in any manner in said case;
(d) Ordering the dismissal of Civil Case No. 67496 of the Court of
First Instance of Manila; and
(e) Ordering the private respondent, Remedios Mago, to pay the
costs.
It is so ordered.
||| (Papa v. Mago, G.R. No. L-27360, [February 28, 1968], 130 PHIL 886-905)

[G.R. No. 96177. January 27, 1993.]

PEOPLE OF THE PHILIPPINES, plaintiff-appellee, vs. MARI


MUSA y HANTATALU, accused-appellant.
The Solicitor General for plaintiff-appellee.
Pablo L. Murillo for accused-appellant.

SYLLABUS

1. CRIMINAL LAW; DANGEROUS DRUGS ACT (R.A. 6425); ILLEGAL SALE


OF MARIJUANA DRUGS; FAMILIARITY BETWEEN BUYER AND SELLER,
NOT MATERIAL. — The contention that the appellant could not have
transacted with Sgt. Ani because they do not know each other is without
merit. The day before the buy-bust operation, Sgt. Ani conducted a test-
buy and he successfully bought a wrapper of marijuana from the
appellant. Through this previous transaction, Sgt. Ani was able to gain
the appellant's confidence for the latter to sell more marijuana to Sgt.
Ani the following day, during the buy-bust operation. Moreover, the Court
has held that what matters is not an existing familiarity between the
buyer and the seller, for quite often, the parties to the transaction may be
strangers, but their agreement and the acts constituting the sale and
delivery of the marijuana.
2. ID.; ID.; ID.; PRESENCE OF OTHER PEOPLE, NOT CRUCIAL. — The
appellant, again to cast doubt on the credibility of Sgt. Ani, argues that it
was impossible for the appellant to sell marijuana while his wife, cousin
and manicurist were present. But the place of the commission of the
crime of selling prohibited drugs has been held to be not crucial and the
presence of other people apart from the buyer and seller will not
necessarily prevent the consummation of the illegal sale. As the Court
observed in People v. Paco, these factors may sometimes camouflage the
commission of the crime. In the instant case, the fact that the other
people inside the appellant's house are known to the appellant may have
given him some assurance that these people will not report him to the
authorities.
3. ID.; ID.; ID.; CASE OF PEOPLE VS. ALE, NOT APPLICABLE IN CASE AT
BAR. — The case of People v. Ale does not apply here because the
policeman in that case testified that he and his companion were certain
that the appellant therein handed marijuana cigarettes to the poseur-
buyer based on the appearance of the cigarette sticks. The Court
rejected this claim. In the case at bar, however, T/Sgt. Belarga did not
positively claim that he saw the appellant hand over marijuana to Sgt.
Ani. What he said was that there was an exchange of certain articles
between the two. Contrary to the contention of the appellant, it was not
impossible for T/Sgt. Belarga to have seen, from a distance of 90-100
meters, Sgt. Ani hand to the appellant "something" and for the latter to
give to the former "something."
4. REMEDIAL LAW; EVIDENCE; CORROBORATIVE EVIDENCE SUPPORTING
DIRECT EVIDENCE; SUFFICIENT TO PROVE THE CRIME COMMITTED. —
Notwithstanding the fact that T/Sgt. Belarga could not have been certain
that what Sgt. Ani received from the appellant was marijuana because of
the distance, his testimony, nevertheless, corroborated the direct
evidence, which the Court earlier ruled to be convincing. The
corroborative testimony of T/Sgt. Belarga strengthens the direct
evidence given by Sgt. Ani. Additionally, the Court has ruled that the fact
that the police officers who accompanied the poseur-buyer were unable
to see exactly what the appellant gave the poseur-buyer because of their
distance or position will not be fatal to the prosecution's case provided
there exists other evidence, direct or circumstantial, e.g., the testimony
of the poseur-buyer, which is sufficient to prove the consummation of the
sale of the prohibited drug.
5. CONSTITUTIONAL LAW; BILL OF RIGHTS; FREEDOM AGAINST
UNREASONABLE SEARCH AND SEIZURE; EVIDENCE OBTAINED IN
VIOLATION THEREOF. — Built into the Constitution are guarantees on the
freedom of every individual against unreasonable searches and seizures.
Furthermore, the Constitution, in conformity with the doctrine laid down
in Stonehill v. Diokno, (G.R. No. L-19550, June 19, 1967, 20 SCRA 383)
declares inadmissible, any evidence obtained in violation of the freedom
from unreasonable searches and seizures.
6. REMEDIAL LAW; CRIMINAL PROCEDURE; SEARCH & SEIZURE; SEARCH
INCIDENTAL TO LAWFUL ARREST. — While a valid search warrant is
generally necessary before a search and seizure may be effected,
exceptions to this rule are recognized. Thus, in Alvero v. Dizon, the Court
stated that "[t]he most important exception to the necessity for a search
warrant is the right of search and seizure as an incident to a lawful
arrest." Rule 126, Section 12 of the Rules of Court expressly authorizes a
warrantless search and seizure incident to a lawful arrest. There is no
doubt that the warrantless search incidental to a lawful arrest authorizes
the arresting officer to make a search upon the person of the person
arrested. As early as 1909, the Court has ruled that "[a]n officer making
an arrest may take from the person arrested and money or property
found upon his person which was used in the commission of the crime or
was the fruit of the crime or which might furnish the prisoner with the
means of committing violence or of escaping, or which may be used as
evidence in the trial of the cause . . ." Hence, in a buy-bust operation
conducted to entrap a drug-pusher, the law enforcement agents may
seize the marked money found on the person of the pusher immediately
after the arrest even without arrest and search warrants.
7. ID.; ID.; ID.; ID.; DOCTRINE OF "PLAIN VIEW". — The warrantless search
and seizure, as an incident to a suspect's lawful arrest, may extend
beyond the person of the one arrested to include the premises or
surroundings under his immediate control. Objects in the "plain view" of
an officer who has the right to be in the position to have that view are
subject to seizure and may be presented as evidence.
8. ID.; ID.; ID.; ID.; ID.; LIMITATION. — The "plain view" doctrine may not,
however, be used to launch unbridled searches and indiscriminate
seizures nor to extend a general exploratory search made solely to find
evidence of defendant's guilt. The "plain view" doctrine is usually applied
where a police officer is not searching for evidence against the accused,
but nonetheless inadvertently comes across an incriminating object. It
has also been suggested that even if an object is observed in "plain
view," the "plain view" doctrine will not justify the seizure of the object
where the incriminating nature of the object is not apparent from the
"plain view" of the object. Stated differently, it must be immediately
apparent to the police that the items that they observe may be evidence
of a crime, contraband, or otherwise subject to seizure.
9. ID.; ID.; ID.; ID.; ID.; NOT APPLICABLE IN CASE AT BAR. — In the instant
case, the appellant was arrested and his person searched in the living
room. Failing to retrieve the marked money which they hoped to find, the
NARCOM agents searched the whole house and found the plastic bag in
the kitchen. The plastic bag was, therefore, not within their "plain view"
when they arrested the appellant as to justify its seizure. The NARCOM
agents had to move from one portion of the house to another before they
sighted the plastic bag. Moreover, when the NARCOM agents saw the
plastic bag hanging in one corner of the kitchen, they had no clue as to
its contents. They had to ask the appellant what the bag contained. When
the appellant refused to respond, they opened it and found the marijuana.
Unlike Ker v. California, where the marijuana was visible to the police
officer's eyes, the NARCOM agents in this case could not have
discovered the inculpatory nature of the contents of the bag had they not
forcibly opened it. Even assuming then, that the NARCOM agents
inadvertently came across the plastic bag because it was within their
"plain view," what may be said to be the object in their "plain view" was
just the plastic bag and not the marijuana. The incriminating nature of
the contents of the plastic bag was not immediately apparent from the
"plain view" of said object. It cannot be claimed that the plastic bag
clearly betrayed its contents, whether by its distinctive configuration, its
transparency, or otherwise, that its contents are obvious to an observer.
We, therefore, hold that under the circumstances of the case, the "plain
view" doctrine does not apply and the marijuana contained in the plastic
bag was seized illegally and cannot be presented in evidence pursuant to
Article III, Section 3(2) of the Constitution.
10. REMEDIAL LAW; EVIDENCE; QUANTUM OF PROOF REQUIRED IN
CRIMINAL; SATISFIED IN CASE AT BAR. — By virtue of the testimonies of
Sgt. Ani and T/Sgt. Belarga and the two wrappings of marijuana sold by
the appellant to Sgt. Ani, among other pieces of evidence, the guilt of the
appellant of the crime charged has been proved beyond reasonable
doubt.

DECISION

ROMERO, J : p

The appellant, Mari Musa, seeks, in this appeal, the reversal of the
decision, dated August 31, 1990, 1 of the Regional Trial Court (RTC) of
Zamboanga City, Branch XII, finding him guilty of selling marijuana in
violation of Article II, Section 4 of Republic Act No. 6425, as amended,
otherwise known as the Dangerous Drugs Act of 1972.
The information filed on December 15, 1989 against the appellant reads:
"That on or about December 14, 1989, in the City of Zamboanga,
Philippines, and within the jurisdiction of this Honorable Court, the
above-named accused, not being authorized by law, did then and
there, wilfully, unlawfully and feloniously sell to one SGT. AMADO
ANI, two (2) wrappers containing dried marijuana leaves, knowing
the same to be a prohibited drug.
CONTRARY TO LAW." 2

Upon his arraignment on January 11, 1990, the appellant pleaded not
guilty. 3
At the trial, the prosecution presented three (3) witnesses, namely: (1)
Sgt. Amado Ani, Jr. of the 9th Narcotics Command (NARCOM) of
Zamboanga City, who acted as poseur-buyer in the buy-bust operation
made against the appellant; (2) T/Sgt. Jesus Belarga, also of the 9th
Narcotics Command of Zamboanga City, who was the NARCOM team
leader of the buy-bust operation; and (3) Athena Elisa P. Anderson, the
Document Examiner and Forensic Chemist of PC-INP Crime Laboratory of
Regional Command (RECOM) 9. The evidence of the prosecution was
summarized by the trial court as follows: LLjur

"Prosecution evidence shows that in the morning of December 13,


1989, T/Sgt. Jesus Belarga, leader of a NARCOTICS COMMAND
(NARCOM) team based at Calarian, Zamboanga City, instructed
Sgt. Amado Ani to conduct surveillance and test buy on a certain
Mari Musa of Suterville, Zamboanga City. Information received
from civilian informer was that this Mari Musa was engaged in
selling marijuana in said place. So Sgt. Amado Ani, another
NARCOM agent, proceeded to Suterville, in company with a
NARCOM civilian informer, to the house of Mari Musa to which
house the civilian informer had guided him. The same civilian
informer had also described to him the appearance of Mari Musa.
Amado Ani was able to buy one newspaper-wrapped dried
marijuana (Exh. 'E') for P10.00. Sgt. Ani returned to the NARCOM
office and turned over the newspaper-wrapped marijuana to T/Sgt.
Jesus Belarga. Sgt. Belarga inspected the stuff turned over to him
and found it to be marijuana.
The next day, December 14, 1989, about 1:30 P.M., a buy-bust was
planned. Sgt. Amado Ani was assigned as the poseur buyer for
which purpose he was given P20.00 (with SN GA955883) by
Belarga. The buy-bust money had been taken by T/Sgt. Jesus
Belarga from M/Sgt. Noh Sali Mihasun, Chief of Investigation
Section, and for which Belarga signed a receipt (Exh. 'L' & 'L-1').
The team under Sgt. Foncargas was assigned as back-up security.
A pre-arranged signal was arranged consisting of Sgt. Ani's raising
his right hand, after he had succeeded to buy the marijuana. The
two NARCOM teams proceeded to the target site in two civilian
vehicles. Belarga's team was composed of Sgt. Belarga, team
leader, Sgt. Amado Ani, poseur buyer, Sgt. Lego and Sgt. Biong.
Arriving at the target site, Sgt. Ani proceeded to the house of Mari
Musa, while the rest of the NARCOM group positioned themselves
at strategic places about 90 to 100 meters from Mari Musa's
house. T/Sgt. Belarga could see what went on between Ani and
suspect Mari Musa from where he was. Ani approached Mari Musa,
who came out of his house, and asked Ani what he wanted. Ani
said he wanted some more stuff. Ani gave Mari Musa the P20.00
marked money. After receiving the money, Mari Musa went back to
his house and came back and gave Amado Ani two newspaper
wrappers containing dried marijuana. Ani opened the two wrappers
and inspected the contents. Convinced that the contents were
marijuana, Ani walked back towards his companions and raised his
right hand. The two NARCOM teams, riding the two civilian
vehicles, sped towards Sgt. Ani. Ani joined Belarga's team and
returned to the house.
At the time Sgt. Ani first approached Mari Musa, there were four
persons inside his house: Mari Musa, another boy, and two women,
one of whom Ani and Belarga later came to know to be Mari Musa's
wife. The second time, Ani with the NARCOM team returned to Mari
Musa's house, the woman, who was later known as Mari Musa's
wife, slipped away from the house. Sgt. Belarga frisked Mari Musa
but could not find the P20.00 marked money with him. Mari Musa
was then asked where the P20.00 was and he told the NARCOM
team he has given the money to his wife (who had slipped away).
Sgt. Belarga also found a plastic bag containing dried marijuana
inside it somewhere in the kitchen. Mari Musa was then placed
under arrest and brought to the NARCOM office. At Suterville, Sgt.
Ani turned over to Sgt. Belarga the two newspaper-wrapped
marijuana he had earlier bought from Mari Musa (Exhs. 'C' & 'D'). LexLib

In the NARCOM office, Mari Musa first gave his name as Hussin
Musa. Later on, Mari Musa gave his true name - Mari Musa. T/Sgt.
Jesus Belarga turned over the two newspaper-wrapped marijuana
(bought at the buy-bust), the one newspaper-wrapped marijuana
(bought at the test-buy) and the plastic bag containing more
marijuana (which had been taken by Sgt. Lego inside the kitchen of
Mari Musa) to the PC Crime Laboratory, Zamboanga City, for
laboratory examination. The turnover of the marijuana specimen to
the PC Crime Laboratory was by way of a letter-request, dated
December 14, 1989 (Exh. 'B'), which was stamped 'RECEIVED' by
the PC Crime Laboratory (Exh. 'E-1') on the same day.
Mrs. Athena Elisa P. Anderson, the Forensic Chemist of the PC
Crime Laboratory, examined the marijuana specimens subjecting
the same to her three tests. All submitted specimens she
examined gave positive results for the presence of marijuana. Mrs.
Anderson reported the results of her examination in her Chemistry
Report D-100-89, dated December 14, 1989, (Exh. 'J', 'J-1', 'J-2', 'J-
3', 'J-4' and 'J-5'). Mrs. Anderson identified in court the two
newspaper wrapped marijuana bought at the buy-bust on
December 14, 1989, through her initial and the weight of each
specimen written with red ink on each wrapper (Exhs. 'C-1' and 'D-
1'). She also identified the one newspaper-wrapped marijuana
bought at the test-buy on December 13, 1989, through her
markings (Exh. 'E-1'). Mrs. Anderson also identified her Chemistry
Report (Exh. 'J' & sub-markings.)
T. Sgt. Belarga identified the two buy-bust newspaper wrapped
marijuana through his initial, the words 'buy-bust' and the words
'December 14, 1989, 2:45 P.M.' (written on Exhs. 'C' and 'D').
Belarga also identified the receipt of the P20 marked money (with
SN GA955883) (Exh. 'L'), dated December 14, 1989, and his
signature thereon (Exh. 'L-1'). He also identified the letter-request,
dated December 14, 1989, addressed to the PC Crime Laboratory
(Exh. 'B') and his signature thereon (Exh. 'B-2') and the stamp of
the PC Crime Laboratory marked 'RECEIVED' (Exh. 'B-1')." 4
For the defense, the following testified as witnesses: (1) the accused-
appellant Mari H. Musa; and (2) Ahara R. Musa, his wife. The trial court
summarized the version of the defense, thus:
"[O]n December 14, 1989, at about 1:30 in the afternoon, Mari Musa
was in his house at Suterville, Zamboanga City. With him were his
wife, Ahara Musa, known as Ara, his one-year old child, a woman
manicurist, and a male cousin named Abdul Musa. About 1:30 that
afternoon, while he was being manicured at one hand, his wife was
inside the one room of their house, putting their child to sleep.
Three NARCOM agents, who introduced themselves as NARCOM
agents, dressed in civilian clothes, got inside Mari Musa's house
whose door was open. The NARCOM agents did not ask permission
to enter the house but simply announced that they were NARCOM
agents. The NARCOM agents searched Mari Musa's house and Mari
Musa asked them if they had a search warrant. The NARCOM
agents were just silent. The NARCOM agents found a red plastic
bag whose contents, Mari Musa said, he did not know. He also did
not know if the plastic bag belonged to his brother, Faisal, who was
living with him, or his father, who was living in another house
about ten arms-length away. Mari Musa, then, was handcuffed and
when Mari Musa asked why, the NARCOM agents told him for
clarification.
Mari Musa was brought in a pick-up, his wife joining him to the
NARCOM Office at Calarian, Zamboanga City. Inside the NARCOM
Office, Mari Musa was investigated by one NARCOM agent which
investigation was reduced into writing. The writing or document
was interpreted to Mari Musa in Tagalog. The document stated that
the marijuana belonged to Mari Musa and Mari Musa was asked to
sign it. But Mari Musa refused to sign because the marijuana did
not belong to him. Mari Musa said he was not told that he was
entitled to the assistance of counsel, although he himself told the
NARCOM agents he wanted to be assisted by counsel.
Mari Musa said four bullets were then placed between the fingers
of his right hand and his fingers were pressed which felt very
painful. The NARCOM agents boxed him and Mari Musa lost
consciousness. While Mari Musa was maltreated, he said his wife
was outside the NARCOM building. The very day he was arrested
(on cross-examination Mari Musa said it was on the next day), Mari
Musa was brought to the Fiscal's Office by three NARCOM agents.
The fiscal asked him if the marijuana was owned by him and he
said "not." After that single question, Mari Musa was brought to
the City Jail. Mari Musa said he did not tell the fiscal that he had
been maltreated by the NARCOM agents because he was afraid he
might be maltreated in the fiscal's office.
cdll

Mari Musa denied the NARCOM agents' charge that he had sold
two wrappers of marijuana to them; that he had received from
them a P20.00 bill which he had given to his wife. He did not sell
marijuana because he was afraid that was against the law and that
the person selling marijuana was caught by the authorities; and he
had a wife and a very small child to support. Mari Musa said he had
not been arrested for selling marijuana before. 5
After trial, the trial court rendered the assailed decision with the
following disposition:
"WHEREFORE, finding accused Mari Musa y Hantatalu guilty
beyond reasonable doubt of selling marijuana and pursuant to Sec.
4, Art II of Rep. Act No. 6425, he is sentenced to life imprisonment
and to pay the fine of P20,000.00, the latter imposed without
subsidiary imprisonment." 6
In this appeal, the appellant contends that his guilt was not proved
beyond reasonable doubt and impugns the credibility of the prosecution
witnesses.
The appellant claims that the testimony of Sgt. Ani, the poseur-buyer, is
not credible because: (1) prior to the buy-bust operation, neither Sgt. Ani
nor the other NARCOM agents were personally known by the appellant or
vice-versa; and (2) there was no witness to the alleged giving of the two
wrappers of marijuana by the appellant to Sgt. Ani.
Sgt. Ani testified that on December 13, 1989, upon instruction by T/Sgt.
Jesus Belarga, he conducted a test-buy operation on the appellant
whereby he bought one wrapper of marijuana for P15.00 from the
latter. 7 He reported the successful operation to T/Sgt. Belarga on the
same day. 8 Whereupon, T/Sgt. Belarga conducted a conference to
organize a buy-bust operation for the following day. 9

On December 14, 1989, at 1:30 p.m., two NARCOM teams in separate


vehicles headed by T/Sgt. Belarga and a certain Sgt. Foncardas went to
the place of operation, which was the appellant's house located in
Laquian Compound, Suterville, Zamboanga City. Sgt. Ani was with the
team of T/Sgt. Belarga, whose other members were Sgts. Lego and
Biong. 10 Sgt. Ani was given a marked P20.00 bill by T/Sgt. Belarga, which
was to be used in the operation.
Upon reaching the place, the NARCOM agents positioned themselves at
strategic places. 11 Sgt. Ani approached the house. Outside the house,
the appellant asked Sgt. Ani what he wanted. Sgt. Ani asked him for
some more marijuana. 12 Sgt. Ani gave him the marked P20.00 bill and the
appellant went inside the house and brought back two paper wrappers
containing marijuana which he handed to Sgt. Ani. 13 From his position,
Sgt. Ani could see that there were other people in the house. 14
After the exchange, Sgt. Ani approached the other NARCOM agents and
made the pre-arranged signal of raising his right hand. 15 The NARCOM
agents, accompanied by Sgt. Ani, went inside the house and made the
arrest. The agents searched the appellant and unable to find the marked
money, they asked him where it was. The appellant said that he gave it to
his wife. 16
The Court, after a careful reading of the record, finds the testimony of
Sgt. Ani regarding the buy-bust operation, which resulted in the
apprehension, prosecution and subsequent conviction of the appellant,
to be direct, lucid and forthright. Being totally untainted by
contradictions in any of the material points, it deserves credence.
The contention that the appellant could not have transacted with Sgt.
Ani because they do not know each other is without merit. The day
before the buy-bust operation, Sgt. Ani conducted a test-buy and he
successfully bought a wrapper of marijuana from the appellant. Through
this previous transaction, Sgt. Ani was able to gain the appellant's
confidence for the latter to sell more marijuana to Sgt. Ani the following
day, during the buy-bust operation. Moreover, the Court has held that
what matters is not an existing familiarity between the buyer and the
seller, for quite often, the parties to the transaction may be strangers,
but their agreement and the acts constituting the sale and delivery of the
marijuana. 17
The appellant, again to cast doubt on the credibility of Sgt. Ani, argues
that it was impossible for the appellant to sell marijuana while his wife,
cousin and manicurist were present. But the place of the commission of
the crime of selling prohibited drugs has been held to be not
crucial 18 and the presence of other people apart from the buyer and
seller will not necessarily prevent the consummation of the illegal sale.
As the Court observed in People v. Paco, 19 these factors may sometimes
camouflage the commission of the crime. In the instant case, the fact
that the other people inside the appellant's house are known to the
appellant may have given him some assurance that these people will not
report him to the authorities.
cdll

The appellant, besides assailing Sgt. Ani's credibility, also questions the
credibility of T/Sgt. Belarga. The appellant submits that since T/Sgt.
Belarga admitted that he was about 90 meters away from Sgt. Ani and
the appellant, he could not have possibly witnessed the sale. The
appellant invokes People v. Ale 20 where the Court observed that from a
distance of 10-15 meters, a policeman cannot distinguish between
marijuana cigarette from ordinary ones by the type of rolling done on the
cigarette sticks. And since T/Sgt. Belarga allegedly did not see the sale,
the appellant contends that the uncorroborated testimony of Sgt. Ani can
not stand as basis for his conviction.
People v. Ale does not apply here because the policeman in that case
testified that he and his companion were certain that the appellant
therein handed marijuana cigarettes to the poseur-buyer based on the
appearance of the cigarette sticks. The Court rejected this claim, stating
that:
"This Court cannot give full credit to the testimonies of the
prosecution witnesses marked as they are with contradictions and
tainted with inaccuracies.
Biñan testified that they were able to tell that the four cigarettes
were marijuana cigarettes because according to him, the rolling of
ordinary cigarettes are different from those of marijuana
cigarettes. (tsn, November 13, 1984, p. 10).
It is however, incredible to believe that they could discern the type
of rolling done on those cigarettes from the distance where they
were observing the alleged sale of more or less 10 to 15 meters." 21
In the case at bar, however, T/Sgt. Belarga did not positively claim that he
saw the appellant hand over marijuana to Sgt. Ani. What he said was that
there was an exchange of certain articles between the two. The relevant
portion of T/Sgt. Belarga's testimony reads: 2 2
Q Now, do you remember whether Sgt. Ani was able to reach the
house of Mari Musa?
A Yes, ma'am.
Q After reaching Mari Musa, did you see what happened (sic)?
A Yes, ma'am.
Q Could you please tell us?
A From our vehicle the stainless owner type jeep where Sgt. Lego,
Sgt. Biong were boarded, I saw that Sgt. Ani proceeded to the
house near the road and he was met by one person and later
known as Mari Musa who was at the time wearing short
pants and later on I saw that Sgt. Ani handed something to
him, thereafter received by Mari Musa and went inside the
house and came back later and handed something to Sgt. Ani.
Contrary to the contention of the appellant, it was not impossible for
T/Sgt. Belarga to have seen, from a distance of 90-100 meters, Sgt. Ani
hand to the appellant "something" and for the latter to give to the
former "something."
Notwithstanding the fact that T/Sgt. Belarga could not have been certain
that what Sgt. Ani received from the appellant was marijuana because of
the distance, his testimony, nevertheless, corroborated the direct
evidence, which the Court earlier ruled to be convincing, presented by
Sgt. Ani on the following material points: (1) T/Sgt. Belarga instructed
Sgt. Ani to conduct a surveillance and test-buy operation on the
appellant at Suterville, Zamboanga City on December 13, 1939; 23 (2) later
that same day, Sgt. Ani went back to their office and reported a
successful operation and turned over to T/Sgt. Belarga one wrapper of
marijuana; 24 (3) T/Sgt. Belarga then organized a team to conduct a buy-
bust operation the following day; 25 (4) on December 14, 1989, T/Sgt.
Belarga led a team of NARCOM agents who went to Suterville,
Zamboanga City; 26 (5) T/Sgt. Belarga gave a P20.00 marked bill to Sgt.
Ani which was to be used in the buy-bust operation; 27 (6) upon the arrival
of the NARCOM agents in Suterville, Zamboanga City, Sgt. Ani proceeded
to the house of the appellant while some agents stayed in the vehicles
and others positioned themselves in strategic places; 28 the appellant
met Sgt. Ani and an exchange of articles took place. 29
The corroborative testimony of T/Sgt. Belarga strengthens the direct
evidence given by Sgt. Ani. Additionally, the Court has ruled that the fact
that the police officers who accompanied the poseur-buyer were unable
to see exactly what the appellant gave the poseur-buyer because of their
distance or position will not be fatal to the prosecution's case 30 provided
there exists other evidence, direct or circumstantial, e.g., the testimony
of the poseur-buyer, which is sufficient to prove the consummation of the
sale of the prohibited drug.cdll

The appellant next assails the seizure and admission as evidence of a


plastic bag containing marijuana which the NARCOM agents found in the
appellant's kitchen. It appears that after Sgt. Ani gave the pre-arranged
signal to the other NARCOM agents, the latter moved in and arrested the
appellant inside the house. They searched him to retrieve the marked
money but didn't find it. Upon being questioned, the appellant said that
he gave the marked money to his wife. 31 Thereafter, T/Sgt. Belarga and
Sgt. Lego went to the kitchen and noticed what T/Sgt. Belarga described
as a "cellophane colored white and stripe hanging at the corner of the
kitchen." 32 They asked the appellant about its contents but failing to get
a response, they opened it and found dried marijuana leaves. At the trial,
the appellant questioned the admissibility of the plastic bag and the
marijuana it contains but the trial court issued an Order ruling that these
are admissible in evidence. 33
Built into the Constitution are guarantees on the freedom of every
individual against unreasonable searches and seizures by providing in
Article III, Section 2, the following:
"The right of the people to be secure in their persons, houses,
papers, and effects against unreasonable searches and seizures of
whatever nature and for any purpose shall be inviolable, and no
search warrant or warrant of arrest shall issue except upon
probable cause to be determined personally by the judge after
examination under oath or affirmation of the complainant and the
witness he may produce, and particularly describing the place to
be searched and the persons or things to be seized."
Furthermore, the Constitution, in conformity with the doctrine laid down
in Stonehill v. Diokno, 34 declares inadmissible, any evidence obtained in
violation of the freedom from unreasonable searches and seizures. 35
While a valid search warrant is generally necessary before a search and
seizure may be effected, exceptions to this rule are recognized. Thus,
in Alvero v. Dizon, 36 the Court stated that "[t]he most important
exception to the necessity for a search warrant is the right of search and
seizure as an incident to a lawful arrest." 37

Rule 126, Section 12 of the Rules of Court expressly authorizes a


warrantless search and seizure incident to a lawful arrest, thus:
SECTION 12. Search incident to lawful arrest. — A person lawfully
arrested may be searched for dangerous weapons or anything
which may be used as proof of the commission of an offense,
without a search warrant.
There is no doubt that the warrantless search incidental to a lawful
arrest authorizes the arresting officer to make a search upon the person
of the person arrested. As early as 1909, the Court has ruled that "[a]n
officer making an arrest may take from the person arrested and money or
property found upon his person which was used in the commission of the
crime or was the fruit of the crime or which might furnish the prisoner
with the means of committing violence or of escaping, or which may be
used as evidence in the trial of the cause." 38 Hence, in a buy-bust
operation conducted to entrap a drug-pusher, the law enforcement
agents may seize the marked money found on the person of the pusher
immediately after the arrest even without arrest and search warrants. 39
In the case at bar, the NARCOM agents searched the person of the
appellant after arresting him in his house but found nothing. They then
searched the entire house and, in the kitchen, found and seized a plastic
bag hanging in a corner. LLjur

The warrantless search and seizure, as an incident to a suspect's lawful


arrest, may extend beyond the person of the one arrested to include the
premises or surroundings under his immediate control. 40 Objects in the
"plain view" of an officer who has the right to be in the position to have
that view are subject to seizure and may be presented as evidence. 41
In Ker v. California, 42 police officers, without securing a search warrant
but having information that the defendant husband was selling marijuana
from his apartment, obtained from the building manager a passkey to
defendants' apartment, and entered it. There they found the defendant
husband in the living room. The defendant wife emerged from the
kitchen, and one of the officers, after identifying himself, observed
through the open doorway of the kitchen, a small scale atop the kitchen
sink, upon which lay a brick-shaped package containing green leafy
substance which he recognized as marijuana. The package of marijuana
was used as evidence in prosecuting defendants for violation of the
Narcotic Law. The admissibility of the package was challenged before
the U.S. Supreme Court, which held, after observing that it was not
unreasonable for the officer to walk to the doorway of the adjacent
kitchen on seeing the defendant wife emerge therefrom, that "the
discovery of the brick of marijuana did not constitute a search, since the
officer merely saw what was placed before him in full view." 43 The U.S.
Supreme Court ruled that the warrantless seizure of the marijuana was
legal on the basis of the "plain view" doctrine and upheld the
admissibility of the seized drugs as part of the prosecution's evidence. 44
The "plain view" doctrine may not, however, be used to launch unbridled
searches and indiscriminate seizures nor to extend a general exploratory
search made solely to find evidence of defendant's guilt. The "plain view"
doctrine is usually applied where a police officer is not searching for
evidence against the accused, but nonetheless inadvertently comes
across an incriminating object. 45 Furthermore, the U.S. Supreme Court
stated the following limitations on the application of the doctrine:
"What the 'plain view' cases have in common is that the police
officer in each of them had a prior justification for an intrusion in
the course of which he came inadvertently across a piece of
evidence incriminating the accused. The doctrine serves to
supplement the prior justification — whether it be a warrant for
another object, hot pursuit, search incident to lawful arrest, or
some other legitimate reason for being present unconnected with
a search directed against the accused — and permits the
warrantless seizure. Of course, the extension of the original
justification is legitimate only where it is immediately apparent to
the police that they have evidence before them; the 'plain view'
doctrine may not be used to extend a general exploratory search
from one object to another until something incriminating at last
emerges." 46
It has also been suggested that even if an object is observed in "plain
view," the "plain view" doctrine will not justify the seizure of the object
where the incriminating nature of the object is not apparent from the
"plain view" of the object. 47 Stated differently, it must be immediately
apparent to the police that the items that they observe may be evidence
of a crime, contraband, or otherwise subject to seizure.
In the instant case, the appellant was arrested and his person searched
in the living room. Failing to retrieve the marked money which they hoped
to find, the NARCOM agents searched the whole house and found the
plastic bag in the kitchen. The plastic bag was, therefore, not within their
"plain view" when they arrested the appellant as to justify its seizure.
The NARCOM agents had to move from one portion of the house to
another before they sighted the plastic bag. Unlike Ker v. California,
where the police officer had reason to walk to the doorway of the
adjacent kitchen and from which position he saw the marijuana, the
NARCOM agents in this case went from room to room with the obvious
intention of fishing for more evidence.
Moreover, when the NARCOM agents saw the plastic bag hanging in one
corner of the kitchen, they had no clue as to its contents. They had to
ask the appellant what the bag contained. When the appellant refused to
respond, they opened it and found the marijuana. Unlike Ker v. California,
where the marijuana was visible to the police officer's eyes, the NARCOM
agents in this case could not have discovered the inculpatory nature of
the contents of the bag had they not forcibly opened it. Even assuming
then, that the NARCOM agents inadvertently came across the plastic bag
because it was within their "plain view," what may be said to be the
object in their "plain view" was just the plastic bag and not the
marijuana. The incriminating nature of the contents of the plastic bag
was not immediately apparent from the "plain view" of said object. It
cannot be claimed that the plastic bag clearly betrayed its contents,
whether by its distinctive configuration, its transparency, or otherwise,
that its contents are obvious to an observer. 48
We, therefore, hold that under the circumstances of the case, the "plain
view" doctrine does not apply and the marijuana contained in the plastic
bag was seized illegally and cannot be presented in evidence pursuant to
Article III, Section 3(2) of the Constitution.
cdrep

The exclusion of this particular evidence does not, however, diminish, in


any way, the damaging effect of the other pieces of evidence presented
by the prosecution to prove that the appellant sold marijuana, in violation
of Article II, Section 4 of the Dangerous Drugs Act of 1972. We hold that
by virtue of the testimonies of Sgt. Ani and T/Sgt. Belarga and the two
wrappings of marijuana sold by the appellant to Sgt. Ani, among other
pieces of evidence, the guilt of the appellant of the crime charged has
been proved beyond reasonable doubt.
WHEREFORE, the appeal is DISMISSED and the judgment of the Regional
Trial Court AFFIRMED.
SO ORDERED.
(People v. Musa y Hantatalu, G.R. No. 96177, [January 27, 1993], 291
|||

PHIL 623-642)
[G.R. No. 145176. March 30, 2004.]

PEOPLE OF THE PHILIPPINES, appellee,vs.SANTIAGO


PERALTA y POLIDARIO (at large),ARMANDO DATUIN JR. y
GRANADOS (at large),ULYSSES GARCIA y TUPAS, MIGUELITO
DE LEON y LUCIANO, LIBRANDO FLORES y CRUZ and
ANTONIO LOYOLA y SALISI,accused,

ULYSSES GARCIA y TUPAS, MIGUELITO DE LEON y LUCIANO,


LIBRANDO FLORES y CRUZ and ANTONIO LOYOLA y
SALISI, appellants.

DECISION
PANGANIBAN, J : p

The right of the accused to counsel demands effective, vigilant and


independent representation. The lawyer's role cannot be reduced to
being that of a mere witness to the signing of an extra-judicial
confession.
The Case
Before the Court is an appeal from the August 21, 2000 Decision 1 of
the Regional Trial Court (RTC) of Manila (Branch 18) in Criminal Case No.
92-112322. Appellants Ulysses Garcia y Tupas, Miguelito de Leon y
Luciano, Librando Flores y Cruz and Antonio Loyola y Salisi, as well as
their co-accused — Santiago Peralta y Polidario and Armando Datuin Jr. y
Granados — were convicted therein of qualified theft. The dispositive
portion of the Decision reads:
"WHEREFORE, the accused, Santiago Peralta y Polidario,
Armando Datuin, Jr. y Granados, Ulysses Garcia y Tupas, Miguelito
De Leon y Luciano, Librando Flores y Cruz and Antonio Loyola y
Salisi, are hereby convicted of the crime of qualified theft of
P194,190.00 and sentenced to suffer the penalty of reclusion
perpetua with all the accessory penalties provided by law, and to
pay the costs. Moreover, all the accused are ordered to pay the
Central Bank of the Philippines, now Bangko Sentral ng Pilipinas,
actual damages in the sum of P194,190.00 with interest thereon at
the legal rate from the date of the filing of this action, November 9,
1992, until fully paid." 2
In an Information dated November 9, 1992, 3 appellants and their co-
accused were charged as follows:
"That sometime in the year 1990 and including November 4,
1992, in the City of Manila, Philippines, the said accused,
conspiring and confederating with others whose true names,
identities and present whereabouts are still unknown and helping
one another, did then and there wilfully, unlawfully and feloniously,
with intent to gain and without the knowledge and consent of the
owner thereof, take, steal and carry away punctured currency
notes due for shredding in the total amount of P194,190.00,
belonging to the Central Bank of the Philippines as represented by
Pedro Labita y Cabriga, to the damage and prejudice of the latter
in the aforesaid sum of P194,190.00 Philippine currency;
"That said accused Santiago Peralta y Polidario, Armando
Datuin, Jr. y Granados, Ulysses Garcia y Tupas, Miguelito de Leon y
Luciano and Antonio Loyola y Salisi committed said offense with
grave abuse of confidence they being at the time employed as
Currency Reviewers, Driver, Currency Assistant I and Money
Counter of the offended party and as such they had free access to
the property stolen." 4
Garcia was arrested on November 4, 1992; and his co-accused, on
November 9, 1992. Appellants, however, obtained two Release Orders
from RTC Vice Executive Judge Corona Ibay-Somera on November 9 and
10, 1992, upon their filing of a cash bond to secure their appearance
whenever required by the trial court. 5
During their arraignment on May 4, 1993, appellants, assisted by
their respective counsels, pleaded not guilty. 6 On September 30, 1998,
the trial court declared that Datuin Jr. and Peralta were at large, because
they had failed to appear in court despite notice. 7
After trial in due course, they were all found guilty and convicted of
qualified theft in the appealed Decision.
The Facts
Version of the Prosecution
The Office of the Solicitor General (OSG) presents the prosecution's
version of the facts as follows:
"About 10:00 o'clock in the morning of November 4, 1992,
Pedro Labita of Central Bank of the Philippines (CBP) [now Bangko
Sentral ng Pilipinas (BSP)] went to the Theft and Robbery Section
of Western Police District Command (WPDC),and filed a complaint
for Qualified Theft against Santiago Peralta, Armando Datuin,
Jr.,Ulysses Garcia, Miguelito de Leon, Librando Flores and Antonio
S. Loyola.
"Pedro Labita submitted to SPO4 Cielito Coronel, the
investigating officer at WPDC, punctured currency notes in P100.00
and P500.00 bills with a face value of Php194,190.00. Said notes
were allegedly recovered by the BSP Cash Department during its
cash counting of punctured currency bills submitted by different
banks to the latter. The punctured bills were rejected by the BSP
money counter machine and were later submitted to the
investigation staff of the BSP Cash Department. As a result of the
investigation, it was determined that said rejected currency bills
were actually punctured notes already due for shredding. These
currency bills were punctured because they were no longer
intended for circulation. Before these notes could be shredded,
they were stolen from the BSP by the above-named accused.
"On the basis of the complaint filed by Pedro Labita, Ulysses
Garcia was apprehended in front of Golden Gate Subdivision, Las
Piñas City, while he was waiting for a passenger bus on his way to
the BSP. Garcia was brought to the police station for investigation.
"On November 4, 5 and 6, 1992, while in the custody of the
police officers, Garcia gave three separate statements admitting
his guilt and participation in the crime charged. He also identified
the other named accused as his cohorts and accomplices and
narrated the participation of each and everyone of them.
"On the basis of Garcia's sworn statements, the other named
accused were invited for questioning at the police station and
were subsequently charged with qualified theft together with
Garcia." 8 (Citations omitted)
Version of the Defense
The defense states its version of the facts in the following manner:
"Accused-appellant Garcia served as a driver of the armored
car of the Central Bank from 1978 to 1994.
"On November 4, 1992, between 7:00 a.m. and 8:00 a.m.,a
man who had identified himself as a police officer arrested
accused-appellant Garcia while waiting for a passenger bus in
front of the Golden Gate Subdivision, Las Piñas City. He was
arrested without any warrant for his arrest. The police officer who
had arrested accused-appellant Garcia dragged the latter across
the street and forced him to ride ...a car.
"While inside the car, he was blindfolded, his hands were
handcuffed behind his back, and he was made to bend with his
chest touching his knees. Somebody from behind hit him and he
heard some of the occupants of the car say that he would be
salvaged if he would not tell the truth. When the occupants of the
car mentioned perforated notes, he told them that he does not
know anything about those notes.
"After the car had stopped, he was dragged out of the car and
...up and down ...the stairs. While being dragged out of the car, he
felt somebody frisk his pocket.
"At a safe house, somebody mentioned to him the names of
his co-accused and he told them that he does not know his co-
accused. ...Whenever he would deny knowing his co-accused,
somebody would box him on his chest. Somebody poured water on
accused-appellant Garcia's nose while lying on the bench. He was
able to spit out the water that had been poured on his nose [at
first],but somebody covered his mouth. As a result, he could not
breath[e].
"When accused-appellant Garcia realized that he could not
bear the torture anymore, he decided to cooperate with the police,
and they stopped the water pouring and allowed him to sit down.
"Accused-appellant Garcia heard people talking and he heard
somebody utter, 'may nakikinig.' Suddenly his two ears were hit
with open palm[s]....As he was being brought down, he felt
somebody return his personal belongings to his pocket. Accused-
appellant Garcia's personal belongings consisted of [his] driver's
license, important papers and coin purse.
"He was forced to ride ...the car still with blindfold. His
blindfold and handcuffs were removed when he was at the office of
police officer Dante Dimagmaliw at the Western Police District,
U.N. Avenue, Manila.
"SPO4 Cielito Coronel asked accused-appellant Garcia about
the latter's name, age and address. The arrival of Mr. Pedro Labita
of the Cash Department, Central Bank of the Philippines,
interrupted the interview, and Mr. Labita instructed SPO4 Coronel
to get accused-appellant Garcia's wallet and examine the contents
thereof. SPO4 Coronel supposedly found three pieces of P100
perforated bill in accused-appellant Garcia's wallet and the former
insisted that they recovered the said perforated notes from
accused-appellant's wallet. SPO4 Coronel took down the statement
of Mr. Labita.
"It was actually Mr. Labita, and not accused-appellant Garcia,
who gave the answers appearing in accused-appellant Garcia's
alleged three sworn statements dated November 4, 1992,
November 5, 1992 and ...November 6, 1992. cASIED

"At or about 6:00 p.m. on November 5, 1992, accused-


appellant Garcia was brought to the cell of the Theft and Robbery
Section of the WPD. At or about 8:00 p.m.,he was brought to the
office of Col. Alladin Dimagmaliw where his co-accused were also
inside. He did not identify his co-accused, but he merely placed his
hands on the shoulders of each of his co-accused, upon being
requested, and Mr. Labita took ...pictures while he was doing the
said act.
"Accused-appellant Garcia came to know Atty. Francisco
Sanchez of the Public Attorney's Office on November 4, 1992, at
the office of police officer Dante Dimagmaliw, when SPO4 Coronel
introduced Atty. Sanchez to accused-appellant Garcia and told him
that Atty. Sanchez would be his lawyer. However, accused-
appellant Garcia did not agree to have Atty. Sanchez to be his
lawyer. Atty. Sanchez left after talking to SPO4 Coronel, and
accused-appellant Garcia had not met Atty. Sanchez anymore
since then. He was not present when Atty. Sanchez allegedly
signed ...the alleged three (3) sworn statements.

"During the hearing of the case on April 6, 2000, Atty.


Sanchez manifested in open court that he did not assist accused-
appellant Garcia when the police investigated accused-appellant
Garcia, and that he signed ...the three (3) sworn statements only
as a witness thereto.
"Accused-appellant Garcia signed the alleged three sworn
statements due to SPO4 Coronel's warning that if he would not do
so, he would again be tortured by water cure.
"SPO[4] Coronel caused the arrest without any warrant of
accused appellants De Leon, Loyola, [Flores] on the basis of the
complaint of Mr. Pedro Labita, and which arrest was effected on
November 5, 1992, by SPO1 Alfredo Silva and SPO1 Redelico.
"SPO4 Coronel, in his letter dated November 6, 1992,
forwarded the case to the Duty Inquest Prosecutor assigned at the
WPDC Headquarters." 9 (Citations omitted)
Ruling of the Trial Court
The trial court found that all the accused used to work for the BSP.
Garcia was a driver assigned to the Security and Transport Department;
while Peralta, Datuin Jr.,De Leon, Flores and Loyola were laborers
assigned to the Currency Retirement Division. Their main task was to
haul perforated currency notes from the currency retirement vault to the
basement of the BSP building for shredding.
On several occasions, during the period 1990-1992, they handed to
Garcia perforated currency notes placed in a coin sack that he, in turn,
loaded in an armored escort van and delivered to someone waiting
outside the premises of the building. The trial court held that the
coordinated acts of all the accused unerringly led to the conclusion that
they had conspired to pilfer the perforated currency notes belonging to
the BSP.
The RTC rejected the disclaimer by Garcia of his own confessions,
as such disclaimer was "an eleventh hour concoction to exculpate
himself and his co-accused." The trial court found his allegations of
torture and coerced confessions unsupported by evidence. Moreover, it
held that the recovery of three pieces of perforated P100 bills from
Garcia's wallet and the flight of Peralta and Datuin Jr. were indicative of
the guilt of the accused.
Hence, this appeal. 10

Issues
In his Brief, Garcia raises the following issues:
"1
The trial court erred in admitting in evidence the alleged
three Sworn Statements of Accused-appellant Garcia and the
alleged three pieces of P100 perforated notes
"2
The trial court erred in finding the accused-appellant guilty of
qualified theft." 11
In their joint Brief, De Leon, Loyola and Flores interpose this
additional assignment of errors:
"1
The trial court erred in admitting in evidence the alleged
three sworn statements of Accused Ulysses Garcia (Exhibits 'I','J'
and 'K') and the alleged three pieces of P100 perforated notes
(Exhibits 'N' to 'N-2') over the objections of the accused-appellants.
"2
The trial court erred in denying the demurrer to evidence of
Accused-appellants De Leon, Loyola and Flores;
"3
The trial court erred in denying the Motion for
Reconsideration of the Order denying the demurrer to evidence;
"4
The trial court erred when it failed to consider the evidence
adduced by the accused-appellants, consisting of exhibits '1','2' to
'2-B','3' and '4' and the testimony of their witness, State Auditor
Esmeralda Elli;
"5
The trial court erred in finding the accused-appellants guilty
of qualified theft." 12
Simplified, the issues are as follows: (1) the sufficiency of the
evidence against appellants, including the admissibility of Garcia's
confessions and of the three perforated P100 currency notes; and (2) the
propriety of the denial of their demurrer to evidence.
The Court's Ruling
The appeal has merit.
First Issue:
Sufficiency of Evidence
The trial court convicted appellants mainly on the strength of the
three confessions given by Garcia and the three perforated P100
currency notes confiscated from him upon his arrest. Appellants,
however, contend that these pieces of evidence are inadmissible.
Extrajudicial Confessions
Appellants aver that the alleged three Sworn Statements of Garcia
were obtained without the assistance of counsel — in violation of his
rights under Article III, Section 12 (1) and (2) of the 1987 Constitution,
which provides thus:
"SECTION 12. (1) Any person under investigation for the
commission of an offense shall have the right to be informed of his
right to remain silent and to have competent and independent
counsel, preferably of his own choice. If the person cannot afford
the services of counsel, he must be provided with one. These
rights cannot be waived except in writing and in the presence of
counsel.
"(2) No torture, force, violence, threat, intimidation, or any
other means which vitiate the free will shall be used against him.
Secret detention places, solitary,incomunicado,or other similar
forms of detention are prohibited."
On the other hand, the OSG contends that counsel, Atty. Francisco
Sanchez III of the Public Attorney's Office, duly assisted Garcia during
the custodial investigation.
It is clear from a plain reading of the three extrajudicial
confessions 13 that Garcia was not assisted by Atty. Sanchez. The
signature of the latter on those documents was affixed after the word
"SAKSI." Moreover, he appeared in court and categorically testified that
he had not assisted Garcia when the latter was investigated by the
police, and that the former had signed the Sworn Statement only as a
witness. 14
The written confessions, however, were still admitted in evidence
by the RTC on the ground that Garcia had expressed in writing his
willingness and readiness to give the Sworn Statements without the
assistance of counsel. The lower court's action is manifest error.
The right to counsel has been written into our Constitution in order
to prevent the use of duress and other undue influence in extracting
confessions from a suspect in a crime. The basic law specifically
requires that any waiver of this right must be made in
writing and executed in the presence of a counsel. In such case, counsel
must not only ascertain that the confession is voluntarily made and that
the accused understands its nature and consequences, but also advise
and assist the accused continuously from the time the first question is
asked by the investigating officer until the signing of the confession.
Hence, the lawyer's role cannot be reduced to being that of a mere
witness to the signing of a pre-prepared confession, even if it indicated
compliance with the constitutional rights of the accused. 15 The accused
is entitled to effective, vigilant and independent counsel. 16
A waiver in writing, like that which the trial court relied upon in the
present case, is not enough. Without the assistance of a counsel, the
waiver has no evidentiary relevance. 17 The Constitution states that
"[a]ny confession or admission obtained in violation of [the aforecited
Section 12] shall be inadmissible in evidence. . . ." Hence, the trial court
was in error when it admitted in evidence the uncounseled confessions
of Garcia and convicted appellants on the basis thereof. The question of
whether he was tortured becomes moot. CADSHI

Perforated Currency Notes


Appellants contend that the three P100 perforated currency notes
(Exhibits "N" to "N-2") allegedly confiscated from Garcia after his arrest
were "fruits of the poisonous tree" and, hence, inadmissible in evidence.
The solicitor general evades the issue and argues, instead, that
appellants waived the illegality of their arrest when they entered a plea.
He further contends that the exclusion from the evidence of the three
punctured currency bills would not alter the findings of the trial court.
The police arrested Garcia without a warrant, while he had merely
been waiting for a passenger bus after being pointed out by the Cash
Department personnel of the BSP. At the time of his arrest, he had not
committed, was not committing, and was not about to commit any crime.
Neither was he acting in a manner that would engender a reasonable
ground to suspect that he was committing a crime. None of the
circumstances justifying an arrest without a warrant under Section 5 of
Rule 113 of the Rules of Court was present.
Hence, Garcia was not lawfully arrested. Nonetheless, not having
raised the matter before entering his plea, he is deemed to have waived
the illegality of his arrest. Note, however, that this waiver is limited to the
arrest. It does not extend to the search made as an incident thereto or to
the subsequent seizure of evidence allegedly found during the search.
The Constitution proscribes unreasonable searches and
seizures 18 of whatever nature. Without a judicial warrant, these are
allowed only under the following exceptional circumstances: (1) a search
incident to a lawful arrest, (2) seizure of evidence in plain view, (3)
search of a moving motor vehicle, (4) customs search, (5) stop and frisk
situations, and (6) consented search. 19
Where the arrest was incipiently illegal, it follows that the
subsequent search was similarly illegal. 20 Any evidence obtained in
violation of the constitutional provision is legally inadmissible in
evidence under the exclusionary rule. 21 In the present case, the
perforated P100 currency notes were obtained as a result of a search
made without a warrant subsequent to an unlawful arrest; hence, they
are inadmissible in evidence.
Moreover, untenable is the solicitor general's argument that
Appellants De Leon, Flores and Loyola waived the illegality of the arrest
and seizure when, without raising objections thereto, they entered a plea
of guilty. It was Garcia who was unlawfully arrested and searched, not
the aforementioned three appellants. The legality of an arrest can be
contested only by the party whose rights have been impaired thereby.
Objection to an unlawful search and seizure is purely personal, and third
parties cannot avail themselves of it. 22

Indeed, the prosecution sufficiently proved the theft of the


perforated currency notes for retirement. It failed, however, to present
sufficient admissible evidence pointing to appellants as the authors of
the crime.
The evidence presented by the prosecution shows that there were
other people who had similar access to the shredding machine area and
the currency retirement vault. 23 Appellants were pinpointed by Labita
because of an anonymous phone call informing his superior of the people
allegedly behind the theft; and of the unexplained increase in their
spending, which was incompatible with their income. Labita, however, did
not submit sufficient evidence to support his allegation.
Without the extrajudicial confession and the perforated currency
notes, the remaining evidence would be utterly inadequate to overturn
the constitutional presumption of innocence.
Second Issue:
Demurrer to Evidence
Appellants contend that the trial court seriously erred when it
denied the demurrer to evidence filed by Appellants Loyola, De Leon and
Flores. Not one of the documents offered by the prosecution and
admitted in evidence by the RTC established the alleged qualified theft
of perforated notes, and not one of the pieces of evidence showed
appellants' participation in the commission of the crime.
On the exercise of sound judicial discretion rests the trial judge's
determination of the sufficiency or the insufficiency of the evidence
presented by the prosecution to establish a prima facie case against the
accused. Unless there is a grave abuse of discretion amounting to lack of
jurisdiction, the trial court's denial of a motion to dismiss may not be
disturbed. 24
As discussed earlier, the inadmissibility of the confessions of Garcia
did not become apparent until after Atty. Francisco had testified in court.
Even if the confiscated perforated notes from the person of the former
were held to be inadmissible, the confessions would still have
constituted prima facie evidence of the guilt of appellants. On that basis,
the trial court did not abuse its discretion in denying their demurrer to
evidence.
WHEREFORE, the assailed Decision is REVERSED and SET ASIDE.
Appellants are hereby ACQUITTED and ordered immediately RELEASED,
unless they are being detained for any other lawful cause. The director of
the Bureau of Corrections is hereby directed to submit his report on the
release of the appellant or the reason for his continued detention within
five (5) days from notice of this Decision. No costs.
SO ORDERED.
(People v. Garcia y Tupas, G.R. No. 145176, [March 30, 2004], 470 PHIL
|||

305-321)

[G.R. No. 83988. September 29, 1989.]


RICARDO C. VALMONTE AND UNION OF LAWYERS AND
ADVOCATES FOR PEOPLE'S RIGHTS
(ULAP), petitioners, vs. GEN. RENATO DE VILLA AND
NATIONAL CAPITAL REGION DISTRICT
COMMAND, respondents.

Ricardo C. Valmonte for himself and his co-petitioners.

SYLLABUS

SARMIENTO, J.,dissenting:
1. CONSTITUTIONAL LAW; CONSTITUTION; BEDROCK OF THE
REGIME OF LAW AND CONSTITUTIONALISM. — The Charter says that
the people enjoy the right of security of person, home, and effects.
(CONST.,art. III, sec. 2.) It is also the bedrock — the right of the people
to be left alone — on which the regime of law and constitutionalism
rest. It is not, as the majority would put it, a matter of "occasional
inconveniences, discomfort and even irritation." (Resolution, 4.) To say
that it is, is — so I submit — to trivialize the plain command of
the Constitution.
2. ID.;SEARCH AND SEIZURE; BURDEN OF PROVING
REASONABLENESS INCUMBENT UPON THE STATE. — While the right
against unreasonable searches and seizures, as my brethren advance,
is a right personal to the aggrieved party, the petitioners, precisely,
have come to Court because they had been, or had felt, aggrieved. I
submit that in that event, the burden is the State's, to demonstrate the
reasonableness of the search. The petitioners, Ricardo Valmonte in
particular, need not, therefore, have illustrated the "details of the
incident" (Resolution, supra, 4) in all their gore and gruesomeness.
3. ID.;ID.;ABSENCE ALONE OF A SEARCH WARRANT MAKES
CHECKPOINT SEARCHES UNREASONABLE. — The absence alone of a
search warrant, as I have averred, makes checkpoint searches
unreasonable, and by itself, subject to constitutional challenges.
(Supra.) As it is, "checkpoints",have become "search warrants" unto
themselves — a roving one at that.
4. ID.;ID.;CASE AT BAR NOT SIMPLY A POLICEMAN ON THE BEAT.
— The American cases the majority refers to involve routine checks
compelled by "probable cause".What we have here, however, is not
simply a policeman on the beat but armed men, CAFGU or Alsa Masa,
who hold the power of life or death over the citizenry, who fire with no
provocation and without batting an eyelash. They likewise shoot you
simply because they do not like your face.
DECISION

PADILLA,J :p

This is a petition for prohibition with preliminary injunction and/or


temporary restraining order, seeking the declaration of checkpoints in
Valenzuela, Metro Manila or elsewhere, as unconstitutional and the
dismantling and banning of the same or, in the alternative, to direct
the respondents to formulate guidelines in the implementation of
checkpoints, for the protection of the people.
Petitioner Ricardo C. Valmonte sues in his capacity as citizen of
the Republic, taxpayer, member of the Integrated Bar of the
Philippines (IBP),and resident of Valenzuela, Metro Manila; while
petitioner Union of Lawyers and Advocates for People's Rights (ULAP)
sues in its capacity as an association whose members are all
members of the IBP.
The factual background of the case is as follows:
On 20 January 1987, the National Capital Region District
Command (NCRDC) was activated pursuant to Letter of Instruction
02/87 of the Philippine General Headquarters, AFP, with the mission of
conducting security operations within its area of responsibility and
peripheral areas, for the purpose of establishing an effective territorial
defense, maintaining peace and order, and providing an atmosphere
conducive to the social, economic and political development of the
National Capital Region. 1 As part of its duty to maintain peace and
order, the NCRDC installed checkpoints in various parts of Valenzuela,
Metro Manila.
Petitioners aver that, because of the installation of said
checkpoints, the residents of Valenzuela are worried of being harassed
and of their safety being placed at the arbitrary, capricious and
whimsical disposition of the military manning the checkpoints,
considering that their cars and vehicles are being subjected to regular
searches and check-ups, especially at night or at dawn, without the
benefit of a search warrant and/or court order. Their alleged fear for
their safety increased when, at dawn of 9 July 1988, Benjamin Parpon,
a supply officer of the Municipality of Valenzuela, Bulacan, was gunned
down allegedly in cold blood by the members of the NCRDC manning
the checkpoint along McArthur Highway at Malinta, Valenzuela, for
ignoring and/or refusing to submit himself to the checkpoint and for
continuing to speed off inspite of warning shots fired in the air.
Petitioner Valmonte also claims that, on several occasions, he had
gone thru these checkpoints where he was stopped and his car
subjected to search/check-up without a court order or search warrant.
Petitioners further contend that the said checkpoints give the
respondents a blanket authority to make searches and/or seizures
without search warrant or court order in violation of
the Constitution; 2 and, instances have occurred where a citizen, while
not killed, had been harassed.
Petitioners' concern for their safety and apprehension at being
harassed by the military manning the checkpoints are not sufficient
grounds to declare the checkpoints as per se illegal. No proof has
been presented before the Court to show that, in the course of their
routine checks, the military indeed committed specific violations
of petitioners' right against unlawful search and seizure or other
rights.
In a case filed by the same petitioner organization, Union of
Lawyers and Advocates for People's Right (ULAP) vs. Integrated
National Police, 3 it was held that individual petitioners who do not
allege that any of their rights were violated are not qualified to bring
the action, as real parties in interest.
The constitutional right against unreasonable searches and
seizures is a personal right invocable only by those whose rights have
been infringed, 4 or threatened to be infringed. What constitutes a
reasonable or unreasonable search and seizure in any particular
case is purely a judicial question, determinable from a consideration
of the circumstances involved. 5
Petitioner Valmonte's general allegation to the effect that he had
been stopped and searched without a search warrant by the military
manning the checkpoints, without more, i.e., without stating the
details of the incidents which amount to a violation of his right against
unlawful search and seizure, is not sufficient to enable the Court to
determine whether there was a violation of Valmonte's right against
unlawful search and seizure. Not all searches and seizures are
prohibited. Those which are reasonable are not forbidden. A
reasonable search is not to be determined by any fixed formula but is
to be resolved according to the facts of each case. 6
Where, for example, the officer merely draws aside the curtain of
a vacant vehicle which is parked on the public fair grounds, 7 or simply
looks into a vehicle, 8or flashes a light therein, 9 these do not
constitute unreasonable search.
The setting up of the questioned checkpoints in Valenzuela (and
probably in other areas) may be considered as a security measure to
enable the NCRDC to pursue its mission of establishing effective
territorial defense and maintaining peace and order for the benefit of
the public. Checkpoints may also be regarded as measures to thwart
plots to destabilize the government, in the interest of public security.
In this connection, the Court may take judicial notice of the shift to
urban centers and their suburbs of the insurgency movement, so
clearly reflected in the increased killings in cities of police and
military men by NPA "sparrow units," not to mention the abundance of
unlicensed firearms and the alarming rise in lawlessness and violence
in such urban centers, not all of which are reported in media, most
likely brought about by deteriorating economic conditions — which all
sum up to what one can rightly consider, at the very least, as
abnormal times. Between the inherent right of the state to protect its
existence and promote public welfare and an individual's right against
a warrantless search which is however reasonablyconducted, the
former should prevail.
True, the manning of checkpoints by the military is susceptible of
abuse by the men in uniform, in the same manner that all
governmental power is susceptible of abuse. But, at the cost of
occasional inconvenience, discomfort and even irritation to the
citizen, the checkpoints during these abnormal times, when conducted
within reasonable limits, are part of the price we pay for an orderly
society and a peaceful community.
Finally, on 17 July 1988, military and police checkpoints in Metro
Manila were temporarily lifted and a review and refinement of the
rules in the conduct of the police and military manning the
checkpoints was ordered by the National Capital Regional Command
Chief and the Metropolitan Police Director. 10
WHEREFORE, the petition is DISMISSED.
SO ORDERED.
(Valmonte v. De Villa, G.R. No. 83988, [September 29, 1989], 258 PHIL
|||

838-848)

S-ar putea să vă placă și